399
[1] Gleim #: 1.1.1 -- Source: CIA 1195 III-12 Answer (A) is correct. TQM is an integrated system that anticipates, meets, and exceeds customers’ needs, wants, and expectations. Answer (B) is incorrect because reduced delivery time is one of many potential activities that need improvement. Answer (C) is incorrect because reduced delivery charges is one of many potential activities that need improvement. Answer (D) is incorrect because increased employee participation is necessary to achieve TQM, but it is not the primary purpose for establishing the program. [2] Gleim #: 1.1.2 -- Source: CIA 1195 III-13 Answer (A) is incorrect because prevention is ordinarily less costly than the combined costs of appraisal, internal failure, and external failure. Answer (B) is correct. Prevention attempts to avoid defective output. Prevention costs include preventive maintenance, employee training, review of equipment design, and evaluation of suppliers. Prevention is less costly than detection and correction of defective output. Answer (C) is incorrect because prevention is ordinarily less costly than the combined costs of appraisal, internal failure, and external failure. Answer (D) is incorrect because prevention is ordinarily less costly than the combined costs of appraisal, internal failure, and external failure. [3] Gleim #: 1.1.3 -- Source: CIA 1195 III-28 Answer (A) is incorrect because other quality control processes can also be expensive. Answer (B) is incorrect because reworking defective items may be possible although costly. Answer (C) is incorrect because no quality control system will be 100% effective. Answer (D) is correct. The process used to produce the goods is not thoroughly reviewed and evaluated for efficiency and effectiveness. Preventing defects and increasing efficiency by improving the production process raises quality standards and decreases costs. [4] Gleim #: 1.1.4 -- Source: CIA 1196 III-24 Answer (A) is correct. Total quality management emphasizes quality as a basic organizational function. TQM is the continuous pursuit of quality in every aspect of organizational activities. One of the basic tenets of TQM is doing it right the first time. Thus, errors should be caught and corrected at the source. Answer (B) is incorrect because total quality management emphasizes discovering errors throughout the process, not during inspection of finished goods. Answer (C) is incorrect because all members of the organization assume responsibility for quality of the products and services. Answer (D) is incorrect because the total quality management philosophy recommends limiting the number of suppliers to create a strong relationship. Gleim's CIA Test Prep: Part III: Business Analysis and Information Technology Answer Explanations (1312 questions) Copyright 2008 Gleim Publications, Inc. Page 1 Printed for Mamdouh Farag

P.3 Answer Explanation

Embed Size (px)

DESCRIPTION

Study

Citation preview

Page 1: P.3 Answer Explanation

[1] Gleim #: 1.1.1 -- Source: CIA 1195 III-12

Answer (A) is correct. TQM is an integrated system that anticipates, meets, and exceeds customers’ needs, wants, andexpectations.

Answer (B) is incorrect because reduced delivery time is one of many potential activities that need improvement.

Answer (C) is incorrect because reduced delivery charges is one of many potential activities that need improvement.

Answer (D) is incorrect because increased employee participation is necessary to achieve TQM, but it is not the primarypurpose for establishing the program.

[2] Gleim #: 1.1.2 -- Source: CIA 1195 III-13

Answer (A) is incorrect because prevention is ordinarily less costly than the combined costs of appraisal, internal failure,and external failure.

Answer (B) is correct. Prevention attempts to avoid defective output. Prevention costs include preventive maintenance,employee training, review of equipment design, and evaluation of suppliers. Prevention is less costly than detection andcorrection of defective output.

Answer (C) is incorrect because prevention is ordinarily less costly than the combined costs of appraisal, internal failure,and external failure.

Answer (D) is incorrect because prevention is ordinarily less costly than the combined costs of appraisal, internal failure,and external failure.

[3] Gleim #: 1.1.3 -- Source: CIA 1195 III-28

Answer (A) is incorrect because other quality control processes can also be expensive.

Answer (B) is incorrect because reworking defective items may be possible although costly.

Answer (C) is incorrect because no quality control system will be 100% effective.

Answer (D) is correct. The process used to produce the goods is not thoroughly reviewed and evaluated for efficiency andeffectiveness. Preventing defects and increasing efficiency by improving the production process raises quality standardsand decreases costs.

[4] Gleim #: 1.1.4 -- Source: CIA 1196 III-24

Answer (A) is correct. Total quality management emphasizes quality as a basic organizational function. TQM is thecontinuous pursuit of quality in every aspect of organizational activities. One of the basic tenets of TQM is doing it rightthe first time. Thus, errors should be caught and corrected at the source.

Answer (B) is incorrect because total quality management emphasizes discovering errors throughout the process, notduring inspection of finished goods.

Answer (C) is incorrect because all members of the organization assume responsibility for quality of the products andservices.

Answer (D) is incorrect because the total quality management philosophy recommends limiting the number of suppliers tocreate a strong relationship.

Gleim's CIA Test Prep: Part III: Business Analysis and Information TechnologyAnswer Explanations

(1312 questions)

Copyright 2008 Gleim Publications, Inc. Page 1Printed for Mamdouh Farag

Page 2: P.3 Answer Explanation

[5] Gleim #: 1.1.5 -- Source: CIA 596 III-29

Answer (A) is incorrect because one of the 14 points recommends elimination of numerical quotas. MBO causesaggressive pursuit of numerical quotas.

Answer (B) is incorrect because informal learning from coworkers serves to entrench bad work habits. One of the 14points stresses proper training of everyone.

Answer (C) is incorrect because another of the 14 points states that quality by final inspection is unnecessary if quality isbuilt in from the start.

Answer (D) is correct. According to management theorist W. Edwards Deming’s well-known 14 points, education andself-improvement are essential. Knowledge is opportunity. Hence, continuous improvement should be everyone’s primarycareer objective.

[6] Gleim #: 1.1.6 -- Source: CIA 596 III-30

Answer (A) is incorrect because hierarchal organization stifles TQM.

Answer (B) is incorrect because TQM works best with teams of people from different specialties.

Answer (C) is correct. TQM advocates replacement of the traditional hierarchal structure with teams of people fromdifferent specialties. This change follows from TQM’s emphasis on empowering employees and teamwork. Employeesshould have proper training, necessary information, and the best tools; be fully engaged in the decision process; andreceive fair compensation. If such empowered employees are assembled in teams of individuals with the required skills,TQM theorists believe they will be more effective than people performing their tasks separately in a rigid structure.

Answer (D) is incorrect because teamwork is essential for TQM.

[7] Gleim #: 1.1.7 -- Source: CMA 1296 3-22

Answer (A) is incorrect because training costs are prevention costs.

Answer (B) is incorrect because the costs of external failure, such as warranty expense, product liability, and customer illwill, arise when problems are discovered after products have been shipped.

Answer (C) is correct. Internal failure costs are incurred when detection of defective products occurs before shipment.Examples of internal failure costs are scrap, rework, tooling changes, and downtime.

Answer (D) is incorrect because prevention costs are incurred to avoid defective output. Examples include preventivemaintenance, employee training, review of equipment design, and evaluation of suppliers.

Gleim's CIA Test Prep: Part III: Business Analysis and Information TechnologyAnswer Explanations

(1312 questions)

Copyright 2008 Gleim Publications, Inc. Page 2Printed for Mamdouh Farag

Page 3: P.3 Answer Explanation

[8] Gleim #: 1.1.8 -- Source: CMA 1295 3-12

Answer (A) is incorrect because carrying cost is not one of the elements of quality costs.

Answer (B) is correct. The following are the four categories of quality costs: prevention, appraisal, internal failure, andexternal failure (lost opportunity). Costs of prevention include attempts to avoid defective output, including employeetraining, review of equipment design, preventive maintenance, and evaluation of suppliers. Appraisal costs include qualitycontrol programs, inspection, and testing. Internal failure costs are incurred when detection of defective products occursbefore shipment, including scrap, rework, tooling changes, and downtime. External failure costs are incurred after theproduct has been shipped, including the costs associated with warranties, product liability, and customer ill will.

Answer (C) is incorrect because training cost is not a category of quality costs.

Answer (D) is incorrect because warranty, product liability, and training are not cost categories identified by SMA 4R.

[9] Gleim #: 1.1.9 -- Source: CMA 1295 3-14

Answer (A) is incorrect because internal failure costs arise after poor quality has been found; statistical quality control isdesigned to detect quality problems.

Answer (B) is incorrect because statistical quality control is not a training cost.

Answer (C) is incorrect because external failure costs are incurred after the product has been shipped, including the costsassociated with warranties, product liability, and customer ill will.

Answer (D) is correct. The following are the four categories of quality costs: prevention, appraisal, internal failure, andexternal failure (lost opportunity). Appraisal costs include quality control programs, inspection, and testing. However,some authorities regard statistical quality and process control as preventive activities because they not only detect faultywork but also allow for adjustment of processes to avoid future defects.

[10] Gleim #: 1.1.10 -- Source: CMA 697 3-28

Answer (A) is incorrect because $786 is the appraisal cost.

Answer (B) is incorrect because $1,154 is the prevention cost.

Answer (C) is correct. The costs of prevention and appraisal are conformance costs that serve as financial measures ofinternal performance. Prevention costs are incurred to prevent defective output. These costs include preventivemaintenance, employee training, review of equipment design, and evaluation of suppliers. Appraisal costs are incurred todetect nonconforming output. They embrace such activities as statistical quality control programs, inspection, and testing.The equipment maintenance cost of $1,154 is a prevention cost. The product testing cost of $786 is an appraisal cost.Their sum is $1,940.

Answer (D) is incorrect because $2,665 includes rework, an internal failure cost.

Gleim's CIA Test Prep: Part III: Business Analysis and Information TechnologyAnswer Explanations

(1312 questions)

Copyright 2008 Gleim Publications, Inc. Page 3Printed for Mamdouh Farag

Page 4: P.3 Answer Explanation

[11] Gleim #: 1.1.11 -- Source: CMA 697 3-27

Answer (A) is incorrect because the costs of warranty claims are readily measurable external failure costs captured by theaccounting system.

Answer (B) is incorrect because the cost of design engineering is a prevention cost that is usually included in cost-of-quality reports.

Answer (C) is incorrect because the cost of supplier evaluations is a prevention cost that is usually included in cost-of-quality reports.

Answer (D) is correct. A cost-of-quality report includes most costs related to quality, including the costs of externalfailure, internal failure, prevention, and appraisal. Lost contribution margins from poor product quality are external failurecosts that normally do not appear on a cost-of-quality report because they are opportunity costs. Opportunity costs are notusually recorded by the accounting system, thereby understating the costs of poor quality. Lost contribution margins fromreduced sales, market share, and sales prices are external failure costs that are also not usually included in a cost-of-quality report.

[12] Gleim #: 1.1.12 -- Source: Publisher

Answer (A) is incorrect because the traditional view of quality treats a certain number of defective units as acceptable.

Answer (B) is incorrect because the traditional view of quality deems that a unit is acceptable if it is within a range ofspecified values.

Answer (C) is incorrect because the robust quality concept is an extension of the zero-defects approach. The goal of robustquality is in every case to reach a target value, not merely a range of acceptable values.

Answer (D) is correct. Conformance is how well a product and its components meet applicable standards. The traditionalview is that conforming products are those with characteristics that lie within an acceptable specified range of values thatincludes a target value. This view also regarded a certain percentage of defective (nonconforming) units as acceptable.The traditional view was superseded by the zero-defects approach that sought to eliminate all nonconforming output. Anextension of this approach is the robust quality concept. Its goal is to reach the target value in every case. The reason isthat hidden quality costs occur when output varies from the target even though the units are within specifications.

[13] Gleim #: 1.1.13 -- Source: Publisher

Answer (A) is correct. PDCA is a “management by fact” or scientific method approach to continuous improvement.PDCA creates a process-centered environment because it involves studying the current process, collecting and analyzingdata to identify causes of problems, planning for improvement, and deciding how to measure improvement (Plan). Theplan is then implemented on a small scale if possible (Do). The next step is to determine what happened (Check). If theexperiment was successful, the plan is fully implemented (Act). The cycle is then repeated using what was learned fromthe preceding cycle.

Answer (B) is incorrect because competitive benchmarking is an ongoing evaluation of the practices of best-in-classorganizations.

Answer (C) is incorrect because quality deployment is the translation of customer requirements into design requirements.

Answer (D) is incorrect because the “quality at the source” concept emphasizes the responsibility of every employee, workgroup, department, or supplier to inspect the work.

Gleim's CIA Test Prep: Part III: Business Analysis and Information TechnologyAnswer Explanations

(1312 questions)

Copyright 2008 Gleim Publications, Inc. Page 4Printed for Mamdouh Farag

Page 5: P.3 Answer Explanation

[14] Gleim #: 1.1.14 -- Source: Publisher

Answer (A) is incorrect because systems development costs are prevention (conformance) costs.

Answer (B) is incorrect because costs of inspecting in-process items are appraisal (conformance) costs.

Answer (C) is correct. Nonconformance costs include internal and external failure costs. External failure costs includeenvironmental costs, e.g., fines for violations of environmental laws and loss of customer goodwill.

Answer (D) is incorrect because costs of quality circles are prevention (conformance) costs.

[15] Gleim #: 1.1.15 -- Source: Publisher

Answer (A) is incorrect because direct labor costs are usually a higher percentage of total costs in service organizations.

Answer (B) is correct. External failure costs arise when problems occur after delivery. They occur because products orservices are nonconforming or otherwise do not satisfy customers. External failure costs in service enterprises are evenmore important than in manufacturing environments. Faulty goods sometimes may be reworked or replaced to acustomer’s satisfaction, but poor service tends to result in a loss of customers.

Answer (C) is incorrect because service activities are usually more labor intensive than in modern manufacturingenvironments. Thus, more efficient labor usage is more likely to be viewed as a threat to employee job security in serviceorganizations.

Answer (D) is incorrect because the badwill resulting from poor service may be even more likely than a defective productto result in loss of customers.

[16] Gleim #: 1.1.16 -- Source: Publisher

Answer (A) is incorrect because, according to the robust quality concept, conformance costs do not necessarily have tocontinue to increase to obtain additional reductions in nonconformance costs.

Answer (B) is incorrect because, according to the robust quality concept, conformance costs do not necessarily have tocontinue to increase to obtain additional reductions in nonconformance costs.

Answer (C) is incorrect because, according to the robust quality concept, conformance costs do not necessarily have tocontinue to increase to obtain additional reductions in nonconformance costs.

Answer (D) is correct. The optimal level of quality costs traditionally has been deemed to occur where the conformancecost curve intercepts the nonconformance cost curve, which corresponds to the minimum point on the total cost curve.Thus, beyond some point, incurrence of prevention and appraisal costs is not cost beneficial. However, the modern robustquality view is that this relationship does not always hold. Improving quality and reducing costs in each category may bepossible if the most efficient prevention methods are applied. For example, selection of a supplier meeting high qualitystandards regarding defect rates and delivery times may drive down not only failure costs but also the prevention andappraisal costs incurred when supplier performance was less reliable. Thus, conformance costs do not have to continue toincrease to obtain additional reductions in nonconformance costs.

Gleim's CIA Test Prep: Part III: Business Analysis and Information TechnologyAnswer Explanations

(1312 questions)

Copyright 2008 Gleim Publications, Inc. Page 5Printed for Mamdouh Farag

Page 6: P.3 Answer Explanation

[17] Gleim #: 1.1.17 -- Source: CMA 693 4-28

Answer (A) is incorrect because it is related to breakeven point, not breakeven time.

Answer (B) is incorrect because the payback period equals investment divided by annual undiscounted net cash inflows.

Answer (C) is incorrect because the payback period is the period required for total undiscounted cash inflows to equaltotal undiscounted cash outflows.

Answer (D) is correct. Breakeven time evaluates the rapidity of new product development. The usual calculationdetermines the period beginning with project approval that is required for the discounted cumulative cash inflows to equalthe discounted cumulative cash outflows. However, it may also be calculated as the point at which discounted cumulativecash inflows on a project equal discounted total cash outflows. The concept is similar to the payback period, but it is moresophisticated because it incorporates the time value of money. It also differs from the payback method because the periodcovered begins at the outset of a project, not when the initial cash outflow occurs.

[18] Gleim #: 1.1.18 -- Source: CIA 594 III-56

Answer (A) is incorrect because the percentage of shipments returned measures quality by the number of defective units.

Answer (B) is correct. The number of parts shipped per day would most likely be used as a measure of the effectivenessand efficiency of shipping procedures, not the quality of the product. This measure does not consider how many of theparts are defective.

Answer (C) is incorrect because the number of defective parts per million measures quality by the number of defectiveunits.

Answer (D) is incorrect because the percentage of products passing quality tests the first time measures quality by thenumber of nondefective products.

[19] Gleim #: 1.1.19 -- Source: CMA 694 3-16

Answer (A) is correct. TQM emphasizes the supplier’s relationship with the customer and recognizes that everyone in aprocess is at some time a customer or supplier of someone else, either within or outside the organization. The costs ofquality include costs of conformance and costs of nonconformance. Costs of conformance include prevention costs andappraisal (inspection) costs. Nonconformance costs are composed of internal failure costs and external failure costs, suchas lost opportunity. Conformance costs (prevention and appraisal) increased substantially, whereas the nonconformancecosts (internal and external failure) decreased. Hence, the increase in conformance costs resulted in a higher-qualityproduct.

Answer (B) is incorrect because prevention costs also increased substantially, which could also have led to higher-qualityproducts.

Answer (C) is incorrect because scrap and rework are internal failure costs, which decreased by 40%.

Answer (D) is incorrect because returns and repairs are external failure costs, which decreased by 48%.

Gleim's CIA Test Prep: Part III: Business Analysis and Information TechnologyAnswer Explanations

(1312 questions)

Copyright 2008 Gleim Publications, Inc. Page 6Printed for Mamdouh Farag

Page 7: P.3 Answer Explanation

[20] Gleim #: 1.1.20 -- Source: CMA 693 3-17

Answer (A) is correct. Manufacturing cycle efficiency equals the time required for value-added production divided bytotal manufacturing lead time. For this order, the total lead time is 12 days (5.0 + 1.5 + 3.0 + 2.5), and the manufacturingcycle efficiency is 25% (3 days of processing ÷ 12).

Answer (B) is incorrect because 13.6% includes the 10 days prior to production in the denominator, a period not includedin the calculation of manufacturing cycle efficiency.

Answer (C) is incorrect because inspection time and move time should be included in the denominator.

Answer (D) is incorrect because the calculation involves dividing the 3 days of processing time by the total of 12 days tocomplete production.

[21] Gleim #: 1.1.21 -- Source: CMA 693 3-18

Answer (A) is incorrect because 7 days excludes the wait time.

Answer (B) is incorrect because 12 days ignores the 10 days of the waiting period prior to the start of production.

Answer (C) is incorrect because 15 days incorporates the wait time but not the production periods.

Answer (D) is correct. The delivery cycle time is defined as the entire time from receipt of the order until delivery of theorder. This period equals 22 days (10.0 + 5.0 + 1.5 + 3.0 + 2.5)

[22] Gleim #: 1.1.22 -- Source: Publisher

Answer (A) is incorrect because $625 equals $.625 times 1,000 units.

Answer (B) is correct. The quality loss constant (k) in the Taguchi quality loss function equation is $10 [$10 loss at thespecification limit ÷ (1 mm)²]. Accordingly, the quality loss for the sample may be determined as follows:

Measurement Actual – TargetUnit in mm (x – T) (x – T)2 k(x – T)2

1 99.0 –1 1.00 $10.002 101.0 +1 1.00 10.003 99.5 –.5 .25 2.504 100.5 +.5 .25 2.50

2.50 $25.00

The average per unit quality loss for the sample is $6.25 ($25 ÷ 4). Consequently, the total estimated loss for 1,000components is $6,250.

Answer (C) is incorrect because $10,000 equals $10 times 1,000 units.

Answer (D) is incorrect because $25,000 is based on the total loss for the sample ($25), not the unit average loss ($6.25).

Gleim's CIA Test Prep: Part III: Business Analysis and Information TechnologyAnswer Explanations

(1312 questions)

Copyright 2008 Gleim Publications, Inc. Page 7Printed for Mamdouh Farag

Page 8: P.3 Answer Explanation

[23] Gleim #: 1.1.23 -- Source: Publisher

Answer (A) is incorrect because they are true statements about benchmarking.

Answer (B) is incorrect because they are true statements about benchmarking.

Answer (C) is incorrect because they are true statements about benchmarking.

Answer (D) is correct. Benchmarking is an ongoing process that entails quantitative and qualitative measurement of thedifference between the company’s performance of an activity and the performance by a best-in-class organization. Thebenchmarking organization against which a firm is comparing itself need not be a direct competitor. The importantconsideration is that the benchmarking organization be an outstanding performer in its industry.

[24] Gleim #: 1.1.24 -- Source: CIA 595 III-22

Answer (A) is incorrect because the labor rate of a competitor is a financial benchmark.

Answer (B) is incorrect because the cost per pound of a product at the company’s most efficient plant is a financialbenchmark.

Answer (C) is incorrect because the cost of a training program is a financial benchmark.

Answer (D) is correct. Benchmarking is a continuous evaluation of the practices of the best organizations in their classand the adaptation of processes to reflect the best of these practices. It requires analysis and measurement of key outputsagainst those of the best organizations. This procedure also involves identifying the underlying key actions and causes thatcontribute to the performance difference. The percentage of orders delivered on time at the company’s most efficient plantis an example of an internal nonfinancial benchmark.

[25] Gleim #: 1.1.25 -- Source: Publisher

Answer (A) is incorrect because market share is an external nonfinancial measure.

Answer (B) is incorrect because delivery performance is an external nonfinancial measure.

Answer (C) is incorrect because customer satisfaction is an external nonfinancial measure.

Answer (D) is correct. Feedback regarding managerial performance may take the form of financial and nonfinancialmeasures that may be internally or externally generated. Moreover, different measures have a long-term or short-termemphasis. Examples of internal nonfinancial measures are product quality, new product development time, andmanufacturing lead time (cycle time).

[26] Gleim #: 1.1.26 -- Source: CIA 597 III-14

Answer (A) is incorrect because customer orientation is difficult to quantify.

Answer (B) is incorrect because the standard specified is vague.

Answer (C) is incorrect because no measure of a positive attitude has been specified for the employee.

Answer (D) is correct. A criterion that requires all customer inquiries to be answered within 7 days of receipt permitsaccurate measurement of performance. The quantitative and specific nature of the appraisal using this standard avoids thevagueness, subjectivity, and personal bias that may afflict other forms of personnel evaluations.

Gleim's CIA Test Prep: Part III: Business Analysis and Information TechnologyAnswer Explanations

(1312 questions)

Copyright 2008 Gleim Publications, Inc. Page 8Printed for Mamdouh Farag

Page 9: P.3 Answer Explanation

[27] Gleim #: 1.1.27 -- Source: CIA 597 III-3

Answer (A) is incorrect because competitive benchmarking is just one tool for implementing TQM.

Answer (B) is correct. TQM is a comprehensive approach to quality. It treats the pursuit of quality as a basicorganizational function that is as important as production or marketing. TQM is the continuous pursuit of quality in everyaspect of organizational activities through a philosophy of doing it right the first time, employee training andempowerment, promotion of teamwork, improvement of processes, and attention to satisfaction of customers, both internaland external. TQM emphasizes the supplier’s relationship with the customer, identifies customer needs, and recognizesthat everyone in a process is at some time a customer or supplier of someone else, either inside or outside of theorganization.

Answer (C) is incorrect because TQM’s primary focus is not profitability.

Answer (D) is incorrect because TQM’s primary focus is not cost reduction.

[28] Gleim #: 1.1.28 -- Source: CIA 597 III-4

Answer (A) is incorrect because TQM emphasizes prevention, not rework. The approach of TQM is to build in and designin quality, not to “fix it in” or “inspect it in.”

Answer (B) is correct. Total quality management emphasizes quality as a basic organizational function. TQM is thecontinuous pursuit of quality in every aspect of organizational activities. One of the basic tenets of TQM is doing it rightthe first time. Thus, errors should be caught and corrected at the source, and quality should be built in (designed in) fromthe start.

Answer (C) is incorrect because TQM emphasizes prevention, not inspection. The approach of TQM is to build in anddesign in quality, not to “fix it in” or “inspect it in.”

Answer (D) is incorrect because TQM emphasizes prevention, not adjustment. The approach of TQM is to build in anddesign in quality, not to “fix it in” or “inspect it in.”

[29] Gleim #: 1.1.29 -- Source: CIA 597 III-24

Answer (A) is incorrect because teams are often inefficient and costly.

Answer (B) is incorrect because high motivation does not directly affect the process improvement that is the key to qualityimprovement.

Answer (C) is correct. TQM promotes teamwork by modifying or eliminating traditional (and rigid) vertical hierarchiesand instead forming flexible groups of specialists. Quality circles, cross-functional teams, and self-managed teams aretypical formats. Teams are an excellent vehicle for encouraging the sharing of ideas and removing process improvementobstacles.

Answer (D) is incorrect because the use of teams with less supervision and reduced staffing may be by-products of TQM,but they are not ultimate objectives.

Gleim's CIA Test Prep: Part III: Business Analysis and Information TechnologyAnswer Explanations

(1312 questions)

Copyright 2008 Gleim Publications, Inc. Page 9Printed for Mamdouh Farag

Page 10: P.3 Answer Explanation

[30] Gleim #: 1.1.30 -- Source: CIA 597 III-96

Answer (A) is incorrect because prevention attempts to avoid defective output, e.g., by employee training, review ofequipment design, preventive maintenance, and evaluation of suppliers.

Answer (B) is correct. The categories of quality costs include conformance costs (prevention and appraisal) andnonconformance costs (internal failure and external failure). Appraisal costs embrace such activities as statistical qualitycontrol programs, inspection, and testing. Thus, the cost of detecting nonconforming products is an appraisal cost.

Answer (C) is incorrect because internal failure costs are incurred when detection of defective products occurs beforeshipment, including scrap, rework, tooling changes, and downtime.

Answer (D) is incorrect because external failure costs are incurred after shipment, including the costs associated withwarranties, product liability, and loss of customer goodwill.

[31] Gleim #: 1.1.31 -- Source: CIA 597 III-97

Answer (A) is incorrect because $643,000 omits scheduled equipment maintenance and includes labor inspection of rawmaterials (an appraisal cost).

Answer (B) is correct. Prevention attempts to avoid defective output, e.g., by employee training, review of equipmentdesign, preventive maintenance, and evaluation of suppliers. Accordingly, the preventive costs equal $701,000 ($275,000design reviews + $180,000 process engineering + $90,000 scheduled maintenance + $156,000 training).

Answer (C) is incorrect because $736,000 includes the cost of product testing equipment (an appraisal cost).

Answer (D) is incorrect because $768,000 includes the cost of product testing equipment and labor inspection of rawmaterials. Both costs are appraisal costs.

[32] Gleim #: 1.1.32 -- Source: CIA 597 III-23

Answer (A) is incorrect because total quality management (TQM) de-emphasizes specialized quality inspectors.

Answer (B) is correct. TQM emphasizes satisfaction of customers, both internal and external. TQM considers thesupplier’s relationship with the customer, identifies customer needs, and recognizes that everyone in a process is at sometime a customer or supplier of someone else, either inside or outside of the organization. Thus, TQM begins with externalcustomer requirements, identifies internal customer-supplier relationships and requirements, and establishes requirementsfor external suppliers.

Answer (C) is incorrect because centralization often needs to be reduced to implement a TQM process.

Answer (D) is incorrect because TQM involves continuous improvement; once a standard is reached, continuousimprovement requires its constant reevaluation.

Gleim's CIA Test Prep: Part III: Business Analysis and Information TechnologyAnswer Explanations

(1312 questions)

Copyright 2008 Gleim Publications, Inc. Page 10Printed for Mamdouh Farag

Page 11: P.3 Answer Explanation

[33] Gleim #: 1.1.33 -- Source: CIA 597 III-20

Answer (A) is correct. TQM is a comprehensive approach to quality. It treats the pursuit of quality as a basicorganizational function that is as important as production or marketing. Because TQM affects every aspect of theorganization’s activities, it permeates the organizational culture. Thus, the cumulative effect of TQM’s continuousimprovement process can attract and hold customers and cannot be duplicated by competitors.

Answer (B) is incorrect because new products can be quickly copied by competitors and therefore do not provide asustained competitive advantage.

Answer (C) is incorrect because TQM does not focus solely on cost reduction.

Answer (D) is incorrect because TQM is only one tool of strategic management.

[34] Gleim #: 1.1.34 -- Source: CIA 1196 III-27

Answer (A) is incorrect because improving the quality of leadership is not the primary goal of quality control circles.

Answer (B) is correct. Quality control circles are used to obtain voluntary input from employees to promote problemsolving. Potential benefits include lower costs, better employer-employee relations, and greater employee commitment.

Answer (C) is incorrect because improved communication is a by-product of quality control circles.

Answer (D) is incorrect because the emergence of team leaders who can be targeted for further leadership development isa by-product of the quality control circles.

[35] Gleim #: 1.1.35 -- Source: CIA 1196 III-23

Answer (A) is correct. The intent of quality control is to ensure that goods and services conform to the designspecifications. Whether the focus is on feedforward, feedback, or concurrent control, the emphasis is on ensuring productor service conformity.

Answer (B) is incorrect because quality control is geared towards satisfying the customer, not upper management.

Answer (C) is incorrect because ensuring the conformance with ISO-9000 specifications is a component of a complianceaudit, not quality control.

Answer (D) is incorrect because determining the appropriate timing of inspections is only one step towards approachingquality control. Consequently, it is not the primary component of the quality control function.

[36] Gleim #: 1.1.36 -- Source: CIA 594 II-40

Answer (A) is incorrect because PERT breaks down a project into a set of events, arranges the events into a strict prioritynetwork, and establishes a completion time for each event.

Answer (B) is incorrect because CPM is used to control large scale projects. Essentially a subset of PERT, CPM uses adeterministic rather than a probabilistic estimate of the time required for task completion.

Answer (C) is correct. Statistical process (quality) control is not used to control large scale projects. Statistical qualitycontrol is a method of determining whether the shipment or production run of units lies within acceptable limits. It is alsoused to determine whether production processes are out of control.

Answer (D) is incorrect because Gantt charts compare scheduled production with actual production. Its control functionlies in its ability to identify variations and thus formulate corrective actions.

Gleim's CIA Test Prep: Part III: Business Analysis and Information TechnologyAnswer Explanations

(1312 questions)

Copyright 2008 Gleim Publications, Inc. Page 11Printed for Mamdouh Farag

Page 12: P.3 Answer Explanation

[37] Gleim #: 1.1.37 -- Source: CIA 1184 III-12

Answer (A) is incorrect because it describes product (not process) quality control.

Answer (B) is correct. Process analysis studies the means of producing a product or service for the purpose of loweringcosts and increasing efficiency while producing items of appropriate quality. The effect of process analysis is to act as apre-control to prevent defects or bad service. It differs from product (or service) quality control, which involves inspectionduring production to eliminate unacceptable results.

Answer (C) is incorrect because it is applicable to process (not product) quality control.

Answer (D) is incorrect because process quality control and product quality control are both important elements in aquality control program.

[38] Gleim #: 1.1.38 -- Source: CIA 594 III-53

Answer (A) is incorrect because TQM has a broader emphasis. It focuses on improving quality, reducing cycle time,providing increased customer satisfaction, and achieving the lowest overall business cost. Reducing the cost of inspectionhelps achieve the lowest overall business cost.

Answer (B) is incorrect because this statement describes participative management.

Answer (C) is incorrect because TQM has a broader emphasis. It focuses on improving quality, reducing cycle time,providing increased customer satisfaction, and achieving the lowest overall business cost. Encouraging cross-functionalteamwork helps achieve the lowest overall business cost.

Answer (D) is correct. TQM establishes quality as an organizational objective and views it as a major component of theorganization’s service to its customers. It emphasizes employee training and commitment, product/service design andproduction, and customer service. Ordinarily, the quality of a product or service is as important to customers as cost andtimeliness. Superior product quality is not attained merely through more inspection, better statistical quality control, andcross-functional teamwork. Manufacturers must make fundamental changes in the way they produce products and do eachjob right the first time.

[39] Gleim #: 1.1.39 -- Source: CIA 596 III-32

Answer (A) is incorrect because customers are internal as well as external.

Answer (B) is incorrect because customers are internal as well as external.

Answer (C) is correct. One of the tenets of TQM is customer orientation, whether the customer is internal or external. Aninternal customer is a member of the organization who relies on another member’s work to accomplish his/her task.

Answer (D) is incorrect because customers are internal as well as external.

Gleim's CIA Test Prep: Part III: Business Analysis and Information TechnologyAnswer Explanations

(1312 questions)

Copyright 2008 Gleim Publications, Inc. Page 12Printed for Mamdouh Farag

Page 13: P.3 Answer Explanation

[40] Gleim #: 1.1.40 -- Source: CIA 596 II-1

Answer (A) is incorrect because turnover of employees is not a financial benchmark.

Answer (B) is incorrect because employee participation in setting budgets is not a financial benchmark.

Answer (C) is correct. Internal benchmarking is the application of best practices in one part of the organization (e.g., ahigh-performing branch store) to its other parts (other branches). This process requires, among other things, use ofquantitative and qualitative measures. A key indicator for financial performance measurement is the amount of bad debtwrite-offs. A high level of bad debt write-offs could indicate fraud, which would compromise the accuracy and reliabilityof financial reports. Bad debt write-offs may result from recording fictitious sales.

Answer (D) is incorrect because the number of suppliers is not a financial benchmark.

[41] Gleim #: 1.1.41 -- Source: CIA 596 III-23

Answer (A) is incorrect because reducing the cost of inspection is a detail of the TQM emphasis.

Answer (B) is incorrect because implementing better statistical quality control techniques is a detail of the TQMemphasis.

Answer (C) is correct. The basic principles of TQM include doing each job right the first time, being customer oriented,committing the company culture to continuous improvement, and building teamwork and employee empowerment.

Answer (D) is incorrect because encouraging cross-functional teamwork is a detail of the TQM emphasis.

[42] Gleim #: 1.1.42 -- Source: CIA 1195 III-98

Answer (A) is correct. The index for May was 40% [($4,000 + $6,000 + $12,000 + $14,000) ÷ $90,000], and the indexfor June was 36% [($5,000 + $5,000 + $15,000 + $11,000) ÷ $100,000].

Answer (B) is incorrect because the index decreased.

Answer (C) is incorrect because the increase in prevention costs was 10% of the increase in labor costs.

Answer (D) is incorrect because the decrease in appraisal costs was 10% of the increase in labor costs.

[43] Gleim #: 1.1.43 -- Source: CIA 1196 III-23

Answer (A) is correct. The intent of quality control is to ensure that goods and services conform to the designspecifications. Whether the focus is on feedforward, feedback, or concurrent control, the emphasis is on ensuring productor service conformity.

Answer (B) is incorrect because quality control is geared towards satisfying the customer, not upper management.

Answer (C) is incorrect because ensuring the conformance with ISO-9000 specifications is a component of a complianceaudit, not quality control.

Answer (D) is incorrect because determining the appropriate timing of inspections is only one step towards approachingquality control. Consequently, it is not the primary component of the quality control function.

Gleim's CIA Test Prep: Part III: Business Analysis and Information TechnologyAnswer Explanations

(1312 questions)

Copyright 2008 Gleim Publications, Inc. Page 13Printed for Mamdouh Farag

Page 14: P.3 Answer Explanation

[44] Gleim #: 1.1.44 -- Source: CMA 693 3-13

Answer (A) is incorrect because prevention costs are incurred in an attempt to avoid defective output.

Answer (B) is correct. Quality-related costs can be subdivided into four categories: external failure costs, internal failurecosts, prevention costs, and appraisal costs. Appraisal costs embrace such activities as statistical quality control programs,inspection, and testing. Thus, the cost of detecting nonconforming individual products is an appraisal cost.

Answer (C) is incorrect because external failure costs arise when quality problems occur after shipment.

Answer (D) is incorrect because opportunity costs are not specifically associated with product quality. Opportunity cost isthe maximum benefit forgone by using a scarce resource for a given purpose; it is the benefit provided by the next best useof that resource.

[45] Gleim #: 1.1.45 -- Source: CMA 694 3-16

Answer (A) is correct. TQM emphasizes the supplier’s relationship with the customer and recognizes that everyone in aprocess is at some time a customer or supplier of someone else, either within or outside the organization. The costs ofquality include costs of conformance and costs of nonconformance. Costs of conformance include prevention costs andappraisal (inspection) costs. Nonconformance costs are composed of internal failure costs and external failure costs, suchas lost opportunity. Conformance costs (prevention and appraisal) increased substantially, whereas the nonconformancecosts (internal and external failure) decreased. Hence, the increase in conformance costs resulted in a higher qualityproduct.

Answer (B) is incorrect because prevention costs also increased substantially, which could also have led to higher qualityproducts.

Answer (C) is incorrect because scrap and rework are internal failure costs, which decreased by 40%.

Answer (D) is incorrect because returns and repairs are external failure costs, which decreased by 48%.

[46] Gleim #: 1.1.46 -- Source: IIA, adapted

Answer (A) is incorrect because prevention costs are incurred to prevent the production of products that do not conform tospecifications.

Answer (B) is correct. Appraisal embraces such activities as statistical quality control programs, inspection, and testing.Appraisal costs are those costs (such as test equipment maintenance and destructive testing) incurred to detect whichproducts do not conform to specifications.

Answer (C) is incorrect because rework costs, a type of failure cost, are incurred when a nonconforming product isdetected and corrections are made.

Answer (D) is incorrect because failure costs are incurred in the repair of nonconforming products.

Gleim's CIA Test Prep: Part III: Business Analysis and Information TechnologyAnswer Explanations

(1312 questions)

Copyright 2008 Gleim Publications, Inc. Page 14Printed for Mamdouh Farag

Page 15: P.3 Answer Explanation

[47] Gleim #: 1.1.47 -- Source: IIA, adapted

Answer (A) is correct. Teams can use the diverse knowledge and skills of all team members. Employee involvementmeans training and empowering employees to harness their creativity for problem solving. Quality control circles are usedto obtain input from employees and to locate the best perspective on problem solving.

Answer (B) is incorrect because teams are often inefficient and costly.

Answer (C) is incorrect because, although employee motivation may be high in teams, the high motivation does not alwaystranslate directly to quality improvement.

Answer (D) is incorrect because, although need for supervision may be reduced, it is not eliminated.

[48] Gleim #: 1.1.48 -- Source: IIA, adapted

Answer (A) is incorrect because this is not a part of the ISO 9000 standards. ISO argues that following the eightmanagement principles that underlie the ISO 9000 standards will lead to improved employee satisfaction.

Answer (B) is incorrect because this is the control environment as defined in the glossary of The IIA’s Standards; there isno direct reference to any such concept in the ISO 9000 standards.

Answer (C) is incorrect because the ISO 9000 approach does not take a risk assessment approach; a risk assessmentapproach is what underlies internal auditing.

Answer (D) is correct. ISO 9001:2000, Quality Management Systems – Requirements, is the standard that provides amodel for quality assurance programs. Organizations are required to monitor information on customer satisfaction as ameasure of performance. This is one of the major changes to the ISO 9000 standards made in the 2000 revision.

[49] Gleim #: 1.1.49 -- Source: IIA, adapted

Answer (A) is incorrect because scheduling production based on capacity utilization ignores other important factors suchas demands.

Answer (B) is incorrect because budgeting maintenance department activities based on previous work orders will notprevent equipment breakdowns and repairs.

Answer (C) is incorrect because standing authorizations of work orders and overtime will not address the problem posed.

Answer (D) is correct. A preventive maintenance program will reduce equipment breakdowns and repairs.

[50] Gleim #: 1.1.50 -- Source: CIA 596 I-20

Answer (A) is incorrect because establishing a quality circle is not a specific quality control procedure, although suchdiscussions may lead to improved procedures.

Answer (B) is incorrect because inspection is a necessary procedure.

Answer (C) is correct. Quality control procedures may be categorized according to the nature of the costs incurred. Anorganization incurs appraisal costs to ensure product quality. These costs include the costs of inspection, testing, andstatistical quality control programs. An organization also should investigate the causes of sales returns, which represent anexternal failure cost. Furthermore, the organization should incur certain prevention costs, e.g., by reviewing equipmentdesign and materials requirements.

Answer (D) is incorrect because establishing a quality circle is not a specific quality control procedure, although suchdiscussions may lead to improved procedures.

Gleim's CIA Test Prep: Part III: Business Analysis and Information TechnologyAnswer Explanations

(1312 questions)

Copyright 2008 Gleim Publications, Inc. Page 15Printed for Mamdouh Farag

Page 16: P.3 Answer Explanation

[51] Gleim #: 1.1.51 -- Source: CIA 596 III-23

Answer (A) is incorrect because reducing the cost of inspection is a detail of the TQM emphasis.

Answer (B) is incorrect because implementing better statistical quality control techniques is a detail of the TQMemphasis.

Answer (C) is correct. The basic principles of TQM include doing each job right the first time, being customer oriented,committing the company culture to continuous improvement, and building teamwork and employee empowerment.

Answer (D) is incorrect because encouraging cross-functional teamwork is a detail of the TQM emphasis.

[52] Gleim #: 1.1.52 -- Source: CIA 594 III-53

Answer (A) is incorrect because TQM has a broader emphasis. It focuses on improving quality, reducing cycle time,providing increased customer satisfaction, and achieving the lowest overall business cost. Reducing the cost of inspectionand encouraging cross-functional teamwork help achieve the lowest overall business cost, and better statistical qualitycontrol techniques help improve quality.

Answer (B) is incorrect because TQM has a broader emphasis. It focuses on improving quality, reducing cycle time,providing increased customer satisfaction, and achieving the lowest overall business cost. Reducing the cost of inspectionand encouraging cross-functional teamwork help achieve the lowest overall business cost, and better statistical qualitycontrol techniques help improve quality.

Answer (C) is correct. TQM establishes quality as an organizational objective and views it as a major component of theorganization’s service to its customers. It emphasizes employee training and commitment, product/service design andproduction, and customer service. Ordinarily, the quality of a product or service is as important to customers as cost andtimeliness. Superior product quality is not attained merely through more inspection, better statistical quality control, andcross-functional teamwork. Manufactures must make fundamental changes in the way they produce products and do eachjob right the first time.

Answer (D) is incorrect because TQM has a broader emphasis. It focuses on improving quality, reducing cycle time,providing increased customer satisfaction, and achieving the lowest overall business cost. Reducing the cost of inspectionand encouraging cross-functional teamwork help achieve the lowest overall business cost, and better statistical qualitycontrol techniques help improve quality.

[53] Gleim #: 1.1.53 -- Source: CIA 1196 III-92

Answer (A) is correct. The total cost of quality equals the sum of prevention costs (quality planning), appraisal costs(testing and inspection), internal failure costs (scrap and rework), and external failure costs (customer complaints andreturns), or $30,000 ($2,800 + $6,400 + $16,800 + $4,000). The quality cost index equals the total costs of quality,divided by sales, times 100%. Thus, the quality cost index for March is 7.5% [($30,000 ÷ $400,000) × 100%].

Answer (B) is incorrect because 6.5% does not include the $4,000 cost of customer complaints.

Answer (C) is incorrect because 22.0% equals the sum of direct labor and testing and inspection costs, divided by sales.

Answer (D) is incorrect because 5.9% does not include testing and inspection cost in quality costs.

Gleim's CIA Test Prep: Part III: Business Analysis and Information TechnologyAnswer Explanations

(1312 questions)

Copyright 2008 Gleim Publications, Inc. Page 16Printed for Mamdouh Farag

Page 17: P.3 Answer Explanation

[54] Gleim #: 1.2.54 -- Source: Publisher

Answer (A) is incorrect because the balanced scorecard approach uses multiple measures.

Answer (B) is correct. The trend in managerial performance evaluation is the balanced scorecard approach. Multiplemeasures of performance permit a determination as to whether a manager is achieving certain objectives at the expense ofothers that may be equally or more important. These measures may be financial or nonfinancial and usually include itemsin four categories: (1) financial; (2) customer; (3) internal business processes; and (4) learning, growth, and innovation.

Answer (C) is incorrect because the balanced scorecard approach includes financial measures.

Answer (D) is incorrect because the balanced scorecard approach includes nonfinancial measures.

[55] Gleim #: 1.2.55 -- Source: Publisher

Answer (A) is incorrect because market share and its trend is a customer measure.

Answer (B) is correct. Customer measures include market share and its trend, service response time, deliveryperformance, warranty returns, expense, complaints, and survey results. Economic value added, or EVA, is a financialmeasure.

Answer (C) is incorrect because service response time is a customer measure.

Answer (D) is incorrect because warranty expense is a customer measure.

[56] Gleim #: 1.2.56 -- Source: CPA 0605 BEC-19

Answer (A) is incorrect because, for the customer perspective, the CSFs may be customer satisfaction and retention rate,dealer and distributor relationships, marketing and selling performance, prompt delivery, and quality.

Answer (B) is incorrect because, for the internal business perspective, the CSFs may be quality, productivity, flexibility ofresponse to changing conditions, operating readiness, and safety.

Answer (C) is correct. The balanced scorecard is an accounting report that connects the firm’s critical success factors(CSFs) determined in a strategic analysis to measures of its performance. CSFs are financial and nonfinancial measures.For the learning, growth, and innovation perspective, the CSFs may be development of new products, promptness of theirintroduction, human resource development, morale, and competence of the workforce. Measures of employee satisfaction,retention, and competence may include R&D personnel qualifications, hours of training, skill set levels attained,personnel turnover, and personnel complaints and survey results.

Answer (D) is incorrect because, for the financial perspective, the CSFs may be sales, fair value of the firm’s stock,profits, and liquidity.

[57] Gleim #: 1.2.57 -- Source: Publisher

Answer (A) is correct. Cycle time is the manufacturing time to complete an order. Thus, cycle time is strictly related tointernal processes. Profitability is a combination of internal and external considerations. Customer satisfaction and marketshare are related to how customers perceive a product and how competitors react.

Answer (B) is incorrect because profitability is a measure that includes external considerations.

Answer (C) is incorrect because customer satisfaction is a measure that includes external considerations.

Answer (D) is incorrect because market share is a measure that includes external considerations.

Gleim's CIA Test Prep: Part III: Business Analysis and Information TechnologyAnswer Explanations

(1312 questions)

Copyright 2008 Gleim Publications, Inc. Page 17Printed for Mamdouh Farag

Page 18: P.3 Answer Explanation

[58] Gleim #: 1.2.58 -- Source: CIA 596 III-20

Answer (A) is incorrect because continuous improvement is important for achieving and maintaining high levels ofperformance.

Answer (B) is incorrect because more and more manufacturers are automating to achieve high quality, deliver customizedproducts on time, minimize inventory, and increase flexibility.

Answer (C) is correct. Innovative companies are customer driven. Because customers demand ever better quality andcompetitors are attempting to provide that quality, continuous improvement (called kaizen by the Japanese) is essential forsuch companies. Thus, the flow of innovative products and services must be continuous. Simply emphasizing existingproducts is not an effective strategy for most organizations.

Answer (D) is incorrect because customer satisfaction is the highest priority according to modern management practice.

[59] Gleim #: 1.2.59 -- Source: IIA, adapted

Answer (A) is incorrect because although a balanced scorecard should be developed with staff in mind, the primary aim isthe alignment of performance measures with strategy.

Answer (B) is incorrect because structure should be created after the scorecard has been developed to ensure thatresponsibilities, competencies and measures are appropriate to achieve the agreed-upon strategies.

Answer (C) is correct. The scorecard is primarily a tool to assist the organization in describing and clarifying its strategyand then alignment of its performance measures to that strategy.

Answer (D) is incorrect because systems are a means to achieving objectives that have been established after thedevelopment of the scorecard.

[60] Gleim #: 1.2.60 -- Source: CIA 594 III-57

Answer (A) is incorrect because the rate of absenteeism does not compare input and output.

Answer (B) is incorrect because the goal of becoming a leading manufacturer concerns effectiveness, not efficiency.

Answer (C) is correct. The number of insurance claims processed per day is a typical measure used in a balancedscorecard. It relates to the critical success factor of productivity. This factor is based on an input-output (efficiency)relationship. An organizational structure is efficient if it facilitates the accomplishment of organizational objectives withminimum resources and fewest unsought consequences. An efficient organizational structure maximizes output for a givenamount of input. Thus, an efficiency measure compares input with output. Insurance claims processed per day relatesoutput (claims processed) to input (a day’s work).

Answer (D) is incorrect because the goal of increasing market share concerns effectiveness, not efficiency.

Gleim's CIA Test Prep: Part III: Business Analysis and Information TechnologyAnswer Explanations

(1312 questions)

Copyright 2008 Gleim Publications, Inc. Page 18Printed for Mamdouh Farag

Page 19: P.3 Answer Explanation

[61] Gleim #: 1.3.61 -- Source: Publisher

Answer (A) is correct. During an on-site visit, the registrar has the right to audit all employees if he or she decides to doso. Employees must have the ability to explain their jobs and show that they are capable of performing properly.

Answer (B) is incorrect because some companies have preliminary audits by registrars to prepare for the official audit.

Answer (C) is incorrect because employees must be able to “say what they do” and demonstrate that they “do what theysay.”

Answer (D) is incorrect because a registrar who is convinced that a quality system conforms to the selected standardissues a certificate describing the scope of the registration. The registration is usually valid for a 3-year period.

[62] Gleim #: 1.3.62 -- Source: Publisher

Answer (A) is incorrect because many foreign countries are beginning to require adoption of ISO 9000 standards as aprerequisite for a company to sell products or services in that country.

Answer (B) is incorrect because many companies view ISO registration as a key to remaining competitive. ISOregistration allows customers to be more comfortable with suppliers’ products and services.

Answer (C) is correct. Market pressure is usually the main driving force for companies that adopt ISO 9000 standards.However, many of the companies that register uncover internal process and quality improvement as a result. ISO 9000forces companies to share information, which leads to a better understanding of who internal customers and users are.

Answer (D) is incorrect because many companies that implement ISO 9000 standards uncover internal process and qualityimprovements.

[63] Gleim #: 1.3.63 -- Source: Publisher

Answer (A) is incorrect because many believe adhering to ISO 14000 standards will reduce monitoring or inspection byregulatory agencies.

Answer (B) is correct. Many European countries already have environmental systems in place, and many individualcountries’ standards are typically more strict than the ISO 14000 standards.

Answer (C) is incorrect because many countries in the European Union have adopted measures similar to the ones in theUS to prevent self-incrimination during voluntary ISO audits.

Answer (D) is incorrect because ISO 14000 establishes internationally recognized standards that are intended to diminishtrade barriers and make it easier to do business across borders.

Gleim's CIA Test Prep: Part III: Business Analysis and Information TechnologyAnswer Explanations

(1312 questions)

Copyright 2008 Gleim Publications, Inc. Page 19Printed for Mamdouh Farag

Page 20: P.3 Answer Explanation

[64] Gleim #: 1.3.64 -- Source: Publisher

Answer (A) is incorrect because one of the most important steps to adhering to ISO 9000 standards is to organize a QMS.A QMS explains the company’s quality control and management’s commitment to quality.

Answer (B) is incorrect because internal audits assure that the company is complying with the documented QMSprocedures and ISO 9000 standards.

Answer (C) is correct. ISO 9000 is a set of generic standards for establishing and maintaining a quality system within acompany. The standards provide no basis for judging the quality of the end product. The marketplace will make thisdetermination on its own. The objective of ISO 9000 standards is to ensure consistent quality.

Answer (D) is incorrect because a registrar must ensure that the company’s quality control system conforms to the selectedstandard.

[65] Gleim #: 1.3.65 -- Source: Publisher

Answer (A) is incorrect because compliance is voluntary but many companies are adopting the standards for competitivereasons or out of fear that the standards will become a requirement in foreign markets.

Answer (B) is incorrect because the ISO rules specify that standards are periodically revised every 5 years in light oftechnological and market developments.

Answer (C) is correct. The objective of ISO 9000 standards is to ensure consistent quality even if the quality is poor. Themarket will determine the quality of the end result.

Answer (D) is incorrect because ISO 9000 standards are generic in nature and only ensure consistent quality in theproduct being produced.

[66] Gleim #: 1.3.66 -- Source: Publisher

Answer (A) is incorrect because not all companies are regulated by government agencies. Also, there is no guarantee thatcompliance with ISO 14000 standards will reduce monitoring and inspection by agencies.

Answer (B) is incorrect because ISO 14000 standards are not currently required by the European Union.

Answer (C) is incorrect because this is a benefit to some companies but not others. Not all companies are subject to theEPA’s audits.

Answer (D) is correct. The main benefit of instituting ISO 14000 standards is internal; companies learn how well theirenvironmental management system operates relative to those of other companies. This is accomplished through thesharing of information required by ISO standards.

Gleim's CIA Test Prep: Part III: Business Analysis and Information TechnologyAnswer Explanations

(1312 questions)

Copyright 2008 Gleim Publications, Inc. Page 20Printed for Mamdouh Farag

Page 21: P.3 Answer Explanation

[67] Gleim #: 1.3.67 -- Source: Publisher

Answer (A) is correct. An important requirement of the ISO 9000 standards is the internal audit system. Internal auditsassure that the company is complying with the documented QMS procedures and ISO 9000 standards.

Answer (B) is incorrect because the registrar evaluates whether the quality control system conforms to the selectedstandard.

Answer (C) is incorrect because the registrar is the external auditor who performs the on-site inspection. The registrarobserves the quality control system and evaluates the conformity to the selected standard.

Answer (D) is incorrect because the internal audit is responsible for ensuring compliance with the QMS and ISO 9000standards. However, management commitment plays an important role in determining the amount of resources dedicatedto quality.

[68] Gleim #: 1.4.68 -- Source: CIA 593 III-64

Answer (A) is incorrect because a perfect negative correlation exists.

Answer (B) is correct. The coefficient of correlation (in standard notation, r) measures the strength of the linearrelationship. The magnitude of r is independent of the scales of measurement of X and Y. Its range is –1.0 to 1.0. A valueof –1.0 indicates a perfectly inverse linear relationship between X and Y. A value of zero indicates no linear relationshipbetween X and Y. A value of +1.0 indicates a perfectly direct relationship between X and Y. As X increases by 1, Yconsistently decreases by 2. Hence, a perfectly inverse relationship exists, and r must be equal to –1.0.

Answer (C) is incorrect because an inverse, not a direct, relationship exists.

Answer (D) is incorrect because a linear relationship exists between X and Y.

[69] Gleim #: 1.4.69 -- Source: CIA 1194 II-46

Answer (A) is incorrect because 1.03 is an impossible value.

Answer (B) is incorrect because –.02 is a very weak correlation coefficient.

Answer (C) is correct. Because the range of values is between –1.0 and 1.0, –.89 suggests a very strong inverserelationship between the independent and dependent variables. A value of –1.0 signifies a perfect inverse relationship,and a value of 1.0 signifies a perfect direct relationship.

Answer (D) is incorrect because .75 is .25 from the maximum value, whereas –.89 is .11 from the minimum value.

Gleim's CIA Test Prep: Part III: Business Analysis and Information TechnologyAnswer Explanations

(1312 questions)

Copyright 2008 Gleim Publications, Inc. Page 21Printed for Mamdouh Farag

Page 22: P.3 Answer Explanation

[70] Gleim #: 1.4.70 -- Source: CIA 595 II-46

Answer (A) is incorrect because cross-sectional regression analysis is inappropriate. The auditor is trying to estimatechanges in a single account balance over time.

Answer (B) is incorrect because regression analysis may still be the most appropriate methodology to estimate interestincome, but the auditor should first understand the factors that may be causing r 2 to decrease. The reason may be asystematic error in the account balance.

Answer (C) is correct. The coefficient of determination (r 2) is the amount of variation in the dependent variable (interestincome) that is explained by the independent variables. In this case, less of the change in interest income is explained bythe model. Thus, some other factor must be causing interest income to change. This change merits audit investigation.

Answer (D) is incorrect because linear regression models are simpler models, but the auditor should be searching for asystematic error in the account balance or applying a more complex model.

[71] Gleim #: 1.4.71 -- Source: CMA 1290 4-27

Answer (A) is incorrect because x is the independent variable.

Answer (B) is incorrect because the dependent variable is y.

Answer (C) is incorrect because the constant coefficient is a.

Answer (D) is correct. In the standard regression equation, b represents the variable coefficient. For example, in a costdetermination regression, y equals total costs, b is the variable cost per unit, x is the number of units produced, and a isfixed cost.

[72] Gleim #: 1.4.72 -- Source: CMA 1290 4-28

Answer (A) is correct. The letter x in the standard regression equation is the independent variable. For example, in aregression to determine the total cost of production, x equals units produced.

Answer (B) is incorrect because the dependent variable is y.

Answer (C) is incorrect because the constant coefficient is a.

Answer (D) is incorrect because r 2 is the coefficient of determination.

[73] Gleim #: 1.4.73 -- Source: CMA 1290 4-29

Answer (A) is incorrect because the budgeted maintenance costs are $3,746.

Answer (B) is incorrect because the budgeted maintenance costs are $3,746.

Answer (C) is incorrect because the budgeted maintenance costs are $3,746.

Answer (D) is correct. Substituting the given data into the regression equation results in a budgeted cost of $3,746(rounded to the nearest dollar).

y = a + bxy = 684.65 + 7.2884(420)y = $3,746

Gleim's CIA Test Prep: Part III: Business Analysis and Information TechnologyAnswer Explanations

(1312 questions)

Copyright 2008 Gleim Publications, Inc. Page 22Printed for Mamdouh Farag

Page 23: P.3 Answer Explanation

[74] Gleim #: 1.4.74 -- Source: CMA 1290 4-30

Answer (A) is correct. The coefficient of determination (r²) measures the percentage of the total variance in cost that canbe explained by the regression equation. If the coefficient of determination is .99724, 99.724% of the variance isexplained by the regression equation. Thus, the values in the regression equation explain virtually the entire amount oftotal cost.

Answer (B) is incorrect because the percentage of the total variance explained by the regression equation is 99.724%,which corresponds to the coefficient of determination (r²), or .99724.

Answer (C) is incorrect because the percentage of the total variance explained by the regression equation is 99.724%,which corresponds to the coefficient of determination (r²), or .99724.

Answer (D) is incorrect because the percentage of the total variance explained by the regression equation is 99.724%,which corresponds to the coefficient of determination (r²), or .99724.

[75] Gleim #: 1.4.75 -- Source: CIA 1195 II-30

Answer (A) is incorrect because a regression coefficient is unrelated to the means of the variables.

Answer (B) is incorrect because, to predict a specific value of sales, the value of the independent variable is multiplied bythe coefficient. The product is then added to the y-intercept value.

Answer (C) is correct. The regression coefficient represents the change in the dependent variable corresponding to a unitchange in the independent variable. Thus, it is the slope of the regression line.

Answer (D) is incorrect because the absolute size of the coefficient bears no necessary relationship to the importance ofthe variable.

[76] Gleim #: 1.4.76 -- Source: CIA 595 II-47

Answer (A) is incorrect because linear time series analysis is inapplicable. It is a simple model that compares data for anindividual store over time.

Answer (B) is correct. Time series data pertain to a given entity over a number of prior time periods. Cross-sectionaldata, however, pertain to different entities for a given time period or at a given time. Thus, cross-sectional regressionanalysis is the most appropriate statistical tool because it compares attributes of all stores’ operating statistics at onemoment in time.

Answer (C) is incorrect because cross tabulations have to be built on a model of expectations. Unless the model is built,the analysis is not useful.

Answer (D) is incorrect because the objective is to compare stores at one moment in time. Multiple regression time seriesanalysis compares the performance of an individual store over a period of time.

[77] Gleim #: 1.4.77 -- Source: CIA 1195 III-11

Answer (A) is correct. The objective of a scatter diagram is to demonstrate correlations. Each observation is representedby a dot on a graph corresponding to a particular value of X (the independent variable) and Y (the dependent variable).

Answer (B) is incorrect because the objective of a histogram is to show frequency distribution in graphic form.

Answer (C) is incorrect because the objective of stratification is to divide a universe of data into homogeneous groups.

Answer (D) is incorrect because regression analysis is used to find trend lines.

Gleim's CIA Test Prep: Part III: Business Analysis and Information TechnologyAnswer Explanations

(1312 questions)

Copyright 2008 Gleim Publications, Inc. Page 23Printed for Mamdouh Farag

Page 24: P.3 Answer Explanation

[78] Gleim #: 1.4.78 -- Source: Publisher

Answer (A) is incorrect because constant variance of the error term is usually met.

Answer (B) is correct. Time series analysis is a regression model in which the independent variable is time. In timeseries analysis, the value of the next time period is frequently dependent on the value of the time period before that.Hence, the error terms are usually correlated or dependent on the prior period; i.e., they are characterized byautocorrelation (serial correlation).

Answer (C) is incorrect because normal distribution of the error term is usually met.

Answer (D) is incorrect because an expected value of the error term equal to zero is usually met.

[79] Gleim #: 1.4.79 -- Source: CIA 589 III-50

Answer (A) is incorrect because cross-sectional regression analysis examines relationships among large amounts of data(e.g., many or different production methods or locations) at a particular moment in time.

Answer (B) is incorrect because regression analysis relates the forecast to changes in particular variables.

Answer (C) is incorrect because, under exponential smoothing, each forecast equals the sum of the last observation timesthe smoothing constant, plus the last forecast times one minus the constant.

Answer (D) is correct. The simple moving-average method is a smoothing technique that uses the experience of the pastN periods (through time period t) to forecast a value for the next period. Thus, the average includes each new observationand discards the oldest observation. The forecast formula for the next period (for time period t+1) is the sum of the last Nobservations divided by N.

[80] Gleim #: 1.4.80 -- Source: CIA 594 II-38

Answer (A) is incorrect because an unweighted average will not give more importance to more recent data.

Answer (B) is correct. Under exponential smoothing, each forecast equals the sum of the last observation times thesmoothing constant, plus the last forecast times one minus the constant. Thus, exponential means that greater weight isplaced on the most recent data, with the weights of all data falling off exponentially as the data age. This feature isimportant because of the organizational changes that affected sales volume.

Answer (C) is incorrect because queuing theory is used to minimize the cost of waiting lines.

Answer (D) is incorrect because linear regression analysis determines the equation for the relationship among variables. Itdoes not give more importance to more recent data.

Gleim's CIA Test Prep: Part III: Business Analysis and Information TechnologyAnswer Explanations

(1312 questions)

Copyright 2008 Gleim Publications, Inc. Page 24Printed for Mamdouh Farag

Page 25: P.3 Answer Explanation

[81] Gleim #: 1.4.81 -- Source: CMA 1293 4-24

Answer (A) is incorrect because $4,320 equals the cost of the items in the fourth batch.

Answer (B) is incorrect because $10,368 is based on the assumption that the cumulative average unit labor cost is reducedby the learning curve percentage with each batch, not each doubling of output.

Answer (C) is incorrect because $2,592 represents the labor cost of 100 units at the unit rate expected after anotherdoubling of production to eight batches.

Answer (D) is correct. The learning curve reflects the increased rate at which people perform tasks as they gainexperience. The time required to perform a given task becomes progressively shorter. Ordinarily, the curve is expressed ina percentage of reduced time to complete a task for each doubling of cumulative production. One common assumption in alearning curve model is that the cumulative average time (and labor cost) per unit is reduced by a certain percentage eachtime production doubles. Given a $120 cost per unit for the first 100 units and a $72 cost per unit when cumulativeproduction doubled to 200 units, the learning curve percentage must be 60% ($72 ÷ $120). If production is again doubledto 400 units (four batches), the average unit labor cost should be $43.20 ($72 × 60%). Hence, total labor cost for 400 unitsis estimated to be $17,280 (400 × $43.20).

[82] Gleim #: 1.4.82 -- Source: CMA 1288 5-19

Answer (A) is correct. Assuming that the cumulative average-time model applies, an 80% learning curve means that thecumulative average time per unit (and labor cost, given a constant labor rate) declines by 20% when unit output doubles inthe early stages of production. The first lot size was 50 units, which was produced at a total cost of $15,400 ($1,500 formaterials + $13,900 for labor and overhead). Materials costs are strictly variable and should remain proportional toproduction. The labor ($8,500) and variable overhead ($4,000) costs (labor-related), however, will be affected by thelearning curve. The average cost per lot for labor and variable overhead after 100 units have been produced should be80% of the costs of the first lot of 50 units. Thus, the average labor and variable overhead cost per 50-unit lot will be$10,000 ($12,500 × 80%). If production doubles again (to a total production of 200 units or four lots of 50 each), thecumulative average cost for labor and variable overhead will be $8,000 per lot ($10,000 × 80%). Given four lots of 50each, at an average cost of $8,000 per lot, the total cost for labor and variable overhead must be $32,000. Adding $6,000for raw materials ($1,500 per 50-unit lot) gives a total variable cost of $38,000 for 200 units. Fixed overhead is 10% oftotal variable cost, so total cost is $41,800. The total cost for the last 150 units is $26,400 ($41,800 – $15,400).

Answer (B) is incorrect because $32,000 is the total cost for labor and variable overhead for 200 units.

Answer (C) is incorrect because $38,000 is the total variable cost for 200 units.

Answer (D) is incorrect because $41,800 is the total cost for 200 units.

[83] Gleim #: 1.4.83 -- Source: CMA 1288 5-20

Answer (A) is incorrect because, with no learning curve effect, estimated total hours would be 4,000 instead of 1,960, achange of more than 50%.

Answer (B) is incorrect because fixed costs applied per lot would decline because they are based on labor hours, whichare declining.

Answer (C) is incorrect because no information about the bid (as opposed to costs) is given. Moreover, a reduction in thelearning curve from 80% to 70% will reduce total costs for the last 150 units from $26,400 to $18,150, a reduction of31.25%.

Answer (D) is correct. The sum of the direct labor hours for the initial lot of 50 units was 1,000. A second lot of 50 wouldreduce the cumulative hours per lot to 700 (1,000 × 70%). A doubling to four lots would reduce the cumulative hours perlot to 490 (700 × 70%). Thus, for an output of 200 units, the total hours worked would be 1,960 (4 × 490). Subtracting the1,000 hours required for the first 50 units from the 1,960-hour total gives 960 hours for the last 150 units. Dividing 960hours by 150 units produces a per-unit time of 6.4 hours.

Gleim's CIA Test Prep: Part III: Business Analysis and Information TechnologyAnswer Explanations

(1312 questions)

Copyright 2008 Gleim Publications, Inc. Page 25Printed for Mamdouh Farag

Page 26: P.3 Answer Explanation

[84] Gleim #: 1.4.84 -- Source: CMA 1294 4-29

Answer (A) is incorrect because integer programming is a variation of linear programming that concerns problems inwhich some variables are not continuous. Integer programming problems are also known as discrete models because thevariables take on discrete, noncontinuous values.

Answer (B) is incorrect because linear programming is a technique used to maximize a revenue or profit function, orminimize a cost function, subject to constraints such as limited resources or minimum (or maximum) levels of productiontime.

Answer (C) is correct. The Monte Carlo simulation method is often used to generate the individual values for a randomvariable. The performance of a quantitative model under uncertainty may be investigated by randomly selecting values foreach variable in the model (based on the probability distribution of each variable) and then calculating the value of thesolution. If this process is performed many times, the distribution of results from the model will be obtained.

Answer (D) is incorrect because regression analysis is used to find an equation for the linear relationships amongvariables.

[85] Gleim #: 1.4.85 -- Source: CMA 690 5-21

Answer (A) is incorrect because expected value analysis is used to determine an anticipated return or cost based uponprobabilities of events and their related outcomes.

Answer (B) is incorrect because linear programming optimizes a function given certain constraints.

Answer (C) is incorrect because PERT is a network technique used to plan and control large projects.

Answer (D) is correct. Sensitivity modeling can be used to determine the outcome of a variety of decisions. A trial-and-error method may be adopted, usually in a computer model, to calculate the sensitivity of the solution (variability ofoutcomes) to changes in a variable.

[86] Gleim #: 1.4.86 -- Source: CIA 1190 III-46

Answer (A) is correct. A Markov chain is a series of events in which the probability of an event depends on theimmediately preceding event. An example is the game of blackjack in which the probability of certain cards being dealt isdependent upon what cards have already been dealt. In the analysis of bad debts, preceding events, such as collections,credit policy changes, and writeoffs, affect the probabilities of future losses.

Answer (B) is incorrect because econometrics forecasts the impact of different economic policies and conditions.

Answer (C) is incorrect because Monte Carlo analysis is a simulation technique that uses random-number procedures tocreate values for probabilistic components.

Answer (D) is incorrect because dynamic programming is a problem-solving approach that breaks a large mathematicalmodel into a number of smaller, manageable problems.

Gleim's CIA Test Prep: Part III: Business Analysis and Information TechnologyAnswer Explanations

(1312 questions)

Copyright 2008 Gleim Publications, Inc. Page 26Printed for Mamdouh Farag

Page 27: P.3 Answer Explanation

[87] Gleim #: 1.4.87 -- Source: CIA 1193 III-67

Answer (A) is correct. Queuing theory is a group of mathematical models for systems involving waiting lines. In general,a queuing system consists of a waiting line and a service facility (a copy center in this case). The objective is to minimizetotal costs, including both service and waiting costs (turnaround time), for a given rate of arrivals.

Answer (B) is incorrect because linear programming optimizes a given objective function subject to constraints.

Answer (C) is incorrect because regression analysis estimates the relation among variables.

Answer (D) is incorrect because game theory is an approach to decision making that considers the actions of competitors.

[88] Gleim #: 1.4.88 -- Source: CMA 688 5-27

Answer (A) is correct. Queuing (waiting-line) models minimize, for a given rate of arrivals, the sum of (1) the cost ofproviding service (including facility costs and operating costs) and (2) the cost of idle resources waiting in line. The lattermay be a direct cost, if paid employees are waiting, or an opportunity cost in the case of waiting customers. Thisminimization occurs at the point where the cost of waiting is balanced by the cost of providing service.

Answer (B) is incorrect because queuing theory minimizes the sum of the costs of waiting and of providing service.

Answer (C) is incorrect because queuing theory minimizes the sum of the costs of waiting and of providing service.

Answer (D) is incorrect because queuing theory minimizes the sum of the costs of waiting and of providing service.

[89] Gleim #: 1.4.89 -- Source: CMA 688 5-28

Answer (A) is incorrect because the queuing model calculates the average percentage of time that a service facility is idle,the probability of a specified number of units in the queue, and the average number of units in the system and the meanlength of the queue.

Answer (B) is incorrect because the queuing model calculates the average percentage of time that a service facility is idle,the probability of a specified number of units in the queue, and the average number of units in the system and the meanlength of the queue.

Answer (C) is correct. Queuing models determine the operating characteristics of a waiting line: the probability that nounits are in the system, the average units in the line, the average units in the system, the average time a unit waits, theaverage time a unit is in the system, the probability that a unit must wait, and the probability of a given number of units inthe system. However, the actual time spent in the queue cannot be determined from the model.

Answer (D) is incorrect because the queuing model calculates the average percentage of time that a service facility is idle,the probability of a specified number of units in the queue, and the average number of units in the system and the meanlength of the queue.

[90] Gleim #: 1.4.90 -- Source: CIA 586 III-15

Answer (A) is incorrect because the number of employees is unlikely to change due to the new system.

Answer (B) is correct. When all customers must wait in a single queue, a decrease in waiting time is possible givenmultiple servers. An added effect is to increase customer satisfaction.

Answer (C) is incorrect because, assuming a Poisson process, the number of customers per teller will not change.

Answer (D) is incorrect because tellers’ duties will not change, so on-the-job training will not improve.

Gleim's CIA Test Prep: Part III: Business Analysis and Information TechnologyAnswer Explanations

(1312 questions)

Copyright 2008 Gleim Publications, Inc. Page 27Printed for Mamdouh Farag

Page 28: P.3 Answer Explanation

[91] Gleim #: 1.4.91 -- Source: CIA 1187 III-39

Answer (A) is incorrect because service by one ticket-seller at a movie theater is an example of a single-channel, single-phase system.

Answer (B) is correct. The drive-through represents a single queue (channel). Because this waiting line has three servicesin series, it may be said to be multiple phase. Another example is the typical factory assembly line. This terminology(channel, phase), however, is not used by all writers on queuing theory.

Answer (C) is incorrect because supermarket checkout lines are a common example of multiple single-phase serversservicing multiple lines.

Answer (D) is incorrect because an example of a multiple-channel, multiple-phase system is a set of supermarket checkoutlines each of which is served in sequence by a cashier and a person who packs grocery bags.

[92] Gleim #: 1.4.92 -- Source: CIA 584 IV-34

Answer (A) is correct. One hundred customers arrive in line per hour and only 60 are serviced per hour. Accordingly, thequeue will expand to infinity during peak periods.

Answer (B) is incorrect because insufficient information is given to determine overall idle time or average customerwaiting time. The question gives only peak period data.

Answer (C) is incorrect because peak customer service is only 60 per hour.

Answer (D) is incorrect because insufficient information is given to determine overall idle time or average customerwaiting time. The question gives only peak period data.

[93] Gleim #: 1.4.93 -- Source: CIA 593 III-69

Answer (A) is correct. If both firms reduce the selling price of Product A, neither will gain sales and the resultant pricewar will cause both firms to earn lower profits. This outcome is inevitable when reduced profit margins do not result in asignificant increase in sales. The effect is a no-win strategy.

Answer (B) is incorrect because both firms will experience lower profits.

Answer (C) is incorrect because both firms will experience lower profits.

Answer (D) is incorrect because both firms will experience lower profits.

Gleim's CIA Test Prep: Part III: Business Analysis and Information TechnologyAnswer Explanations

(1312 questions)

Copyright 2008 Gleim Publications, Inc. Page 28Printed for Mamdouh Farag

Page 29: P.3 Answer Explanation

[94] Gleim #: 1.4.94 -- Source: CIA 1192 III-98

Answer (A) is incorrect because the binomial distribution is a discrete distribution in which each trial has just twooutcomes.

Answer (B) is incorrect because the chi-square distribution is a continuous distribution used to measure the fit betweenactual data and the theoretical distribution.

Answer (C) is correct. Queuing models assume that arrivals follow a Poisson process: the events (arrivals) areindependent, any number of events must be possible in the interval of time, the probability of an event is proportional tothe length of the interval, and the probability of more than one event is negligible if the interval is sufficiently small. If λis the average number of events in a given interval, k is the number of events, and e is the natural logarithm (2.71828...),the probability of k is

λ k e - λf(k) =

k !

Answer (D) is incorrect because service time has an exponential distribution. This distribution gives the probability ofzero events in a given interval, i.e., the probability of a specified time between arrivals.

[95] Gleim #: 1.4.95 -- Source: CIA 1192 III-99

Answer (A) is incorrect because car 3 must wait for 4 minutes.

Answer (B) is incorrect because car 3 must wait for 4 minutes.

Answer (C) is correct. Car 1 is at the attached window and will require 3 minutes to service. Car 2 must wait for car 1 tobe serviced (3 minutes in the queue + 3 minutes to be serviced = 6 minutes). Car 3 arrived at the attached window 2minutes after cars 1 and 2. It must wait 1 minute for car 1 to be serviced and 3 minutes for car 2 to be serviced, a waitingtime of 4 minutes.

Answer (D) is incorrect because car 3 must wait for 4 minutes.

[96] Gleim #: 1.4.96 -- Source: CIA 1192 III-100

Answer (A) is incorrect because car 4 must wait 4 minutes.

Answer (B) is incorrect because car 4 must wait 4 minutes.

Answer (C) is correct. Car 4 arrives at the just-vacated island window 4 minutes after car 3. It must wait 4 minutes forcar 3 to be serviced.

Answer (D) is incorrect because car 4 must wait 4 minutes.

Gleim's CIA Test Prep: Part III: Business Analysis and Information TechnologyAnswer Explanations

(1312 questions)

Copyright 2008 Gleim Publications, Inc. Page 29Printed for Mamdouh Farag

Page 30: P.3 Answer Explanation

[97] Gleim #: 1.4.97 -- Source: CMA 1288 5-6

Answer (A) is incorrect because deterministic decision making is based upon fixed (nonprobabilistic) inputs into thedecision process.

Answer (B) is incorrect because maximax is a criterion that maximizes the maximum possible profit.

Answer (C) is incorrect because expected value decision making is a risk-neutral process that selects the strategy thatshould maximize value in the long run.

Answer (D) is correct. In game theory, the minimax decision criterion selects the strategy that will minimize themaximum possible loss. It is a technique used by a risk-averse player. The maximin criterion, which chooses the strategywith the maximum minimum payoff, gives the same results as the minimax procedure.

[98] Gleim #: 1.4.98 -- Source: CMA 688 5-19

Answer (A) is incorrect because maximax is adopted by risk seekers.

Answer (B) is incorrect because the minimum regret criterion is used by a player who wishes to minimize the effect of abad decision in either direction.

Answer (C) is incorrect because the insufficient reason (Laplace) criterion applies when the decision maker cannot assignprobabilities to the states of nature arising after a decision.

Answer (D) is correct. A rational economic decision maker (one completely guided by objective criteria) will useexpected monetary value to maximize gains under conditions of risk because (s)he is risk-neutral (the utility of a gainequals the disutility of a loss of the same absolute amount). Expected value represents the long-term average payoff forrepeated trials. The best choice is the one having the highest expected value (sum of the products of the possible outcomesand their respective probabilities).

[99] Gleim #: 1.4.99 -- Source: CIA 1191 III-100

Answer (A) is incorrect because the minimize regret rule selects the action that minimizes the maximum opportunity cost.

Answer (B) is incorrect because the maximize utility rule is not a decision rule.

Answer (C) is correct. The maximin rule determines the minimum payoff for each decision and then chooses the decisionwith the maximum minimum payoff. It is a conservative criterion adopted by risk-averse players, that is, those for whomthe disutility of a loss exceeds the utility of an equal gain.

Answer (D) is incorrect because the maximax rule selects the choice that provides the greatest payoff if the most favorablestate of nature occurs.

Gleim's CIA Test Prep: Part III: Business Analysis and Information TechnologyAnswer Explanations

(1312 questions)

Copyright 2008 Gleim Publications, Inc. Page 30Printed for Mamdouh Farag

Page 31: P.3 Answer Explanation

[100] Gleim #: 1.4.100 -- Source: CIA 588 III-45

Answer (A) is correct. Game theory is a mathematical approach to decision making when confronted with an enemy orcompetitor. Games are classified according to the number of players and the algebraic sum of the payoffs. In a two-playergame, if the payoff is given by the loser to the winner, the algebraic sum is zero, and the game is a zero-sum game.However, if it is possible for both players to profit, the game is a positive-sum game. In this situation, the sum of thepayoffs for each combination of strategies is zero. For example, if X takes no action and Y chooses limited advertising,X’s payoff is –1 and Y’s is 1.

Answer (B) is incorrect because, in a cooperative game, the players are permitted to negotiate and form bindingagreements prior to the selection of strategies. In addition, in such games, the payoffs in one or more of the cells will notsum to zero.

Answer (C) is incorrect because the prisoner’s dilemma is a special outcome of a partly competitive game. In these games,each player has a strategy that dominates all others, and the outcome of each player’s choice of the dominant strategy isless favorable to both players than some other outcome.

Answer (D) is incorrect because games against nature are formulations of problems in which only one player chooses astrategy, and the set of outcomes and payoffs is not influenced by the selection.

[101] Gleim #: 1.4.101 -- Source: CIA 1191 III-96

Answer (A) is incorrect because L1 is the choice based on the maximin criterion.

Answer (B) is incorrect because L2 is the choice based on the minimax regret criterion.

Answer (C) is incorrect because L3 is the choice based on the Laplace criterion.

Answer (D) is correct. Under the maximax criterion, the decision maker selects the choice that maximizes the maximumprofit. The maximum profits for the five locations are

Location L1 L2 L3 L4 L5Maximum Profit 15 21 17 26 29

The location with the greatest potential profit is L5.

[102] Gleim #: 1.4.102 -- Source: CIA 1191 III-97

Answer (A) is incorrect because L1 is the choice based on the maximin criterion.

Answer (B) is correct. Under the minimax regret criterion, the decision maker selects the choice that minimizes themaximum regret (opportunity cost). The maximum regret for each location is determined from the opportunity loss matrix. The maximum regret for each location is the highest number in each column as indicated below.

Location L1 L2 L3 L4 L5Maximum Regret 14 9 12 10 13

The location with the minimum regret is L2. If demand is low, L2 has a payoff of –2, whereas L1 has a payoff of 7.

Answer (C) is incorrect because L3 is the choice based on the Laplace criterion.

Answer (D) is incorrect because L5 is the choice based on the maximax criterion.

Gleim's CIA Test Prep: Part III: Business Analysis and Information TechnologyAnswer Explanations

(1312 questions)

Copyright 2008 Gleim Publications, Inc. Page 31Printed for Mamdouh Farag

Page 32: P.3 Answer Explanation

[103] Gleim #: 1.4.103 -- Source: CIA 1191 III-98

Answer (A) is correct. The opportunity loss matrix is as follows:

Location L1 L2 L3 L4 L5

Demand: High 14 =29–15

8 =29–21

12 =29–17

3 =29–26

0 = 29-29

Medium 2 =14–12

6 =14–8

0 =14–14

4 =14–10

10 =14–4

Low 0 = 7–7 9 =7–(–2)

3 = 7–4 10 =7–(–3)

13 =7–(–6)

The expected opportunity loss from selecting location L4 is 5.50 [(3 × 0.3) + (4 × 0.4) + (10 × 0.3)].

Answer (B) is incorrect because 7.90 is the expected opportunity loss from selecting location L5.

Answer (C) is incorrect because 7.50 is the expected opportunity loss from selecting location L2.

Answer (D) is incorrect because 5.00 is the expected opportunity loss from selecting location L1.

[104] Gleim #: 1.4.104 -- Source: Publisher

Answer (A) is incorrect because $(15,200) is the annual amount the company will lose without a generator.

Answer (B) is incorrect because $(1,267) is the monthly amount the company will lose without a generator.

Answer (C) is correct. Each outage costs $800, but this expense can be avoided by paying $1,000 per month ($12,000 forthe year). The expected-value approach uses the probability distribution derived from past experience to determine theaverage expected outages per month.

3 ÷ 12 × 0 = 0.02 ÷ 12 × 1 = 0.166674 ÷ 12 × 2 = 0.666673 ÷ 12 × 3 = 0.75000

1.58334

The company can expect to have, on average, 1.58334 outages per month. At $800 per outage, the expected cost is$1,266.67. Thus, paying $1,000 to avoid an expense of $1,266.67 saves $266.67 per month, or $3,200 per year.

Answer (D) is incorrect because $7,200 is the savings amount if two outages occur per month.

[105] Gleim #: 1.4.105 -- Source: CMA 697 4-22

Answer (A) is incorrect because $85,000 is the sales estimate with the highest probability.

Answer (B) is incorrect because $84,000 is the expected value of sales.

Answer (C) is incorrect because $68,000 is 80% of the sales estimate with the highest probability.

Answer (D) is correct. The expected value is calculated by weighting each sales estimate by the probability of itsoccurrence. Consequently, the expected value of sales is $84,000 [$60,000 × .25) + ($85,000 × .40) + ($100,000 × .35)].Cost of goods sold is therefore $67,200 (80% × $84,000).

Gleim's CIA Test Prep: Part III: Business Analysis and Information TechnologyAnswer Explanations

(1312 questions)

Copyright 2008 Gleim Publications, Inc. Page 32Printed for Mamdouh Farag

Page 33: P.3 Answer Explanation

[106] Gleim #: 1.4.106 -- Source: CMA 1293 4-26

Answer (A) is incorrect because the expected value of perfect information is the difference between the expected profitunder certainty and the profit from the best decision under uncertainty.

Answer (B) is incorrect because the expected value of perfect information is the excess of the total conditional profitsunder certainty over the profit from the best decision under uncertainty.

Answer (C) is incorrect because there is no expected opportunity loss under conditions of certainty.

Answer (D) is correct. Perfect information permits certainty that a future state of nature will occur. The expected value ofperfect information determines the maximum amount a decision maker is willing to pay for information. It is thedifference between the expected value without perfect information, that is, the expected value of the best action underuncertainty and the expected value under certainty. Under certainty, a decision maker knows in each case which state ofnature will occur and can act accordingly.

[107] Gleim #: 1.4.107 -- Source: CMA 1289 5-20

Answer (A) is incorrect because 4,000 pretzels assumes each outcome is equally likely.

Answer (B) is correct. The calculation using an expected value approach weights each possible sales volume by itsprobability. Thus, the estimated demand is 4,400 pretzels.

Volume Probability Result2,000 × .10 = 2003,000 × .15 = 4504,000 × .20 = 8005,000 × .35 = 1,7506,000 × .20 = 1,200

Expected value 4,400

Answer (C) is incorrect because 5,000 pretzels is the estimated demand using a deterministic approach based on the mostlikely outcome.

Answer (D) is incorrect because 4,400 pretzels is among the responses given.

[108] Gleim #: 1.4.108 -- Source: CMA 1289 5-21

Answer (A) is incorrect because 4,000 pretzels assumes each outcome is equally likely.

Answer (B) is incorrect because 4,400 pretzels is the estimated demand using an expected value approach.

Answer (C) is correct. A deterministic approach assumes that a value is known with certainty. If that value is deemed tobe the most likely outcome, assumed demand will be 5,000 pretzels, the volume with the highest probability (35%).

Answer (D) is incorrect because 6,000 pretzels is merely the greatest demand.

Gleim's CIA Test Prep: Part III: Business Analysis and Information TechnologyAnswer Explanations

(1312 questions)

Copyright 2008 Gleim Publications, Inc. Page 33Printed for Mamdouh Farag

Page 34: P.3 Answer Explanation

[109] Gleim #: 1.4.109 -- Source: CMA 1292 4-22

Answer (A) is incorrect because the most the vendor can make is $2,500 per day.

Answer (B) is correct. The vendor would like to sell coffee on cold days ($2,000) and soft drinks on hot days ($2,500).Hot days are expected 40% of the time. Hence, the probability is 40% of making $2,500 by selling soft drinks. The chanceof making $2,000 by selling coffee is 60%. The payoff equation is:

.4($2,500) + .6($2,000) = $2,200

Answer (C) is incorrect because the least the vendor could make by having perfect information is $2,000 on cold days.

Answer (D) is incorrect because the least the vendor could make by having perfect information is $2,000 on cold days.

[110] Gleim #: 1.4.110 -- Source: CMA 1292 4-23

Answer (A) is incorrect because the vendor would pay $600 for perfect information, but the forecasts are only 50%accurate.

Answer (B) is correct. If the weather is hot and coffee is served, the vendor earns $1,900. If the vendor knows theweather will be hot, (s)he would sell soft drinks and make $2,500, a $600 increase. Thus, the vendor should be willing topay up to $600 for perfect information regarding hot weather. However, if the forecasts are only 50% accurate, theinformation is not perfect. Accordingly, the vendor should be willing to pay only $300 (the $600 potential increase inprofits × 50%) for the sometimes accurate forecasts.

Answer (C) is incorrect because the most the vendor could profit from perfect information on hot days would be $600($2,500 – $1,900).

Answer (D) is incorrect because perfect information is worth $600, but information that is 50% accurate warrants only a$300 payment.

[111] Gleim #: 1.4.111 -- Source: CIA 1196 I-29

Answer (A) is incorrect because a classical variables sampling estimate does not help quantify the reasons for the changein production costs.

Answer (B) is incorrect because a ratio analysis of raw inventory could only identify one possible cause for the costincrease.

Answer (C) is correct. Regression analysis extends correlation to find an equation for the linear relationship amongvariables. The behavior of a dependent variable, such as cost of production, is explained in terms of one or moreindependent variables (for example, raw material costs, employees, overtime). Thus, multiple regression analysisdetermines functional relationships among quantitative variables.

Answer (D) is incorrect because like ratio analysis, a linear regression addresses only one possible cause of the costincrease.

Gleim's CIA Test Prep: Part III: Business Analysis and Information TechnologyAnswer Explanations

(1312 questions)

Copyright 2008 Gleim Publications, Inc. Page 34Printed for Mamdouh Farag

Page 35: P.3 Answer Explanation

[112] Gleim #: 1.4.112 -- Source: CIA 591 III-44

Answer (A) is incorrect because a positive correlation coefficient implies that one variable increases (decreases) as theother increases (decreases). The data clearly do not support this conclusion.

Answer (B) is correct. A correlation coefficient of -1.00 implies a perfect inverse correlation; that is, the observations fallexactly along a straight line and the value of one variable increases (decreases) as the other decreases (increases). In theexample to the left, the equation of the straight line is

y = 200 – 4xor x = 50 – 1y

4

Answer (C) is incorrect because a positive correlation coefficient implies that one variable increases (decreases) as theother increases (decreases). The data clearly do not support this conclusion.

Answer (D) is incorrect because a correlation coefficient of zero implies that the two variables are unrelated. The dataclearly indicate that the two variables move in opposite directions.

[113] Gleim #: 1.4.113 -- Source: CIA 595 III-97

Answer (A) is incorrect because $3,729,968 results from improperly calculating the collections of April and March creditsales. The 2% uncollectible amount should not be removed from the credit sales before the collection percentage isapplied.

Answer (B) is correct. The cash receipts for April equal April’s cash sales (30% × $4,000,000 = $1,200,000), 40% ofApril’s credit sales, and 58% of March’s credit sales. Consequently, total cash receipts equal $3,781,600 [$1,200,000 +(40% × 70% × $4,000,000) + (58% × 70% × $3,600,000)].

Answer (C) is incorrect because $4,025,200 includes the collection of May credit sales rather than March credit sales.

Answer (D) is incorrect because $4,408,000 improperly calculates the collections of March credit sales. The calculationtreats the entire sales figure for March as credit sales.

[114] Gleim #: 1.4.114 -- Source: CIA 1193 III-67

Answer (A) is correct. Two basic costs are involved in queuing (waiting-line) models: (1) the cost of providing service(including facility costs and operating costs), and (2) the cost of idle resources waiting in line. The latter may be a directcost if paid employees are waiting, or an opportunity cost in the case of waiting customers. The objective of queuingtheory is to minimize the total cost of the system, including both service and waiting costs, for a given rate of arrivals.This minimization occurs at the point at which cost of waiting is balanced by the cost of providing service. This companywishes to reduce the total of waiting costs (turnaround time) and the cost of copy services.

Answer (B) is incorrect because linear programming is a mathematical technique for optimizing a given objective functionsubject to constraints.

Answer (C) is incorrect because regression analysis is a statistical procedure for estimating the relation between variables.

Answer (D) is incorrect because game theory is a mathematical approach to decision-making in which each decision-maker takes into account the courses of action of competitors.

Gleim's CIA Test Prep: Part III: Business Analysis and Information TechnologyAnswer Explanations

(1312 questions)

Copyright 2008 Gleim Publications, Inc. Page 35Printed for Mamdouh Farag

Page 36: P.3 Answer Explanation

[115] Gleim #: 1.4.115 -- Source: CIA 1192 III-95

Answer (A) is incorrect because ARIMA modeling is a times-series technique.

Answer (B) is incorrect because exponential smoothing is a time-series model.

Answer (C) is correct. Two basic costs are involved in queuing (waiting-line) models: (1) the cost of providing service(including facility costs and operating costs), and (2) the cost of idle resources waiting in line. The latter may be a directcost if paid employees are waiting, or an opportunity cost in the case of waiting customers. The objective of the queuingtheory is to minimize the total cost of the system, including both service and waiting costs, for a given rate of arrivals.This minimization occurs at the point where the cost of waiting is balanced by the cost of providing service.

Answer (D) is incorrect because linear programming optimizes outputs given scarce resources.

[116] Gleim #: 1.4.116 -- Source: CIA 1192 III-96

Answer (A) is correct. Simulation is a technique for experimenting with logical/mathematical models using a computer.Despite the power of mathematics, many problems cannot be solved by known analytical methods because of the behaviorof the variables and the complexity of their interactions. However, the performance of a quantitative model underuncertainty may be investigated by randomly selecting values for each of the variables in the model (based on theprobability distribution of each variable) and then calculating the value of the solution. If this process is performed a largenumber of times, the distribution of results from the model will be obtained.

Answer (B) is incorrect because ARIMA modeling is a time-series technique.

Answer (C) is incorrect because linear programming optimizes outputs given scarce resources.

Answer (D) is incorrect because differential calculus is used to establish optimization points.

[117] Gleim #: 1.4.117 -- Source: CIA 1192 III-97

Answer (A) is incorrect because statistical inference refers to estimation or hypothesis testing.

Answer (B) is incorrect because hypothesis testing relates to the testing of specific null hypotheses.

Answer (C) is incorrect because confidence coefficients are an integral part of statistical estimation and hypothesis testing.

Answer (D) is correct. Validation is a step in the simulation procedure. Some assurance is needed that the results of theexperiment will be realistic. This assurance requires validation of the model -- often using historical data. If the modelgives results equivalent to what actually happened, the model is historically valid. There is still some risk, however, thatchanges could make the model invalid for the future.

Gleim's CIA Test Prep: Part III: Business Analysis and Information TechnologyAnswer Explanations

(1312 questions)

Copyright 2008 Gleim Publications, Inc. Page 36Printed for Mamdouh Farag

Page 37: P.3 Answer Explanation

[118] Gleim #: 1.4.118 -- Source: CIA 1193 III-71

Answer (A) is correct. Game (or decision) theory is a mathematical approach to decision making when confronted with anenemy or competitor. Games are classified according to the number of players and the algebraic sum of the payoffs. In atwo-person game, if the payoff is given by the loser to the winner, the algebraic sum is zero and the game is called a zero-sum game. If it is possible for both players to profit, however, the game is a positive-sum game. Mathematical modelshave been developed to select optimal strategies for certain simple games.

Answer (B) is incorrect because probability theory is a mathematical technique used to express quantitatively thelikelihood of occurrence of an event.

Answer (C) is incorrect because linear programming is a mathematical technique for optimizing a given objective subjectto certain constraints.

Answer (D) is incorrect because sensitivity analysis is a method for studying the effects of changes in one or morevariables on the results of a decision model.

[119] Gleim #: 1.4.119 -- Source: CIA 592 III-64

Answer (A) is correct. Simulation is a technique in which a probabilistic process is first modeled. The inputs to themodel are then varied a large number of times to estimate the distribution of possible outcomes from the model of thevariable of interest. Simulations that use a random-number procedure to generate values for the inputs are referred to asMonte Carlo simulations.

Answer (B) is incorrect because linear programming is a mathematical technique for maximizing or minimizing a givenobjective subject to certain constraints.

Answer (C) is incorrect because correlation analysis is a statistical procedure for studying the relations among variables.

Answer (D) is incorrect because differential analysis is a method used for decision-making that compares differences incosts (and revenues) of two or more possibilities.

[120] Gleim #: 1.4.120 -- Source: CIA 1190 III-41

Answer (A) is incorrect because it is part of the validation process.

Answer (B) is incorrect because it is part of the validation process.

Answer (C) is incorrect because it is part of the validation process.

Answer (D) is correct. The Monte Carlo technique is used in a simulation to generate the individual values for a randomvalue. An essential step in the simulation procedure is to validate the mathematical model used. This process involves notonly searching for errors but also verifying the assumptions. It also should provide some assurance that the results of theexperiment will be realistic. This assurance is often obtained using historical data. If the model gives results equivalent towhat actually happened, the model is historically valid. There is still some risk, however, that changes could make themodel invalid for the future. The model should not be implemented until this validation process is complete.

Gleim's CIA Test Prep: Part III: Business Analysis and Information TechnologyAnswer Explanations

(1312 questions)

Copyright 2008 Gleim Publications, Inc. Page 37Printed for Mamdouh Farag

Page 38: P.3 Answer Explanation

[121] Gleim #: 1.4.121 -- Source: CIA 593 III-65

Answer (A) is correct. Simulation (Monte Carlo simulation) is a technique for experimenting with logical/mathematicalmodels using a computer. Despite the power of mathematics, many problems cannot be solved by known analyticalmethods because of the behavior of the variables and the complexity of their interactions. However, the performance of aquantitative model under uncertainty may be investigated by randomly selecting values for each variable in the model(based on its probability distribution) and then calculating the value of the solution. If this process is performed manytimes, the distribution of results from the model will be obtained.

Answer (B) is incorrect because least squares is a prediction and estimation technique using a single dependent and singleor multiple independent variables.

Answer (C) is incorrect because queuing theory is a waiting-line technique used to balance desirable service levels againstthe cost of providing more service.

Answer (D) is incorrect because exponential smoothing is a forecasting technique using arbitrary weights.

[122] Gleim #: 1.4.122 -- Source: CIA 594 III-60

Answer (A) is correct. Simulation is a technique for experimenting with logical/mathematical models using a computer.The simulation procedure has five steps: define the objectives, formulate the model, validate the model, design theexperiment, and conduct the simulation and evaluate the results. A simulation uses the laws of probability to generatevalues for random variables. Thus, simulation models are probabilistic, not deterministic.

Answer (B) is incorrect because simulation modeling samples the operation of a system.

Answer (C) is incorrect because simulation models mathematically estimate what performance would be under variousconditions.

Answer (D) is incorrect because simulation models are by definition stochastic or probabilistic models.

[123] Gleim #: 1.4.123 -- Source: CIA 593 III-66

Answer (A) is correct. After a problem has been formulated into any mathematical model, it may be subjected tosensitivity analysis. Sensitivity analysis is a method for studying the effects of changes in one or more variables on theresults of a decision model.

Answer (B) is incorrect because statistical estimation involves the estimation of parameters.

Answer (C) is incorrect because statistical hypothesis testing calculates the conditional probability that both thehypothesis is true and the sample results have occurred.

Answer (D) is incorrect because a time-series study involves forecasting data over time.

Gleim's CIA Test Prep: Part III: Business Analysis and Information TechnologyAnswer Explanations

(1312 questions)

Copyright 2008 Gleim Publications, Inc. Page 38Printed for Mamdouh Farag

Page 39: P.3 Answer Explanation

[124] Gleim #: 1.4.124 -- Source: CIA 1194 III-58

Answer (A) is correct. Linear programming is a mathematical technique for planning resource allocation that optimizes agiven objective function that is subject to certain constraints. In this case, the maximum investment is constrained by a70% limit on either investment choice.

Answer (B) is incorrect because capital budgeting is used to analyze and evaluate long-term capital investments.

Answer (C) is incorrect because differential analysis is used for decision making when differences in costs (revenues) fortwo or more options are compared.

Answer (D) is incorrect because queuing theory is used to minimize the sum of the costs of waiting lines and servicingwaiting lines when items arrive randomly at a service point and are serviced sequentially.

[125] Gleim #: 1.4.125 -- Source: CIA 1194 III-61

Answer (A) is correct. Simulation is a technique used to describe the behavior of a real-world system over time. Thistechnique usually employs a computer program to perform the simulation computations. Sensitivity analysis examineshow outcomes change as the model parameters change.

Answer (B) is incorrect because linear programming is a mathematical technique for optimizing a given objective functionsubject to certain constraints.

Answer (C) is incorrect because correlation analysis is a statistical procedure for studying the relation between variables.

Answer (D) is incorrect because differential analysis is used for decision making that compares differences in costs(revenues) of two or more options.

[126] Gleim #: 1.4.126 -- Source: CIA 1196 III-8

Answer (A) is incorrect because, in time series analysis, the cyclical fluctuation in business activity is usually incorporatedas an index number in the forecasting model.

Answer (B) is correct. Internal allocations of costs relate to costs already incurred, that is, to sunk costs. Sunk costs arenot relevant to decision making, for example, to forecasting future purchases.

Answer (C) is incorrect because, in time series analysis, the seasonal variation is usually incorporated as an index numberin the forecasting model.

Answer (D) is incorrect because an econometric model is an application of statistical methods to economic problems. Sucha model is used as a forecasting tool.

[127] Gleim #: 1.4.127 -- Source: CMA 697 4-26

Answer (A) is correct. Regression analysis is used to find an equation for the linear relationship among variables. Thebehavior of the dependent variable is explained in terms of one or more independent variables. Regression analysis isoften used to estimate a dependent variable (such as cost) given a known independent variable (such as production).

Answer (B) is incorrect because regression results are limited to observations within the relevant range.

Answer (C) is incorrect because regression analysis does not use constraint functions.

Answer (D) is incorrect because the dependent variable is estimated using regression analysis.

Gleim's CIA Test Prep: Part III: Business Analysis and Information TechnologyAnswer Explanations

(1312 questions)

Copyright 2008 Gleim Publications, Inc. Page 39Printed for Mamdouh Farag

Page 40: P.3 Answer Explanation

[128] Gleim #: 1.4.128 -- Source: CMA 1285 5-27

Answer (A) is incorrect because -0.73 signifies a strong negative correlation.

Answer (B) is correct. The correlation coefficient can vary from -1 to +1. A -1 relationship indicates a perfect negativecorrelation, and a +1 relationship indicates a perfect positive correlation. A zero correlation coefficient would indicate noassociation between the variables. Thus, the correlation coefficient that is nearest to zero would indicate the weakestlinear association. Of the options given in the question, the correlation coefficient that is nearest to zero is -0.11.

Answer (C) is incorrect because 0.12 is a slightly stronger correlation.

Answer (D) is incorrect because 0.35 is a considerably stronger correlation.

[129] Gleim #: 1.4.129 -- Source: CMA 1289 5-14

Answer (A) is incorrect because the coefficient is negative if the relationship between the variables is inverse.

Answer (B) is incorrect because the coefficient relates the two variables to each other.

Answer (C) is incorrect because the size of the coefficient varies between -1.0 and +1.0.

Answer (D) is correct. The coefficient of correlation (r) measures the strength of the linear relationship between thedependent and independent variables. The magnitude of r is independent of the scales of measurement of x and y. Thecoefficient lies between -1.0 and +1.0. A value of zero indicates no linear relationship between the x and y variables. Avalue of +1.0 indicates a perfectly direct relationship, and a value of -1.0 indicates a perfectly inverse relationship.

[130] Gleim #: 1.4.130 -- Source: CMA 1293 4-25

Answer (A) is incorrect because adding a seasonality factor to, or subtracting it from, a forecast based on trend analysis isa means of adjusting for seasonality.

Answer (B) is incorrect because seasonality factors cannot be ignored; they are reflected in the data and must beconsidered for a model to be accurate.

Answer (C) is correct. Time series analysis relies on past experience. Changes in the value of a variable may have severalpossible components including secular trends, cyclical variation, seasonality, and random variation. Seasonal variationsare common in many businesses. A variety of methods exist for including seasonal variations in a forecasting model, butmost methods use a seasonal index. Alternatively, seasonal variations can be removed from data by using a weightedaverage of several time periods instead of data from individual periods.

Answer (D) is incorrect because the seasonality adjustment for a single season’s data may be an increase or a decrease.

Gleim's CIA Test Prep: Part III: Business Analysis and Information TechnologyAnswer Explanations

(1312 questions)

Copyright 2008 Gleim Publications, Inc. Page 40Printed for Mamdouh Farag

Page 41: P.3 Answer Explanation

[131] Gleim #: 1.4.131 -- Source: CMA 696 4-20

Answer (A) is incorrect because expected value is the probabilistically weighted average of the outcomes of an action.

Answer (B) is incorrect because learning curve analysis quantifies how labor costs decline as employees learn their jobsthrough repetition.

Answer (C) is correct. Sensitivity analysis determines how a result varies with changes in a given variable or parameterin a mathematical decision model. For example, in a present value analysis, a manager might first calculate the netpresent value or internal rate of return assuming that a new asset has a 10-year life. The NPV or IRR can then berecalculated using a 5-year life to determine how sensitive the result is to the change in the assumption.

Answer (D) is incorrect because regression, or least squares, analysis determines the average change in the dependentvariable given a unit change in one or more independent variables.

[132] Gleim #: 1.4.132 -- Source: CIA 597 I-31

Answer (A) is incorrect because simple linear regression is based on just one independent variable.

Answer (B) is incorrect because ratio analysis measure changes but do not explain them.

Answer (C) is incorrect because trend analysis measure changes but do not explain them.

Answer (D) is correct. Regression analysis develops an equation to explain the behavior of a dependent variable (forexample, investment income) in terms of one or more independent variables (for example, market risk and the risks ofparticular investments). Multiple regression analysis is the best approach because it allows the auditor to regress thechange in investment income on more than one independent variable.

[133] Gleim #: 1.4.133 -- Source: CIA R98 III-35

Answer (A) is incorrect because the Delphi technique is an approach in which the manager solicits opinions on a problemfrom experts, summarizes the opinions, and feeds the summaries back to the experts (without revealing any of theparticipants to each other). The process is reiterated until the opinions converge on an optimal solution.

Answer (B) is incorrect because under exponential smoothing, each forecast equals the sum of the last observation timesthe smoothing constant, plus the last forecast times one minus the constant. Thus, exponential means that greater weightis placed on the most recent data, with the weights of all data falling off exponentially as the data age.

Answer (C) is incorrect because regression analysis is used to fit a linear trend line to a dependent variable based on oneor more independent variables.

Answer (D) is correct. Linear programming is a technique used to optimize an objective function, that is, to maximize arevenue or profit function or to minimize a cost function, subject to constraints, e.g., limited (scarce) resources orminimum/maximum levels of production, performance, etc. In business, linear programming is used for planning resourceallocations. Managers are often faced with problems of selecting the most profitable or least costly way to use availableresources.

Gleim's CIA Test Prep: Part III: Business Analysis and Information TechnologyAnswer Explanations

(1312 questions)

Copyright 2008 Gleim Publications, Inc. Page 41Printed for Mamdouh Farag

Page 42: P.3 Answer Explanation

[134] Gleim #: 1.4.134 -- Source: CIA R98 III-36

Answer (A) is incorrect because regression (least squares) analysis extends correlation to find an equation for the linearrelationship among variables. The behavior of the dependent variable is explained in terms of one or more independentvariables. Thus, regression analysis determines functional relationships among quantitative variables.

Answer (B) is incorrect because, under exponential smoothing, each forecast equals the sum of the last observation timesthe smoothing constant, plus the last forecast times one minus the constant. Thus, exponential means that greater weightis placed on the most recent data, with the weights of all data falling off exponentially as the data age.

Answer (C) is correct. The Delphi technique is an approach in which the manager solicits opinions on a problem fromexperts, summarizes the opinions, and feeds the summaries back to the experts (without revealing any of the participantsto each other). The process is reiterated until the opinions converge on an optimal solution. Thus, the Delphi technique isa qualitative, not a quantitative, method.

Answer (D) is incorrect because a moving average is the simplest form of smoothing. Each forecast is based on a fixednumber of prior observations.

[135] Gleim #: 1.4.135 -- Source: IIA, adapted

Answer (A) is incorrect because of the following computation: 0.1($10) + 0.2($10) + 0.5($10) + 0.2($10) = $10.

Answer (B) is incorrect because of the following computation: 0.1($0) + 0.2($20) + 0.5($20) + 0.2($20) = $18.

Answer (C) is correct. To achieve the maximum expected profit, 30 yachts should be built. For each level of production,multiply the probability of demand by the expected profit. The computation is: 0.1(-$10) + 0.2($10) + 0.5($30) + 0.2($30) = $22.

Answer (D) is incorrect because of the following computation: 0.1(-$30) + 0.2(-$10) + 0.5($10) + 0.2($50) = $10.

[136] Gleim #: 1.4.136 -- Source: IIA, adapted

Answer (A) is incorrect because knowing the behavior of business cycles can be valuable when forecasting the requiredpurchases of inventory.

Answer (B) is correct. Forecasts are the basis for business plans. Models are used in the forecasting process to makedecisions that optimize future results. Internal accounting allocations of costs to different segments of the company arearbitrary assignments of already incurred costs that do not have anything to do with forecasting demand.

Answer (C) is incorrect because understanding seasonal variations in demand for the product can be valuable whenforecasting the required purchases of inventory.

Answer (D) is incorrect because using econometric models can be valuable when forecasting the required purchases ofinventory.

Gleim's CIA Test Prep: Part III: Business Analysis and Information TechnologyAnswer Explanations

(1312 questions)

Copyright 2008 Gleim Publications, Inc. Page 42Printed for Mamdouh Farag

Page 43: P.3 Answer Explanation

[137] Gleim #: 1.4.137 -- Source: IIA, adapted

Answer (A) is incorrect because the Delphi technique is a qualitative forecasting method that obtains forecasts throughgroup consensus.

Answer (B) is incorrect because exponential smoothing is a forecasting technique that uses past time series values toarrive at forecasted values.

Answer (C) is incorrect because regression analysis is a statistical technique used to develop forecasts based on therelationship between two or more variables.

Answer (D) is correct. Linear programming is a mathematical technique for maximizing or minimizing a given objectivesubject to certain constraints. It is the correct technique to optimize the problem of limited resources.

[138] Gleim #: 1.5.138 -- Source: CIA 1195 III-15

Answer (A) is incorrect because more detailed information is not available. The Pareto diagram does not focus on the totalquantity of computer complaints.

Answer (B) is incorrect because complaints about CD-ROMs and software are infrequent.

Answer (C) is correct. Complaints based on lack of user knowledge and hardware problems are by far the most frequentaccording to this chart. Consequently, the company should devote its resources primarily to these issues.

Answer (D) is incorrect because cost information is not provided.

[139] Gleim #: 1.5.139 -- Source: CIA 1195 III-16

Answer (A) is incorrect because the chart does not display arithmetic means, relative frequencies, or medians of each typeof complaint.

Answer (B) is incorrect because the chart does not display arithmetic means, relative frequencies, or medians of each typeof complaint.

Answer (C) is incorrect because the chart does not display arithmetic means, relative frequencies, or medians of each typeof complaint.

Answer (D) is correct. This Pareto diagram depicts the frequencies of complaints in absolute terms. It displays the actualnumber of each type of complaint. The chart does not display arithmetic means, relative frequencies, or medians of eachtype of complaint.

[140] Gleim #: 1.5.140 -- Source: CMA 689 5-25

Answer (A) is incorrect because the critical path is not shown on a Gantt chart.

Answer (B) is incorrect because linear programming is used to determine an optimal product mix.

Answer (C) is incorrect because a Gantt chart shows the activities to be completed but not their sequencing.

Answer (D) is correct. A Gantt or bar chart is sometimes used in conjunction with PERT or CPM to show the progress ofa special project. Time is shown on the horizontal axis, the length of a bar equals the length of an activity, and shadingindicates the degree of completion. However, the Gantt chart is not as sophisticated as PERT or CPM in that it does notreflect the relationships among the activities or define a critical path.

Gleim's CIA Test Prep: Part III: Business Analysis and Information TechnologyAnswer Explanations

(1312 questions)

Copyright 2008 Gleim Publications, Inc. Page 43Printed for Mamdouh Farag

Page 44: P.3 Answer Explanation

[141] Gleim #: 1.5.141 -- Source: CIA 586 III-23

Answer (A) is incorrect because the project is ahead of schedule, but activity D has not yet been started, much lesscompleted.

Answer (B) is correct. Assuming that (1) each of the bars represents the expected time necessary to complete an activityand (2) the shaded regions represent the portions completed, activity A has been completed as scheduled and activities Band C are ahead of schedule. Consequently, the project is ahead of schedule.

Answer (C) is incorrect because the project is ahead of schedule, but activity D has not yet been started, much lesscompleted.

Answer (D) is incorrect because the project is ahead of schedule, but activity D has not yet been started, much lesscompleted.

[142] Gleim #: 1.5.142 -- Source: CMA 694 4-1

Answer (A) is incorrect because the most likely time estimate should be included in the formula.

Answer (B) is incorrect because the most likely time estimate should be included in the formula.

Answer (C) is correct. PERT was developed to aid managers in controlling large, complex projects. PERT analysisincludes probabilistic estimates of activity completion times. Three time estimates are made: optimistic, most likely, andpessimistic. The time estimates for an activity are assumed to approximate a beta probability distribution. PERTapproximates the mean of the beta distribution by dividing the sum of the optimistic time, the pessimistic time, and fourtimes the most likely time by six.

Answer (D) is incorrect because all time estimates are not weighted equally.

[143] Gleim #: 1.5.143 -- Source: CMA 688 5-13

Answer (A) is correct. PERT analysis includes probabilistic estimates of activity completion times. Three time estimatesare made: optimistic, most likely, and pessimistic. The time estimates for an activity are assumed to approximate a betaprobability distribution. In contrast to the normal distribution, this distribution has finite endpoints (the optimistic andpessimistic estimates) and is unimodal; that is, it has only one mode (the most likely time). PERT approximates the meanof the beta distribution by dividing the sum of the optimistic time, the pessimistic time, and four times the most likelytime (the mode) by six. The standard deviation is approximated by dividing the difference between the pessimistic andoptimistic times by six. The basis for the latter approximation is that various probability distributions have tails that lieabout plus or minus three standard deviations from the mean. For example, 99.9% of observations in the normaldistribution are expected to lie within this range. Accordingly, if the pessimistic and optimistic times are 21 and 9 weeks,respectively, the standard deviation is 2 weeks (12 ÷ 6).

Answer (B) is incorrect because 6 is the approximate number of standard deviations between the tails of the normaldistribution.

Answer (C) is incorrect because 9 weeks is the pessimistic estimate.

Answer (D) is incorrect because 12 weeks is the difference between the optimistic and the pessimistic estimates.

Gleim's CIA Test Prep: Part III: Business Analysis and Information TechnologyAnswer Explanations

(1312 questions)

Copyright 2008 Gleim Publications, Inc. Page 44Printed for Mamdouh Farag

Page 45: P.3 Answer Explanation

[144] Gleim #: 1.5.144 -- Source: CMA 688 5-17

Answer (A) is incorrect because 7 is the average standard deviation.

Answer (B) is correct. The mean time for the critical path is simply the sum of the means of the activity times. However,the standard deviation equals the square root of the sum of the variances (squares of the standard deviations) of the timesfor activities on the critical path. The standard deviation of the project completion time (time for the critical path) istherefore the square root of 100 (6 2 + 82), or 10.

Answer (C) is incorrect because 14 is the sum of the standard deviations.

Answer (D) is incorrect because 48 is the product of the standard deviations.

[145] Gleim #: 1.5.145 -- Source: CMA 1290 4-7

Answer (A) is incorrect because the expected time of the critical path is 13.0 days.

Answer (B) is correct. The critical path is the longest path. The longest path in the diagram is A-D-E, which requires 13days (5.5 + 7.5) based on expected times.

Answer (C) is incorrect because the expected time of the critical path is 13.0 days.

Answer (D) is incorrect because the expected time of the critical path is 13.0 days.

[146] Gleim #: 1.5.146 -- Source: CMA 1290 4-8

Answer (A) is correct. The critical (longest) path is A-D-E, which has an expected time of 13 days (5.5 + 7.5). However,to decrease the project’s completion time by 1.5 days, paths A-B-C-E (4.5 + 1.0 + 6.5 = 12 days) and A-B-D-E (4.5 + .5 +7.5 = 12.5 days) as well as A-D-E must also be shortened. Hence, A-D-E must be reduced by 1.5 days, A-B-C-E by .5 day,and A-B-D-E by 1.0 day. The only way to decrease A-D-E by 1.5 days is to crash activity AD (5.5 expective time – 4.0crash time = 1.5 days). Crashing DE results in a 1.0-day saving (7.5 – 6.5) only. Crashing AB is the efficient way toreduce both A-B-C-E and A-B-D-E by the desired amount of time because it is part of both paths. The incremental cost ofcrashing AB is $1,000 ($4,000 crash cost – $3,000 normal cost) to shorten the completion time by 1.0 day (4.5 – 3.5). Thealternatives for decreasing both A-B-C-E and A-B-D-E are more costly.

Answer (B) is incorrect because crashing activity DE saves only 1.0 day (7.5 – 6.5) on the critical path and does notreduce the time needed for A-B-C-E.

Answer (C) is incorrect because crashing AD does not reduce the time necessary to complete A-B-C-E or A-B-D-E.

Answer (D) is incorrect because AB and CE are not on the critical path.

[147] Gleim #: 1.5.147 -- Source: CMA 694 4-3

Answer (A) is incorrect because eliminating an activity with slack will not reduce the total time of the project.

Answer (B) is incorrect because the activity with the lowest unit crash cost may not be on the critical path.

Answer (C) is incorrect because the time reduction should be related to its cost. The maximum time reduction may not becost effective.

Answer (D) is correct. When making a cost-time trade-off, the first activity to be crashed (have its completion timeaccelerated) is one on the critical path. To select an activity on another path would not reduce the total time of completion.The initial activity chosen should be the one with the completion time that can be accelerated at the lowest possible costper unit of time saved.

Gleim's CIA Test Prep: Part III: Business Analysis and Information TechnologyAnswer Explanations

(1312 questions)

Copyright 2008 Gleim Publications, Inc. Page 45Printed for Mamdouh Farag

Page 46: P.3 Answer Explanation

[148] Gleim #: 1.5.148 -- Source: CMA 695 4-12

Answer (A) is incorrect because uncertainty is reflected in the use of probabilistic estimates of completion times.

Answer (B) is incorrect because the difference between the latest starting time and earliest finishing time is irrelevant.

Answer (C) is incorrect because the path with the largest amount of time associated with it is the critical path.

Answer (D) is correct. PERT diagrams are free-form networks showing each activity in a large project as a line betweenevents. The critical path is the longest path in time through the network. That path is critical in that, if any activity on thecritical path takes longer than expected, the entire project will be delayed. Paths that are not critical have slack time.Slack is the number of days an activity can be delayed without forcing a delay for the entire project.

[149] Gleim #: 1.5.149 -- Source: CIA 1196 III-96

Answer (A) is incorrect because the project cannot be completed in less than 8 months.

Answer (B) is incorrect because the project cannot be completed in less than 8 months.

Answer (C) is correct. The longest, or critical, path in the network from node (A) to node (F) is path A-C-D-F. All otherpaths are shorter than path A-C-D-F, so the activities along those paths can be completed before the activities along pathA-C-D-F. Thus, the shortest time to complete the project is 8 months (3 + 3 + 2).

Answer (D) is incorrect because no path through the network requires 14 months.

[150] Gleim #: 1.5.150 -- Source: CIA 596 III-100

Answer (A) is incorrect because the critical path method (CPM) is intended to identify bottlenecks in a network and henceidentify the longest path.

Answer (B) is correct. Network models are used to solve managerial problems pertaining to project scheduling,information systems design, and transportation systems design. Networks consisting of nodes and arcs may be created torepresent in graphic form problems related to transportation, assignment, and transshipment. The shortest-route, minimalspanning tree, and maximal flow problems are other applications of network models. A shortest-route algorithmminimizes total travel time from one site to each of the other sites in a transportation system.

Answer (C) is incorrect because the maximal flow algorithm maximizes throughput in networks with distinct entry (sourcenode) and exit (sink node) points. Examples of applications are highway transportation systems and oil pipelines. Flowsare limited by capacities of the arcs (e.g., highways or pipes).

Answer (D) is incorrect because the minimal spanning tree algorithm identifies the set of connecting branches having theshortest combined length. A spanning tree is a group of branches (arcs) that connects each node in the network to everyother node. An example problem is the determination of the shortest telecommunications linkage among users at remotesites and a central computer.

Gleim's CIA Test Prep: Part III: Business Analysis and Information TechnologyAnswer Explanations

(1312 questions)

Copyright 2008 Gleim Publications, Inc. Page 46Printed for Mamdouh Farag

Page 47: P.3 Answer Explanation

[151] Gleim #: 1.5.151 -- Source: CIA 597 III-100

Answer (A) is incorrect because 11 days is the shortest, not the longest, time to completion.

Answer (B) is incorrect because 14 days is not the completion time of a path to completion.

Answer (C) is correct. The two paths through the network are ACE (5 + 4 + 6 = 15 days) and BDE (3 + 2 + 6 = 11 days).The critical or longest path is A-C-E. Hence, the earliest completion time is 15 days.

Answer (D) is incorrect because 20 days is the sum of all of the activity times.

[152] Gleim #: 1.5.152 -- Source: CIA 594 III-61

Answer (A) is incorrect because a dummy activity is one that consumes no time but establishes precedence amongactivities. It is used specifically in project management.

Answer (B) is incorrect because the latest finish is the latest that an activity can finish without causing delay in thecompletion of the project.

Answer (C) is incorrect because optimistic time is the time for completing a project if all goes well.

Answer (D) is correct. Project management concerns managing teams assigned to special projects. Lumpy demand refersto periodic demand for a product or service that increases in large, lumpy increments.

[153] Gleim #: 1.5.153 -- Source: CIA 592 III-69

Answer (A) is incorrect because the path length of A-C is 13.

Answer (B) is incorrect because the path length of B-E is 8.

Answer (C) is correct. The critical path is the longest path because it defines the minimum duration of the project. A-D-E(4 + 6 + 5 = 15) is the critical path.

Answer (D) is incorrect because B-D-C is not a path. The predecessor of C is A.

[154] Gleim #: 1.5.154 -- Source: CIA 1191 III-37

Answer (A) is incorrect because an activity with slack may nevertheless be essential to the overall project.

Answer (B) is incorrect because it is not a backup activity.

Answer (C) is correct. Slack is the free time associated with each activity. In other words, paths that are not critical haveslack time. Slack represents unused resources that can be diverted to the critical path.

Answer (D) is incorrect because time is involved in a slack activity.

Gleim's CIA Test Prep: Part III: Business Analysis and Information TechnologyAnswer Explanations

(1312 questions)

Copyright 2008 Gleim Publications, Inc. Page 47Printed for Mamdouh Farag

Page 48: P.3 Answer Explanation

[155] Gleim #: 1.5.155 -- Source: CMA 689 5-24

Answer (A) is incorrect because CPM specifies the activity times (uses deterministic estimates).

Answer (B) is incorrect because CPM but not PERT uses activity costs and considers crash times.

Answer (C) is incorrect because a less significant difference between PERT and CPM is that PERT uses probabilisticestimates of completion times. CPM times are deterministic. The 1:4:1 method is typically used in PERT. Under thismethod, the most optimistic and pessimistic estimates are weighted equally, but the most likely estimate is weighted fourtimes more heavily than the others.

Answer (D) is correct. Both PERT and CPM are network analysis techniques. But CPM was developed independently ofPERT and is widely used in the construction industry. CPM may be thought of as a subset of PERT. Like PERT, it is anetwork technique, but, unlike PERT, it uses deterministic time and cost estimates. Its advantages include cost estimatesplus the concept of "crash" efforts and costs. Activity times are estimated for normal effort and crash effort. Crash time isthe time to complete an activity assuming that all available resources were devoted to the task (overtime, extra crew, etc.).Activity costs are also estimated for normal and crash efforts. These estimates allow the project manager to estimate thecosts of completing the project if some of the activities are completed on a crash basis. The network diagram isconstructed in the same manner as PERT diagrams. Once the diagram is constructed, the critical paths are found fornormal and crash times. More than one critical path may exist for each diagram.

[156] Gleim #: 1.5.156 -- Source: Publisher

Answer (A) is incorrect because PERT is a control tool as well as a planning tool.

Answer (B) is correct. PERT (Program Evaluation Review Technique) is applied in the management of complex projects.It analyzes the project in terms of its component activities and determines their sequencing and timing. Thus, itsystematically combines planning with control.

Answer (C) is incorrect because PERT is a planning tool.

Answer (D) is incorrect because PERT is a planning tool.

[157] Gleim #: 1.5.157 -- Source: CIA 597 III-98

Answer (A) is correct. When making a cost-time trade-off, the first activity to be crashed (have its completion timeaccelerated) is one on the critical path. To select an activity on another path would not reduce the total time of completion.The activity chosen should have a completion time that can be accelerated at the lowest possible cost per unit of timesaved.

Answer (B) is incorrect because the Delphi technique is a qualitative forecasting approach.

Answer (C) is incorrect because material-requirements planning is an inventory model.

Answer (D) is incorrect because a branch-and-bound solution is an integer programming solution.

Gleim's CIA Test Prep: Part III: Business Analysis and Information TechnologyAnswer Explanations

(1312 questions)

Copyright 2008 Gleim Publications, Inc. Page 48Printed for Mamdouh Farag

Page 49: P.3 Answer Explanation

[158] Gleim #: 1.5.158 -- Source: IIA, adapted

Answer (A) is correct. Activity times are estimated for normal effort and crash effort. Crashing is the process of addingresources to shorten activity times on the critical path in project scheduling.

Answer (B) is incorrect because the Delphi technique is a qualitative forecasting approach.

Answer (C) is incorrect because ABC analysis is an inventory model.

Answer (D) is incorrect because the branch-and-bound solution is an integer programming solution.

[159] Gleim #: 1.6.159 -- Source: CIA 1195 I-66

Answer (A) is incorrect because budgetary comparison is a typical example of a monitoring control.

Answer (B) is incorrect because investigation of exceptions is a monitoring control used by lower-level management todetermine when their operations may be out of control.

Answer (C) is correct. Monitoring assesses the quality of internal control over time. Management considers whetherinternal control is properly designed and operating as intended and modifies it to reflect changing conditions. Monitoringmay be in the form of separate, periodic evaluations or of ongoing monitoring. Ongoing monitoring occurs as part ofroutine operations. It includes management and supervisory review, comparisons, reconciliations, and other actions bypersonnel as part of their regular activities. However, reconciling batch control totals is a processing control.

Answer (D) is incorrect because internal auditing is a form of monitoring. It serves to evaluate management’s othercontrols.

[160] Gleim #: 1.6.160 -- Source: Publisher

Answer (A) is correct. The theory of constraints (TOC) is a short-term approach to managing bottlenecks (bindingconstraints) in production and distribution processes. Its basic principle is that short-term profit maximization requiresmaximizing the contribution margin of the binding constraint (the throughput contribution).

Answer (B) is incorrect because reengineering an approach to business process analysis that entails process innovationand core process redesign. Instead of improving existing procedures, it finds new ways of doing things.

Answer (C) is incorrect because rationalization is yet another approach to business process analysis. It is the streamliningof procedures to make automation more efficient.

Answer (D) is incorrect because a micromotion study is a method used for work management, and it requires videotapingthe performance of a job.

[161] Gleim #: 1.6.161 -- Source: Publisher

Answer (A) is incorrect because $50,000 is the incremental cost.

Answer (B) is incorrect because $36,000 is the incremental throughput contribution.

Answer (C) is correct. Operation 2 is the bottleneck because it is functioning at its capacity. The incremental annualthroughput contribution (revenues – direct materials costs) from adding workers to Operation 2 is $36,000 [500 units ×($120 unit price – $48 DM per unit)]. Because the cost of the additional workers is $50,000, the change in operatingincome is $(14,000).

Answer (D) is incorrect because $(20,000) is based on the assumption that an additional $12 per unit of fixed costs willbe applied.

Gleim's CIA Test Prep: Part III: Business Analysis and Information TechnologyAnswer Explanations

(1312 questions)

Copyright 2008 Gleim Publications, Inc. Page 49Printed for Mamdouh Farag

Page 50: P.3 Answer Explanation

[162] Gleim #: 1.6.162 -- Source: Publisher

Answer (A) is correct. X’s offer should be accepted because its cost is $40,000 (1,000 units × $40), but the increase inthroughput contribution is $72,000 [1,000 units × ($120 unit price – $48 DM per unit)]. Hence, the relevant cost of X’soffer is less than the incremental throughput contribution. X’s offer effectively increases the capacity of the bottleneckoperation. Y’s offer should be rejected because, even though its $7 unit price is less than the $8 unit operating cost(excluding direct materials) for Operation 1, it will result in the incurrence of additional costs with no increase inthroughput contribution, given that Operation 2 is already producing at its 150,000-unit capacity.

Answer (B) is incorrect because X’s offer but not Y’s offer is acceptable.

Answer (C) is incorrect because X’s offer but not Y’s offer is acceptable.

Answer (D) is incorrect because X’s offer but not Y’s offer is acceptable.

[163] Gleim #: 1.6.163 -- Source: Publisher

Answer (A) is incorrect because Company J can sell only 45 units of A-1.

Answer (B) is incorrect because Company J can produce only 75 units of A-2 if it produces 45 units of the more profitableA-1.

Answer (C) is correct. A-1’s throughput contribution margin per unit of the scarce resource (the internal bindingconstraint) is $150 ($150 UCM ÷ 1 machining hour). A-2’s throughput contribution margin per unit of the scarce resourceis $100 ($300 UCM ÷ 3 machine hours). Consequently, Company J should produce as much A-1 as it can sell (45 units).If Company J adds 90 machine hours to increase the capacity of Operation 1 to 270 hours (180 + 90), it cannot produceadditional units of A-1 because the external binding constraint has not been relaxed. However, it can produce additionalunits of A-2. Given that the UCM per machine hour of A-2 is $100 and that the cost is $80 per hour, adding capacity toOperation 1 is profitable. Thus, Company J should use 45 machine hours to produce 45 units of A-1. The remaining225 machine hours (270 – 45) should be used to produce 75 units (225 ÷ 3 hours) of A-2. The latter amount is within theexternal binding constraint.

Answer (D) is incorrect because Company J should produce as much of A-1 as it can sell.

[164] Gleim #: 1.6.164 -- Source: CIA 1192 III-41

Answer (A) is correct. A model consisting of a system of functions may be used to optimize an objective function. If thefunctions in the model are all linear, the model is a linear programming model. Linear programming is a technique todetermine optimal resource allocation. Several solution methods are available to solve linear programming problems. Thegraphical method, the easiest technique, is limited to simple problems. Here, the graph consists of three lines, eachrepresenting a production constraint. The lines connecting points 3, 4, 6, and 7 bound the feasible solution region. Productmixes of X and Y that lie outside this boundary cannot be produced and/or sold because the demand constraint (line 3,4),the labor constraint (line 4,6), and the material constraint (line 6,7) are binding.

Answer (B) is incorrect because points 1, 5, and 8 are outside the boundary.

Answer (C) is incorrect because points 2, 5, and 8 are outside the boundary.

Answer (D) is incorrect because point 5 is outside the boundary.

Gleim's CIA Test Prep: Part III: Business Analysis and Information TechnologyAnswer Explanations

(1312 questions)

Copyright 2008 Gleim Publications, Inc. Page 50Printed for Mamdouh Farag

Page 51: P.3 Answer Explanation

[165] Gleim #: 1.6.165 -- Source: CIA 1192 III-42

Answer (A) is correct. A profit line has negative slope because the profit from sales of one product increases as the profitfrom sales of the other product declines. Moving the profit line rightward (while maintaining its slope) to the last point inthe feasible region determines the solution.

Answer (B) is incorrect because the last point in the feasible solution region touched by a profit line determines thesolution.

Answer (C) is incorrect because the last point in the feasible solution region touched by a profit line determines thesolution.

Answer (D) is incorrect because the last point in the feasible solution region touched by a profit line determines thesolution.

[166] Gleim #: 1.6.166 -- Source: Publisher

Answer (A) is incorrect because the $4,380 of contribution margin yielded by 146 units of X and 0 units of Y is less thanthe $5,600 yielded by 120 units of X and 40 units of Y.

Answer (B) is incorrect because the $5,000 of contribution margin yielded by 0 units of X and 100 units of Y is less thanthe $5,600 yielded by 120 units of X and 40 units of Y.

Answer (C) is correct. Linear programming is a technique used to maximize a contribution margin function or tominimize a cost function, subject to constraints such as scarce resources or minimum/maximum levels of production.Thus, linear programming is often used for planning resource allocations. In this problem, the equation to be maximized,called the objective function, is: $30X + $50Y. This equation is to be maximized subject to the constraints on materials.The two constraint functions are:

Material A: 6X + 12Y < 1,200Material B: 12X + 8Y < 1,760

One way to solve this problem is to graph the constraint lines and determine the feasible area. The optimal productionlevel is at an extreme point within the feasible area. The graph shows that a production level of 120 units of X and 40units of Y is a feasible production level that maximizes the contribution margin.

Answer (D) is incorrect because 40 units of X and 120 units of Y is not within the feasible production area. It violates theconstraint on Material A.

[167] Gleim #: 1.6.167 -- Source: Publisher

Answer (A) is incorrect because this equation multiplies the time of production rather than dividing it.

Answer (B) is correct. The constraint function that expresses the number of units to be produced is R + W = 96, but thatis not one of the answer choices. Another constraint is that the total quantities of red (R) and white (W) units must beproduced in 20 or fewer days at a rate of 4 red units per day and 7 white units per day. Thus, the time constraint is (R ÷ 4)+ (W ÷ 7) < 20.

Answer (C) is incorrect because the total production must be 96.

Answer (D) is incorrect because 96 is total production.

Gleim's CIA Test Prep: Part III: Business Analysis and Information TechnologyAnswer Explanations

(1312 questions)

Copyright 2008 Gleim Publications, Inc. Page 51Printed for Mamdouh Farag

Page 52: P.3 Answer Explanation

[168] Gleim #: 1.6.168 -- Source: CIA 1191 III-33

Answer (A) is incorrect because the objective function is to maximize the total contribution margin (4A + 5B). Thisamount is not expressed as an inequality.

Answer (B) is incorrect because this equation (A < 0) is nonsensical.

Answer (C) is correct. The linear programming solution is subject to constraints on the availability of machine hours inboth centers. For example, products A and B require 2.5 and 4 hours per unit, respectively, in Machine Center 1, but only60 hours are available. Hence, the optimal production of A and B to the following constraint:

2.5A + 4B < 60

Answer (D) is incorrect because the hours available in the two machine centers cannot be added together. Also, the leftside of the equation states the objective function, not a constraint.

[169] Gleim #: 1.6.169 -- Source: CIA 1196 III-23

Answer (A) is correct. The intent of quality control is to ensure that goods and services conform to the designspecifications. Whether the focus is on feedforward, feedback, or concurrent control, the emphasis is on ensuring productor service conformity.

Answer (B) is incorrect because quality control is geared towards satisfying the customer, not upper management.

Answer (C) is incorrect because ensuring the conformance with ISO-9000 specifications is a component of a complianceaudit, not quality control.

Answer (D) is incorrect because determining the appropriate timing of inspections is only one step towards approachingquality control. Consequently, it is not the primary component of the quality control function.

[170] Gleim #: 1.6.170 -- Source: CIA 593 III-63

Answer (A) is incorrect because 3X + 7Y is a constraint.

Answer (B) is incorrect because 2X + Y is a constraint.

Answer (C) is correct. The objective function is the function to be optimized. This firm wishes to maximize profits on thesales of two products (X and Y). Based on profits per unit ($8 and $6, respectively), the objective function is 8X + 6Y.

Answer (D) is incorrect because 5X + 8Y is the sum of the constraints.

[171] Gleim #: 1.6.171 -- Source: IIA, adapted

Answer (A) is incorrect because the original data should be divided by the seasonal factor, not increased.

Answer (B) is incorrect because the original data should be divided by the seasonal factor, not reduced.

Answer (C) is incorrect because the original data should be divided by the seasonal factor, not multiplied.

Answer (D) is correct. Seasonal variations are common in many businesses. To remove the effect of seasonal variationfrom a time series, the original data (with the four trends) is divided by the seasonal norm.

Gleim's CIA Test Prep: Part III: Business Analysis and Information TechnologyAnswer Explanations

(1312 questions)

Copyright 2008 Gleim Publications, Inc. Page 52Printed for Mamdouh Farag

Page 53: P.3 Answer Explanation

[172] Gleim #: 1.6.172 -- Source: IIA, adapted

Answer (A) is incorrect because control self-assessment involving only sales representatives is not the most appropriatemethodology. Even process-based CSA involving these representatives will not be as effective as a review of the fullorganizational context of these costs.

Answer (B) is incorrect because benchmarking has some applicability but it is not as effective as a full review of all theprocesses related to these costs.

Answer (C) is correct. A business process review (BPR) assesses the performance of administrative, financial, and otherprocesses, such as those within the procurement and payables functions. BPR considers process effectiveness andefficiency, including the presence of appropriate controls, to mitigate business risk. It seeks to achieve improvements insuch critical measures of performance as cost, quality, service, speed, and customer satisfaction. Because the objective isto control cellular phone costs, BPR is the appropriate tool.

Answer (D) is incorrect because performance measurement and design of the budgeting and forecasting processes do notaddress cost control.

[173] Gleim #: 2.1.1 -- Source: CMA 686 5-29

Answer (A) is incorrect because $48 equals total annual cost divided by total orders from the previous year.

Answer (B) is incorrect because $35 equals total annual cost minus the costs associated with the shipping clerks, dividedby total orders from the previous year.

Answer (C) is incorrect because $24 includes the costs associated with shipping.

Answer (D) is correct. The incremental cost of additional orders equals the variable costs incurred in purchasing andreceiving inventory (but not costs associated with shipping). Using the high-low method of analysis, these variable costscan be computed by determining the apparent variable costs for the two given levels of activity. For 160 orders, therelevant costs with a variable element include purchasing clerks, $1,750; supplies, $400; and receiving clerks, $2,200.The total is $4,350. For 100 orders, these costs are $1,250, $260, and $1,700, respectively, for a total of $3,210. As thenumber of orders increased by 60, the costs increased by $1,140. Consequently, the variable or incremental costs per ordermust have been $19 ($1,140 ÷ 60 orders).

[174] Gleim #: 2.1.2 -- Source: CMA 686 5-30

Answer (A) is correct. The annual carrying costs are $12,750 for the warehouse, $1,500 for insurance, and $2,250 forproperty taxes. These costs total $16,500. In addition, the company desires a 12% after-tax return on investments. Becausethe tax rate is 40%, the 12% after-tax return equals a 20% before-tax return (12% ÷ 60%). A 20% return on the $75,000average investment in inventory is $15,000. Hence, the total carrying cost is $31,500 ($16,500 + $15,000). This amount is42% of the $75,000 investment in inventory.

Answer (B) is incorrect because 37% does not consider insurance and property taxes.

Answer (C) is incorrect because 34% is based on a 12% before-tax return.

Answer (D) is incorrect because 22% equals carrying costs other than return on investment divided by average inventory.

Gleim's CIA Test Prep: Part III: Business Analysis and Information TechnologyAnswer Explanations

(1312 questions)

Copyright 2008 Gleim Publications, Inc. Page 53Printed for Mamdouh Farag

Page 54: P.3 Answer Explanation

[175] Gleim #: 2.1.3 -- Source: CIA 597 III-91

Answer (A) is correct. The objective of the EOQ model is to find an optimal order quantity that balances carrying andordering costs. Only variable costs should be considered. The EOQ is the point where the ordering cost and carrying costcurves intersect. It corresponds to the minimum point on the total inventory cost curve.

Answer (B) is incorrect because the price (purchase costs) is not directly incorporated into the EOQ model.

Answer (C) is incorrect because neither the price nor quality costs are incorporated into the EOQ model.

Answer (D) is incorrect because stockout costs are not directly incorporated into the EOQ model.

[176] Gleim #: 2.1.4 -- Source: CMA 1290 4-9

Answer (A) is incorrect because the purchasing manager’s salary is a fixed cost. The EOQ model includes variable costsonly.

Answer (B) is incorrect because advertising is not an ordering or carrying cost.

Answer (C) is incorrect because the cost of shipping to customers is a selling expense.

Answer (D) is correct. The determination of the economic order quantity balances the variable costs of ordering andcarrying inventory. Factors in the equation include the cost of placing an order, unit carrying cost, and annual demand inunits. Carrying costs include storage costs, handling costs, insurance, property taxes, obsolescence, and the opportunitycost of investing capital in inventory. Thus, the return on capital that is forgone when it is invested in inventory should beconsidered.

[177] Gleim #: 2.1.5 -- Source: CMA 1294 4-8

Answer (A) is correct. The basic EOQ model equals the square root of the quotient of (1) the product of twice the demandtimes the cost per order, (2) divided by the periodic carrying cost. Hence, the model is relatively insensitive to error. Agiven percentage error in a value results in a lower percentage change in the EOQ.

Answer (B) is incorrect because the EOQ model can be used regardless of the relationship between carrying and holdingcosts.

Answer (C) is incorrect because product demand and ordering costs must be known with some certainty.

Answer (D) is incorrect because an EOQ model can be used with any type of system.

[178] Gleim #: 2.1.6 -- Source: CMA 1294 4-6

Answer (A) is incorrect because carrying costs typically decline in JIT companies. Less inventory is on hand.

Answer (B) is correct. The objective of JIT is to reduce carrying costs by eliminating inventories and increasing thedeliveries made by suppliers. Ideally, shipments of raw materials are received just in time to be incorporated into themanufacturing process. The focus of quality control under JIT is the prevention of quality problems. Quality control isshifted to the supplier. JIT companies typically do not inspect incoming goods; the assumption is that receipts are ofperfect quality. Suppliers are limited to those who guarantee perfect quality and prompt delivery.

Answer (C) is incorrect because, in a JIT system, materials are delivered directly to the production line ready for insertionin the finished product.

Answer (D) is incorrect because the need for communication with the vendor is greater. Orders and deliveries must bemade on short notice, sometimes several times a day.

Gleim's CIA Test Prep: Part III: Business Analysis and Information TechnologyAnswer Explanations

(1312 questions)

Copyright 2008 Gleim Publications, Inc. Page 54Printed for Mamdouh Farag

Page 55: P.3 Answer Explanation

[179] Gleim #: 2.1.7 -- Source: CIA 597 III-94

Answer (A) is incorrect because the supplier may seek a concession on the selling price that will raise purchasing costs,but the manufacturing company’s stockout costs will increase.

Answer (B) is incorrect because the cost of quality is not necessarily affected by a JIT system.

Answer (C) is incorrect because fewer purchase orders are processed by the manufacturer, so the ordering costs are likelyto decrease. However, the cost of quality is not necessarily affected by a JIT system.

Answer (D) is correct. The objective of a JIT system is to reduce carrying costs by eliminating inventories and increasingthe deliveries made by suppliers. Ideally, shipments are received just in time to be incorporated into the manufacturingprocess. This system increases the risk of stockout costs because the inventory buffer is reduced or eliminated.

[180] Gleim #: 2.1.8 -- Source: CMA 1289 5-16

Answer (A) is incorrect because AB is the time between receipt of the last order and the placing of the next order.

Answer (B) is incorrect because AE is the safety stock.

Answer (C) is incorrect because AF represents the quantity of inventory that will be used during the reorder lead time.

Answer (D) is correct. The quantity of inventory on hand is represented by the y axis and time by the x axis. The reorderlead time is represented by the line segment BC.

[181] Gleim #: 2.1.9 -- Source: CMA 1289 5-17

Answer (A) is incorrect because AB is the time between the receipt of the last order and the placing of the next order.

Answer (B) is correct. Quantities of inventory are shown along the y axis. Safety stock is represented by the line AE.

Answer (C) is incorrect because AC is the time to consume the EOQ.

Answer (D) is incorrect because BC is the reorder lead time.

[182] Gleim #: 2.1.10 -- Source: CMA 1289 5-18

Answer (A) is incorrect because DE represents the total inventory on hand just after an order has been received.

Answer (B) is incorrect because BC is the reorder lead time.

Answer (C) is correct. Time is shown along the x axis. The line segment AC depicts the time to consume an entire order(to reduce the inventory to the safety stock).

Answer (D) is incorrect because AE is the safety stock.

Gleim's CIA Test Prep: Part III: Business Analysis and Information TechnologyAnswer Explanations

(1312 questions)

Copyright 2008 Gleim Publications, Inc. Page 55Printed for Mamdouh Farag

Page 56: P.3 Answer Explanation

[183] Gleim #: 2.1.11 -- Source: CMA 692 4-23

Answer (A) is incorrect because a safety stock of 0 units has a total expected cost of $21,000.

Answer (B) is incorrect because a safety stock of 100 units has a total expected cost of $13,100.

Answer (C) is incorrect because a safety stock of 300 units has a total expected cost of $3,600.

Answer (D) is correct. The total expected cost of safety stock equals the sum of the expected annual stockout cost and theexpected annual carrying cost. Annual expected stockout cost equals the cost per occurrence ($1,750), times theprobability of a stockout per cycle, times the number of cycles (24). Annual expected carrying cost of a safety stock equalsthe unit carrying cost ($5) times the number of units. Hence, a safety stock of 400 units has the lowest total expected cost.

Expected Expected TotalUnits Carrying Stockout Cost Stockout Cost ExpectedHeld Cost Per Cycle for 24 Cycles Cost

0 $ 0 $875.00 $21,000 $21,000100 500 525.00 12,600 13,100200 1,000 245.00 5,880 6,880300 1,500 87.50 2,100 3,600400 2,000 17.50 420 2,420

[184] Gleim #: 2.1.12 -- Source: CMA 1293 4-9

Answer (A) is incorrect because 5,000 units does not allow for safety stock.

Answer (B) is correct. The reorder point is the quantity on hand when an order is placed. With a 20-day normal leadtime, a safety stock of 1,750 units, and usage of 250 units per day, an order should be placed when 27 days of inventoryare on hand, a total of 6,750 units (250 units × 27 days).

Answer (C) is incorrect because 1,750 units covers only safety stock.

Answer (D) is incorrect because 5,250 units includes only 1 day of safety stock.

[185] Gleim #: 2.1.13 -- Source: CMA 694 4-23

Answer (A) is incorrect because a single production run indicates an EOQ of 5,000 units. The carrying costs of $25,000[$10 × ($5,000 ÷ 2)] would exceed the $1,000 of setup costs.

Answer (B) is incorrect because two production runs correspond to an EOQ of 2,500 units and an average inventory of1,250 units. The resulting $12,500 of carrying costs would exceed the $2,000 of setup costs.

Answer (C) is incorrect because four production runs correspond to an EOQ of 1,250 units and an average inventory of625 units. The resulting $6,250 of carrying costs would exceed the $4,000 of setup costs.

Answer (D) is correct. The EOQ minimizes the sum of carrying and setup costs. The EOQ is the amount at whichcarrying costs are equal to setup costs. Thus, plugging the data into the EOQ formula results in the following:

Thus, if each lot consists of 1,000 units, five production runs per year are needed to meet the 5,000-unit demand. At thislevel, setup costs will total $5,000 (5 × $1,000). Carrying costs will also equal $5,000 ($10 per unit carrying cost ×average inventory of 500 units). Accordingly, total costs are minimized at $10,000.

Gleim's CIA Test Prep: Part III: Business Analysis and Information TechnologyAnswer Explanations

(1312 questions)

Copyright 2008 Gleim Publications, Inc. Page 56Printed for Mamdouh Farag

Page 57: P.3 Answer Explanation

[186] Gleim #: 2.1.14 -- Source: CMA 694 4-22

Answer (A) is incorrect because the cost of maintaining an average inventory of 625 units is $6,250.

Answer (B) is incorrect because $5,000 is based upon an EOQ of 1,000 units and an average inventory of 500 units.

Answer (C) is correct. Given four production runs and an annual demand of 5,000 units, each production run mustgenerate 1,250 units. Inventory will total 1,250 units at the completion of each run but will decline to zero just prior to thenext run. Thus, the average inventory is 625 units (1,250 ÷ 2), and the total carrying cost is $6,250 (625 units × $10).

Answer (D) is incorrect because $12,500 is based on the maximum inventory level.

[187] Gleim #: 2.1.15 -- Source: CIA, adapted

Answer (A) is incorrect because just-in-time (JIT) production approaches increase the need for reliable suppliers. Wheninventories are at low or nonexistent levels, supplier performance is critical.

Answer (B) is correct. Just-in-time (JIT) production approaches can reduce the cost of production. It lowers or eliminatesinventory costs and requires production improvements to eliminate product defects.

Answer (C) is incorrect because just-in-time (JIT) production increases flexibility. Production is pulled by demand.

Answer (D) is incorrect because JIT requires workers to be multiskilled and independent. Such traits are needed in a pullsystem.

[188] Gleim #: 2.1.16 -- Source: IIA, adapted

Answer (A) is correct. If there is no monetary amount in the database for existing inventory, inventory will beunderstated. Moreover, inadequate edit checks or uncontrolled borrow/paybacks could cause negative quantities to bereported. This condition would cause inventory to be understated. If the amount ordered exceeds requirements, the resultis an increase in inventory. However, by itself, this condition would not cause inventory to be understated or overstated.Also, if lead times are longer than delivery times, the effect is an increase in inventory but not necessarily a misstatement.

Answer (B) is incorrect because, if there is no monetary amount in the database for existing inventory, inventory will beunderstated. Moreover, inadequate edit checks or uncontrolled borrow/paybacks could cause negative quantities to bereported. This condition would cause inventory to be understated. If the amount ordered exceeds requirements, the resultis an increase in inventory. However, by itself, this condition would not cause inventory to be understated or overstated.Also, if lead times are longer than delivery times, the effect is an increase in inventory but not necessarily a misstatement.

Answer (C) is incorrect because, if there is no monetary amount in the database for existing inventory, inventory will beunderstated. Moreover, inadequate edit checks or uncontrolled borrow/paybacks could cause negative quantities to bereported. This condition would cause inventory to be understated. If the amount ordered exceeds requirements, the resultis an increase in inventory. However, by itself, this condition would not cause inventory to be understated or overstated.Also, if lead times are longer than delivery times, the effect is an increase in inventory but not necessarily a misstatement.

Answer (D) is incorrect because, if there is no monetary amount in the database for existing inventory, inventory will beunderstated. Moreover, inadequate edit checks or uncontrolled borrow/paybacks could cause negative quantities to bereported. This condition would cause inventory to be understated. If the amount ordered exceeds requirements, the resultis an increase in inventory. However, by itself, this condition would not cause inventory to be understated or overstated.Also, if lead times are longer than delivery times, the effect is an increase in inventory but not necessarily a misstatement.

Gleim's CIA Test Prep: Part III: Business Analysis and Information TechnologyAnswer Explanations

(1312 questions)

Copyright 2008 Gleim Publications, Inc. Page 57Printed for Mamdouh Farag

Page 58: P.3 Answer Explanation

[189] Gleim #: 2.1.17 -- Source: CIA 1193 II-9

Answer (A) is correct. The effect of this policy is to increase inventory holding (carrying) costs. Using the longest leadtime avoids stockout costs but ensures that many items will be delivered long before they are needed.

Answer (B) is incorrect because rotating assignments would not directly affect holding costs.

Answer (C) is incorrect because approval requirements would not increase holding costs.

Answer (D) is incorrect because this would address holding costs for finished goods, but not for raw materials andsubassemblies.

[190] Gleim #: 2.1.18 -- Source: CIA 595 III-32

Answer (A) is incorrect because JIT systems can require significant computer resources, but they can also be maintainedmanually.

Answer (B) is incorrect because contracts may have to be renegotiated with strict delivery and quality specifications, butthese changes usually occur over extended periods.

Answer (C) is correct. JIT minimizes inventory by relying on coordination with suppliers to provide deliveries when theyare needed for production. Consequently, work stoppages at suppliers or transportation disruptions can cause almostimmediate work stoppages at the manufacturer plant.

Answer (D) is incorrect because JIT can be implemented over an extended period or a shorter time frame depending onthe manufacturer’s immediate needs.

[191] Gleim #: 2.1.19 -- Source: CIA 596 III-98

Answer (A) is incorrect because 1,400 wood screws assumes that one wood screw is used per banana hook.

Answer (B) is correct. The company needs 1,800 banana hooks (2,000 – 200) and therefore 1,800 swag hooks (1 × 1,800)and 3,600 wood screws (2 × 1,800). Given that 300 swag hooks and 400 wood screws are on hand, the company mustobtain 1,500 swag hooks (1,800 – 300) and 3,200 wood screws (3,600 – 400).

Answer (C) is incorrect because 1,700 swag hooks and 3,600 wood screws would be needed if no banana hooks were incurrent inventory.

Answer (D) is incorrect because 1,800 swag hooks would be needed if no swag hooks were in current inventory. Also,3,600 wood screws would be needed if no banana hooks were in current inventory.

[192] Gleim #: 2.1.20 -- Source: CIA 594 III-66

Answer (A) is incorrect because the basic EOQ model does not include safety stock.

Answer (B) is correct. The purpose of the EOQ model is to minimize the sum of inventory order costs and holding costs.The EOQ equals the square root of: twice the annual demand multiplied by the variable cost per order, divided by the unitperiodic holding cost.

Answer (C) is incorrect because, in the EOQ model, costs, not quantities, are to be minimized.

Answer (D) is incorrect because quantity demanded is a variable in the model, but order costs, not demand costs, arerelevant. Backlogs are customer orders that cannot be filled immediately because of stockouts. Backlog costs are notquantified in the model.

Gleim's CIA Test Prep: Part III: Business Analysis and Information TechnologyAnswer Explanations

(1312 questions)

Copyright 2008 Gleim Publications, Inc. Page 58Printed for Mamdouh Farag

Page 59: P.3 Answer Explanation

[193] Gleim #: 2.1.21 -- Source: CIA 593 III-70

Answer (A) is incorrect because regression analysis is used to fit a linear trend line to a dependent variable based on oneor more independent variables.

Answer (B) is correct. The primary business application of differential calculus is to identify the maxima or minima ofcurvilinear functions. In business and economics, these are the points of revenue or profit maximization (maxima) or costminimization (minima). The EOQ results from differentiating the total cost with regard to order quantity.

Answer (C) is incorrect because Markov process models are used to study the evolution of certain systems over repeatedtrials.

Answer (D) is incorrect because queuing theory is a waiting-line technique used to balance desirable service levels againstthe cost of providing more service.

[194] Gleim #: 2.1.22 -- Source: CIA 590 III-45

Answer (A) is correct. The EOQ formula is

If: D is the demand, or number of units used per year. A is the cost of placing one order. K is the cost of carrying one item for 1 year.

Answer (B) is incorrect because 447 is the EOQ when demand is 10,000 units per year.

Answer (C) is incorrect because 483 results when annual demand is 11,664 units.

Answer (D) is incorrect because 500 results when annual demand is 12,500 units.

[195] Gleim #: 2.1.23 -- Source: CIA 1190 III-42

Answer (A) is incorrect because service will improve.

Answer (B) is correct. Safety stocks are amounts held in excess of forecasted demand to avoid the losses associated withstockouts. Holding safety stocks improves the level of service to customers at the expense of increased holding costs.

Answer (C) is incorrect because ordering costs will not increase. The fixed costs of the ordering department will beunaffected. Also, the department’s variable costs should not change because the EOQ will be the same. However, in thefirst year, an additional order may be necessary to increase the safety stock.

Answer (D) is incorrect because delivery will increase under the new expedited delivery policy. Moreover, increasing thesafety stock increases carrying costs.

Gleim's CIA Test Prep: Part III: Business Analysis and Information TechnologyAnswer Explanations

(1312 questions)

Copyright 2008 Gleim Publications, Inc. Page 59Printed for Mamdouh Farag

Page 60: P.3 Answer Explanation

[196] Gleim #: 2.1.24 -- Source: CIA 1193 IV-26

Answer (A) is incorrect because long, not short, lead times prompt a more frequent review.

Answer (B) is incorrect because high, not low, stockout costs prompt a more frequent review.

Answer (C) is incorrect because expensive, not inexpensive, items prompt a more frequent review.

Answer (D) is correct. The ABC system is a method for controlling inventories that divides inventory items into threegroups:

Group A -- high-dollar value items, which account for a small portion(perhaps 10%) of the total inventory usage

Group B -- medium-dollar value items, which may account for about 20%

of the total inventory items Group C -- low-dollar value items, which account for the remaining

70% of sales or usage

The ABC system permits the proper degree of managerial control to be identified and exercised over each group. Group Aitems are reviewed on a regular basis. Group B items may not have to be reviewed as often as group A items, but moreoften than group C items. For group C, extensive use of models and records is not cost effective. It is cheaper to orderlarge quantities infrequently. The ABC method therefore reduces the safety-stock investment because high-value items arefrequently monitored and medium-value items are monitored more often than inexpensive items. Frequent review canprevent stockouts and decrease inventory levels, and the cost of such review is minimized if it is limited to high- ormedium-value items.

[197] Gleim #: 2.1.25 -- Source: CIA 1193 IV-25

Answer (A) is incorrect because the economic order quantity is a decision model that focuses on the trade-off betweencarrying and ordering costs.

Answer (B) is correct. Materials requirements planning (MRP) is usually a computer-based information system designedto plan and control raw materials used in a production setting. It assumes that estimated demand for materials isreasonably accurate and that suppliers can deliver based upon this accurate schedule. It is crucial that delivery delays beavoided because, under MRP, production delays are almost unavoidable if the materials are not on hand. An MRP systemuses a parts list, often called a bill of materials, and lead times for each type of material to obtain materials just as theyare needed for planned production.

Answer (C) is incorrect because linear programming is a decision model concerned with allocating scarce resources tomaximize profit or minimize costs.

Answer (D) is incorrect because just-in-time purchasing involves the purchase of goods such that delivery immediatelyprecedes demand or use.

Gleim's CIA Test Prep: Part III: Business Analysis and Information TechnologyAnswer Explanations

(1312 questions)

Copyright 2008 Gleim Publications, Inc. Page 60Printed for Mamdouh Farag

Page 61: P.3 Answer Explanation

[198] Gleim #: 2.1.26 -- Source: CIA 592 III-62

Answer (A) is incorrect because safety stock is the inventory maintained in order to reduce the number of stockoutsresulting from higher-than-expected demand during lead time.

Answer (B) is incorrect because the economic order quantity is the order quantity that minimizes total inventory costs.

Answer (C) is correct. JIT is a manufacturing philosophy popularized by the Japanese that combines purchasing,production, and inventory control. As with MRP, minimization of inventory is a goal; however, JIT also encompasseschanges in the production process itself. An emphasis on quality and a “pull” of materials related to demand are keydifferences between JIT and MRP. The factory is organized so as to bring materials and tools close to the point of userather than keeping them in storage areas. A key element of the JIT system is reduction or elimination of waste ofmaterials, labor, factory space, and machine usage. Minimizing inventory is the key to reducing waste. When a part isneeded on the production line, it arrives just in time, not before. Daily deliveries from suppliers are the ultimate objective,and some Japanese users have been able to get twice-daily deliveries.

Answer (D) is incorrect because the master budget is the detailed financial plan for the next period.

[199] Gleim #: 2.1.27 -- Source: CIA 595 III-98

Answer (A) is incorrect because the safety stock is not included in the basic EOQ formula.

Answer (B) is correct. The basic EOQ formula is used to minimize the total of inventory carrying and ordering costs. Thebasic EOQ equals the square root of a fraction consisting of a numerator equal to the product of twice the unit periodicdemand and the variable cost per order and a denominator equal to the unit periodic carrying cost.

Answer (C) is incorrect because the selling price of the item is not included in the basic EOQ formula.

Answer (D) is incorrect because the lead time for delivery is not included in the basic EOQ formula.

[200] Gleim #: 2.1.28 -- Source: CIA 595 III-99

Answer (A) is incorrect because $750 results from subtracting instead of adding the cost of holding safety stock.

Answer (B) is incorrect because $1,250 ignores safety stock.

Answer (C) is correct. Given that demand is constant and the EOQ is 2,500 units, the average inventory level withoutregard to safety stock is 1,250 units (2,500 ÷ 2). Adding safety stock results in an average level of 1,750 units (1,250 +500). Given also that annual holding costs are 25% of average inventory and that unit cost is $4, total annual holding costis $1,750 [(1,750 units × $4) × 25%].

Answer (D) is incorrect because $2,250 results from double counting the cost of holding safety stock.

[201] Gleim #: 2.1.29 -- Source: CIA 595 III-100

Answer (A) is incorrect because $1,250 equals the annual holding cost of the average inventory excluding safety stock.

Answer (B) is incorrect because $2,400 assumes an EOQ of 3,000 units.

Answer (C) is correct. Total annual demand is 48,000 units (4,000 per month × 12). Hence, total annual ordering costsequal $2,880 [$150 cost per order × (48,000 units ÷ 2,500 EOQ)].

Answer (D) is incorrect because $3,600 assumes an EOQ of 2,000 units.

Gleim's CIA Test Prep: Part III: Business Analysis and Information TechnologyAnswer Explanations

(1312 questions)

Copyright 2008 Gleim Publications, Inc. Page 61Printed for Mamdouh Farag

Page 62: P.3 Answer Explanation

[202] Gleim #: 2.1.30 -- Source: CIA 1195 III-66

Answer (A) is correct. Safety stock is inventory maintained to reduce the number of stockouts resulting from higher-than-expected demand during lead time. Maintaining a safety stock avoids the costs of stockouts, e.g., lost sales and customerdissatisfaction.

Answer (B) is incorrect because a just-in-time inventory system involves the purchase of materials and production ofcomponents immediately preceding their use.

Answer (C) is incorrect because materials requirements planning is a system for scheduling production and controlling thelevel of inventory for components with dependent demand.

Answer (D) is incorrect because a master production schedule is a statement of the timing and amounts of individualitems to be produced.

[203] Gleim #: 2.1.31 -- Source: CIA 1195 III-97

Answer (A) is correct. If D decreases by 36%, that is, from 100% to 64%, the EOQ will decrease by 20%.

Answer (B) is incorrect because the new EOQ decreases to 80% of its former value.

Answer (C) is incorrect because 6% is the square root of 36%.

Answer (D) is incorrect because 6% is the square root of 36%.

[204] Gleim #: 2.1.32 -- Source: CIA 1196 III-95

Answer (A) is correct. When the order size is 2,000 units, the variable inventory costs are CU2,000 {[(2,000 units ×CU1) ÷ 2] + [(20,000 units ÷ 2,000 units) × CU100]}. When the order size is 4,000 units, the variable inventory costs areCU2,500 {[(4,000 units × CU1) ÷ 2] + [(20,000 units ÷ 4,000 units) × CU100]}. The increase in inventory costs is CU500(CU2,500 – CU2,000).

Answer (B) is incorrect because a CU1,000 increase omits the change in the number of orders.

Answer (C) is incorrect because increasing the order size increases total costs. The EOQ is 2,000 units.

Answer (D) is incorrect because a CU900 increase results from an order size of 5,000 units.

[205] Gleim #: 2.1.33 -- Source: CMA 691 4-4

Answer (A) is correct. A decrease in inventory ordering costs should decrease the economic order quantity. The effect isthat more orders can be made (of smaller quantities) without increasing costs. Accordingly, in the EOQ model, orderingcost is a numerator value.

Answer (B) is incorrect because the reorder point is based on lead time, not the EOQ model.

Answer (C) is incorrect because the EOQ does not increase when inventory order costs decrease.

Answer (D) is incorrect because the holding cost percentage will always be identical to the ordering cost percentage inaccordance with the fundamental calculus underlying the EOQ model.

Gleim's CIA Test Prep: Part III: Business Analysis and Information TechnologyAnswer Explanations

(1312 questions)

Copyright 2008 Gleim Publications, Inc. Page 62Printed for Mamdouh Farag

Page 63: P.3 Answer Explanation

[206] Gleim #: 2.1.34 -- Source: CMA 1294 4-7

Answer (A) is incorrect because interest on invested capital is considered in the basic EOQ model.

Answer (B) is incorrect because inventory obsolescence is considered in the basic EOQ model.

Answer (C) is incorrect because public warehouse rental charges is considered in the basic EOQ model.

Answer (D) is correct. The basic EOQ model minimizes the sum of ordering (or setup) and carrying costs. Included in theformula are annual demand, ordering (or setup) costs, and carrying costs. Carrying costs include warehousing costs,insurance, spoilage, obsolescence, and interest on invested capital. The cost of the inventory itself and any quantitydiscounts lost on inventory purchases are not components of the EOQ model.

[207] Gleim #: 2.1.35 -- Source: CMA 1286 5-10

Answer (A) is incorrect because a CPM system is a project management and scheduling technique that determines thelongest time path from the first to the last event for a project.

Answer (B) is correct. Materials requirements planning (MRP) is usually considered a computer-based informationsystem designed to plan and control raw materials used in a production setting. It assumes that estimated demand formaterials is reasonably accurate and that suppliers can deliver based upon this accurate schedule. It is crucial that deliverydelays be avoided because, under MRP, production delays are almost unavoidable if the materials are not on hand. AnMRP system uses a parts list, often called a bill of materials, and lead times for each type of material to obtain materialsjust as they are needed for planned production.

Answer (C) is incorrect because the EOQ model does not adjust for the dependent demand for the components of aninventory item.

Answer (D) is incorrect because the main purpose of the ABC system is to increase control over items accounting for themost profit.

[208] Gleim #: 2.1.36 -- Source: CMA 1286 5-11

Answer (A) is correct. The ABC method of inventory control requires management to exert greatest control over the Aclassification items, which usually include a relatively small percentage of total items but a high percentage of the dollarvolume. This method is analogous to the 80/20 rule, which says, for instance, that 20% of the customers account for 80%of the profit.

Answer (B) is incorrect because the EOQ model is intended to minimize the sum of holding and ordering costs.

Answer (C) is incorrect because a just-in-time system attempts to reduce holding costs by scheduling deliveries ofmaterials as closely as possible to when they are needed in production.

Answer (D) is incorrect because an MRP system recognizes that the demand for one inventory item creates a dependentdemand for the components of that item.

Gleim's CIA Test Prep: Part III: Business Analysis and Information TechnologyAnswer Explanations

(1312 questions)

Copyright 2008 Gleim Publications, Inc. Page 63Printed for Mamdouh Farag

Page 64: P.3 Answer Explanation

[209] Gleim #: 2.1.37 -- Source: CMA 688 5-22

Answer (A) is incorrect because the average inventory level increases by the amount of the safety stock.

Answer (B) is incorrect because the average inventory level increases by the amount of the safety stock.

Answer (C) is incorrect because the average inventory level increases by the amount of the safety stock.

Answer (D) is correct. If there is no safety stock, the average level of inventory will be one-half of the economic orderquantity. For example, if the EOQ is 500, the basic model predicts that the average inventory level will be 250. Thecompany will have 500 units immediately after a purchase and zero immediately before the receipt of the next purchase(replenishment is assumed to be instantaneous). However, safety stock increases the average inventory level by theamount of the safety stock. The modified EOQ model assumes that safety stock will never be used. Thus, if a safety stockof 100 is carried by the company in the example, the average inventory level will increase to 350. Inventory will be 600units immediately upon receipt of a purchase and 100 units immediately before the receipt of the next purchase.

[210] Gleim #: 2.1.38 -- Source: CIA R98 III-32

Answer (A) is correct. Materials requirements planning (MRP) is a system that translates a production schedule intorequirements for each component needed to meet the schedule. It is usually implemented in the form of a computer-basedinformation system designed to plan and control raw materials used in production. It assumes that forecasted demand isreasonably accurate and that suppliers can deliver based upon this accurate schedule. MRP is a centralized push-throughsystem; output based on forecasted demand is pushed through to the next department or to inventory.

Answer (B) is incorrect because regression analysis is used to fit a linear trend line to a dependent variable based on oneor more independent variables.

Answer (C) is incorrect because capital budgeting is the process of planning expenditures for long-lived assets. It involveschoosing among investment proposals using a ranking procedure.

Answer (D) is incorrect because linear programming is a decision model concerned with allocating scarce resources tomaximize profit or minimize costs.

[211] Gleim #: 2.1.39 -- Source: CIA, adapted

Answer (A) is incorrect because the movement of parts can escape being recorded with any identification method.

Answer (B) is correct. A reason to use bar codes rather than other means of identification is to record the movement ofparts with minimal labor costs.

Answer (C) is incorrect because each vendor has its own part-numbering scheme, which is unlikely to correspond to thebuyer’s scheme.

Answer (D) is incorrect because each vendor has its own identification method, although vendors in the same industryoften cooperate to minimize the number of bar-code systems that they use.

Gleim's CIA Test Prep: Part III: Business Analysis and Information TechnologyAnswer Explanations

(1312 questions)

Copyright 2008 Gleim Publications, Inc. Page 64Printed for Mamdouh Farag

Page 65: P.3 Answer Explanation

[212] Gleim #: 2.1.40 -- Source: CIA, adapted

Answer (A) is incorrect because the supplier may ask for a concession in its selling price, which would raise themanufacturer’s purchasing costs. However, the manufacturing company will be receiving fewer materials at any point intime, increasing the likelihood of stockout and thereby resulting in an increase in stockout costs.

Answer (B) is incorrect because the supplier may ask for a concession in its selling price, which would raise themanufacturer’s purchasing costs. However, the cost of quality would not necessarily be affected by the just-in-timepurchasing system.

Answer (C) is incorrect because with fewer purchase orders being processed by the manufacturer, the ordering costs arelikely to decrease. However, the cost of quality would not necessarily be affected by the just-in-time purchasing system.

Answer (D) is correct. In this situation, the company will be receiving fewer materials at any point in time, increasing thelikelihood of stockout and thereby resulting in an increase in a reduction in the carrying cost. At the same time, theaverage inventory will be less, resulting in a reduction in the carrying cost.

[213] Gleim #: 2.1.41 -- Source: CIA, adapted

Answer (A) is incorrect because forecasting models involve projecting data over time or developing regression modelswhen time series data are not available.

Answer (B) is correct. An economic order quantity (EOQ) sensitivity analysis involves varying the holding costs per unitand/or the order costs to determine how much the changes affect the optimal EOQ.

Answer (C) is incorrect because critical path method involves project scheduling.

Answer (D) is incorrect because decision analysis involves selecting the best option from alternatives.

[214] Gleim #: 2.1.42 -- Source: CIA 590 IV-51

Answer (A) is incorrect because total ordering costs will increase.

Answer (B) is incorrect because total ordering costs will increase.

Answer (C) is correct. Inventory carrying costs can sometimes be transferred to suppliers. If a seller has good enoughcontrol of demand schedules to know exactly when goods are needed, orders can be placed so that goods arrive no earlierthan when actually needed. This practice relies on a supplier who is willing to take the responsibility for storing theneeded inventory and shipping it to arrive on time. Suppliers are more willing to provide this type of service when theyhave many competitors.

Answer (D) is incorrect because total carrying costs are reduced.

[215] Gleim #: 2.1.43 -- Source: CIA 594 IV-37

Answer (A) is correct. The annual opportunity cost of carrying inventory equals the average inventory level times the costper unit of inventory times the cost of capital. The average inventory level is the order quantity divided by 2. Thus, theannual opportunity cost of carrying inventory is 150 [(500 ÷ 2) × 5 × .12].

Answer (B) is incorrect because 180 is obtained by using the total annual quantity rather than the average inventory leveland by not multiplying by the unit price.

Answer (C) is incorrect because 300 is obtained by using the order size rather than the average inventory level.

Answer (D) is incorrect because 900 is based on the total annual quantity rather than the average inventory level.

Gleim's CIA Test Prep: Part III: Business Analysis and Information TechnologyAnswer Explanations

(1312 questions)

Copyright 2008 Gleim Publications, Inc. Page 65Printed for Mamdouh Farag

Page 66: P.3 Answer Explanation

[216] Gleim #: 2.1.44 -- Source: CIA, adapted

Answer (A) is incorrect because warranties are not a part of inventory valuation.

Answer (B) is incorrect because vendor pricing policies have no effect on inventory valuation until goods are purchased.The price at the time of purchase is the only price that matters in inventory valuation, and changes in vendor pricingpolicies would not necessarily affect valuation.

Answer (C) is correct. The amount of inventory loss through shrinkage directly affects inventory valuation. Inventoryshrinkage must be considered in risk models involving inventory valuation.

Answer (D) is incorrect because sales forecasts do not affect inventory valuation.

[217] Gleim #: 2.1.45 -- Source: CIA 596 I-4

Answer (A) is incorrect because stockout costs are directly affected by the quantity on hand.

Answer (B) is incorrect because seasonal variations in demand directly affect the minimum quantity available.

Answer (C) is correct. The basic economic order quantity (EOQ) model is based on the following variables: demand,variable cost per purchase order, and variable unit carrying cost. Thus, minimum stocking levels do not affect the EOQ.

Answer (D) is incorrect because storage space and potential obsolescence directly affect stocking levels.

[218] Gleim #: 2.2.46 -- Source: Publisher

Answer (A) is incorrect because the supply chain consists of the flows that may occur across the functions in anorganization’s value chain or separate organizations.

Answer (B) is correct. The supply chain consists of flows from sources of (a) raw materials, (b) components, (c) finishedgoods, (d) services, or (e) information through intermediaries to ultimate consumers. These flows and the related activitiesmay occur across the functions in an organization’s value chain (R&D, design, production, marketing, distribution, andcustomer service). These flows and the related activities also may occur across separate organizations.

Answer (C) is incorrect because logistics is a branch of military science. It addresses the procurement, maintenance, andtransportation of military materials, facilities, and personnel.

Answer (D) is incorrect because the phrase “integrated chain” is not a technical term.

[219] Gleim #: 2.2.47 -- Source: Publisher

Answer (A) is incorrect because the need to purchase or manufacture goods in cost-efficient batches is a cause of thebullwhip, or whiplash, effect.

Answer (B) is incorrect because purchases in anticipation of future price increases cause the bullwhip, or whiplash, effect.

Answer (C) is incorrect because rationing by suppliers or manufacturers and hoarding by manufacturers or retailers causethe bullwhip, or whiplash, effect.

Answer (D) is correct. Sharing information about sales, inventory, pricing, advertising campaigns, and sales forecasts byall functions and organizations in the supply chain moderates demand uncertainty for all parties. The desired results are(a) minimization of inventories held by suppliers, manufacturers, and retailers; (b) avoidance of stockouts; (c) fewer rushorders; and (d) production as needed by retailers.

Gleim's CIA Test Prep: Part III: Business Analysis and Information TechnologyAnswer Explanations

(1312 questions)

Copyright 2008 Gleim Publications, Inc. Page 66Printed for Mamdouh Farag

Page 67: P.3 Answer Explanation

[220] Gleim #: 2.2.48 -- Source: Publisher

Answer (A) is correct. Sharing information about sales, inventory, pricing, advertising campaigns, and sales forecasts byall functions and organizations in the supply chain moderates demand uncertainty for all parties. The desired results are(a) minimization of inventories held by suppliers, manufacturers, and retailers; (b) avoidance of stockouts; (c) fewer rushorders; and (d) production as needed by retailers.

Answer (B) is incorrect because minimization of inventories held by all parties in the supply chain is a desired result ofsharing information.

Answer (C) is incorrect because avoidance of stockouts is a desired result of sharing information.

Answer (D) is incorrect because incompatibility of the information systems of the parties is a difficulty faced by supply-chain management, not a desired result of sharing information.

[221] Gleim #: 2.2.49 -- Source: Publisher

Answer (A) is incorrect because a consignee is also an intermediary between sellers and buyers.

Answer (B) is incorrect because a broker is an intermediary.

Answer (C) is correct. A distribution channel is a series of interdependent marketing institutions that facilitate thetransfer of a product from producer (seller) to consumer (buyer). Intermediaries include merchant middlemen, agents,brokers, consignees, and facilitating intermediaries.

Answer (D) is incorrect because a consumer is a buyer, not an intermediary.

[222] Gleim #: 2.2.50 -- Source: Publisher

Answer (A) is incorrect because, in a horizontal distribution system, two or more companies at one level of the channelwork together.

Answer (B) is correct. In vertical distribution systems, producers, wholesalers, and retailers act as a unified system.Channel conflict is managed through common ownership, contractual relationships, or administration by one or a fewdominant channel members. Horizontal distribution systems consist of two or more companies at one level of the channelworking together to exploit new opportunities, such as the introduction of ATMs in supermarkets. The joint nature ofhorizontal distribution efforts is the tool for managing channel conflict. In a multichannel system, a single firm sets up twoor more channels to reach one or more customer segments. Because such a system is managed by a single firm, channelconflicts can be evaluated and managed internally.

Answer (C) is incorrect because a unilateral distribution system is not a channel structure.

Answer (D) is incorrect because, in a multichannel system, a single firm sets up two or more channels.

[223] Gleim #: 2.2.51 -- Source: CIA 597 IV-80

Answer (A) is correct. Marketing intermediaries assist companies in promoting, selling, and distributing their goods andservices to ultimate consumers. For example, travel agents access an airline’s computerized reservation system and makereservations for their customers without ever taking title to the ticket.

Answer (B) is incorrect because jobbers buy from manufacturers, then resell the products.

Answer (C) is incorrect because distributors, or wholesalers, usually have selective or exclusive distribution rights.

Answer (D) is incorrect because facilitating agents assist in functions other than buying, selling, or transferring title.

Gleim's CIA Test Prep: Part III: Business Analysis and Information TechnologyAnswer Explanations

(1312 questions)

Copyright 2008 Gleim Publications, Inc. Page 67Printed for Mamdouh Farag

Page 68: P.3 Answer Explanation

[224] Gleim #: 2.2.52 -- Source: Publisher

Answer (A) is incorrect because physical distribution is the efficient movement of finished products to consumers.

Answer (B) is incorrect because physical distribution is the efficient movement of finished products to consumers.

Answer (C) is correct. Physical distribution is the efficient management of supply chains. It controls value-added flowsfrom suppliers to consumers.

Answer (D) is incorrect because physical distribution is the efficient movement of finished products to consumers.

[225] Gleim #: 2.2.53 -- Source: Publisher

Answer (A) is incorrect because shopping goods are usually sold through selective distribution.

Answer (B) is incorrect because specialty goods are usually sold through selective or exclusive distribution.

Answer (C) is correct. Convenience goods are consumer goods and services that are usually low-priced and widelyavailable. Consumers buy them often and with a minimum of comparison and effort. Examples are soap and newspapers.Producers of convenience goods ordinarily use intensive distribution to sell their products through a large number of retailor wholesale units.

Answer (D) is incorrect because unsought goods, e.g., life insurance, are often marketed through personal selling.

[226] Gleim #: 2.2.54 -- Source: CIA 597 IV-78

Answer (A) is incorrect because the introduction of a distributor will affect the number of contacts in the marketingchannel.

Answer (B) is incorrect because the number of contacts declined from 16 to 8.

Answer (C) is incorrect because the number of contacts decreased.

Answer (D) is correct. The number of contacts without a distributor is 16 (four producers × four customers). The numberwith a distributor is 8 (four producers + four customers). Thus, the effort required of producers and consumers is reducedby the distributor, thereby increasing marketing efficiency.

[227] Gleim #: 2.3.55 -- Source: CIA 1194 IV-78

Answer (A) is incorrect because adding a standard markup to the cost of the product is cost-plus pricing.

Answer (B) is incorrect because determining the price at which the product will earn a target profit is target profit pricing.

Answer (C) is correct. Buyer-based pricing involves basing prices on the product’s perceived value rather than on theseller’s cost. Nonprice variables in the marketing mix augment the perceived value. For example, a cup of coffee mayhave a higher price at an expensive restaurant than at a fast-food outlet.

Answer (D) is incorrect because basing prices on competitors’ prices is going-rate pricing.

Gleim's CIA Test Prep: Part III: Business Analysis and Information TechnologyAnswer Explanations

(1312 questions)

Copyright 2008 Gleim Publications, Inc. Page 68Printed for Mamdouh Farag

Page 69: P.3 Answer Explanation

[228] Gleim #: 2.3.56 -- Source: CIA 1194 IV-79

Answer (A) is incorrect because, if no buyers want the product at a high price, this marketing strategy is inappropriate.

Answer (B) is correct. Market-skimming pricing is used when a new product is introduced at the highest price possiblegiven the benefits of the product. For market skimming to work, the product must appear to be worth its price, the costs ofproducing a small volume cannot be so high that they eliminate the advantage of charging more, and competitors cannotenter the market and undercut the price.

Answer (C) is incorrect because, if competitors can easily enter the market, they can undercut the price.

Answer (D) is incorrect because the product quality and image must support a high price.

[229] Gleim #: 2.3.57 -- Source: CIA 596 IV-78

Answer (A) is incorrect because this graph describes the familiar, negatively sloped relation between price charged andthe resulting demand level for normal goods.

Answer (B) is incorrect because the demand curve can be linear or curvilinear.

Answer (C) is correct. Over some intermediate range of prices, the reaction to a price increase for prestige goods is anincrease, not a decrease, in the quantity demanded. Within this range, the demand curve is upward sloping. The reason isthat consumers interpret the higher price to indicate a better or more desirable product. Above some price level, therelation between price and quantity demanded will again become negatively sloped.

Answer (D) is incorrect because this demand curve has the same basic shape as the demand curve for prestige goods, butit bends the wrong way. As prices increase, quantity demanded first falls and then rises in this graph.

[230] Gleim #: 2.3.58 -- Source: CIA 595 IV-74

Answer (A) is incorrect because, in freight absorption pricing, the selling company absorbs all or part of the actual freightcharges. Customers are not charged actual delivery costs.

Answer (B) is correct. In uniform delivered pricing, the company charges the same price, inclusive of shipping costs, toall customers, regardless of their location. This price is the company’s average actual freight cost. Thus, both nearby anddistant customers are charged the same amount. This policy is easy to administer, permits the company to advertise oneprice nationwide, and facilitates marketing to faraway customers.

Answer (C) is incorrect because, in zone pricing, differential freight charges are set for customers on the basis of theirlocation. Customers are not charged actual average freight costs.

Answer (D) is incorrect because, in FOB-origin pricing, each customer pays its actual freight costs.

[231] Gleim #: 2.3.59 -- Source: CIA 1195 IV-75

Answer (A) is incorrect because cash discounts encourage prompt payment.

Answer (B) is incorrect because quantity discounts encourage large-volume purchases.

Answer (C) is incorrect because functional or trade discounts are provided to channel members in return for theperformance of certain functions, such as selling, storing, and record keeping.

Answer (D) is correct. Seasonal discounts are designed to smooth production by the selling firm. For example, a skimanufacturer offers seasonal discounts to retailers in the spring and summer to encourage early ordering.

Gleim's CIA Test Prep: Part III: Business Analysis and Information TechnologyAnswer Explanations

(1312 questions)

Copyright 2008 Gleim Publications, Inc. Page 69Printed for Mamdouh Farag

Page 70: P.3 Answer Explanation

[232] Gleim #: 2.3.60 -- Source: CIA 1195 IV-76

Answer (A) is correct. A by-product is a product of relatively minor importance generated during the production of one ormore other products. Its production entails no additional costs. Any amount received above the storage and delivery costsfor a by-product allows the seller to reduce the main product’s price to make it more competitive.

Answer (B) is incorrect because optional products are offered for sale along with the main product. They are unlikely tohave a zero production cost, so the seller must receive a price above their storage and delivery costs.

Answer (C) is incorrect because captive products must be used along with the main product, such as film for use with acamera. Sellers often make their profits on the captive products rather than on the main product, which is sold at a lowprice. The captive products therefore will be priced well above the storage and delivery costs.

Answer (D) is incorrect because product bundles are combinations of products sold together at a reduced price, such asseason tickets for a theater. Products are bundled to promote the sale of certain items that consumers might not otherwisepurchase. The combined price of the bundle must be low enough to encourage consumers to buy the bundle but mustrecover production costs and provide some profit for the seller, so the price must exceed storage and delivery costs.

[233] Gleim #: 2.3.61 -- Source: CMA 1296 4-6

Answer (A) is incorrect because full-cost pricing promotes price stability. It limits the ability to cut prices.

Answer (B) is incorrect because full-cost pricing provides evidence that the company is not violating antitrust laws againstpredatory pricing.

Answer (C) is correct. A target price is the expected market price of a product, given the company’s knowledge of itscustomers and competitors. Hence, under target pricing, the sales price is known before the product is developed.Subtracting the unit target profit margin determines the long-term unit target cost. If cost-cutting measures do not permitthe product to be made at or below the target cost, it will be abandoned.

Answer (D) is incorrect because full-cost pricing has the advantage of recovering the full long-term costs of the product. Inthe long term, all costs are relevant.

[234] Gleim #: 2.3.62 -- Source: Publisher

Answer (A) is incorrect because collusive pricing involves a conspiracy to set higher prices.

Answer (B) is incorrect because dumping is defined under U.S. law as sale by a non-U.S. company in the U.S. market of aproduct below its market value in the country where it was produced. Such sale is illegal if it threatens material injury to aU.S. industry.

Answer (C) is correct. Predatory pricing is intentionally pricing below cost to eliminate competition and reduce supply.Federal statutes and many state laws prohibit the practice. The U.S. Supreme Court has held that pricing is predatorywhen two conditions are met: (1) the seller’s price is below “an appropriate measure of its costs,” and (2) it has areasonable prospect of recovering the resulting loss through higher prices or greater market share.

Answer (D) is incorrect because price discrimination involves charging different prices to different customers foressentially the same product without a legal justification.

Gleim's CIA Test Prep: Part III: Business Analysis and Information TechnologyAnswer Explanations

(1312 questions)

Copyright 2008 Gleim Publications, Inc. Page 70Printed for Mamdouh Farag

Page 71: P.3 Answer Explanation

[235] Gleim #: 2.3.63 -- Source: Publisher

Answer (A) is incorrect because $32.00 equals the current full cost per unit minus the new unit target price.

Answer (B) is incorrect because $38.40 is the unit target operating income.

Answer (C) is correct. Unit target operating income is $38.40 ($320 unit target price × 12%). Hence, the unit target fullcost is $281.60 ($320 – $38.40). The current full cost per unit is $352.00 [($13,200,000 CGS + $7,920,000 other valuechain operating costs) ÷ 60,000 units sold], so the necessary reduction in the full cost per unit is $70.40 ($352.00 –$281.60).

Answer (D) is incorrect because $80.00 equals the change in the unit price.

[236] Gleim #: 2.3.64 -- Source: Publisher

Answer (A) is incorrect because $15 is the budgeted unit manufacturing cost.

Answer (B) is incorrect because $31 is the budgeted unit life-cycle cost.

Answer (C) is incorrect because $36 is the budgeted unit whole-life cost.

Answer (D) is correct. Whole-life costs include after-purchase costs (operating, support, repair, and disposal) incurred bycustomers as well as life-cycle costs (R&D, design, manufacturing, marketing, distribution, and research). Hence, thebudgeted unit whole-life cost is $36 [($2,000,000 + $3,000,000 + $1,200,000 + $1,000,000) ÷ 200,000 units], and thebudgeted unit selling price is $45 ($36 × 125%).

[237] Gleim #: 2.3.65 -- Source: CIA, adapted

Answer (A) is incorrect because a cost leader is the lowest cost producer in the industry as a whole.

Answer (B) is correct. A cost focus strategy aims at cost leadership in a particular segment, such as a regional market or aspecialty product line. The rationale for a focus strategy is that the narrower market can be better served.

Answer (C) is incorrect because cost differentiation aims at providing a product at different costs in different marketsegments.

Answer (D) is incorrect because cost containment aims at controlling costs related to a particular product/market but notnecessarily producing at the lowest possible cost.

Gleim's CIA Test Prep: Part III: Business Analysis and Information TechnologyAnswer Explanations

(1312 questions)

Copyright 2008 Gleim Publications, Inc. Page 71Printed for Mamdouh Farag

Page 72: P.3 Answer Explanation

[238] Gleim #: 2.3.66 -- Source: Publisher

Answer (A) is incorrect because guerilla warfare consists of numerous small attacks designed to reduce the strength of thetarget, e.g., by ad campaigns, carefully chosen price decreases, and lawsuits. Such warfare ordinarily must be followed bya different (and stronger) type of attack if the challenge is to succeed.

Answer (B) is incorrect because the bypass attack directs the assault against markets other than those where thecompetitive target is strong. It may involve diversification of products or geographic markets. It may also entail developingnext-generation technology so as to move the competition to an arena where the challenger is in a stronger position.

Answer (C) is incorrect because a frontal attack directly pits the firm’s products, prices, promotions, and methods ofdistribution against the target’s.

Answer (D) is correct. A flank attack may be directed at a geographic or segmental weakness of the target (anunderserved market) or an unmet need (such as the desire for more healthful fast food). A flank attack succeeds whenmarket segments shift. The result is a gap in need fulfillment that the attacker can convert into a strong position in aprofitable segment.

[239] Gleim #: 2.3.67 -- Source: Publisher

Answer (A) is incorrect because, as the firm most likely to gain, the leader should attempt to increase total demand, forexample, by attracting new users, encouraging new uses, and promoting increased use.

Answer (B) is incorrect because a position defense strengthens the firm’s brand power.

Answer (C) is incorrect because a preemptive defense anticipates an attack, such as by targeting particular competitorsbefore they can launch assaults, flooding the market with products for every segment and niche, or sending market signalsindicating ways in which the leader intends to anticipate attacks.

Answer (D) is correct. Constant innovation to improve products and services, control costs, and increase distributioneffectiveness is the basis for a good offensive strategy. The leader must continuously improve the value offered tocustomers.

[240] Gleim #: 2.3.68 -- Source: CIA, adapted

Answer (A) is incorrect because poor product quality is evident during the introduction stage of the product life cycle.

Answer (B) is correct. In the growth stage, sales and profits increase rapidly, cost per customer decreases, customers areearly adopters, new competitors enter an expanding market, new product models and features are introduced, andpromotion spending declines or remains stable. The firm enters new market segments and distribution channels andattempts to build brand loyalty and achieve the maximum share of the market. Thus, prices are set to penetrate the market,distribution channels are extended, and the mass market is targeted through advertising. The strategy is to advance bythese means and by achieving economies of productive scale.

Answer (C) is incorrect because competitors are most numerous and products become less differentiated during thematurity stage of the product life cycle. In this stage, imitators have entered the market and competitors have learnedwhich technologies and features are successful.

Answer (D) is incorrect because the quality of the products becomes more variable and products are less differentiatedduring the decline stage of the product life cycle.

Gleim's CIA Test Prep: Part III: Business Analysis and Information TechnologyAnswer Explanations

(1312 questions)

Copyright 2008 Gleim Publications, Inc. Page 72Printed for Mamdouh Farag

Page 73: P.3 Answer Explanation

[241] Gleim #: 2.3.69 -- Source: CIA, adapted

Answer (A) is incorrect because a decline in the firm’s purchases–resulting in a decline in the firm’s inventory levels–isnot the first symptom. It will occur only when production declines as a result of a drop in sales.

Answer (B) is correct. The sales of most product types and brands eventually decrease permanently. This decline may beslow or rapid. This first symptom of the decline stage of a product’s life cycle triggers such other effects as price cutting,narrowing of the product line, and reduction in promotion budgets.

Answer (C) is incorrect because a decline in production costs may be due to many factors, e.g., new plant technology orthe increased availability of raw materials. Moreover, production costs may decrease in any stage of a product’s life cycleand not specifically in the decline stage.

Answer (D) is incorrect because a change in prices is a marketing decision. It is an action that may be taken in thematurity stage to compete in the market. Moreover, a decrease in the product’s prices is a response to a permanent declinein sales.

[242] Gleim #: 2.3.70 -- Source: CIA, adapted

Answer (A) is correct. The introduction stage is characterized by slow sales growth and lack of profits because of the highexpenses of promotion and selective distribution to generate awareness of the product and encourage customers to try it.Thus, the per-customer cost is high. Competitors are few, basic versions of the product are produced, and higher-incomecustomers (innovators) are usually targeted. Cost-plus prices are charged. They may initially be high to permit costrecovery when unit sales are low. The strategy is to infiltrate the market, plan for financing to cope with losses, buildsupplier relations, increase production and marketing efforts, and plan for competition.

Answer (B) is incorrect because, during the introduction stage, little competition exists. Competitors tend not to enter themarket until they have greater assurance of profits.

Answer (C) is incorrect because no mass market is available during the introduction stage.

Answer (D) is incorrect because, by definition, not many alternatives are available during the introduction stage of aninnovative product.

[243] Gleim #: 2.3.71 -- Source: CIA, adapted

Answer (A) is incorrect because, during the introduction stage, pre-unit costs of production are high and little competitionexists. Hence, prices are at their highest.

Answer (B) is incorrect because, during the growth stage, prices will be lower than during the introduction stage, but notas low as during the maturity stage. In the growth stage, costs are dropping and competitors are being added, but costs arenot at their minimum and competitors are not at their maximum.

Answer (C) is correct. During the maturity stage, competition is at its greatest and costs are at their lowest. Moreover,firms are engaged in competitive price-cutting measures, resulting in some of the lowest prices seen during a product’slife cycle.

Answer (D) is incorrect because, during the decline stage, price-cutting predominates as firms struggle to maintain salesvolume in the face of a permanent decrease in demand. However, late in the decline stage, there are few competitors, soprices can be raised. In addition, pre-unit costs are on the rise because volume is declining, resulting in higher prices.

Gleim's CIA Test Prep: Part III: Business Analysis and Information TechnologyAnswer Explanations

(1312 questions)

Copyright 2008 Gleim Publications, Inc. Page 73Printed for Mamdouh Farag

Page 74: P.3 Answer Explanation

[244] Gleim #: 2.3.72 -- Source: CIA, adapted

Answer (A) is incorrect because production volume is low during the introduction stage. Although costs are also highduring this period, low volume reduces the opportunities for cost reductions.

Answer (B) is correct. During the growth stage, the opportunity for cost reductions is at its maximum because productionvolume is increasing at a high rate. Thus, fixed costs are being spread over more units of production, and the benefits ofthe learning curve are being realized.

Answer (C) is incorrect because production volume changes little during the maturity stage. The result is less opportunityfor cost reductions.

Answer (D) is incorrect because costs per unit typically rise during the decline stage as production volume declines.

[245] Gleim #: 2.3.73 -- Source: CIA, adapted

Answer (A) is incorrect because few competitors exist during the introduction stage, and quality is sometimes poor.

Answer (B) is incorrect because buyers are less concerned with price and quality during the growth stage than in thematurity stage.

Answer (C) is correct. The maturity stage is the ideal time for advertising lower prices and superior quality because thisis the period during a product’s life when competition is greatest. Due to the availability of many alternatives orsubstitutes, a firm has reasons to set itself apart. Because price and quality are both concerns of customers during thematurity stage, it is ideal for the firm to differentiate its product by advertising low prices and higher quality.

Answer (D) is incorrect because few competitors exist during the decline stage. Moreover, prices may rise late in thedecline stage for the remaining firms as per-unit costs increase.

[246] Gleim #: 2.3.74 -- Source: Publisher

Answer (A) is incorrect because multinational firms also police their distributors and change the product to prevent graymarket activity.

Answer (B) is incorrect because multinational firms also change the product to prevent gray market activity.

Answer (C) is incorrect because multinational firms also raise prices charged to lower-cost distributors to prevent graymarket activity.

Answer (D) is correct. In a gray market, products imported from one country to another are sold by persons trying to makea profit from the difference in retail prices between the two countries. These activities clearly lower the profits in somemarkets of the multinational firm that was the initial seller. One response is to monitor the practices of distributors andretaliate if necessary. A second response is to charge higher prices to the low-cost distributors to reduce their incentives toparticipate in a gray market. A third response is to differentiate products sold in different countries, e.g., by adapting theproduct or offering distinct service features.

Gleim's CIA Test Prep: Part III: Business Analysis and Information TechnologyAnswer Explanations

(1312 questions)

Copyright 2008 Gleim Publications, Inc. Page 74Printed for Mamdouh Farag

Page 75: P.3 Answer Explanation

[247] Gleim #: 2.3.75 -- Source: Publisher

Answer (A) is incorrect because dumping is sale below cost or at less than the price charged in the home market.

Answer (B) is incorrect because price discrimination involves illegally selling the same products at different prices todifferent customers.

Answer (C) is correct. In a gray market, products imported from one country to another are sold by persons trying to makea profit from the difference in retail prices between the two countries. In essence, the seller firm in this case wasexploiting a price difference between markets.

Answer (D) is incorrect because black market operations are illegal.

[248] Gleim #: 2.3.76 -- Source: Publisher

Answer (A) is incorrect because an arm’s-length price is what a competitor would charge in that market.

Answer (B) is incorrect because a cost-plus price does not necessarily trigger higher import duties.

Answer (C) is incorrect because, if the transfer price is too low, import duties would be lower and taxes would be higher.

Answer (D) is correct. A transfer price is the price charged by one subunit of a firm to another. When the subsidiary-buyer is in a foreign country, the higher the transfer, the higher the potential tariffs. However, the tax levied on asubsequent sale by the subsidiary will be lower because of its higher acquisition cost.

[249] Gleim #: 2.3.77 -- Source: Publisher

Answer (A) is correct. A firm may set a cost-based price in each market with a standard markup. In a region or countrywhere costs are high, this strategy may result in prices that are too high to be competitive within the local market.

Answer (B) is incorrect because a uniform pricing policy may overprice the product in some markets and underprice it inothers.

Answer (C) is incorrect because charging what consumers can afford in each country may create a gray market.

Answer (D) is incorrect because dumping often entails charging a below-cost price.

[250] Gleim #: 2.3.78 -- Source: Publisher

Answer (A) is incorrect because price elasticity of demand is the relationship of total revenue to a change in price. Ifdemand is price elastic, a price increase results in lower revenue.

Answer (B) is incorrect because dumping is sale at a price below cost or below the price in the home country.

Answer (C) is incorrect because, in a gray market, products imported from one country to another are sold by personstrying to make a profit from the difference in retail prices between the two countries.

Answer (D) is correct. Price escalation is caused by an accumulation of additional costs, e.g., currency fluctuations;transportation expenses; profits earned by importers, wholesalers, and retailers; and import duties.

Gleim's CIA Test Prep: Part III: Business Analysis and Information TechnologyAnswer Explanations

(1312 questions)

Copyright 2008 Gleim Publications, Inc. Page 75Printed for Mamdouh Farag

Page 76: P.3 Answer Explanation

[251] Gleim #: 2.3.79 -- Source: Publisher

Answer (A) is incorrect because the price is based on buyer perceptions in market-based pricing, rather than simply beingset at a high price.

Answer (B) is correct. Price skimming is the practice of setting an introductory price relatively high to attract buyers whoare not concerned about price and to recover research and development costs rather quickly. The opposite approach is touse penetration pricing, which involves setting a relatively low price to gain deep market penetration quickly.

Answer (C) is incorrect because penetration pricing sets a relatively low price to gain deep market penetration quickly.

Answer (D) is incorrect because pricing by intermediaries does not describe the setting of a high introductory price.

[252] Gleim #: 2.3.80 -- Source: Publisher

Answer (A) is incorrect because a physical distribution system includes transportation.

Answer (B) is correct. Physical distribution (market logistics) involves planning, implementing, and controlling themovement of materials and final goods to meet customer needs while earning a profit. Physical distribution systemscoordinate suppliers, purchasing agents, marketers, channels, and customers. They include warehousing, transportation,and retail outlets.

Answer (C) is incorrect because a physical distribution system includes the location of retail outlets.

Answer (D) is incorrect because a physical distribution system includes warehousing.

[253] Gleim #: 2.3.81 -- Source: CIA 1194 IV-79

Answer (A) is incorrect because it refers to penetration pricing.

Answer (B) is correct. Price skimming is used when a new product is introduced at the highest price possible given thebenefits of the product. For market skimming to work, the product must appear to be worth its price, the costs ofproducing a small volume cannot be so high that they eliminate the advantage of charging more, and competitors cannotenter the market and undercut the price.

Answer (C) is incorrect because it refers to zone pricing.

Answer (D) is incorrect because it relates to pricing by intermediaries.

[254] Gleim #: 2.3.82 -- Source: CIA 1195 IV-75

Answer (A) is incorrect because cash discounts encourage prompt payment.

Answer (B) is incorrect because quantity discounts encourage large volume purchases.

Answer (C) is incorrect because functional or trade discounts are provided to channel members in return for theperformance of certain functions, such as selling, storing, and record keeping.

Answer (D) is correct. Seasonal discounts are designed to smooth production by the selling firm. For example, a skimanufacturer offers seasonal discounts to retailers in the spring and summer to encourage early ordering.

Gleim's CIA Test Prep: Part III: Business Analysis and Information TechnologyAnswer Explanations

(1312 questions)

Copyright 2008 Gleim Publications, Inc. Page 76Printed for Mamdouh Farag

Page 77: P.3 Answer Explanation

[255] Gleim #: 2.3.83 -- Source: Publisher

Answer (A) is incorrect because it refers to FOB-origin pricing.

Answer (B) is correct. A seller that uses uniform delivered pricing charges the same price, inclusive of shipping, to allcustomers regardless of their location.

Answer (C) is incorrect because it refers to freight-absorption pricing.

Answer (D) is incorrect because it refers to zone pricing.

[256] Gleim #: 2.3.84 -- Source: Publisher

Answer (A) is correct. Basing-point pricing charges each customer the freight costs incurred from a specified city to thedestination, regardless of the actual point of origin of the shipment.

Answer (B) is incorrect because zone pricing sets differential freight charges for customers on the basis of their location.

Answer (C) is incorrect because FOB-origin pricing charges each customer its actual freight costs.

Answer (D) is incorrect because cost-plus pricing is not a geographical pricing method.

[257] Gleim #: 2.3.85 -- Source: Publisher

Answer (A) is correct. Product-bundle pricing entails selling combinations of products at a price lower than the combinedprices of the individual items. This strategy promotes sales of items consumers might not otherwise buy.

Answer (B) is incorrect because by-product pricing refers to the practice of accepting prices at any amount in excess ofstoring and delivering the by-products.

Answer (C) is incorrect because value pricing refers to the practice of redesigning products to improve quality withoutraising prices, or offering the same quality at lower prices.

Answer (D) is incorrect because collusive pricing occurs when companies conspire to restrict output and set artificiallyhigh prices.

[258] Gleim #: 2.3.86 -- Source: Publisher

Answer (A) is incorrect because it is pricing products below cost. It may be done both domestically and in other countries.

Answer (B) is incorrect because price discrimination is charging different prices to different customers.

Answer (C) is correct. Dumping, which refers to selling below cost in other countries, is an inappropriate pricing tacticthat may trigger retaliatory tariffs and other sanctions.

Answer (D) is incorrect because collusive pricing occurs when companies conspire to restrict output and set artificiallyhigh prices.

Gleim's CIA Test Prep: Part III: Business Analysis and Information TechnologyAnswer Explanations

(1312 questions)

Copyright 2008 Gleim Publications, Inc. Page 77Printed for Mamdouh Farag

Page 78: P.3 Answer Explanation

[259] Gleim #: 2.3.87 -- Source: Publisher

Answer (A) is incorrect because it is a pricing objective.

Answer (B) is incorrect because it is a pricing objective.

Answer (C) is incorrect because it is a pricing objective.

Answer (D) is correct. Pricing objectives include

profit maximization, which assumes that all firms select the price that results in the highest profit1.target margin maximization, which is stated as a percentage ratio of profits to sales2.volume-oriented objectives, which refers to setting prices to meet target sales volumes or market shares3.image oriented objectives, which refers to setting prices to enhance the consumer’s perception of the firm’smerchandise mix

4.

stabilization objectives, which refers to setting prices to maintain a stable relationship between the firm’s prices andthe industry leader’s prices

5.

Price setting is not a factor in maximizing production.

[260] Gleim #: 2.3.88 -- Source: Publisher

Answer (A) is incorrect because it equals the current full cost per unit minus the new unit target price.

Answer (B) is incorrect because it is the unit target operating income.

Answer (C) is correct. Unit target operating income is $125 (10% × $1,250 target price). Thus, the unit target full cost is$1,125 ($1,250 – $125). The current full cost per unit is $1,350 [($8,000,000 cost of goods sold + $5,500,000 operatingcosts excluding production) ÷ 10,000 units sold], so the necessary reduction in full cost per unit is $225 ($1,350 –$1,125).

Answer (D) is incorrect because it is the change in unit price.

[261] Gleim #: 2.3.89 -- Source: CIA 1194 IV-78

Answer (A) is incorrect because adding a standard markup to the cost of the product is cost-plus pricing.

Answer (B) is incorrect because determining the price at which the product will earn a target profit is target-profit pricing.

Answer (C) is correct. Market-based pricing involves basing prices on the product’s perceived value rather than on theseller’s cost. Nonprice variables in the marketing mix augment the perceived value. For example, a cup of coffee mayhave a higher price at an expensive restaurant than at a fast food outlet.

Answer (D) is incorrect because basing prices on competitors’ prices is going-rate pricing.

Gleim's CIA Test Prep: Part III: Business Analysis and Information TechnologyAnswer Explanations

(1312 questions)

Copyright 2008 Gleim Publications, Inc. Page 78Printed for Mamdouh Farag

Page 79: P.3 Answer Explanation

[262] Gleim #: 2.3.90 -- Source: Publisher

Answer (A) is incorrect because it refers to Kaizen.

Answer (B) is incorrect because it refers to cost-plus pricing.

Answer (C) is incorrect because value added engineering is not directly related to marketing mix strategies.

Answer (D) is correct. Value engineering is a means of reaching targeted cost levels. It is a systematic approach toassessing all aspects of the value chain cost buildup for a product: R&D, design of products, design of processes,production, marketing, distribution, and customer service. The purpose is to minimize costs without sacrificing customersatisfaction.

[263] Gleim #: 2.3.91 -- Source: Publisher

Answer (A) is correct. By-product pricing usually sets prices at any amount in excess of storing and delivering by-products. Such prices allow the seller to reduce the costs and therefore the prices of the main products.

Answer (B) is incorrect because captive-product pricing involves products that must be used with a main product.

Answer (C) is incorrect because product-bundle pricing entails selling combinations of products at a price lower than thecombined prices of individual items.

Answer (D) is incorrect because value pricing entails redesigning products to improve quality without raising prices oroffering the same quality at lower prices.

[264] Gleim #: 2.3.92 -- Source: Publisher

Answer (A) is correct. Predatory pricing is the practice of pricing products below cost to destroy competitors. A price ispredatory if it is below an appropriate measure of costs, and the seller has a reasonable prospect of recovering its losses inthe future through higher prices or greater market share.

Answer (B) is incorrect because pricing products at different prices for different customers refers to price discrimination.

Answer (C) is incorrect because conspiring to restrict output and set artificially high prices is collusive pricing.

Answer (D) is incorrect because selling below cost in other countries is dumping.

[265] Gleim #: 2.3.93 -- Source: Publisher

Answer (A) is incorrect because it refers to the illegal practice of conspiring to restrict output and set artificially highprices.

Answer (B) is correct. Under peak-load pricing, prices vary directly with capacity usage. Thus, when idle capacity isavailable, that is, when demand falls, the price of a product or service tends to be higher given a peak-load pricingapproach.

Answer (C) is incorrect because it involves basing prices on the product’s perceived value.

Answer (D) is incorrect because it involves making a cost determination and setting a price that will recover costs andprovide a return on investment.

Gleim's CIA Test Prep: Part III: Business Analysis and Information TechnologyAnswer Explanations

(1312 questions)

Copyright 2008 Gleim Publications, Inc. Page 79Printed for Mamdouh Farag

Page 80: P.3 Answer Explanation

[266] Gleim #: 2.3.94 -- Source: Publisher

Answer (A) is incorrect because it equals the current full cost per unit minus the new unit target price.

Answer (B) is incorrect because it is the unit target operating income.

Answer (C) is incorrect because it is the change in unit price.

Answer (D) is correct. Unit target operating income is $1.20 (15% × $8 target price). Thus, the unit target full cost is$6.80 ($8 – $1.20). The current full cost per unit is $8.50 [$515,000 cost of goods sold + $335,000 operating costsexcluding production) ÷ 100,00 units sold], so the necessary reduction in full cost per unit is $1.70 ($8.50 – $6.80).

[267] Gleim #: 2.3.95 -- Source: Publisher

Answer (A) is correct. Collusive pricing is illegal. It involves two or more competitors conspiring to restrict output andcharge artificially high prices.

Answer (B) is incorrect because predatory pricing occurs when a company sells products below cost to destroycompetitors.

Answer (C) is incorrect because price discrimination occurs when different prices are charged to different customers.

Answer (D) is incorrect because market-based pricing bases prices on a product’s perceived value rather than on cost.

[268] Gleim #: 2.3.96 -- Source: CIA 595 IV-74

Answer (A) is incorrect because, in freight absorption pricing, the selling company absorbs all or part of the actual freightcharges. Customers are not charged actual delivery costs.

Answer (B) is correct. In uniform delivered pricing, the company charges the same price, inclusive of shipping costs, toall customers regardless of their location. This price is the company’s average actual freight cost. Thus, both nearby anddistant customers are charged the same amount. This policy is easy to administer, permits the company to advertise oneprice nationwide, and facilitates marketing to faraway customers.

Answer (C) is incorrect because, in zone pricing, differential freight charges are set for customers on the basis of theirlocation. Customers are not charged actual average freight costs.

Answer (D) is incorrect because, in FOB-origin pricing, each customer pays its actual freight costs.

[269] Gleim #: 2.3.97 -- Source: Publisher

Answer (A) is incorrect because it is not discriminatory to charge everyone the same from the same point.

Answer (B) is incorrect because zone pricing differs by region of the country to which a shipment is made.

Answer (C) is correct. Basing-point pricing charges each customer the freight costs incurred from a specified city to thedestination regardless of the actual point of origin of the shipment.

Answer (D) is incorrect because FOB-origin pricing charges each customer its actual freight costs.

Gleim's CIA Test Prep: Part III: Business Analysis and Information TechnologyAnswer Explanations

(1312 questions)

Copyright 2008 Gleim Publications, Inc. Page 80Printed for Mamdouh Farag

Page 81: P.3 Answer Explanation

[270] Gleim #: 2.3.98 -- Source: Publisher

Answer (A) is incorrect because predatory prices are prices set below cost to destroy competitors.

Answer (B) is incorrect because this means charging the going rate in the market.

Answer (C) is correct. Product-bundle pricing is a strategy to promote items consumers might not otherwise buy byselling combinations of products at a price lower than the prices of the individual items combined.

Answer (D) is incorrect because captive-product pricing involves charging a low price for a main product (such as arazor), while razor blades are sold at a high markup. Once a person has a razor, he has to buy blades.

[271] Gleim #: 2.3.99 -- Source: Publisher

Answer (A) is incorrect because the Japanese concept of continuous improvement is known as “Kaizen.”

Answer (B) is correct. Value engineering is a means of reaching targeted cost levels. Value engineering requiresdistinguishing between cost incurrence and locked-in costs, with emphasis on controlling costs at the design stage. It is notJapanese.

Answer (C) is incorrect because the Japanese concept of continuous improvement is known as “Kaizen.”

Answer (D) is incorrect because the Japanese concept of continuous improvement is known as “Kaizen.”

[272] Gleim #: 2.3.100 -- Source: Publisher

Answer (A) is incorrect because $32.00 equals the current full cost per unit minus the new unit target price.

Answer (B) is incorrect because because $38.40 is the unit target operating income.

Answer (C) is correct. Unit target operating income is $38.40 (12% × $320 unit target price). Hence, the unit target fullcost is $281.60 ($320 – $38.40). The current full cost per unit is $352.00 [($13,200,000 CGS + $7,920,000 other valuechain operating costs) &divide; 60,000 units sold], so the necessary reduction in the full cost per unit is $70.40 ($352.00 –$281.60).

Answer (D) is incorrect because $80.00 equals the change in the unit price.

[273] Gleim #: 2.3.101 -- Source: Publisher

Answer (A) is incorrect because it is a discount offered by the hardware company.

Answer (B) is incorrect because it is a discount offered by the hardware company.

Answer (C) is incorrect because it is a discount offered by the hardware company.

Answer (D) is correct. Cash discounts encourage prompt payment, improve cash flows, and avoid bad debts. Quantitydiscounts encourage large volume purchases. Trade (functional) discounts are offered to other members of the marketingchannel for performing certain services, such as selling. Seasonal discounts are offered for sales out of season. They helpsmooth production. Allowances such as trade-ins reduce list prices.

Gleim's CIA Test Prep: Part III: Business Analysis and Information TechnologyAnswer Explanations

(1312 questions)

Copyright 2008 Gleim Publications, Inc. Page 81Printed for Mamdouh Farag

Page 82: P.3 Answer Explanation

[274] Gleim #: 2.3.102 -- Source: CIA, adapted

Answer (A) is incorrect because there are no additional costs incurred other than storage and delivery for the developmentof this product. Therefore, the manufacturer will make a profit on any price over the cost of storage and delivery.

Answer (B) is incorrect because optional products are those offered for sale along with the main product. They areunlikely to have zero production cost so the seller must receive a price above the storage and delivery costs for suchproducts.

Answer (C) is correct. Captive products are those that must be used along with the main product. Sellers often make theirmoney on the captive products, rather than on the main product that is sold at a low price. The captive products willtherefore be priced well above the storage and delivery costs.

Answer (D) is incorrect because product bundles are combinations of products sold together at a reduced price, such asseason tickets for a theater. Products are bundled in order to promote the sale of certain items that consumers might nototherwise purchase. The combined price of the bundle must be low enough to encourage consumers to buy the bundle, butmust recover production costs and provide some profit for the seller. The price must exceed storage and delivery costs.

[275] Gleim #: 2.3.103 -- Source: CIA, adapted

Answer (A) is incorrect because market leader strategies are employed by the major competitors that dominate a market.

Answer (B) is incorrect because market challenger strategies are followed by runner-up companies that aggressively attackcompetitors to get more market share.

Answer (C) is incorrect because market follower strategies are used by runner-up companies that follow competitor’sproduct offers, pricing, and market programs.

Answer (D) is correct. Specializing in serving customers overlooked or ignored by major competitors is a market nichestrategy. This strategy specializes along market, customer, product, or marketing mix lines.

[276] Gleim #: 2.3.104 -- Source: Publisher

Answer (A) is incorrect because penetration pricing is a new product pricing method in which an introductory price is setrelatively low to attract as many customers as possible.

Answer (B) is incorrect because going-rate pricing is a method of pricing based largely on competitors’ prices.

Answer (C) is correct. Cost-based pricing is a pricing method in which a standard markup or target profit is added to thecost of the product in determining the product’s sales price.

Answer (D) is incorrect because price skimming sets the introductory price relatively high to attract buyers who are notconcerned with price.

Gleim's CIA Test Prep: Part III: Business Analysis and Information TechnologyAnswer Explanations

(1312 questions)

Copyright 2008 Gleim Publications, Inc. Page 82Printed for Mamdouh Farag

Page 83: P.3 Answer Explanation

[277] Gleim #: 2.3.105 -- Source: Publisher

Answer (A) is incorrect because it is a true statement about cost-based pricing.

Answer (B) is incorrect because it is a true statement about cost-based pricing.

Answer (C) is correct. Cost-based pricing begins with a cost determination followed by setting a price that will recoverthe value chain costs and provide the desired return on investment. A cost-plus price equals the cost plus a markup. Costmay be defined in many ways. Most companies use either absorption manufacturing cost or total cost when calculating theprice. Variable costs may be used as the basis for cost, but then fixed costs must be covered by the markup.

Answer (D) is incorrect because it is a true statement about cost-based pricing.

[278] Gleim #: 2.4.106 -- Source: Publisher

Answer (A) is correct. Evaluations reinforce accomplishments, help in assessing employee strengths and weaknesses,provide motivation, assist in employee development, permit the organization to assess its human resource needs, and serveas a basis for wage increases. Nondiscriminatory benefits are given to everyone in the organization in equal amounts,regardless of title, pay, or achievement of objectives.

Answer (B) is incorrect because it is a purpose of a performance evaluation.

Answer (C) is incorrect because it is a purpose of a performance evaluation.

Answer (D) is incorrect because it is a purpose of a performance evaluation.

[279] Gleim #: 2.4.107 -- Source: Publisher

Answer (A) is incorrect because a judgmental evaluation is based on nonverifiable, subjective criteria.

Answer (B) is correct.The halo effect occurs when the appraiser judges one or a few employee traits and carries over this judgment to theevaluation of the employee’s other traits.

1.

The converse of the halo effect is the horn effect. It occurs when the manager allows one negative trait to influencethe evaluation of other traits.

2.

Answer (C) is incorrect because projection is the process of attributing one’s own traits to another person.

Answer (D) is incorrect because an objective evaluation uses verifiable, often quantitative criteria.

[280] Gleim #: 2.4.108 -- Source: CIA 596 II-33

Answer (A) is incorrect because personal beliefs alone are not an appropriate basis for managerial action.

Answer (B) is incorrect because personal beliefs alone are not an appropriate basis for managerial action.

Answer (C) is correct. The only legitimate grounds on which the supervisor may take action is the employee’s behavior.Personal beliefs, such as those on religious and political matters, cannot be the basis of personnel actions. Discriminationon the basis of personal beliefs could expose the organization to legal action.

Answer (D) is incorrect because personal beliefs alone are not an appropriate basis for managerial action.

Gleim's CIA Test Prep: Part III: Business Analysis and Information TechnologyAnswer Explanations

(1312 questions)

Copyright 2008 Gleim Publications, Inc. Page 83Printed for Mamdouh Farag

Page 84: P.3 Answer Explanation

[281] Gleim #: 2.4.109 -- Source: Publisher

Answer (A) is correct. The personnel director should tailor the discussion to the listener by describing how theemployee’s department evaluates performance and what is expected of him or her. The director should also obtainfeedback from the employee to determine if everything is clearly understood.

Answer (B) is incorrect because discussing information that is irrelevant to the new employee’s appraisal informationwould confuse the new employee.

Answer (C) is incorrect because discussing information that is irrelevant to the new employee’s appraisal informationwould confuse the new employee.

Answer (D) is incorrect because discussing information that is irrelevant to the new employee’s appraisal informationwould confuse the new employee.

[282] Gleim #: 2.4.110 -- Source: Publisher

Answer (A) is incorrect because an advantage of separating appraisals from compensation increases is that more emphasisis placed on long-term objectives.

Answer (B) is incorrect because this separation does not deprive money of its motivational power, but it does emphasizeother rewards, such as feelings of achievement and the recognition of superiors.

Answer (C) is correct. The employee may not be motivated immediately by a good appraisal because of the delay inreceipt of any monetary reward. The evaluation also may not be taken as seriously by the employee if compensation is notcorrelated with performance.

Answer (D) is incorrect because an advantage of separating performance evaluations from compensation-increasedecisions is that the employee’s good performance can be separated from the overall company’s bad financialperformance.

[283] Gleim #: 2.4.111 -- Source: Publisher

Answer (A) is incorrect because reduced opportunity for advancement may motivate employees to find jobs elsewhere.

Answer (B) is correct. Fringe benefits (e.g., pensions and profit-sharing plans) may be contingent on duration ofemployment. Accordingly, they motivate employees to remain with the company.

Answer (C) is incorrect because job simplification may result in boredom, lessened job satisfaction, and a higher turnoverrate.

Answer (D) is incorrect because abolition of a mandatory retirement age would be more appropriate.

Gleim's CIA Test Prep: Part III: Business Analysis and Information TechnologyAnswer Explanations

(1312 questions)

Copyright 2008 Gleim Publications, Inc. Page 84Printed for Mamdouh Farag

Page 85: P.3 Answer Explanation

[284] Gleim #: 2.4.112 -- Source: Publisher

Answer (A) is correct. Human resource or human asset accounting attempts to measure the value, and the changes invalue, of the organization’s investment in human assets. Although this “asset” is enormously valuable (sometimesestimated at two or three times the annual payroll), it is not shown in balance sheets or accounted for in earningsstatements. One experimental measurement approach is a “present value” of human resources. Another is a “cost”approach, with financial investments (training, customer goodwill, etc.) offset by reductions (e.g., retirement).

Answer (B) is incorrect because this traditional accounting method does not value the organization’s human assets.

Answer (C) is incorrect because this traditional accounting method does not value the organization’s human assets.

Answer (D) is incorrect because human resource planning merely audits the skills of the current employees and forecastsneeds.

[285] Gleim #: 2.4.113 -- Source: Publisher

Answer (A) is correct. The 360-degree performance appraisal is a model for employee assessment in the age ofteamwork. It is based on giving workers feedback from peers, customers, supervisors, and those who work for theemployee. Feedback is typically provided anonymously and is usually subjective.

Answer (B) is incorrect because this is a nonsense term.

Answer (C) is incorrect because the critical incident technique is based on having a list of critical aspects of a job againstwhich to compare the employee’s performance; it would usually not be effective in a team situation.

Answer (D) is incorrect because BARS is based on specific job-related behaviors.

[286] Gleim #: 2.4.114 -- Source: Publisher

Answer (A) is correct. A leniency error is a manager’s failure to give a negative evaluation because of fear of damaging agood working relationship with an employee.

Answer (B) is incorrect because the recency effect means that the employee’s most recent behavior overshadows overallperformance.

Answer (C) is incorrect because the halo effect means the manager’s judgment on one positive trait affects the rating onother traits.

Answer (D) is incorrect because a contrast error is the tendency to rate people relative to other people, withoutconsideration of performance standards.

[287] Gleim #: 2.4.115 -- Source: CIA 1192 III-10

Answer (A) is incorrect because internal promotions usually lead to increased motivation among employees.

Answer (B) is incorrect because internal promotions are less expensive. The firm can avoid the expenses associated withan executive search and certain training costs.

Answer (C) is incorrect because it is more difficult to identify proven performers from among outside candidates thaninternal candidates.

Answer (D) is correct. Hiring an internal candidate can lead to social inbreeding. Many firms look to external candidatesfor certain jobs because they bring a fresh perspective to the organization’s problems and may have more up-to-datetraining or education.

Gleim's CIA Test Prep: Part III: Business Analysis and Information TechnologyAnswer Explanations

(1312 questions)

Copyright 2008 Gleim Publications, Inc. Page 85Printed for Mamdouh Farag

Page 86: P.3 Answer Explanation

[288] Gleim #: 2.4.116 -- Source: Publisher

Answer (A) is incorrect because age may not be asked in a recruiting interview.

Answer (B) is incorrect because number of children may not be asked in a recruiting interview.

Answer (C) is incorrect because the kind of military discharge cannot be asked in a recruiting interview.

Answer (D) is correct. In a recruiting interview, one may not ask about the applicant’s marital status, number of children,height and weight, criminal record (except for the purpose of giving a security clearance), kind of military discharge, orage.

[289] Gleim #: 2.4.117 -- Source: Publisher

Answer (A) is incorrect because this is not an example of the halo effect.

Answer (B) is correct. The halo effect diminishes the accuracy of an evaluation when a manager’s judgment on onepositive trait affects the rating of other traits.

Answer (C) is incorrect because the recency effect is when the most recent behavior overshadows overall performance.

Answer (D) is incorrect because the central tendency effect is when all personnel who work together are rated within thesame narrow range.

[290] Gleim #: 2.4.118 -- Source: Publisher

Answer (A) is incorrect because the evaluation system should be relevant.

Answer (B) is incorrect because the evaluation system should be significant.

Answer (C) is incorrect because the evaluation system should be unbiased.

Answer (D) is correct. The system should be relevant, unbiased, significant, and practical. The evaluation system shouldbe as objective, easy to use, clearly understood, and efficient as possible, but it is not necessary that it be highlystructured.

[291] Gleim #: 2.4.119 -- Source: CIA, adapted

Answer (A) is incorrect because the applicant is providing the transcript, leading to a loss of independence. In addition,the transcript is unofficial, making it very easy to change the information and send a photocopy of the altered transcript.

Answer (B) is correct. Calling the last place of employment for candidates to verify information represents anindependent verification of employment. The hiring organization is performing the verification process.

Answer (C) is incorrect because there is nothing to prevent the applicants from writing the letters themselves, puttingfraudulent return address information on the letters, and mailing them.

Answer (D) is incorrect because if an applicant is going to lie about information, there is no reason to believe that theapplicant will not sign the applicant’s own name to the fraudulent information. This is not an independent verification.

Gleim's CIA Test Prep: Part III: Business Analysis and Information TechnologyAnswer Explanations

(1312 questions)

Copyright 2008 Gleim Publications, Inc. Page 86Printed for Mamdouh Farag

Page 87: P.3 Answer Explanation

[292] Gleim #: 2.4.120 -- Source: CIA 588 III-12

Answer (A) is correct. Behavior-oriented performance evaluation rewards the behavior that is desired by management.Behavior control involves examining work processes rather than work output.

Answer (B) is incorrect because the goal-oriented approach measures how well the employee attained the objectives orgoals set by management.

Answer (C) is incorrect because a trait-oriented approach tends to reward what the supervisor thinks of the employeerather than the job the employee did.

Answer (D) is incorrect because an employee-oriented approach would focus on who did the job.

[293] Gleim #: 3.1.1 -- Source: CIA 597 IV-1

Answer (A) is incorrect because historical cost is used to measure property, plant, and equipment and most inventories.

Answer (B) is incorrect because some inventories are measured at current (replacement) cost.

Answer (C) is incorrect because short-term receivables and some inventories are reported at net realizable value.

Answer (D) is correct. The measurement basis most commonly adopted by entities in preparing their financial statementsis historical cost. However, it is usually combined with other measurement bases (attributes). The attribute used tomeasure a long-term receivable or payable is the present or discounted value of its future cash flows.

[294] Gleim #: 3.1.2 -- Source: CIA 593 IV-25

Answer (A) is correct. Matching is the simultaneous or combined recognition of revenues and expenses resulting directlyand jointly from the same transactions or other events. Expenses should be associated with the revenues that they help tocreate. Because the catalogs are still on hand at the balance sheet date, they will not contribute to an inflow of economicbenefits until the next period. Hence, the cost should be deferred and matched with the revenues of the following period.

Answer (B) is incorrect because the revenue recognition principle determines the period in which revenue is recognized.

Answer (C) is incorrect because reliable information is free of error and bias and is representationally faithful.

Answer (D) is incorrect because the cost principle states that cost is the usual basis for recording most assets andliabilities.

[295] Gleim #: 3.1.3 -- Source: CIA 1192 IV-37

Answer (A) is correct. Income includes revenue and gains. Their essential nature is the same, and they are not treated asseparate financial statement elements. Revenues occur in the course of ordinary activities. Gains may or may not occur inthe course of ordinary activities. For example, gains may occur from the sale of noncurrent assets. Thus, the gain on thesale of a plant asset is not an operating item and should be classified in an income statement with separate operating andnonoperating sections in the other revenues and gains section.

Answer (B) is incorrect because the asset sold was not stock in trade.

Answer (C) is incorrect because extraordinary items are not reported in the financial statements or notes.

Answer (D) is incorrect because the transaction is not the correction of an error in the financial statements of a priorperiod.

Gleim's CIA Test Prep: Part III: Business Analysis and Information TechnologyAnswer Explanations

(1312 questions)

Copyright 2008 Gleim Publications, Inc. Page 87Printed for Mamdouh Farag

Page 88: P.3 Answer Explanation

[296] Gleim #: 3.1.4 -- Source: CIA 595 IV-29

Answer (A) is correct. Expenses include losses. Their essential nature is the same, and they are not treated as separatefinancial statement elements. Losses may or may not occur in the course of ordinary activities. For example, they mayresult from nonreciprocal transactions (e.g., theft), reciprocal transactions (e.g., a sale of plant assets), or from holdingassets or liabilities. Losses are typically displayed separately.

Answer (B) is incorrect because no restitution will be made. Thus, recording the item as a receivable, then writing it off,is not consistent with the substance of the event.

Answer (C) is incorrect because, although some inventory shrinkage is expected in the normal course of processing, fraudis abnormal. Hence, the item should be recorded as a loss.

Answer (D) is incorrect because losses are included in the determination of profit or loss.

[297] Gleim #: 3.1.5 -- Source: CIA 591 IV-44

Answer (A) is incorrect because a going concern should report assets at their undepreciated historical cost. Whenliquidation appears imminent, historical cost is inappropriate for balance sheet reporting.

Answer (B) is incorrect because an entity facing liquidation is expected to dispose of its assets in a “forced” or“distressed” sale and is unlikely to realize the fair value amount. The net realizable value of the assets is the appropriateamount for reporting purposes.

Answer (C) is correct. When liquidation is imminent, and the going concern assumption is no longer valid, the mostappropriate valuation method for assets is realizable value, which is the amount of cash currently obtainable by sale in anorderly disposal.

Answer (D) is incorrect because current cost is only appropriate when the going concern assumption is applicable and theeffects of changing prices are to be measured and reported in the financial statements.

[298] Gleim #: 3.1.6 -- Source: CIA 1190 IV-27

Answer (A) is correct. The objectives of financial reporting are concerned with the underlying goals and purposes ofaccounting. They are to provide information that (1) is useful to those making investment and credit decisions, assumingthat those individuals have a reasonable understanding of business and economic activities; (2) is helpful to current andpotential investors and creditors and other users in assessing the amount, timing, and uncertainty of future cash flows; and(3) discloses economic resources, claims to those resources, and the changes therein.

Answer (B) is incorrect because assessing the adequacy of internal control is a function of internal auditing, not financialreporting.

Answer (C) is incorrect because evaluating management results compared with standards is a function of internalauditing, not financial reporting.

Answer (D) is incorrect because providing information on compliance with established procedures is a function of internalauditing, not financial reporting.

Gleim's CIA Test Prep: Part III: Business Analysis and Information TechnologyAnswer Explanations

(1312 questions)

Copyright 2008 Gleim Publications, Inc. Page 88Printed for Mamdouh Farag

Page 89: P.3 Answer Explanation

[299] Gleim #: 3.1.7 -- Source: CIA 590 IV-25

Answer (A) is incorrect because the economic entity assumption is that every entity’s affairs are separate from those of itsowners.

Answer (B) is correct. Every business is assumed to be a going concern that will continue operating indefinitely. Thus,liquidation values are not important. For example, if an entity is not a going concern, its intangible assets are reported atliquidation values, not at historical cost net of amortization.

Answer (C) is incorrect because the monetary unit assumption provides that all transactions and events can be measuredin terms of money.

Answer (D) is incorrect because the historical cost principle deems cost to be the most objective and reliable measure.

[300] Gleim #: 3.1.8 -- Source: CIA 592 IV-29

Answer (A) is incorrect because the going-concern principle relates to circumstances in which there is doubt as to theviability of the enterprise.

Answer (B) is incorrect because relevance and reliability are two of the principal qualitative characteristics of informationin financial statements. Information is relevant if it permits users to predict the outcome of future events or confirm orcorrect their prior expectations. Reliability provides assurance that the information is reasonably free from error and biasand represents what it purports to represent. Thus, reliable information must be neutral, that is, free from error and bias.

Answer (C) is incorrect because comparability is a principal qualitative characteristic. Financial statements must becomparable for the same entity over time and also among different entities. Information is relevant if it permits users topredict the outcome of future events or confirm or correct their prior expectations.

Answer (D) is correct. In principle, wasting assets should be capitalized and depreciated. However, the effect on thefinancial statements of expensing rather than capitalizing and depreciating the staplers is clearly not material given thatthey cost 1,000 and the enterprise has total assets of 100,000,000. The choice of treatment is not likely to influence thedecisions of financial statement users. The balance between benefit and cost is a pervasive constraint, not a qualitativecharacteristic. The benefits should exceed the cost of information. Specifically, the cost of producing the information aboutdepreciation expense over 10 years for the staplers probably is higher than the benefits of the information for decisionmaking. Thus, the expedient procedure of expensing the 1,000 should be followed.

[301] Gleim #: 3.1.9 -- Source: CIA 1191 IV-29

Answer (A) is incorrect because direct labor cost is a product (inventoriable) cost that is normally not recognized untilsale.

Answer (B) is incorrect because direct labor cost is a product (inventoriable) cost that is normally not recognized untilsale.

Answer (C) is incorrect because direct labor cost is a product (inventoriable) cost that is normally not recognized untilsale.

Answer (D) is correct. Recognition of expenses occurs concurrently with recognition of increases in liabilities ordecreases in assets. Expenses are recognized if the costs are directly associated with the earning of particular incomeitems. This process is often called matching. Matching is simultaneous or combined recognition of the revenues andexpenses that result directly and jointly from the same transactions or other events. This direct relationship is exemplifiedby the sale of a product. This transaction results in revenue (sales revenue) for receipt of cash or a receivable, therecognition of an expense (cost of sales) for the sacrifice of the product to a customer, and a decrease in inventory. Thedirect labor cost of manufacturing the product is absorbed by the finished goods inventory and is not recognized until sale.

Gleim's CIA Test Prep: Part III: Business Analysis and Information TechnologyAnswer Explanations

(1312 questions)

Copyright 2008 Gleim Publications, Inc. Page 89Printed for Mamdouh Farag

Page 90: P.3 Answer Explanation

[302] Gleim #: 3.1.10 -- Source: CIA 1192 IV-38

Answer (A) is incorrect because not all losses are extraordinary items. They are not reported in the financial statements ornotes.

Answer (B) is incorrect because losses may be immaterial, and most expenses are material.

Answer (C) is correct. Expenses are defined as “decreases in economic benefits during the accounting period in the formof outflows or depletions of assets or incurrences of liabilities that result in decreases in equity, other than those relatingto distributions to equity participants.” Expenses include losses. Thus, losses are not treated as separate elements.Expenses include items arising in the course of ordinary activities. Losses meet the definition of expenses but may or maynot occur in the course of ordinary activities. For example, losses may result from the sale of noncurrent assets or fromnatural disasters.

Answer (D) is incorrect because some expenses cannot be prevented, or at least not in the short run. Some losses can beprevented.

[303] Gleim #: 3.1.11 -- Source: CIA 1196 IV-36

Answer (A) is incorrect because administrative expense is a cash charge.

Answer (B) is incorrect because interest expense is a cash charge.

Answer (C) is incorrect because income tax expense is a cash charge.

Answer (D) is correct. Depreciation expense is a noncash charge. It represents the allocation of the historical cost ofassets to the time periods in which they are used to generate revenues.

[304] Gleim #: 3.2.12 -- Source: CIA 597 IV-12

Answer (A) is incorrect because 525,000 deducts rather than adds the 60,000 increase in receivables.

Answer (B) is incorrect because 555,000 deducts the increase in receivables and adds the increase in unearned revenue.

Answer (C) is correct. The amount of service revenue for the year on an accrual basis equals 645,000 (600,000 cashcollected – 15,000 unearned revenue + 60,000 increase in accounts receivable).

Answer (D) is incorrect because 675,000 adds the 15,000 increase in unearned revenue.

[305] Gleim #: 3.2.13 -- Source: CIA 1195 IV-3

Answer (A) is incorrect because adjusting entries are made prior to closing.

Answer (B) is correct. The order of the steps in the accounting cycle is identification and measurement of transactionsand other events required to be recognized, journalization, posting from the journals to the ledgers, the development of atrial balance, adjustments to produce an adjusted trial balance, statement presentation, closing, taking a postclosing trialbalance (optional), and making reversing entries (optional).

Answer (C) is incorrect because reversing entries are made after adjustments and closing entries.

Answer (D) is incorrect because posting is done prior to adjusting.

Gleim's CIA Test Prep: Part III: Business Analysis and Information TechnologyAnswer Explanations

(1312 questions)

Copyright 2008 Gleim Publications, Inc. Page 90Printed for Mamdouh Farag

Page 91: P.3 Answer Explanation

[306] Gleim #: 3.2.14 -- Source: CIA 597 IV-4

Answer (A) is incorrect because the existing balance in the prepaid insurance account should be expensed.

Answer (B) is correct. The insurance policy in effect at the beginning of the year expired and was renewed during theyear. Hence, the entire 3,600 beginning balance in the prepaid insurance account should be expensed. The amount of theprepayment on the new policy to be expensed is 1,500 [3 months elapsed × (18,000 ÷ 36-month duration of the policy)].The debit balance in prepaid insurance at year-end is therefore 16,500 (18,000 – 1,500), and total insurance expense is5,100 (3,600 + 1,500).

Answer (C) is incorrect because insurance expense should be recognized for the last 3 months of the year.

Answer (D) is incorrect because prepaid insurance should be credited and insurance expense debited for 3,600.

[307] Gleim #: 3.2.15 -- Source: CIA 597 IV-5

Answer (A) is incorrect because the income summary account is used for closing entries, not adjusting entries.

Answer (B) is incorrect because the entry shown is a reversing entry that could be made at the beginning of thesubsequent period.

Answer (C) is correct. An expense incurred but not yet paid is an accrued expense. The adjusting entry to record anaccrued expense of 1,700 for unpaid salaries and wages is to debit an expense account and credit a liability account.

Answer (D) is incorrect because the debit should be to an expense account.

[308] Gleim #: 3.2.16 -- Source: CIA 597 IV-6

Answer (A) is incorrect because an expense account should be credited in a closing entry, and the income summaryaccount should be debited.

Answer (B) is correct. All income statement account balances are closed either to a summary account (such as incomesummary or revenue and expense summary) or to retained earnings. The normal balance of an expense account is a debit;therefore, an expense account is credited in a closing entry.

Answer (C) is incorrect because an expense account should not be closed to a revenue account. Such offsetting is not goodpractice.

Answer (D) is incorrect because an expense account should be credited in a closing entry.

[309] Gleim #: 3.2.17 -- Source: CIA 596 IV-5

Answer (A) is correct. Assuming an account entitled “dividends” or “dividends declared” is debited when dividendspayable is credited on the declaration date, it must be closed at the end of the period. The balance in this account is closeddirectly to retained earnings. The effect of declaring and paying dividends is to reduce retained earnings, so the entry is adebit to retained earnings and a credit to dividends.

Answer (B) is incorrect because the closing entry should credit dividends and debit retained earnings.

Answer (C) is incorrect because dividends is closed directly to retained earnings.

Answer (D) is incorrect because dividends is closed directly to retained earnings by a credit.

Gleim's CIA Test Prep: Part III: Business Analysis and Information TechnologyAnswer Explanations

(1312 questions)

Copyright 2008 Gleim Publications, Inc. Page 91Printed for Mamdouh Farag

Page 92: P.3 Answer Explanation

[310] Gleim #: 3.2.18 -- Source: CIA 1195 IV-4

Answer (A) is incorrect because reversing entries are optional.

Answer (B) is incorrect because reversing entries are made at the beginning of the next accounting period.

Answer (C) is incorrect because reversing entries are the exact opposite of the adjustments made in the previous period.

Answer (D) is correct. Reversing entries are made at the beginning of a period to reverse the effects of adjusting entriesmade at the end of the preceding period. They are optional entries made for the sake of convenience in recording thetransactions of the period. In order for reversing entries to reverse the prior adjustments, they must be the exact oppositeof the adjustments made in the previous period.

[311] Gleim #: 3.2.19 -- Source: CIA 592 IV-53

Answer (A) is incorrect because 93,300 is the present value of 200,000 to be received in 8 years.

Answer (B) is incorrect because 200,000 is the present value, not the future value, of 200,000 invested today.

Answer (C) is incorrect because the addition of the present and future values has no accounting meaning.

Answer (D) is correct. The interest factor for the future value of a present sum is equal to the reciprocal of the interestfactor for the present value of a future sum. Thus, the future value is 428,724 [(200,000 ÷ 93,300) × 200,000].

[312] Gleim #: 3.2.20 -- Source: CIA 1192 IV-55

Answer (A) is incorrect because 39,150 is based on dividing (270,000 × 1.16) by 8 (years).

Answer (B) is incorrect because 43,200 is 16% of 270,000.

Answer (C) is correct. The periodic payment is found by dividing the amount to be accumulated (300,000 price – 30,000down payment = 270,000) by the interest factor for the present value of an ordinary annuity for 8 years at 16%.Consequently, the payment is 62,160 (270,000 ÷ 4.3436).

Answer (D) is incorrect because 82,350 reflects multiplication by the present value of a sum due (.305) instead of dividingby the present value of an annuity (4.3436).

[313] Gleim #: 3.2.21 -- Source: CPA 1191 I-9

Answer (A) is incorrect because 250,000 does not take into account the interest.

Answer (B) is incorrect because 215,500 is computed using the future value factor of an ordinary annuity instead of anannuity in advance (annuity due).

Answer (C) is correct. The depositor wishes to have 1,000,000 at the end of a 4-year period (from 9/1/Year 1 to9/1/Year 5). The amount will be generated from four equal annual payments (an annuity) to be made starting at thebeginning of the 4-year period. The annual payment for this annuity can be calculated in advance by dividing the desiredfuture amount of 1,000,000 by the factor for the future amount of an annuity in advance of 1 at 10% for four periods. Eachannual deposit should therefore equal 195,700 (1,000,000 ÷ 5.11).

Answer (D) is incorrect because 684,930 is computed using the future value factor of 1 instead of the future value factorof an annuity in advance.

Gleim's CIA Test Prep: Part III: Business Analysis and Information TechnologyAnswer Explanations

(1312 questions)

Copyright 2008 Gleim Publications, Inc. Page 92Printed for Mamdouh Farag

Page 93: P.3 Answer Explanation

[314] Gleim #: 3.2.22 -- Source: CIA 1190 III-44

Answer (A) is incorrect because 30,034.33 cannot be derived from any of the time value factors given.

Answer (B) is incorrect because 43,329.44 results from incorrectly dividing by the factor for the present value of anordinary annuity for 4 periods.

Answer (C) is correct. The current investment is the present value of the given future amount. It equals the future amountmultiplied by the factor for the present value of 1 for four periods at 10%. Accordingly, the current investment is93,810.05 (137,350 × .6830).

Answer (D) is incorrect because 201,094.14 results from incorrectly using the factor for the future value of 1 for 4 periods.

[315] Gleim #: 3.2.23 -- Source: CIA 594 IV-3

Answer (A) is correct. The accounting cycle may be summarized as consisting of nine steps: record the period’stransactions in the appropriate journals, post to the ledger(s) from the journals, prepare an unadjusted trial balance,prepare and post adjusting journal entries, prepare an adjusted trial balance, prepare the financial statements, prepare andpost the closing entries, take a post-closing trial balance, and prepare reversing entries (optional).

Answer (B) is incorrect because the preparation of reversing entries is the last step in the accounting cycle.

Answer (C) is incorrect because the adjusted trial balance is prepared after adjusting entries are made.

Answer (D) is incorrect because the post-closing trial balance is prepared after adjusting entries and the adjusted trialbalance are completed.

[316] Gleim #: 3.2.24 -- Source: CIA 1194 IV-28

Answer (A) is incorrect because 5,000 is the interest expense under the cash basis.

Answer (B) is correct. The interest expense for the first year equals the 5,000 payment made on October 1 plus the threemonths’ accrued interest at year-end, or 7,500 (5,000 + 2,500).

Answer (C) is incorrect because 10,000 is the interest expense for a full year.

Answer (D) is incorrect because 12,500 is equal to 15 months of interest.

[317] Gleim #: 3.2.25 -- Source: CIA 595 IV-1

Answer (A) is correct. To convert from the cash basis (cash receipts) to the accrual basis (net sales), the increase in netaccounts receivable must be added to cash receipts from customers.

Answer (B) is incorrect because a decrease in receivables would be subtracted from cash receipts.

Answer (C) is incorrect because changes in accounts payable are not included in the calculation of net sales.

Answer (D) is incorrect because changes in accounts payable are not included in the calculation of net sales.

Gleim's CIA Test Prep: Part III: Business Analysis and Information TechnologyAnswer Explanations

(1312 questions)

Copyright 2008 Gleim Publications, Inc. Page 93Printed for Mamdouh Farag

Page 94: P.3 Answer Explanation

[318] Gleim #: 3.2.26 -- Source: CIA 595 IV-3

Answer (A) is correct. The order of the steps in the accounting cycle is identification and measurement of transactionsand other events required to be recognized, journalization, posting from the journals to the ledgers, the development of atrial balance, adjustments to produce an adjusted trial balance, statement presentation, closing, taking a post-closing trialbalance (optional), and making reversing entries (optional).

Answer (B) is incorrect because, of the steps listed, journalization occurs second.

Answer (C) is incorrect because, of the steps listed, posting occurs third.

Answer (D) is incorrect because the identification and measurement of transactions is the first step in the accountingcycle.

[319] Gleim #: 3.2.27 -- Source: CIA 595 IV-4

Answer (A) is correct. Because the initial entry recorded the entire purchase to a nominal account, the year-end adjustingentry must debit a prepaid asset to reflect the remaining supplies on hand of 15,000. The adjusting entry must also credit(reduce) expenses for the 15,000 of supplies that were not used during the period.

Answer (B) is incorrect because the reversing entry is to debit expense and credit a prepaid asset.

Answer (C) is incorrect because the appropriate amount is 15,000, not 35,000.

Answer (D) is incorrect because the closing entry would have been to debit expense and credit a prepaid asset if the initialentry had been to a real account.

[320] Gleim #: 3.2.28 -- Source: CIA R98 IV-36

Answer (A) is incorrect because 46,650 is the present value of 100,000 discounted at 10% for 8 years.

Answer (B) is incorrect because 100,000 is the present value of 214,360 discounted at 10% for 8 years.

Answer (C) is incorrect because 114,360 is the interest earned.

Answer (D) is correct. The present value of a future amount equals that amount divided by the interest factor for thegiven discount period and discount rate. The interest factor for the present value of 1 due in 8 years and discounted at 10%is 2.1436 (100,000 ÷ 46,650). Thus, the future value of 100,000 must be 214,360 (2.1436 × 100,000).

[321] Gleim #: 3.2.29 -- Source: CIA R98 IV-41

Answer (A) is incorrect because the cost at the date of purchase is the present value of an ordinary annuity.

Answer (B) is incorrect because the cost at the date of purchase is the present value of an ordinary annuity.

Answer (C) is incorrect because the cost at the date of purchase is the present value of an ordinary annuity.

Answer (D) is correct. The cost of the machine is the present value of the annuity (the series of equal periodic payments).The annuity is an ordinary annuity because the payments begin one period after the date of purchase. The cost thereforeequals the periodic payment times the interest factor for an ordinary annuity of one currency unit for a specified number ofequal periodic payments discounted at a given interest rate.

Gleim's CIA Test Prep: Part III: Business Analysis and Information TechnologyAnswer Explanations

(1312 questions)

Copyright 2008 Gleim Publications, Inc. Page 94Printed for Mamdouh Farag

Page 95: P.3 Answer Explanation

[322] Gleim #: 3.2.30 -- Source: CIA 1196 IV-4

Answer (A) is correct. An entry debiting prepaid property taxes and crediting property tax expense is needed at thebalance sheet date when prepayments are initially recorded in an expense account. This adjusting entry defers expenserecognition to the future period in which the property taxes apply.

Answer (B) is incorrect because the entry records an asset.

Answer (C) is incorrect because the entry records an asset.

Answer (D) is incorrect because the entry records an unexpired cost (an asset).

[323] Gleim #: 3.2.31 -- Source: CIA 597 IV-3

Answer (A) is incorrect because the accrual of revenue increases assets, increases equity, and has no effect on liabilities.

Answer (B) is correct. The journal entry to accrue revenue requires a debit to a receivable account and a credit to arevenue account. Thus, the accrual of revenue increases assets and equity.

Answer (C) is incorrect because the accrual of revenue increases assets, increases equity, and has no effect on liabilities.

Answer (D) is incorrect because the accrual of revenue increases assets, increases equity, and has no effect on liabilities.

[324] Gleim #: 3.3.32 -- Source: CIA 1195 IV-27

Answer (A) is incorrect because the going concern assumption is that the business will have an indefinite life.

Answer (B) is incorrect because the monetary unit assumption is that money is the common denominator by whicheconomic activity is conducted and that the monetary unit provides an appropriate basis for accounting measurement andanalysis.

Answer (C) is incorrect because the historical cost principle reflects the practice that many assets and liabilities areaccounted for and reported on the basis of acquisition price.

Answer (D) is correct. Recognition of revenue occurs when the flow of future economic benefits to the enterprise isprobable and such benefits are reliably measurable. Recording advance payments as a liability reflects a determinationthat the receipt of future economic benefits is not sufficiently certain to merit revenue recognition, given that theenterprise has not yet performed its obligations.

[325] Gleim #: 3.3.33 -- Source: CIA 1194 IV-24

Answer (A) is incorrect because the recognition of income from the sale of the extended warranty is deferred until theextended warranty period begins.

Answer (B) is correct. Because warranty costs are expected to be incurred evenly over the life of the warranty contracts,the income should be recognized on the straight-line basis over the life of the extended warranty contract.

Answer (C) is incorrect because the income should be recognized evenly over the life of the contract. It is not related tothe timing of the claims.

Answer (D) is incorrect because income is recognized over the life of the warranty, not at expiration.

Gleim's CIA Test Prep: Part III: Business Analysis and Information TechnologyAnswer Explanations

(1312 questions)

Copyright 2008 Gleim Publications, Inc. Page 95Printed for Mamdouh Farag

Page 96: P.3 Answer Explanation

[326] Gleim #: 3.3.34 -- Source: CIA 596 IV-4

Answer (A) is incorrect because 145,000 equals Year 2 revenue from Year 2 sales.

Answer (B) is incorrect because 165,000 adds, rather than subtracts, the Year 2 deferred revenue and subtracts, ratherthan adds, the Year 1 receipts for which revenue was earned in Year 2.

Answer (C) is incorrect because 175,000 equals the cash receipts for Year 2.

Answer (D) is correct. The sales revenue earned in Year 2 equals Year 2 cash receipts, minus any receipts in Year 2 forwhich the revenue was deferred, plus the revenue earned from cash receipts in Year 1, or 185,000 (175,000 – 30,000 +40,000).

[327] Gleim #: 3.3.35 -- Source: CIA 590 IV-26

Answer (A) is incorrect because the certainty and measurability criteria are not met when the customer places an order.

Answer (B) is correct. Income, which includes revenue and gains, should not be recognized until an increase in futureeconomic benefits related to an increase in an asset or a decrease in a liability is sufficiently certain and can be measuredreliably. The most common time at which these two conditions are met is when the product or merchandise is delivered orservices are rendered to customers. Thus, the entity has substantially accomplished what it must do to be entitled to futureeconomic benefits when it serves the luncheon. It should then accrue a receivable and revenue.

Answer (C) is incorrect because the date for billing is a matter of administrative procedure and convenience. The revenueshould be recognized at the date the service was performed.

Answer (D) is incorrect because the revenue should be recognized at the point of performance of the service. To wait untilthe receivable is collected is to ignore the accrual basis of accounting, which is identified in the Framework for thePreparation and Presentation of Financial Statements as an underlying assumption of financial accounting.

[328] Gleim #: 3.3.36 -- Source: CIA 595 IV-11

Answer (A) is incorrect because sales and cost of sales are recognized in proportion to cash collections.

Answer (B) is correct. Under the installment method, the gross profit on sales (sales – cost of sales) is not recognizeduntil cash is collected. The proportion of cash collected on the sales during the accounting period determines theproportion of the gross profit on those sales that is recognized during the period. Hence, both sales and cost of sales aredeferred.

Answer (C) is incorrect because only the gross profit is deferred on sales for which cash has not yet been collected.

Answer (D) is incorrect because only the gross profit is deferred on sales for which cash has not yet been collected.

[329] Gleim #: 3.3.37 -- Source: CIA 597 IV-10

Answer (A) is incorrect because the accrual basis recognizes revenue on the date of the installment sale.

Answer (B) is incorrect because the installment basis recognizes revenue in proportion to the cash collections.

Answer (C) is correct. Under the cost-recovery method, no revenue is recognized until cash payments by the buyer exceedthe seller’s cost of the merchandise sold. This method is appropriate when collection of the revenue is very uncertain.

Answer (D) is incorrect because, after the cash collections equal the cost of sales, revenue is to be recognized for anyfurther collections.

Gleim's CIA Test Prep: Part III: Business Analysis and Information TechnologyAnswer Explanations

(1312 questions)

Copyright 2008 Gleim Publications, Inc. Page 96Printed for Mamdouh Farag

Page 97: P.3 Answer Explanation

[330] Gleim #: 3.3.38 -- Source: CIA 1190 IV-28

Answer (A) is correct. Recognition of an element of financial statements (e.g., income, which includes revenue and gains)requires that two criteria be met. It must be probable that any future economic benefit associated with the item will flowto or from the entity, and the cost or value of the item must be measurable with reliability. The usual procedures forincome recognition, e.g., that income be earned, reflect these criteria. Thus, income is recognized when an increase infuture economic benefits is associated with an increase in an asset or a decrease in a liability. However, the entity has notsubstantially completed what it must do to be entitled to the benefits represented by the advance payment, and the receiptof future economic benefits is not sufficiently certain to merit income recognition. Accordingly, a liability should berecognized because the entity has a current obligation arising from a past event that will require an outflow of economicbenefits, that is, to deliver the software or to refund the customer’s money. Thus, a liability for 100,000 and revenue of 0should be recognized for Year 1.

NOTE: This analysis assumes that the sale of the software is a sale either of goods or of services for which theappropriate conditions have not been met. Under IAS 18, Revenue Recognition, revenue is recognized for a sale of goodswhen the entity has transferred the significant risks and rewards of ownership, the entity has neither continuingmanagerial involvement to an extent associated with ownership nor effective control over the goods, the amount can bereliably measured, it is probable that the economic benefits will flow to the entity, and transaction costs can be reliablymeasured. For a sale of services, revenue is recognized when revenue can be reliably measured, it is probable that theeconomic benefits will flow to the entity, the stage of completion can be reliably measured, and the costs incurred and thecosts to complete can be reliably measured.

Answer (B) is incorrect because income, which includes revenue and gains, should not be recognized until an increase infuture economic benefits related to an increase in an asset or a decrease in a liability is sufficiently certain and can bemeasured reliably.

Answer (C) is incorrect because income, which includes revenue and gains, should not be recognized until an increase infuture economic benefits related to an increase in an asset or a decrease in a liability is sufficiently certain and can bemeasured reliably.

Answer (D) is incorrect because income, which includes revenue and gains, should not be recognized until an increase infuture economic benefits related to an increase in an asset or a decrease in a liability is sufficiently certain and can bemeasured reliably.

[331] Gleim #: 3.3.39 -- Source: CIA 1193 IV-28

Answer (A) is incorrect because revenue is not recognized until progress has been made toward completion.

Answer (B) is correct. Under the percentage-of-completion method, revenues and expenses are recognized based on thestage of completion at the balance sheet date if the outcome of the contract can be estimated reliably. For a fixed-pricecontract, the outcome can be estimated reliably if (1) total revenue can be measured reliably, (2) it is probable that theeconomic benefits of the contract will flow to the enterprise, (3) contract costs to complete and stage of completion can bemeasured reliably, and (4) contract costs can be clearly identified and measured reliably so that actual and estimated costscan be compared.

Answer (C) is incorrect because the cash basis is inappropriate. An accrual method, that is, the percentage-of-completionmethod, should be used.

Answer (D) is incorrect because the completed-contract method is not a permissible method.

Gleim's CIA Test Prep: Part III: Business Analysis and Information TechnologyAnswer Explanations

(1312 questions)

Copyright 2008 Gleim Publications, Inc. Page 97Printed for Mamdouh Farag

Page 98: P.3 Answer Explanation

[332] Gleim #: 3.3.40 -- Source: CIA 593 IV-27

Answer (A) is incorrect because a sufficient degree of certainty that economic benefits will flow to the enterprise does notexist when the reservations are booked or confirmed.

Answer (B) is incorrect because a sufficient degree of certainty that economic benefits will flow to the enterprise does notexist when the reservations are booked or confirmed.

Answer (C) is incorrect because a sufficient degree of certainty that economic benefits will flow to the enterprise does notexist when the ticket is issued.

Answer (D) is correct. Recognition of an element of financial statements (e.g., income, which includes revenue and gains)requires that two criteria be met. It must be probable that any future economic benefit associated with the item will flowto or from the enterprise, and the cost or value of the item must be measurable with reliability. The usual procedures forincome recognition, e.g., that income be earned, reflect these criteria. Thus, income is recognized when an increase infuture economic benefits is associated with an increase in an asset or a decrease in a liability. The recognition criteria arenot met with respect to the collections from sales of airline tickets until the receipt of future economic benefits issufficiently certain. Such certainty exists when the airline performs the contracted-for service, that is, when the relatedflights occur. The critical event in the earning process for the airline is the delivery of the service to the customer, whichoccurs when the related flight takes place.

[333] Gleim #: 3.3.41 -- Source: CIA 1192 IV-27

Answer (A) is incorrect because a liability is recognized when the cash is collected prior to the rendition of the service.

Answer (B) is correct. Income, which includes revenue and gains, should not be recognized until an increase in futureeconomic benefits related to an increase in an asset or a decrease in a liability is sufficiently certain and can be measuredreliably. The most common time at which these two conditions are met is when the product or merchandise is delivered orservices are rendered to customers. In the situation presented, the performance of the service (monthly spraying) is sosignificant to creating a sufficient certainty of an increase in future economic benefits that revenue should not berecognized until delivery occurs.

Answer (C) is incorrect because revenue from services rendered is recognized when the services have been performed. Aportion of the services is performed monthly. Thus, a portion of the related revenue should be recognized monthly ratherthan waiting until the entire contract year is complete.

Answer (D) is incorrect because a portion of the related revenue should be recognized monthly rather than waiting untilthe entire fiscal year is complete.

[334] Gleim #: 3.3.42 -- Source: CIA 1191 IV-40

Answer (A) is incorrect because profit recognized in Year 2 is 550.

Answer (B) is incorrect because profit recognized in Year 2 is 550.

Answer (C) is correct. At the end of Year 1, total cost was expected to be 400 (100 incurred + 300 estimated cost tocomplete), and estimated total profit was 600 (1,000 price – 400 estimated total cost). Hence, the amount of profitrecognized in Year 1 was 150 [600 × (100 cost incurred ÷ 400 estimated total cost)]. The project was completed in Year 2at an additional cost of 200. Actual profit was therefore 700 (1,000 – 300 actual total cost). Profit recognized in year 2 is550 (700 total – 150 recognized in Year 1).

Answer (D) is incorrect because profit recognized in Year 2 is 550.

Gleim's CIA Test Prep: Part III: Business Analysis and Information TechnologyAnswer Explanations

(1312 questions)

Copyright 2008 Gleim Publications, Inc. Page 98Printed for Mamdouh Farag

Page 99: P.3 Answer Explanation

[335] Gleim #: 3.3.43 -- Source: CIA 590 IV-31

Answer (A) is incorrect because the revenue recognition criteria for sales of goods are not met in a consignment until theconsignee sells the goods to a third party.

Answer (B) is incorrect because the revenue recognition criteria for sales of goods are not met in a consignment until theconsignee sells the goods to a third party.

Answer (C) is incorrect because the revenue recognition criteria for sales of goods are not met in a consignment until theconsignee sells the goods to a third party.

Answer (D) is correct. Under a consignment arrangement, the consignor ships merchandise to the consignee who acts asagent for the consignor in selling the goods. The goods are in the physical possession of the consignee but remain theproperty of the consignor and are included in the consignor’s inventory. Revenue and the related cost of goods sold fromthese consigned goods should only be recognized by the consignor when the merchandise is sold and delivered to the finalcustomer. Accordingly, recognition occurs when notification is received that the consignee has sold the goods. Only thenare the recognition criteria for a sale of goods satisfied: the enterprise has transferred the significant risks and rewards ofownership, the enterprise has neither continuing managerial involvement to an extent associated with ownership noreffective control over the goods, the amount can be reliably measured, it is probable that the economic benefits will flowto the enterprise, and transactional costs can be reliably measured.

[336] Gleim #: 3.3.44 -- Source: CIA 1189 IV-29

Answer (A) is incorrect because there is no basis for such an allocation.

Answer (B) is incorrect because, when the certificates are sold, future economic benefits are neither sufficiently certainnor reliably measurable. The store has not yet performed its obligations created by the sales of gift certificates.

Answer (C) is incorrect because revenue is also recognized when certificates are redeemed.

Answer (D) is correct. Income, which includes revenue and gains, should not be recognized until an increase in futureeconomic benefits related to an increase in an asset or a decrease in a liability is sufficiently certain and can be measuredreliably. These criteria are met when the certificates are redeemed or expire because the liability for redemptions will thendecrease and economic benefits will be reliably measurable.

[337] Gleim #: 3.3.45 -- Source: CIA 1194 IV-8

Answer (A) is incorrect because 3,000,000 is based on the engineering estimate of the stage of completion.

Answer (B) is correct. Under the cost-to-cost approach to determining the stage of completion of the contract, the stage ofcompletion equals contract costs incurred to date divided by the most recent estimate of total contract costs. Hence, therevenue to be recognized in the first year is 4,705,882 [(4,000,000 ÷ 8,500,000) × 10,000,000]. This amount equals costsincurred plus recognized profit.

Answer (C) is incorrect because 5,000,000 is based on the original estimate of contract costs.

Answer (D) is incorrect because 6,000,000 is based on the percentage of total revenue billed to date.

Gleim's CIA Test Prep: Part III: Business Analysis and Information TechnologyAnswer Explanations

(1312 questions)

Copyright 2008 Gleim Publications, Inc. Page 99Printed for Mamdouh Farag

Page 100: P.3 Answer Explanation

[338] Gleim #: 3.3.46 -- Source: CIA 1194 IV-9

Answer (A) is incorrect because an excess of billings over the sum of costs and recognized profits is a liability, and1,000,000 is the amount of cash collected.

Answer (B) is incorrect because an excess of billings over the sum of costs and recognized profits is a liability.

Answer (C) is incorrect because 1,000,000 equals cash collected.

Answer (D) is correct. The gross amount due from (to) customers for contract work is an asset (liability). If the amount ofcosts incurred plus recognized profits minus recognized losses exceeds progress billings, the entity reports an asset. If theamount of progress billings exceeds costs incurred plus recognized profits minus recognized losses, the entity reports aliability. At the end of Year 1, the company had recognized 2,000,000 of revenue (costs to date + recognized profit) andhad submitted billings of 6,000,000. Thus, the excess billings equal 4,000,000. Because the billings exceed revenuerecognized, this amount is listed as a current liability. It represents deferred revenue. Given a 10 million fixed price and8.5 million of total costs, the assumption that 2 million of revenue was recognized under the percentage-of-completionmethod (cost-to-cost basis) necessarily includes the assumption that the stage of completion was 20% (2 million ÷ 10million), that recognized profit was 300,000 [20% × (10 million – 8.5 million)], and that costs to date were 1.7 million(20% × 8.5 million).

[339] Gleim #: 3.3.47 -- Source: CIA 1194 IV-25

Answer (A) is correct. Under the cash basis of accounting, revenue is recognized when the cash is received. Because thecustomers paid cash for the maintenance contracts in the first year, that is the year in which the revenue should berecognized.

Answer (B) is incorrect because, if the accrual basis is used, revenue received for the maintenance contracts is recognizedafter the first year. The services are provided and the revenue recognition criteria are met after year one.

Answer (C) is incorrect because, under cash-basis accounting, the revenue received in the first year is recognized in thefirst year.

Answer (D) is incorrect because accrual-basis accounting recognizes the revenue in year two. Services will be provided inyear two.

[340] Gleim #: 3.3.48 -- Source: CIA 1194 IV-26

Answer (A) is incorrect because the matching principle requires the recognition of related revenues and expenses in thesame accounting period.

Answer (B) is correct. An asset classified under property, plant, and equipment is measured initially at cost. This amountincludes the purchase price and any directly attributable costs of bringing the asset to working condition for its intendeduse. Directly attributable costs include costs of, for example, site preparation, initial delivery and handling, installation,professional fees (e.g., those of architects and engineers), and dismantling and removing the asset and restoring the site.The purchase price is determined by adding any import fees and nonrefundable purchase taxes and subtracting any tradediscounts and rebates.

Answer (C) is incorrect because the going-concern assumption is that the enterprise will continue in existenceindefinitely.

Answer (D) is incorrect because revenue recognition principles refer to the timing of revenue recognition, not to thevaluation basis for property, plant, and equipment.

Gleim's CIA Test Prep: Part III: Business Analysis and Information TechnologyAnswer Explanations

(1312 questions)

Copyright 2008 Gleim Publications, Inc. Page 100Printed for Mamdouh Farag

Page 101: P.3 Answer Explanation

[341] Gleim #: 3.3.49 -- Source: CIA 1196 IV-11

Answer (A) is incorrect because (100,000) is the difference between costs incurred and collections.

Answer (B) is incorrect because 100,000 is the difference between billings and costs incurred.

Answer (C) is incorrect because 200,000 is the difference between billings and collections.

Answer (D) is correct. When the outcome of a contract can be reasonably estimated, the percentage-of-completion methodrecognizes revenue based on the stage of completion of the contract. One typical method for estimating the stage ofcompletion is the calculation of ratio of the contract costs incurred to date to the estimated total costs. The percentage-of-completion at year-end on the cost-to-cost basis is 35% (700,000 ÷ 2,000,000). The profit for Year 1 is the anticipatedprofit on the contract times the completion percentage. Thus, profit for Year 1 is 350,000 [(3,000,000 – 2,000,000) ×35%].

[342] Gleim #: 3.3.50 -- Source: CIA 1196 IV-30

Answer (A) is incorrect because 3,000 does not include the 40 barrels consigned in July.

Answer (B) is incorrect because 4,000 does not include the 40 barrels consigned in July or deduct the 5 barrels returned.

Answer (C) is correct. Under a consignment arrangement, the consignor ships merchandise to the consignee who acts asagent for the consignor in selling the goods. The goods are in the physical possession of the consignee but remain theproperty of the consignor and are included in the consignor’s inventory. Revenue and the related cost of goods sold fromthese consigned goods should only be recognized by the consignor when the merchandise is sold and delivered to the finalcustomer. Accordingly, recognition occurs when notification is received that the consignee has sold the goods. Only thenare the recognition criteria for a sale of goods satisfied: the enterprise has transferred the significant risks and rewards ofownership, the enterprise has neither continuing managerial involvement to an extent associated with ownership noreffective control over the goods, the amount can be reliably measured, it is probable that the economic benefits will flowto the enterprise, and transactional costs can be reliably measured. In this consignment arrangement, 30 barrels have beenpaid for, so the revenue is recognized for these barrels. The 5 barrels returned are not included in unearned revenuebecause they constitute a return of consigned goods. Accordingly, the amount of inappropriately recognized revenue is11,000 [(40 consigned + 50 consigned – 30 paid for – 5 returns) × 200].

Answer (D) is incorrect because 12,000 does not reflect the 5 barrels returned.

[343] Gleim #: 3.3.51 -- Source: CIA 1193 IV-30

Answer (A) is correct. The prepayment does not meet the income recognition criteria because the future inflow ofeconomic benefits is not sufficiently certain given that the entity has not done what is required to be entitled to thosebenefits. Thus, the amount received in advance is considered a liability (deferred income) because it represents anobligation to perform a service in the future (an outflow of economic benefits) arising from a past transaction.

Answer (B) is incorrect because the income is not earned. The exterminator has not performed the related services for thecustomer.

Answer (C) is incorrect because accrued income has met the recognition criteria but has not been received.

Answer (D) is incorrect because the customer has a prepaid expense (expense paid but not incurred).

Gleim's CIA Test Prep: Part III: Business Analysis and Information TechnologyAnswer Explanations

(1312 questions)

Copyright 2008 Gleim Publications, Inc. Page 101Printed for Mamdouh Farag

Page 102: P.3 Answer Explanation

[344] Gleim #: 3.3.52 -- Source: CIA 597 IV-14

Answer (A) is correct. Recognition of an element of financial statements (e.g., income, which includes revenue and gains)requires that two criteria be met. It must be probable that any future economic benefit associated with the item will flowto or from the enterprise, and the cost or value of the item must be measurable with reliability. The usual procedures forincome recognition, e.g., that income be earned, reflect these criteria. Thus, income is recognized when an increase infuture economic benefits is associated with an increase in an asset or a decrease in a liability. However, the enterprise hasnot substantially completed what it must do to be entitled to the benefits represented by the advance payment, and thereceipt of future economic benefits is not sufficiently certain to merit income recognition. Accordingly, a liability shouldbe recognized because the entity has a current obligation arising from a past event that will require an outflow ofeconomic benefits, that is, to deliver goods or to refund the customer’s money. The delivery of goods is to take placewithin a year of the balance-sheet date; therefore, the obligation is expected to be settled in the normal course of theoperating cycle or is due to be settled within 12 months.

Answer (B) is incorrect because the obligation is current.

Answer (C) is incorrect because a contra account reduces the valuation of the related account.

Answer (D) is incorrect because the payment has not met the income recognition criteria.

[345] Gleim #: 3.3.53 -- Source: CIA 597 IV-28

Answer (A) is correct. The collections represent a liability (deferred income). The income recognition criteria are not metwith respect to the season ticket collections until the receipt of future economic benefits is sufficiently certain. Suchcertainty exists with respect to collections related to the games played as of the balance sheet date. Accordingly, the entityshould recognize 1 million as income in Year 1 [(3,000,000 ÷ 6 months) × 2 months]. The remaining 2 million should bereported as a liability.

Answer (B) is incorrect because the liability at the end of Year 1 is 2 million.

Answer (C) is incorrect because the enterprise should report income of 1 million in Year 1 and 2 million in Year 2.

Answer (D) is incorrect because the enterprise should report income of 1 million in Year 1 and 2 million in Year 2.

[346] Gleim #: 3.3.54 -- Source: CIA, adapted

Answer (A) is incorrect because the going concern assumption is that the business will have a long life. This does notrelate directly to the practice of recording unearned revenues as liabilities.

Answer (B) is incorrect because the monetary unit assumption is that money is the common denominator by whicheconomic activity is conducted, and that the monetary unit provides an appropriate basis for accounting measurement andanalysis. It does not relate directly to the practice of recording unearned revenues as liabilities.

Answer (C) is incorrect because the historic cost principle is the requirement that most assets and liabilities be accountedfor and reported on the basis of acquisition price. It does not relate directly to the practice of recording unearned revenuesas liabilities.

Answer (D) is correct. Since the amount received in cash has not yet been earned, it is appropriate to record the advancepayment as a liability of the company. This is an example of the revenue recognition principle, which states that revenueshould not be recognized until it is earned.

Gleim's CIA Test Prep: Part III: Business Analysis and Information TechnologyAnswer Explanations

(1312 questions)

Copyright 2008 Gleim Publications, Inc. Page 102Printed for Mamdouh Farag

Page 103: P.3 Answer Explanation

[347] Gleim #: 3.3.55 -- Source: CIA R98 IV-39

Answer (A) is incorrect because the transferor must recognize cash and either a sale or a borrowing.

Answer (B) is correct. An entity derecognizes a financial asset (or a part thereof) when it loses control of the contractualrights the asset represents. Control is lost when the entity realizes the rights to the benefits of the contract, the rightsexpire, or the entity surrenders the rights. Moreover, whether loss of control has occurred depends on the positions of boththe transferor and the transferee. Control is not lost, for example, if the entity may reacquire the asset (unless it is readilyobtainable in the market or the price is fair value) or when the transferor is entitled and obligated to repurchase or redeemat a price giving the transferee a lender’s return. Control ordinarily is lost when the transferee may obtain the benefits ofthe asset, for example, if the transferee may freely sell or pledge the full fair value of the asset. When the conditions aboveare met, the transfer of receivables with recourse is accounted for as a sale, with the proceeds of the sale reduced by thefair value of the recourse obligation (a new financial liability).

Answer (C) is incorrect because certain conditions must be met before a sale may be recognized.

Answer (D) is incorrect because a sale may be recognized if certain conditions are met.

[348] Gleim #: 3.3.56 -- Source: CIA 597 IV-13

Answer (A) is incorrect because 720,000 is the year-end balance of installment accounts receivable.

Answer (B) is incorrect because 288,000 applies the gross profit percentage to the sum of cost of sales and general andadministrative expenses.

Answer (C) is correct. The gross profit on installment sales is 360,000 (1,200,000 installment sales – 840,000 cost ofinstallment sales). Accordingly, the gross profit percentage is 30% (360,000 gross profit ÷ 1,200,000 installment sales),and the amount of gross profit deferred at the end of the current year is 216,000 [30% × (1,200,000 installment sales –480,000 collections)]. General and administrative expenses have no effect on the computation of realized gross profit ordeferred gross profit. They are to be classified as operating expenses on the income statement in the period in which theyare incurred.

Answer (D) is incorrect because 144,000 treats general and administrative expenses as costs of installment sales.

[349] Gleim #: 3.4.57 -- Source: CIA 590 IV-32

Answer (A) is correct. The IASs do not require a particular income statement format, although, at a minimum, certainline items must be presented. The single-step income statement provides one grouping for income items and one forexpense items. The “single step” is the one subtraction necessary to arrive at profit or loss. The multiple-step incomestatement matches operating income and expenses separately from nonoperating items.

Answer (B) is incorrect because both formats separate income tax expense and administrative expenses.

Answer (C) is incorrect because both formats separate cost of goods sold expense and administrative expenses.

Answer (D) is incorrect because intraperiod income tax allocation procedures must be applied to both formats.Extraordinary items are not reported.

Gleim's CIA Test Prep: Part III: Business Analysis and Information TechnologyAnswer Explanations

(1312 questions)

Copyright 2008 Gleim Publications, Inc. Page 103Printed for Mamdouh Farag

Page 104: P.3 Answer Explanation

[350] Gleim #: 3.4.58 -- Source: CIA 1196 IV-5

Answer (A) is incorrect because the entity did have a profit.

Answer (B) is incorrect because 6,000 mistakenly deducts the 72,000 and adds the 75,000.

Answer (C) is incorrect because 9,000 is the difference between beginning and ending equity without considering thecapital transactions with, and distributions to, the owner.

Answer (D) is correct. Profit or loss may be derived using the basic accounting equation (assets = liabilities + equity).Equity at 1/1/Year 1 was 90,000 (210,000 of assets – 120,000 of liabilities). Equity at 12/31/Year 1 was 99,000 (270,000– 171,000). Because owner transactions decreased assets by 3,000 (72,000 investment – 75,000 withdrawals), profit musthave been 12,000 [99,000 – (90,000 – 3,000)].

[351] Gleim #: 3.4.59 -- Source: CIA 596 IV-44

Answer (A) is incorrect because 90 is the gross margin.

Answer (B) is incorrect because 135 is 150% of the gross margin.

Answer (C) is incorrect because 150 assumes profit before tax equals 50% of profit.

Answer (D) is correct. Profit equals sales minus cost of sales, G&A expenses, interest, and tax. Given a 50% tax rate,profit before tax must have been 20 [10 profit ÷ (1.0 – .5 tax rate)]. Accordingly, profit before interest and tax must havebeen 40 (20 profit before tax + 20 interest), and the gross margin (sales – cost of sales) must have been 90 (40 profitbefore interest and tax + 50 G&A expenses). If the gross margin is 50% of sales, sales equals 180 (90 gross margin ÷ .5).

[352] Gleim #: 3.4.60 -- Source: CIA 1193 IV-33

Answer (A) is incorrect because interest payments are classified as an operating or financing outflow on the statement ofcash flows.

Answer (B) is correct. Interest incurred is classified as interest expense on the income statement, which in turn reducesequity on the balance sheet by reducing retained earnings. According to IAS 7, cash payments for interest made by anenterprise that is not a financial institution may be classified on the statement of cash flows as an outflow of cash fromoperating or financing activities.

Answer (C) is incorrect because credit card interest charges reduce equity.

Answer (D) is incorrect because credit card interest charges reduce equity.

Gleim's CIA Test Prep: Part III: Business Analysis and Information TechnologyAnswer Explanations

(1312 questions)

Copyright 2008 Gleim Publications, Inc. Page 104Printed for Mamdouh Farag

Page 105: P.3 Answer Explanation

[353] Gleim #: 3.4.61 -- Source: CIA 1192 IV-32

Answer (A) is incorrect because only the direct method supplies information about major classes of gross cash receiptsand payments related to operating activities.

Answer (B) is correct. The statement of cash flows may report cash flows from operating activities in either an indirect ora direct format. The direct format reports the major classes of operating cash receipts and cash payments as gross amounts.The indirect presentation adjusts profit or loss to the same amount of net cash from operating activities that would bedetermined in accordance with the direct method. To arrive at this amount, the indirect method adjusts profit or loss forthe effects of noncash transactions, deferrals or accruals of past or future operating cash flows, and income or expenserelated to financing or investing activities.

Answer (C) is incorrect because the direct method, rather than the indirect method, supplies information about majorclasses of gross cash receipts and payments related to operating activities.

Answer (D) is incorrect because the direct method reports major classes of gross cash receipts and payments fromoperating activities.

[354] Gleim #: 3.4.62 -- Source: CIA 1195 IV-34

Answer (A) is incorrect because payment of cash dividends is a use of cash for an operating or financing activity.

Answer (B) is correct. According to IAS 7, dividends paid may be treated as a cash outflow from financing activitiesbecause they are a cost of obtaining resources from owners. However, they may also be treated as operating items to helpdetermine the enterprise’s ability to pay dividends from operating cash flows.

Answer (C) is incorrect because payment of cash dividends is a use of cash for an operating or financing activity.

Answer (D) is incorrect because payment of cash dividends is a use of cash for an operating or financing activity.

[355] Gleim #: 3.4.63 -- Source: CIA 596 IV-10

Answer (A) is incorrect because 70,000 fails to add to profit the reduction in accounts receivable.

Answer (B) is correct. Profit is adjusted to determine the net cash from operations. The payment of cash dividends isregarded as a cash flow from a financing activity. Hence, it is not a reconciling item. However, the decrease in accountsreceivable (220,000 – 200,000 = 20,000) during the period represents a cash inflow (collections of pre-Year 2 receivables)not reflected in Year 2 profit. Moreover, the decrease in payables (160,000 – 80,000 = 80,000) indicates a cash outflow(payment of pre-Year 2 liabilities) that also is not reflected in Year 2 net income. Accordingly, net cash from operationswas 90,000 (150,000 + 20,000 – 80,000).

Answer (C) is incorrect because 150,000 is profit.

Answer (D) is incorrect because 210,000 subtracts the reduction in receivables and adds the reduction in payables.

Gleim's CIA Test Prep: Part III: Business Analysis and Information TechnologyAnswer Explanations

(1312 questions)

Copyright 2008 Gleim Publications, Inc. Page 105Printed for Mamdouh Farag

Page 106: P.3 Answer Explanation

[356] Gleim #: 3.4.64 -- Source: CIA 1196 IV-1

Answer (A) is incorrect because 154,000 subtracts the 6,000 increase in prepaid salaries instead of adding it.

Answer (B) is correct. An increase in prepaid salaries indicates that salaries expense is less than the cash paid forsalaries. An increase in salaries payable indicates that salaries expense is more than the cash paid for salaries. Thus, theamount of cash payments for salaries was 166,000 (190,000 salaries expense + 6,000 increase in prepaid salaries – 30,000increase in salaries payable).

Answer (C) is incorrect because 214,000 subtracts the increase in prepaid salaries instead of adding it and adds the30,000 increase in salaries payable instead of subtracting it.

Answer (D) is incorrect because 226,000 adds the 30,000 increase in salaries payable instead of subtracting it.

[357] Gleim #: 3.4.65 -- Source: CIA 593 IV-44

Answer (A) is incorrect because the increase in prepaid expenses requires a deduction from profit in the reconciliation.

Answer (B) is incorrect because the increase in prepaid expenses requires a deduction from profit in the reconciliation.

Answer (C) is incorrect because amortization of premium on bonds payable requires a deduction from profit in thereconciliation.

Answer (D) is correct. An increase in prepaid expenses indicates that cash outlays for expenses exceeded the relatedexpense incurred; thus, profit exceeded net cash from operating activities, and a deduction is needed in the reconciliation.Also, the amortization of premium on bonds payable causes a reduction of interest expense but does not increase cash;therefore, profit exceeds net cash from operating activities, and a deduction is needed in the reconciliation.

[358] Gleim #: 3.4.66 -- Source: CIA 1196 IV-13

Answer (A) is correct. Cost of goods sold equals beginning inventory, plus purchases, minus ending inventory. Todetermine cost of goods sold, purchases must be calculated. Purchases equal 167,000 (49,000 ending accounts payable +180,000 payments to suppliers – 62,000 beginning accounts payable). Thus, cost of goods sold equals 147,000 (120,000beginning inventory + 167,000 purchases – 140,000 ending inventory).

Answer (B) is incorrect because 160,000 results from assuming that 180,000 of cash payments to suppliers equaledpurchases.

Answer (C) is incorrect because 167,000 equals purchases.

Answer (D) is incorrect because 180,000 is the amount of cash payments to suppliers.

[359] Gleim #: 3.4.67 -- Source: CIA 1196 IV-14

Answer (A) is correct. Cash collections from customers equals beginning accounts receivable, plus sales revenue, minusaccounts written off, minus ending accounts receivable. In Year 2, cash collections from customers were 341,000 (95,000+ 338,000 – 3,000 – 89,000).

Answer (B) is incorrect because 338,000 is the sales revenue for the year.

Answer (C) is incorrect because 344,000 includes the 3,000 of accounts written off.

Answer (D) is incorrect because 335,000 is sales revenue minus accounts written off.

Gleim's CIA Test Prep: Part III: Business Analysis and Information TechnologyAnswer Explanations

(1312 questions)

Copyright 2008 Gleim Publications, Inc. Page 106Printed for Mamdouh Farag

Page 107: P.3 Answer Explanation

[360] Gleim #: 3.4.68 -- Source: CIA 1196 IV-15

Answer (A) is correct. The cost of PPE disposed of is 20,000 (295,000 beginning PPE + 65,000 acquisitions – 340,000ending PPE). The accumulated depreciation is 13,000 (102,000 beginning accumulated depreciation + 30,000 depreciationexpense – 119,000 ending accumulated depreciation). Thus, the carrying amount of PPE disposed of is 7,000 (20,000 costof PPE – 13,000 accumulated depreciation).

Answer (B) is incorrect because 17,000 is the difference between ending and beginning accumulated depreciation.

Answer (C) is incorrect because 20,000 is the cost of the PPE disposed.

Answer (D) is incorrect because 32,000 results from using the change in the PPE account without acquisitions minus theaccumulated depreciation.

[361] Gleim #: 3.4.69 -- Source: CIA 1196 IV-16

Answer (A) is incorrect because 8,000 is the beginning interest payable.

Answer (B) is correct. The interest payable credited in Year 1 was 15,000 (20,000 interest expense – 5,000 amortizedbond discount). Thus, the cash interest payment was 12,000 (8,000 beginning interest payable + 15,000 interest payablecredited in Year 1 – 11,000 ending interest payable).

Answer (C) is incorrect because 20,000 is the interest expense for Year 1.

Answer (D) is incorrect because 25,000 is the interest expense plus the amortized discount.

[362] Gleim #: 3.4.70 -- Source: CIA 595 IV-2

Answer (A) is correct. To convert from the cash basis (cash payments) to the accrual basis (cost of goods sold), anincrease in accounts payable must be added to cash payments for goods to determine net purchases. Net purchases is thenadjusted for the change in inventory to determine cost of goods sold.

Answer (B) is incorrect because a decrease in accounts payable must be subtracted from, not added to, cash payments.

Answer (C) is incorrect because an increase in inventory must be subtracted from, not added to, cash payments tocalculate cost of goods sold.

Answer (D) is incorrect because a decrease in inventory must be added to, not subtracted from, cash payments to calculatecost of goods sold.

[363] Gleim #: 3.4.71 -- Source: CIA 1193 IV-34

Answer (A) is incorrect because the component is to be terminated (discontinued).

Answer (B) is correct. A separate major line of business or geographical operating area that is distinct for operational andreporting purposes is a component of an entity. Thus, its disposal qualifies to be reported.

Answer (C) is incorrect because extraordinary items are not reported in the financial statements or notes. They are notclearly distinct from ordinary activities.

Answer (D) is incorrect because disclosures begin with the current period, i.e., when the criteria for classification of theDO as held for sale are met.

Gleim's CIA Test Prep: Part III: Business Analysis and Information TechnologyAnswer Explanations

(1312 questions)

Copyright 2008 Gleim Publications, Inc. Page 107Printed for Mamdouh Farag

Page 108: P.3 Answer Explanation

[364] Gleim #: 3.4.72 -- Source: CIA 591 IV-38

Answer (A) is correct. Costs and expenses other than product costs should be either charged to income in interim periodsas incurred or allocated among interim periods based upon the benefits received. Accordingly, costs such as advertisingshould be deferred in an interim period if the benefits extend beyond that period; otherwise, they should be expensed asincurred. But such a determination is difficult, and deferral raises the additional issue of how the deferred costs should beallocated among quarters. Thus, many entities expense the costs as incurred even though they may benefit other interimperiods in the same annual period.

Answer (B) is incorrect because the only product costs appropriate to expense in an interim period are the ones related tothe revenue transactions recognized in the same interim period.

Answer (C) is incorrect because the annual depreciation amount is an estimate. The depreciation amount for an interimperiod is simply a pro rata amount of the annual estimate.

Answer (D) is incorrect because extraordinary items are not reported in interim or annual statements.

[365] Gleim #: 3.4.73 -- Source: CIA 1188 IV-33

Answer (A) is correct. Depreciation and amortization are noncash expenses and are added to profit. A decrease inreceivables indicates that cash collections exceed sales on an accrual basis, so it is added to profit. To account for thedifference between cost of goods sold (a deduction from profit) and cash paid to suppliers, a two-step adjustment of profitis necessary. The difference between CGS and purchases is the change in inventory. The difference between purchasesand the amount paid to suppliers is the change in accounts payable. Accordingly, the conversion of CGS to cash paid tosuppliers requires deducting the inventory increase and adding the accounts payable increase. An increase in plant assetsindicates an acquisition of plant assets, causing a decrease in cash, so it is deducted. An increase in share capitalrepresents a cash inflow and is added to profit. A decrease in short-term notes payable is deducted from profit because itreflects a cash outflow. Thus, cash increased by 11,000 (70,000 profit + 14,000 + 1,000 + 2,000 – 9,000 + 4,000 – 47,000+ 31,000 – 55,000).

Answer (B) is incorrect because 17,000 results from subtracting the amortization and the decrease in receivables andadding the increase in inventories.

Answer (C) is incorrect because 54,000 results from adjusting profit for the increase in plant assets and the increase inshare capital only.

Answer (D) is incorrect because 69,000 results from failing to make the adjustments for receivables, inventories, notespayable, and accounts payable.

[366] Gleim #: 3.4.74 -- Source: CIA 1192 IV-26

Answer (A) is incorrect because the economic entity assumption provides that economic activity can be identified with aparticular unit of accountability.

Answer (B) is incorrect because the monetary unit assumption provides that all transactions and events can be measuredin terms of a common denominator, for instance, the euro.

Answer (C) is incorrect because the materiality assumption simply implies that items of insignificant value may beexpensed rather than capitalized and depreciated or amortized. The difference in treatment is not large enough toinfluence users if the item is not material.

Answer (D) is correct. A basic feature of financial accounting is that the entity is assumed to be a going concern in theabsence of evidence to the contrary. The going concern concept is based on the empirical observation that many entitieshave an indefinite life. The reporting entity is assumed to have a life long enough to fulfill its objectives and commitmentsand therefore to depreciate wasting assets over their useful lives.

Gleim's CIA Test Prep: Part III: Business Analysis and Information TechnologyAnswer Explanations

(1312 questions)

Copyright 2008 Gleim Publications, Inc. Page 108Printed for Mamdouh Farag

Page 109: P.3 Answer Explanation

[367] Gleim #: 3.4.75 -- Source: CIA 590 IV-35

Answer (A) is incorrect because items described as extraordinary are not presented on the face of the income statement orin the notes.

Answer (B) is incorrect because items described as extraordinary are not presented on the face of the income statement orin the notes.

Answer (C) is incorrect because items described as extraordinary are not presented on the face of the income statement orin the notes.

Answer (D) is correct. According to IAS 1, “An entity shall not present any items of income and expense as extraordinaryitems, either on the face of the income statement or in the notes.”

[368] Gleim #: 3.4.76 -- Source: CIA 1191 IV-42

Answer (A) is correct. Gains and losses similar to those that would not be deferred at year-end should not be deferred tolater interim periods of the same year. Hence, the extraordinary gain should not be prorated. However, no items of incomeor expense are presented as extraordinary on the income statement or in the notes.

Answer (B) is incorrect because the gain should be recognized in full in the second quarter.

Answer (C) is incorrect because the gain should be recognized in full in the second quarter.

Answer (D) is incorrect because the gain should be recognized in full in the second quarter.

[369] Gleim #: 3.4.77 -- Source: CMA 692 1-26

Answer (A) is incorrect because the cash flow for the period is greater than profit given noncash expenses in the form ofdepreciation and bond discount amortization.

Answer (B) is incorrect because 66,000 does not reflect the noncash expense for depreciation.

Answer (C) is incorrect because the 5,000 of depreciation and the 6,000 for amortization should be added back to, notsubtracted from, income.

Answer (D) is correct. To determine cash flow for the period, all noncash expenses should be added back to profit.Adding the 5,000 of depreciation and the 6,000 of discount amortization to the 60,000 of profit produces a cash flow of71,000.

[370] Gleim #: 3.4.78 -- Source: CIA 1192 IV-30

Answer (A) is incorrect because the statement of income is prepared on an accrual basis and is not meant to report cashflows.

Answer (B) is incorrect because the statement of changes in equity is prepared on the accrual basis.

Answer (C) is correct. A statement of cash flows provides information about the cash receipts and cash payments of anenterprise during a period. This information helps investors, creditors, and other users to assess the enterprise’s ability togenerate cash and cash equivalents and the needs of the enterprise to use those cash flows. Historical cash flow dataindicate the amount, timing, and certainty of future cash flows. It is also a means of verifying past cash flow assessmentsand of determining the relationship between profits and net cash flows and the effects of changing prices.

Answer (D) is incorrect because the balance sheet reports on financial position at a moment in time.

Gleim's CIA Test Prep: Part III: Business Analysis and Information TechnologyAnswer Explanations

(1312 questions)

Copyright 2008 Gleim Publications, Inc. Page 109Printed for Mamdouh Farag

Page 110: P.3 Answer Explanation

[371] Gleim #: 3.4.79 -- Source: CIA 1191 IV-32

Answer (A) is incorrect because depreciation does not involve an inflow or outflow of cash. The purchase and the sale ofproperty, plant, and equipment constitute investing activities, but the process of depreciating such assets is not defined asan investing activity. Depreciation is a noncash operating expense.

Answer (B) is incorrect because depreciation is a noncash operating expense.

Answer (C) is correct. Net cash flow from (used by) operating activities may be reported indirectly by removing fromprofit or loss the effects of (1) deferrals of past operating cash flows, (2) accruals of expected future operating cash flows,(3) income or expense items related to financing and investing cash flows, and (4) noncash transactions. Theseadjustments include such items as depreciation, amortization of patents, amortization of bond discount and bond premium,gains and losses, changes during the period in trade receivables, changes in inventory, and changes in accounts payableand accrued liabilities. In the reconciliation of profit or loss to net cash from (used by) operations, depreciation (a noncashexpense) is added to profit or loss.

Answer (D) is incorrect because depreciation should be added.

[372] Gleim #: 3.4.80 -- Source: CIA 592 IV-35

Answer (A) is incorrect because the balance sheet does not include periodic profit or loss or depreciation expense.

Answer (B) is incorrect because the income statement does not have captions for operating and financing activities.

Answer (C) is correct. A statement of cash flows is a required financial statement. It provides information about cashreceipts and payments by reporting the cash effects of an enterprise’s operating, investing, and financing activities.Related disclosures report the effects of noncash investing and financing activities. If the statement is presented using theindirect method, it will reconcile profit or loss to net cash from (used in) operating activities. Depreciation, a noncashexpense, is included in this presentation.

Answer (D) is incorrect because the statement of changes in equity does not include captions for operating and investingactivities and depreciation.

[373] Gleim #: 3.4.81 -- Source: CIA 592 IV-36

Answer (A) is incorrect because a fundamental error is no longer recognized under IFRSs.

Answer (B) is incorrect because extraordinary items are not recognized.

Answer (C) is correct. A discontinued operation (DO) is a component of an entity that has been disposed of or meets thecriteria for classification as held for sale. It is (1) a separate major line of business or geographical operating area, (2) partof a single plan to dispose of such a line or area, or (3) a subsidiary acquired solely for resale. A component of an entityconsists of operations and cash flows that are clearly distinguishable from the rest of the entity for financial reporting aswell as operationally. A single amount is disclosed on the face of the income statement equal to the sum of (1) after-taxprofit (loss) on DOs and (2) after-tax gain (loss) on (a) remeasurement of noncurrent assets (or disposal groups) classifiedas held for sale at fair value minus cost to sell or (b) disposal of the assets or disposal groups that constituted the DO.

Answer (D) is incorrect because accounting policies guide the preparation and presentation of financial statements.

Gleim's CIA Test Prep: Part III: Business Analysis and Information TechnologyAnswer Explanations

(1312 questions)

Copyright 2008 Gleim Publications, Inc. Page 110Printed for Mamdouh Farag

Page 111: P.3 Answer Explanation

[374] Gleim #: 3.4.82 -- Source: CIA 592 IV-34

Answer (A) is incorrect because 33 is the profit effect excluding interest revenue of 14 [.01 × (1,500 – 100)].

Answer (B) is correct. The gross profit margin is 33 1/3% [(1,500 – 1,000) ÷ 1,500], so the amount of profit from the 100down payment recognizable in Year 1 is 33 (rounded). Interest accrued on the 1,400 (1,500 – 100) balance for 1 month is14. Consequently, the effect on profit is 47 (33 + 14).

Answer (C) is incorrect because 67 is the cost of goods sold [(1,000 ÷ 1,500) × 100].

Answer (D) is incorrect because 114 {100 + [.01 × (1,500 – 100)]} is the total revenue, not the profit.

[375] Gleim #: 3.4.83 -- Source: CIA 1189 IV-9

Answer (A) is incorrect because the expenses relating to the current year need to be recognized.

Answer (B) is correct. The 72,000 rental fee should be recognized as expense evenly over the 36-month duration of thelease. In Year 3, 22,000 should be debited to current expenses (11 months × 72,000 ÷ 36), and 50,000 should be deferredas prepaid expense.

Answer (C) is incorrect because 24,000 is the expense recognized for 12 months instead of the 11 months from January31, Year 3 to December 31, Year 3.

Answer (D) is incorrect because only those expenditures related to the current year should be expensed.

[376] Gleim #: 3.4.84 -- Source: CIA 1194 IV-27

Answer (A) is incorrect because the balance sheet approach estimates bad debts as a portion of overdue accountsestimated to be uncollectible. Under this approach, the amount estimated to be uncollectible is 50,000 (100,000 × 50%).

Answer (B) is incorrect because the balance sheet approach estimates bad debts as a portion of overdue accountsestimated to be uncollectible. Under this approach, the amount estimated to be uncollectible is 50,000 (100,000 × 50%).

Answer (C) is correct. Using the income statement approach, the bad debt expense is determined using a percentage oftotal credit sales. Thus, bad debt expense is 20,000 (400,000 credit sales × 5% estimated bad debt rate).

Answer (D) is incorrect because 50,000 is the estimated bad debt expense using the balance sheet approach.

[377] Gleim #: 3.4.85 -- Source: CIA 591 IV-52

Answer (A) is incorrect because 92,400 equals the increase in after-tax profit.

Answer (B) is incorrect because 140,000 is the increase in pre-tax profit.

Answer (C) is correct. The increase in pre-tax profit is 140,000 (400,000 cash sales increase – 180,000 nondepreciationexpenses increase – 80,000 depreciation). Thus, taxes will increase by 47,600 (34% × 140,000), and the increase in netcash inflows will be 172,400 (400,000 – 180,000 – 47,600).

Answer (D) is incorrect because 220,000 equals cash sales minus expenses other than depreciation.

Gleim's CIA Test Prep: Part III: Business Analysis and Information TechnologyAnswer Explanations

(1312 questions)

Copyright 2008 Gleim Publications, Inc. Page 111Printed for Mamdouh Farag

Page 112: P.3 Answer Explanation

[378] Gleim #: 3.4.86 -- Source: CIA 590 IV-55

Answer (A) is incorrect because 8 million includes next year’s payments on this year’s sales in the estimation of thisyear’s cash inflows.

Answer (B) is correct. The cash inflows from last year’s credit sales are estimated to be 3,000,000 (10% × 30,000,000).The cash inflows from this year’s sales are expected to be 36,000,000 (90% × 40,000,000), a total cash inflow of39,000,000 for the current year. Ignoring depreciation, which is a noncash expense, cash outflows are estimated at51,000,000. Hence, the net cash outflow is anticipated to be 12,000,000 (39,000,000 – 51,000,000).

Answer (C) is incorrect because 15 million excludes the 3 million cash inflow that resulted from last year’s credit sales.

Answer (D) is incorrect because 17 million includes depreciation when estimating cash outflows.

[379] Gleim #: 3.4.87 -- Source: CIA 597 IV-32

Answer (A) is incorrect because a description of the company’s primary business and segments, a discussion of thecompany’s operating results, and a discussion of the future prospects of the company are included.

Answer (B) is incorrect because a description of the company’s primary business and segments, a discussion of thecompany’s operating results, and a discussion of the future prospects of the company are included.

Answer (C) is incorrect because a description of the company’s primary business and segments, a discussion of thecompany’s operating results, and a discussion of the future prospects of the company are included.

Answer (D) is correct. The MD&A section addresses capital resources, liquidity, and operating results. Managementmust also identify trends and discuss significant events and uncertainties. Thus, the MD&A section typically includes adescription of the company’s primary business and segments. It reviews the operating results of the company, providing abreakdown of net sales and income by segment. It also contains prospective information on economic trends and marketchanges, and their potential effects on the company’s future performance.

[380] Gleim #: 3.4.88 -- Source: Publisher

Answer (A) is correct. Essentially the same reporting methods should be used for interim and annual financialstatements. However, the preparation of interim financial reports ordinarily requires a greater use of estimates. Forexample, IAS 34 states that complete inventory-taking and valuation may not be required at interim dates. Estimatesbased on sales margins may suffice.

Answer (B) is incorrect because income tax expense is based on the estimated annual effective income tax rate.

Answer (C) is incorrect because the same accounting policies should be applied in the interim statements as in the annualstatements. Accordingly, an extraordinary item is to be reported in the interim period in which the gain or loss occurred.

Answer (D) is incorrect because the same accounting policies should be applied in the interim statements as in the annualstatements. Thus, BEPS and DEPS must be reported.

Gleim's CIA Test Prep: Part III: Business Analysis and Information TechnologyAnswer Explanations

(1312 questions)

Copyright 2008 Gleim Publications, Inc. Page 112Printed for Mamdouh Farag

Page 113: P.3 Answer Explanation

[381] Gleim #: 3.4.89 -- Source: CIA 594 IV-1

Answer (A) is incorrect because the balance of accumulated depreciation is higher in the tax-basis financial statements.

Answer (B) is incorrect because depreciation expense is a noncash charge. The cash balance is unaffected by thedepreciation method used.

Answer (C) is correct. Because the tax basis uses an accelerated method, depreciation expense and accumulateddepreciation will be greater. Moreover, taxable income will be lower than financial net income. Consequently, tax-basisretained earnings will be less than that in the general purpose financial statements.

Answer (D) is incorrect because the historical cost of property, plant, and equipment is recorded in the gross property,plant, and equipment account. This amount is unaffected by depreciation.

[382] Gleim #: 3.4.90 -- Source: CIA 594 IV-5

Answer (A) is incorrect because 800,000 uses the beginning balance of inventory.

Answer (B) is correct. The year-end total assets can be determined by summing all of the assets and deductingaccumulated depreciation (including the current year’s depreciation). Total accumulated depreciation at the end of thesecond year is 120,000 [(600,000 ÷ 10 years) × 2 years]. Total assets equal 890,000 (80,000 cash + 100,000 A/R +230,000 EI + 600,000 gross property, plant, and equipment – 120,000 accumulated depreciation).

Answer (C) is incorrect because 950,000 omits second-year depreciation from the calculation.

Answer (D) is incorrect because 1,010,000 omits total accumulated depreciation from the calculation.

[383] Gleim #: 3.4.91 -- Source: CIA 594 IV-4

Answer (A) is incorrect because the debt has been outstanding for only 6 months, so accrued interest is only 50,000.

Answer (B) is incorrect because the debt pays annual interest on July 1, and no cash outlay is required at year-end.

Answer (C) is incorrect because accrued interest is 50,000. Also, interest expense is debited and interest payable iscredited.

Answer (D) is correct. The debt was issued on July 1 and has been outstanding for only 6 months. Interest expense equalsthe face amount of the debt multiplied by the interest rate and the fraction of the year the debt was outstanding [1,000,000× 10% × (6 ÷ 12) = 50,000]. Because interest is payable on July 1, 6 months’ interest is accrued and expensed in thecurrent period. The payable is also recognized in the current period. Thus, the adjusting entry should be

Interest expense 50,000Interest payable 50,000

Gleim's CIA Test Prep: Part III: Business Analysis and Information TechnologyAnswer Explanations

(1312 questions)

Copyright 2008 Gleim Publications, Inc. Page 113Printed for Mamdouh Farag

Page 114: P.3 Answer Explanation

[384] Gleim #: 3.4.92 -- Source: CIA 594 IV-6

Answer (A) is correct. Current period pretax net income equals 280,000 (750,000 sales – 200,000 CGS – 60,000depreciation – 10,000 interest – 200,000 administrative expenses). Thus, after-tax net income credited to retainedearnings equals 140,000 [(1.0 – .5) × 280,000].

Answer (B) is incorrect because income taxes are omitted and the journal entry is reversed.

Answer (C) is incorrect because administrative expenses were omitted.

Answer (D) is incorrect because administrative expenses were omitted and the journal entry is reversed.

[385] Gleim #: 3.4.93 -- Source: CIA 1191 IV-14

Answer (A) is incorrect because the entity will need to borrow 6,000.

Answer (B) is correct. This entity will collect 80,000 (50,000 from September credit sales + 30,000 from October cashsales) in October. To reach its targeted cash balance on October 31, it will have to borrow 6,000.

September 30 cash balance 23,000Collections 80,000Disbursements (94,000) (14,000)Ending balance 9,000Necessary borrowing 6,000Targeted ending balance 15,000

Answer (C) is incorrect because the entity will need to borrow 6,000.

Answer (D) is incorrect because the entity will need to borrow 6,000.

[386] Gleim #: 3.4.94 -- Source: Publisher

Answer (A) is incorrect because the write-down is expensed in the period in which the writedown occurs.

Answer (B) is incorrect because the loss is ordinary.

Answer (C) is correct. If a separate expense account is not used, the ending inventory will be reduced directly and theresult will be an increase in cost of goods sold. No separate disclosure of the inventory writedown will appear in theincome statement. The effect is to hide the loss in cost of goods sold. If the separate expense account is used, it appears onthe income statement as a deduction from gross profit (sales – cost of goods sold). One advantage is that cost of goods soldis not misstated.

Answer (D) is incorrect because the writedown is not treated as a selling or administrative expense.

Gleim's CIA Test Prep: Part III: Business Analysis and Information TechnologyAnswer Explanations

(1312 questions)

Copyright 2008 Gleim Publications, Inc. Page 114Printed for Mamdouh Farag

Page 115: P.3 Answer Explanation

[387] Gleim #: 3.4.95 -- Source: CIA 1195 IV-12

Answer (A) is incorrect because 891,667 uses a gross profit percentage of 40%.

Answer (B) is incorrect because 1,100,000 calculates profit before tax as profit divided by the tax rate and also calculatessales as gross profit divided by the tax rate.

Answer (C) is correct. Gross profit equals 30% of sales. Thus, dividing gross profit by 30% yields sales. Gross profitequals profit before interest and tax (PBIT) plus administrative expense (given as 40,000) and depreciation (500,000 fixedassets ÷ 10 years = 50,000). Interest equals 10% of the long-term debt, or 100,000 (10% × 1,000,000). Profit before taxequals 166,667 [100,000 profit ÷ (1.0 – 40% tax rate)]. Hence, PBIT must equal 266,667 (166,667 + 100,000), and grossprofit must be 356,667 (266,667 PBIT + 40,000 administrative expense + 50,000 depreciation). Sales is therefore1,118,890 (356,667 gross profit ÷ 30% gross profit rate).

Answer (D) is incorrect because 1,466,667 calculates profit before tax as profit divided by the tax rate.

[388] Gleim #: 3.4.96 -- Source: CIA 1195 IV-13

Answer (A) is incorrect because 1,000,000 equals working capital minus long-term debt.

Answer (B) is correct. Return on total assets (3%) equals profit (100,000) divided by total assets. Consequently, totalassets equals 3,333,333 (100,000 ÷ .03). Current assets must therefore equal 3,033,333 (3,333,333 total assets – 300,000other assets).

Answer (C) is incorrect because 3,333,333 is the amount of total assets.

Answer (D) is incorrect because 3,633,333 is the sum of total and other assets.

[389] Gleim #: 3.4.97 -- Source: CMA 1290 1-30

Answer (A) is incorrect because after-tax profit will increase by 6,000.

Answer (B) is correct. Manufacturing and selling costs exclusive of bad debts equal 70% of sales. Hence, the gross profiton the 100,000 increase in sales will be 30,000 (30% × 100,000). Assuming 15,000 of bad debts and 5,000 of collectionexpense, the increase in pre-tax profit will be 10,000 (30,000 – 20,000). Consequently, after-tax profit will increase by6,000 [10,000 – (40% × 10,000)].

Answer (C) is incorrect because after-tax profit will increase by 6,000.

Answer (D) is incorrect because after-tax profit will increase by 6,000.

Gleim's CIA Test Prep: Part III: Business Analysis and Information TechnologyAnswer Explanations

(1312 questions)

Copyright 2008 Gleim Publications, Inc. Page 115Printed for Mamdouh Farag

Page 116: P.3 Answer Explanation

[390] Gleim #: 3.4.98 -- Source: CIA 1196 IV-37

Answer (A) is correct. Cash provided by operations equals profit, plus depreciation, minus the increase in accountsreceivable, minus the decrease in accounts payable. The cash provided is 219,000 [294,000 + 50,000 – (300,000 –200,000) – (275,000 – 250,000)]. An increase in receivables is a noncash component of profit. A decrease in accountspayable is added when adjusting cost of goods sold to reflect cash paid to suppliers. Thus, it is subtracted when adjustingprofit to arrive at cash provided by operations.

Answer (B) is incorrect because 244,000 does not consider the changes in accounts receivable and payable and subtractsdepreciation.

Answer (C) is incorrect because 344,000 does not consider the changes in accounts receivable and payable.

Answer (D) is incorrect because 469,000 adds rather than subtracts the increase in receivables and the decrease inpayables.

[391] Gleim #: 3.4.99 -- Source: CIA 1196 IV-38

Answer (A) is incorrect because 49 days uses accounts receivable from Year 1 instead of Year 2.

Answer (B) is incorrect because 52 days is the average collection period for Year 1.

Answer (C) is correct. The average collection period for Year 2 is 73 days [300,000 accounts receivable ÷ (1,500,000sales ÷ 365 days)].

Answer (D) is incorrect because 78 days uses Year 1 annual sales.

[392] Gleim #: 3.5.100 -- Source: CIA 1191 IV-34

Answer (A) is incorrect because the net method requires a sales discount forfeited but not a sales discount account.

Answer (B) is incorrect because the net method requires a sales discount forfeited but not a sales discount account.

Answer (C) is incorrect because the net method requires a sales discount forfeited.

Answer (D) is correct. The gross method accounts for receivables at their face amount. If a discount is taken, a salesdiscount is recorded and classified as an offset to sales in the income statement to yield net sales. The net method recordsreceivables net of the applicable discount. If the payment is not received during the discount period, an interest revenueaccount, such as sales discounts forfeited, is credited at the end of the discount period or when the payment is received.Accordingly, the application of the net method requires a sales discount forfeited but not a sales discount account.

Gleim's CIA Test Prep: Part III: Business Analysis and Information TechnologyAnswer Explanations

(1312 questions)

Copyright 2008 Gleim Publications, Inc. Page 116Printed for Mamdouh Farag

Page 117: P.3 Answer Explanation

[393] Gleim #: 3.5.101 -- Source: CIA 1193 IV-41

Answer (A) is incorrect because 67,000 results from subtracting the write-offs and the bad debt expense from the sum ofnet income and beginning net accounts receivable.

Answer (B) is incorrect because 68,500 assumes a zero balance in the beginning allowance account and deducts bad debtexpense from the sum of profit and beginning net accounts receivable.

Answer (C) is incorrect because 68,000 deducts bad debt expense from the sum of profit and beginning net accountsreceivable.

Answer (D) is correct. The cash collected equals net sales adjusted for the change in net accounts receivable (gross A/R –allowance for bad debts). An increase in net accounts receivable implies that cash collected was less than net sales.Hence, cash collected was 70,000 (100,000 – 30,000 increase in net A/R). Write-offs (debit the allowance, credit A/R) donot affect the computation of cash collected because the allowance and gross accounts receivable are reduced by the sameamount. Moreover, recognition of bad debt expense (debit bad debt expense, credit the allowance) is not included in thiscalculation because it is already reflected in the net accounts receivable balance.

[394] Gleim #: 3.5.102 -- Source: CIA 1196 IV-33

Answer (A) is incorrect because 1,200 results from subtracting the recoveries instead of adding them.

Answer (B) is incorrect because 1,800 results from subtracting bad debt expense from the allowance account.

Answer (C) is correct. Under the allowance method, uncollectible accounts are written off by a debit to the allowanceaccount and a credit to accounts receivable. The 500 of recovered bad debts is accounted for by a debit to accountsreceivable and a credit to the allowance account. The 2,000 bad debt expense is also credited to the allowance account.The amount of accounts receivable written off as uncollectible is 2,200 [5,000 ending allowance – (4,700 beginningallowance + 500 recoveries + 2,000 bad debt expense)].

Answer (D) is incorrect because 2,800 results from subtracting the recoveries and bad debt expense from the allowanceaccount.

[395] Gleim #: 3.5.103 -- Source: CIA 1196 IV-42

Answer (A) is incorrect because 22.86% is the proportion of receivables in the 0-to-30-day age group at the end ofSeptember.

Answer (B) is incorrect because 28.57% is the proportion of receivables in the 61-to-90-day age group at the end ofSeptember.

Answer (C) is correct. Receivables from August sales still outstanding at the end of September are in the 31-to-60-dayage group. This group represents 48.57% of total receivables [170 ÷ (100 + 170 + 80)].

Answer (D) is incorrect because 71.43% is the proportion of outstanding receivables that are from 0 to 60 days old at theend of September.

Gleim's CIA Test Prep: Part III: Business Analysis and Information TechnologyAnswer Explanations

(1312 questions)

Copyright 2008 Gleim Publications, Inc. Page 117Printed for Mamdouh Farag

Page 118: P.3 Answer Explanation

[396] Gleim #: 3.5.104 -- Source: CIA 596 IV-1

Answer (A) is correct. The rate of gross profit on Year 2 installment sales is 20% [(5,000 of Year 2 installment sales –4,000 cost of Year 2 installment sales) ÷ 5,000 of Year 2 installment sales].

Answer (B) is incorrect because 40% is the gross profit on Year 1 installment sales.

Answer (C) is incorrect because 50% is the gross profit on Year 3 installment sales.

Answer (D) is incorrect because 80% is the ratio of the cost of Year 2 installment sales to Year 2 installment sales.

[397] Gleim #: 3.5.105 -- Source: CIA 596 IV-2

Answer (A) is correct. In Year 1, cash receipts were 2,000 from Year 1 installment sales. The gross profit realized is thegross profit on the portion of sales for which payment has been received. This amount equals the Year 1 gross profitpercentage multiplied by the cash receipts, or 800 {[(10,000 – 6,000) ÷ 10,000] × 2,000}.

Answer (B) is incorrect because 2,000 is the amount of cash receipts during Year 1 on Year 1 installment sales.

Answer (C) is incorrect because 3,200 is the amount of the total gross profit on Year 1 installment sales that is deferred tofuture periods.

Answer (D) is incorrect because 4,000 is the total gross profit on Year 1 installment sales.

[398] Gleim #: 3.5.106 -- Source: CIA 596 IV-3

Answer (A) is incorrect because 2,000 is the realized gross profit on Year 3 sales.

Answer (B) is incorrect because 3,000 equals total receipts for Year 2 and Year 3 on Year 2 sales.

Answer (C) is correct. The total gross profit on Year 3 sales is 10,000 (20,000 sales – 10,000 cost), and the amountrealized is 2,000 {[(20,000 – 10,000) ÷ 20,000] × 4,000 of Year 3 cash receipts}. Accordingly, the amount deferred is8,000 (10,000 – 2,000).

Answer (D) is incorrect because 10,000 is the total gross profit on Year 3 sales.

[399] Gleim #: 3.5.107 -- Source: CIA 595 IV-12

Answer (A) is incorrect because, under the cost-recovery method, profit is recognized in the second year when cashpayments by the buyer exceed the seller’s cost of merchandise.

Answer (B) is incorrect because 5,000 is the profit to be recognized without consideration of the payment received in thefirst year.

Answer (C) is correct. The profit recognized in the second year equals the cumulative payments received minus theseller’s cost, or 15,000 [(10,000 + 45,000) – 40,000].

Answer (D) is incorrect because 45,000 is the payment received in the second year.

Gleim's CIA Test Prep: Part III: Business Analysis and Information TechnologyAnswer Explanations

(1312 questions)

Copyright 2008 Gleim Publications, Inc. Page 118Printed for Mamdouh Farag

Page 119: P.3 Answer Explanation

[400] Gleim #: 3.5.108 -- Source: CIA 1195 IV-15

Answer (A) is correct. One condition for recognition of revenue from the sale of goods is the transfer of the significantrisks and rewards of ownership. Retention of significant risk may occur when, for example, the buyer may rescind thepurchase for a reason stipulated in the contract, and the buyer is uncertain about the probability of return. However, if theentity can reliably estimate future returns and recognizes a liability for returns based on experience and other pertinentinformation, revenue may be recognized at the time of sale if the other conditions for revenue recognition are also met.

Answer (B) is incorrect because the risks and rewards of ownership must be transferred.

Answer (C) is incorrect because this contingency is an example of retention of significant risk.

Answer (D) is incorrect because returns may be material if they can be reliably estimated.

[401] Gleim #: 3.5.109 -- Source: CMA 1287 3-26

Answer (A) is incorrect because the proceeds of the sale are reduced by the fair value of the recourse obligation.

Answer (B) is incorrect because the proceeds of the sale are reduced by the fair value of the recourse obligation.

Answer (C) is incorrect because the proceeds of the sale are reduced by the fair value of the recourse obligation.

Answer (D) is correct. The entity derecognizes financial assets if it has transferred substantially all of the risks andrewards of ownership. Derecognition also is appropriate when the entity neither has transferred nor retained substantiallyall the risks and rewards of ownership, providing the entity does not have control. After derecognition, periodic profit orloss will include the difference between the carrying amount transferred and the proceeds, plus or minus any prioradjustment reflecting the fair value of the asset that had been reported in equity. If a new financial asset is created or anew financial liability is assumed, the calculation is adjusted for the fair value of the asset or liability. Thus, the proceedsof the sale are reduced by the fair value of the recourse obligation (a new financial liability). When the transfer does notmeet these criteria, the transfer is accounted for as a collateralized borrowing.

[402] Gleim #: 3.5.110 -- Source: Publisher

Answer (A) is incorrect because a financial asset should be derecognized despite the transferor’s right to reacquire it if theasset is readily obtainable in the market.

Answer (B) is incorrect because, when the transferee has the ability unilaterally to sell the entire asset to an unrelatedthird party without transfer restrictions, the transferor lacks control.

Answer (C) is incorrect because a financial asset should be derecognized despite the transferor’s option to reacquire it atfair value at the date of reacquisition.

Answer (D) is correct. An entity derecognizes a financial asset if it has transferred substantially all of the risks andrewards of ownership. Derecognition also is appropriate when the entity neither has transferred nor retained substantiallyall the risks and rewards of ownership, providing the entity does not have control. A transferor does not derecognize thefinancial asset if it is entitled and obligated to repurchase or redeem the asset, and the terms of this transaction in effectallow the transferee to obtain a lender’s return on the assets it receives in exchange for the transferred financial asset.

Gleim's CIA Test Prep: Part III: Business Analysis and Information TechnologyAnswer Explanations

(1312 questions)

Copyright 2008 Gleim Publications, Inc. Page 119Printed for Mamdouh Farag

Page 120: P.3 Answer Explanation

[403] Gleim #: 3.5.111 -- Source: Publisher

Answer (A) is correct. Financial assets or liabilities at fair value through profit or loss include those held for trading.Regardless of intent, a financial asset is held for trading if it is included in a portfolio with a recent pattern of short-termprofit taking. Derivatives also are deemed to be held for trading unless they are designated and effective as hedginginstruments.

Answer (B) is incorrect because held-to-maturity investments have fixed or determinable payments and a fixed maturity.Moreover, the entity must have a positive intent and ability to hold such investments to maturity.

Answer (C) is incorrect because loans and receivables are not derivatives.

Answer (D) is incorrect because available-for-sale financial assets are nonderivatives. They do not fall within one of theother classifications.

[404] Gleim #: 3.5.112 -- Source: Publisher

Answer (A) is incorrect because given that the hedge was fully effective, the 55,000 gain should be recognized in theperiod in which the forecasted transaction affects earnings.

Answer (B) is correct. A cash flow hedge is a hedge of an exposure to variability in the cash flows of a recognized assetor liability or a forecast transaction. The accounting treatment of a cash flow hedge varies for the effective and ineffectiveportions. The effective portion initially is reported directly in equity. It is reclassified into profit or loss when the forecasttransaction affects profit or loss. The ineffective portion is immediately included in profit or loss. This hedge has noineffective portion. Given that the sale occurred in Year 2, the 30,000 gain in Year 1 is recognized directly in equity inYear 1. It is reclassified and included in profit or loss in Year 2. Thus, Year 2 profit or loss includes the 30,000reclassified from equity and the 25,000 gain attributable to the increase in fair value in Year 2. Given that the hedge wasfully effective, the 55,000 gain should be recognized in the period in which the forecast transaction affects profit or loss.

Answer (C) is incorrect because given that the hedge was fully effective, the 55,000 gain should be recognized in theperiod in which the forecasted transaction affects earnings.

Answer (D) is incorrect because given that the hedge was fully effective, the 55,000 gain should be recognized in theperiod in which the forecasted transaction affects earnings.

[405] Gleim #: 3.5.113 -- Source: Publisher

Answer (A) is incorrect because the corporation should recognize profit or loss for period 1 of 2,000. The increase in fairvalue of the derivative exceeds the decrease in PV of the cash flows by 2,000. The derivative is adjusted to fair value by a50,000 debit, equity is credited for 48,000, and profit or loss is credited for 2,000.

Answer (B) is correct. The effective portion of a cash flow hedge of a forecast transaction is included in equity untilperiods in which the forecast transaction affects profit or loss. At the end of period 3, the net change in the hedgingderivative’s fair value is 16,000 (50,000 + 47,000 – 81,000), and the change in the PV of the expected cash flows on theforecast transaction is –19,000 (80,000 – 48,000 – 51,000). Thus, the hedge is effective at the end of period 3 to the extentit offsets 16,000 of the net 19,000 decrease in the cash flows of the forecast transaction that are expected to occur inperiod 4.

Answer (C) is incorrect because the entry for period 2 is to debit the derivative for 47,000, debit profit or loss for 2,000,and credit equity for 49,000 (50,000 + 47,000 – 48,000 credit in period 1).

Answer (D) is incorrect because, at the end of period 2, equity should have a credit balance of 97,000 (the extent of thehedge’s effectiveness).

Gleim's CIA Test Prep: Part III: Business Analysis and Information TechnologyAnswer Explanations

(1312 questions)

Copyright 2008 Gleim Publications, Inc. Page 120Printed for Mamdouh Farag

Page 121: P.3 Answer Explanation

[406] Gleim #: 3.5.114 -- Source: Publisher

Answer (A) is incorrect because available-for-sale and held-for-trading financial assets are measured at fair value unlessthey do not have quoted market prices in an active market and their fair values cannot be reliably measured.

Answer (B) is incorrect because available-for-sale and held-for-trading financial assets are measured at fair value unlessthey do not have quoted market prices in an active market and their fair values cannot be reliably measured.

Answer (C) is correct. Subsequent measurement of financial assets is normally at fair value. Thus, derivative, available-for-sale, and held-for-trading financial assets are measured at fair value. Loans and receivables and held-to-maturityinvestments are measured at amortized cost using the effective interest rate method. Unquoted equity instruments whosefair value is not reliably measurable are reported at cost.

Answer (D) is incorrect because available-for-sale and held-for-trading financial assets are measured at fair value unlessthey do not have quoted market prices in an active market and their fair values cannot be reliably measured.

[407] Gleim #: 3.5.115 -- Source: Publisher

Answer (A) is incorrect because a gain or loss on a financial asset or liability classified as at fair value through profit orloss is recognized in profit or loss.

Answer (B) is correct. A gain or loss on an available-for-sale financial asset that is not part of a hedge is recognizeddirectly in equity through the statement of changes in equity except for impairment losses and foreign exchange gains andlosses. The accumulated remeasurement gain or loss is included in profit or loss when the asset is derecognized.

Answer (C) is incorrect because a gain or loss on an available-for-sale financial asset is normally recorded directly inequity.

Answer (D) is incorrect because a gain or loss on an available-for-sale financial asset is normally recorded directly inequity.

[408] Gleim #: 3.5.116 -- Source: CIA 1191 IV-39

Answer (A) is incorrect because the excess of the reacquisition price over the net carrying amount of the old bonds isrecognized in full in profit or loss for the period.

Answer (B) is incorrect because the excess of the reacquisition price over the net carrying amount of the old bonds isrecognized in full in profit or loss for the period.

Answer (C) is incorrect because an item of profit or loss is never classified as extraordinary.

Answer (D) is correct. An entity should remove a financial liability from its balance sheet only when it is extinguished,for example, when the creditor is paid. The difference between the carrying amount and the amount paid should beincluded in profit or loss for the period.

Gleim's CIA Test Prep: Part III: Business Analysis and Information TechnologyAnswer Explanations

(1312 questions)

Copyright 2008 Gleim Publications, Inc. Page 121Printed for Mamdouh Farag

Page 122: P.3 Answer Explanation

[409] Gleim #: 3.5.117 -- Source: Publisher

Answer (A) is incorrect because payment to a third party such as a trust (also known as an in-substance defeasance) doesnot by itself extinguish the obligation absent a legal release.

Answer (B) is incorrect because the terms should be substantially different.

Answer (C) is correct. Derecognition of a financial liability (or a part) occurs only by means of extinguishment. Thiscondition is satisfied only when the debtor pays the creditor or is legally released from primary responsibility either by thecreditor or through the legal process. An extinguishment and derecognition of the old debt and recognition of new debtoccurs when the borrower and lender exchange debt instruments with substantially different terms, that is, when therespective discounted cash flows differ by at least 10%.

Answer (D) is incorrect because payment to a third party such as a trust (also known as an in-substance defeasance) doesnot by itself extinguish the obligation absent a legal release.

[410] Gleim #: 3.5.118 -- Source: CIA 1192 IV-44

Answer (A) is incorrect because the liability and receivables should not be increased by the 10% interest rate.

Answer (B) is incorrect because the receivables should not be increased by the 10% interest rate.

Answer (C) is incorrect because the liability should not be increased by the 10% interest rate.

Answer (D) is correct. An entity may derecognize a financial liability when it is extinguished. This condition is met whenthe debtor discharges the debt by paying the creditor, such as with cash, other financial assets (e.g., receivables), goods, orservices. Consequently, the liability should be debited for its 100,000 balance. Receivables with a 90,000 balance aregiven up, so that account should be credited. The difference is a gain.

[411] Gleim #: 3.5.119 -- Source: Publisher

Answer (A) is incorrect because 30,000 is the difference between the sum of the existing liabilities and the face amount ofthe note with modified terms. Moreover, a gain should be recognized.

Answer (B) is incorrect because 30,000 is the difference between the sum of the existing liabilities and the face amount ofthe note with modified terms. Moreover, a gain should be recognized.

Answer (C) is correct. Derecognition of a financial liability (or a part) occurs only by means of extinguishment. Thiscondition is satisfied only when the debtor pays the creditor or is legally released from primary responsibility either by thecreditor or through the legal process. An extinguishment and derecognition of the old debt and recognition of new debtoccurs when the borrower and lender exchange debt instruments with substantially different terms, that is, when therespective discounted cash flows differ by at least 10%. A substantial modification of terms is also accounted for as anextinguishment. The difference between the carrying amount (including unamortized costs) of a liability (or part) that hasbeen extinguished or transferred and the amount paid is included in profit or loss. This transaction qualifies as anextinguishment based on a substantial modification of terms because the discounted cash flow from the old debt (130,000due immediately) and the new debt (given as 84,018) differ by at least 10%. Hence, the amount included by Debtor inprofit or loss at the date of the restructuring is a 45,982 gain (130,000 – 84,018), that is, the difference between thecarrying amount extinguished and the amount paid (the present value of the new debt instrument determined bydiscounting the cash outflows at the prevailing rate for similar instruments of an issuer with a similar credit rating). Theentry is to debit the extinguished liability for accrued interest and principal (130,000), debit discount on note payable(15,982), credit note payable (100,000), and credit gain (45,982).

Answer (D) is incorrect because the terms were substantially different. Thus, an extinguishment occurred.

Gleim's CIA Test Prep: Part III: Business Analysis and Information TechnologyAnswer Explanations

(1312 questions)

Copyright 2008 Gleim Publications, Inc. Page 122Printed for Mamdouh Farag

Page 123: P.3 Answer Explanation

[412] Gleim #: 3.5.120 -- Source: CIA 597 IV-19

Answer (A) is incorrect because the carrying amount should be reduced, and the charge should be included in profit orloss.

Answer (B) is incorrect because the carrying amount should be reduced, and the charge should be included in profit orloss.

Answer (C) is incorrect because the carrying amount should be reduced, and the charge should be included in profit orloss.

Answer (D) is correct. A financial asset, such as a lender’s mortgage receivable, is impaired if its carrying amount at thebalance sheet date exceeds its estimated recoverable amount. If it is probable that all amounts due on such a held-to-maturity investment (a financial asset carried at amortized cost) cannot be collected, the loss should be included in profitor loss. The loss equals the difference between the carrying amount and the present value of the expected future cashflows discounted at the original effective interest rate. The carrying amount of the asset should be reduced to its estimatedrecoverable amount directly or by crediting an allowance account.

[413] Gleim #: 3.5.121 -- Source: Publisher

Answer (A) is incorrect because the contract meets the definition of a firm commitment. Thus, it cannot be a forecasttransaction.

Answer (B) is incorrect because the contract meets the definition of a firm commitment. Thus, it cannot be a forecasttransaction.

Answer (C) is correct. A firm commitment is a binding agreement for the exchange of a specified quantity of resources ata specified price on a specified future date(s). A forecast transaction is an uncommitted but anticipated future transaction(IAS 39).

Answer (D) is incorrect because the contract meets the definition of a firm commitment. Thus, it cannot be a forecasttransaction.

[414] Gleim #: 3.5.122 -- Source: Publisher

Answer (A) is incorrect because 10,000 and 40,000 reflect the undiscounted changes in the spot rate from 11/15/Yr 1 to12/31/Year 1 and from 11/15/Yr 1 to 02/15/Yr 2, respectively.

Answer (B) is correct. This hedge is a foreign currency fair value hedge because it hedges a foreign currency exposure ofan unrecognized firm commitment. IAS 39 requires recognition of the forward contract receivable as an asset at fair value,with the changes in fair value recognized in profit or loss. IAS 39 requires recognition of the changes in the fair value ofthe firm commitment that are attributable to the changes in exchange rates. The changes in fair value are recognized inprofit or loss and as entries to a liability. Fair values should reflect changes in the forward exchange rates on a net-present-value basis. Thus, the forward contract receivable should be debited and a gain credited for 19,600 at 12/31/Yr 1.A loss should be debited and a firm commitment liability should be credited in the same amount at the same date. At2/15/Yr 2, a further 10,400 forward contract gain and firm commitment loss should be recorded. Because the changes invalue of both the forward-exchange contract and the U.S. dollar equivalent of the firm commitment are based on changesin forward rates, the hedge is completely effective. The changes in fair values (19,600 and 10,400) of the forward contractreceivable (gains) and the firm commitment (losses) offset each other in the income statement.

Answer (C) is incorrect because 19,600 and 10,400 reflect the income statement effects.

Answer (D) is incorrect because 20,000 and 30,000 reflect the undiscounted changes in the forward rates.

Gleim's CIA Test Prep: Part III: Business Analysis and Information TechnologyAnswer Explanations

(1312 questions)

Copyright 2008 Gleim Publications, Inc. Page 123Printed for Mamdouh Farag

Page 124: P.3 Answer Explanation

[415] Gleim #: 3.5.123 -- Source: Publisher

Answer (A) is incorrect because 350,000 is the amount that should be recognized if the equipment had been delivered on11/15/Yr 1.

Answer (B) is correct. The equipment should be recorded at 360,000. This amount equals 390,000 (FC1,000,000 x 0.39spot rate at 2/15/Yr 2) minus the 30,000 balance in the firm commitment liability account. The entry is to debit equipmentfor 360,000, debit the firm commitment liability for 30,000, and credit a payable for 390,000. On the same date, Hectorwill debit the payable for 390,000, credit the forward contract receivable for 30,000, and credit cash for 360,000. Thelatter entry reflects settlement of the payable and of the forward contract.

Answer (C) is incorrect because 390,000 is the amount that should be recognized if the firm commitment had not beenhedged.

Answer (D) is incorrect because 420,000 equals 390,000 plus the 30,000 balance in the firm commitment liabilityaccount.

[416] Gleim #: 3.5.124 -- Source: CMA 683 1-13

Answer (A) is incorrect because the proceeds would be determined by reducing the 110,000 by 9.5% (8% reserve + 1.5%commission) and then reducing that amount by the interest expense [16% × (60 ÷ 360)].

Answer (B) is incorrect because the proceeds would be determined by reducing the 110,000 by 9.5% (8% reserve + 1.5%commission) and then reducing that amount by the interest expense [16% × (60 ÷ 360)].

Answer (C) is correct. The factor will hold out 8,800 (8% × 110,000) as a reserve against returns and allowances and1,650 (1.5% × 110,000) as a commission. That leaves 99,550 to be advanced to the seller. However, interest at the rate of16% annually is also to be withheld. For 60 days that interest would amount to approximately 2,655 (assuming a 360-dayyear). The proceeds to be given to the seller equal 96,895 (99,550 – 2,655).

Answer (D) is incorrect because the proceeds would be determined by reducing the 110,000 by 9.5% (8% reserve + 1.5%commission) and then reducing that amount by the interest expense [16% × (60 ÷ 360)].

[417] Gleim #: 3.5.125 -- Source: Publisher

Answer (A) is incorrect because initial measurement of financial assets and liabilities is at fair value. Subsequentmeasurement of assets or liabilities held for trading and of certain other designated items is at fair value through profit orloss.

Answer (B) is incorrect because unquoted equity instruments whose fair value is not reliably measurable are reported atcost.

Answer (C) is correct. According to IAS 39, subsequent measurement of financial assets (including derivatives that areassets) is at fair value with certain exceptions. Loans and receivables and held-to-maturity investments are measured atamortized cost using the effective interest method. Unquoted equity instruments whose fair value is not reliablymeasurable are reported at cost.

Answer (D) is incorrect because a gain or loss on an available-for-sale financial asset is recognized directly in equity untilthe asset is derecognized. Available-for-sale financial assets and liabilities are nonderivatives not designated as such ornot falling within one of the other categories: (1) items at fair value through profit or loss, (2) held-to-maturity items, or(3) loans and receivables.

Gleim's CIA Test Prep: Part III: Business Analysis and Information TechnologyAnswer Explanations

(1312 questions)

Copyright 2008 Gleim Publications, Inc. Page 124Printed for Mamdouh Farag

Page 125: P.3 Answer Explanation

[418] Gleim #: 3.5.126 -- Source: CIA 593 IV-42

Answer (A) is correct. Determination of the imputed interest rate is made at the time the debt instrument is issued.Subsequent to initial recognition, most financial liabilities are measured at amortized cost using the effective interestmethod. Exceptions are those (1) classified as at fair value through profit or loss or (2) arising when a transfer of afinancial asset does not qualify for derecognition or is accounted for on a continuing involvement basis (IAS 39).Consequently, a change in the prevailing market rate does not affect the measurement of the noninterest-bearing notegiven for the land.

Answer (B) is incorrect because any subsequent changes in prevailing interest rates are ignored.

Answer (C) is incorrect because determination of the imputed interest rate is made at the time the debt instrument isissued.

Answer (D) is incorrect because determination of the imputed interest rate is made at the time the debt instrument isissued, and any subsequent changes in prevailing interest rates are ignored.

[419] Gleim #: 3.5.127 -- Source: CIA 1193 IV-44

Answer (A) is incorrect because the value of the land will be overstated if it is recorded at future value; present value isthe proper valuation.

Answer (B) is incorrect because interest should be recognized on the note.

Answer (C) is incorrect because the proper discount rate is the prevailing rate for similar notes, not necessarily the primerate.

Answer (D) is correct. When a financial liability is initially recognized, the entity should measure it at its cost, that is, thefair value of the consideration received. The fair value ordinarily is determined by reference to the transaction price orother market prices. If these prices are not reliably determinable, the fair value is estimated by discounting the future cashpayments at an imputed rate equal to the prevailing market rate for a similar instrument (e.g., similar as to currency, term,and type of rate) of an issuer with a similar credit rating.

[420] Gleim #: 3.5.128 -- Source: Publisher

Answer (A) is incorrect because the effective portion of gains and losses on this hedge is reported as a component ofequity.

Answer (B) is incorrect because the effective portion of gains and losses on these hedges is included in equity.

Answer (C) is correct. A fair value hedge includes a hedge of an exposure to changes in the fair value of a recognizedasset or liability or an unrecognized firm commitment. Such a hedge minimizes the risk associated with fixed cash flows.Gains and losses arising from changes in fair value of a derivative classified as a fair value hedge are included in thedetermination of profit or loss in the period of change. They are offset by losses or gains on the hedged item attributable tothe risk being hedged. Thus, profit or loss of the period of change is affected only by the net gain or loss attributable to theineffective portion of the hedge.

Answer (D) is incorrect because the effective portion of gains and losses on these hedges is included in equity.

Gleim's CIA Test Prep: Part III: Business Analysis and Information TechnologyAnswer Explanations

(1312 questions)

Copyright 2008 Gleim Publications, Inc. Page 125Printed for Mamdouh Farag

Page 126: P.3 Answer Explanation

[421] Gleim #: 3.5.129 -- Source: Publisher

Answer (A) is incorrect because a financial instrument does not involve the delivery of a product.

Answer (B) is correct. A firm commitment is an agreement with an unrelated party, binding on both parties and usuallylegally enforceable, that specifies all significant terms and includes a disincentive for nonperformance.

Answer (C) is incorrect because a forecast transaction is a transaction that is expected to occur for which no firmcommitment exists.

Answer (D) is incorrect because the purchase commitment is an exposure to risk, not a hedge of an exposure to risk.

[422] Gleim #: 3.5.130 -- Source: Publisher

Answer (A) is incorrect because the effect on profit or loss is equal to the ineffective portion of the hedge.

Answer (B) is correct. A hedge of an exposure to changes in the fair value of a recognized asset or liability is classified asa fair value hedge. Gains and losses arising from changes in fair value of a derivative classified as a fair value hedge areincluded in the determination of profit or loss in the period of change. They are offset by losses or gains on the hedgeditem attributable to the risk being hedged. Thus, profit or loss of the period of change is affected only by the net gain orloss attributable to the ineffective aspect of the hedge. The ineffective portion is equal to 25,000 (350,000 – 325,000).

Answer (C) is incorrect because the increase in the fair value of the derivative is a gross effect.

Answer (D) is incorrect because the decrease in the fair value of the hedged assets is a gross effect.

[423] Gleim #: 3.6.131 -- Source: CIA 597 IV-15

Answer (A) is incorrect because 40,000 is based on a 25% markup on sales.

Answer (B) is correct. Cost of goods sold equals beginning inventory, plus purchases (including freight-in), minus endinginventory. Given that sales reflect 125% of cost, cost of goods sold must equal 720,000 (900,000 sales ÷ 1.25).Consequently, the beginning inventory must have been 85,000 (720,000 CGS + 80,000 EI – 690,000 purchases – 25,000freight-in).

Answer (C) is incorrect because 110,000 omits the freight-in from the computation of cost of goods available for sale.

Answer (D) is incorrect because 265,000 uses the sales figure for cost of goods sold.

[424] Gleim #: 3.6.132 -- Source: CIA 597 IV-16

Answer (A) is incorrect because LIFO approximates current cost of goods sold.

Answer (B) is incorrect because FIFO approximates current cost for ending inventory, and LIFO approximates current costof goods sold.

Answer (C) is correct. FIFO assigns the most recent purchase prices to ending inventory and the earliest purchase pricesto cost of goods sold. LIFO uses the earliest acquisition costs to price the ending inventory. It is not permitted by theIFRSs. Thus, FIFO approximates current cost for ending inventory, and LIFO approximates current cost of goods sold.

Answer (D) is incorrect because FIFO approximates current cost for ending inventory.

Gleim's CIA Test Prep: Part III: Business Analysis and Information TechnologyAnswer Explanations

(1312 questions)

Copyright 2008 Gleim Publications, Inc. Page 126Printed for Mamdouh Farag

Page 127: P.3 Answer Explanation

[425] Gleim #: 3.6.133 -- Source: CIA 596 IV-11

Answer (A) is incorrect because LIFO yields the lowest profit. It is not permitted by the IFRSs.

Answer (B) is correct. Profit will be higher when cost of goods sold is lower, other factors held constant. Cost of goodssold equals beginning inventory, plus purchases, minus ending inventory. Accordingly, cost of goods sold will be lowestwhen the ending inventory is highest. Ending inventory is highest under FIFO because the older, less expensive items aredeemed to have been sold, leaving the more expensive items in the ending inventory.

Answer (C) is incorrect because weighted average averages inventory, so it results in a lower profit than FIFO.

Answer (D) is incorrect because, under specific identification, the newest (most expensive) items are not necessarily inthe ending inventory. The result is a higher cost of goods sold and lower profit than under FIFO.

[426] Gleim #: 3.6.134 -- Source: CIA 1193 IV-31

Answer (A) is incorrect because 1,000,000 excludes the goods shipped FOB destination.

Answer (B) is incorrect because 1,082,000 excludes the goods shipped FOB destination and fails to exclude the goodsshipped FOB shipping point.

Answer (C) is correct. The goods shipped FOB shipping point should be counted in the buyer’s, not the seller’s,inventory because title and risk of loss pass at the time and place of shipment. These goods were properly excluded fromending inventory. The goods shipped FOB destination were improperly excluded from the seller’s ending inventory. Thetitle and risk of loss did not pass until the time and place where the goods reached their destination and were dulytendered. Thus, the correct ending inventory is 1,122,000 (1,000,000 beginning balance + 122,000 goods shipped FOBdestination).

Answer (D) is incorrect because 1,204,000 includes the goods shipped FOB shipping point.

[427] Gleim #: 3.6.135 -- Source: CIA 1191 IV-31

Answer (A) is correct. The entry to record a write-down is a debit to inventory over and short and a credit to inventory.This amount is reported as an adjustment of cost of goods sold or as an other expense on the income statement.

Answer (B) is incorrect because a difference between a physical count and a perpetual inventory balance is common.Reasons include normal and expected shrinkage, breakage, shoplifting, and faulty record keeping. Thus, it is not anextraordinary item.

Answer (C) is incorrect because a difference between a physical count and a perpetual inventory balance is common.Reasons include normal and expected shrinkage, breakage, shoplifting, and faulty record keeping. Thus, it is not anextraordinary item.

Answer (D) is incorrect because, although the debit to cost of goods sold is acceptable, the credit should be to inventory.Also, any appropriation of retained earnings would also have to involve the unappropriated retained earnings account.

Gleim's CIA Test Prep: Part III: Business Analysis and Information TechnologyAnswer Explanations

(1312 questions)

Copyright 2008 Gleim Publications, Inc. Page 127Printed for Mamdouh Farag

Page 128: P.3 Answer Explanation

[428] Gleim #: 3.6.136 -- Source: CIA 1192 IV-43

Answer (A) is incorrect because no effect implies that NRV is at least as high as cost.

Answer (B) is correct. Inventories are measured at the lower of cost or net realizable value (NRV). NRV equals sellingprice minus completion and selling costs. Given that historical cost is 91 and NRV is 85 (price of 100 – 10 completioncost – 5 selling cost), the effect on per-unit gross profit is a reduction of 6. This amount is the writedown expensed.

Answer (C) is incorrect because a reduction of 26 results from treating normal gross profit as if it were a completion orselling cost.

Answer (D) is incorrect because an increase is permissible only to the extent of a prior writedown.

[429] Gleim #: 3.6.137 -- Source: CIA 596 IV-17

Answer (A) is incorrect because 3,050 is the ending inventory under the specific identification method.

Answer (B) is correct. Under the FIFO method, the 1,700 units of ending inventory are valued at the most recent prices.Ending inventory is assumed to include 1,000 units purchased November 24, 500 units purchased May 5, and 200 unitspurchased January 12. Hence, the ending inventory is 3,150 [(1,000 × 1.65) + (500 × 2.20) + (200 × 2.00)].

Answer (C) is incorrect because 3,230 is the ending inventory under the weighted-average method.

Answer (D) is incorrect because 3,430 is the ending inventory under the LIFO method.

[430] Gleim #: 3.6.138 -- Source: CIA 596 IV-18

Answer (A) is incorrect because 1,320 is the cost of goods sold under the LIFO method.

Answer (B) is incorrect because 1,520 is the cost of goods sold under the weighted-average method.

Answer (C) is incorrect because 1,600 is the cost of goods sold under the FIFO method.

Answer (D) is correct. Of the 800 units sold during the period, the 300 units sold on March 15 were purchased onJanuary 12 at a cost of 2.00 per unit. The remaining 500 units were purchased on May 5 at a cost of 2.20 per unit. Thecost of goods sold under the specific identification method is therefore 1,700 [(300 units × 2.00) + (500 units × 2.20)].

[431] Gleim #: 3.6.139 -- Source: CIA 1196 IV-22

Answer (A) is incorrect because 2,755 equals sales minus purchases.

Answer (B) is correct. The FIFO method assumes that the first goods purchased are the first goods sold and that endinginventory consists of the latest purchases. Moreover, whether the inventory system is periodic or perpetual does not affectFIFO valuation. The cost of goods sold is 2,445 {beginning inventory (200 units × 3.50) + purchases [(250 units × 3.30) +(200 units × 3.10) + (350 units × 3.00)] – ending inventory (250 units × 3.00)}. Thus, the gross profit for Year 1 usingFIFO is 2,805 [sales (750 units × 7.00) – cost of goods sold of 2,445].

Answer (C) is incorrect because 2,854 uses a weighted-average ending inventory and part of the cost of goods soldcalculation.

Answer (D) is incorrect because 2,920 uses a periodic LIFO inventory value.

Gleim's CIA Test Prep: Part III: Business Analysis and Information TechnologyAnswer Explanations

(1312 questions)

Copyright 2008 Gleim Publications, Inc. Page 128Printed for Mamdouh Farag

Page 129: P.3 Answer Explanation

[432] Gleim #: 3.6.140 -- Source: CIA 1196 IV-9

Answer (A) is incorrect because 8,400 neglects to subtract the net markdowns to compute the merchandise inventory atretail at December 31, Year 1.

Answer (B) is incorrect because 8,000 is the inventory at retail at December 31, Year 1.

Answer (C) is incorrect because 6,000 is computed by including the net markdowns in the cost-to-retail ratio.

Answer (D) is correct. The version of the retail method that approximates a lower-of-average-cost-or-NRV valuationincludes markups but not markdowns in the cost-to-retail ratio. Thus, the cost of the inventory at December 31, Year 1, is5,600.

Cost RetailInventory, January 1, Year 1 9,000 13,000Purchases 33,000 46,000Markups, net 1,000 42,000 60,000Sales (48,000)Markdowns, net (4,000)Inventory at retail, December 31, Year 1 8,000Cost-to-retail ratio (42,000 ÷ 60,000) × .70Inventory at cost, December 31, Year 1 5,600

[433] Gleim #: 3.6.141 -- Source: CIA 1195 I-27

Answer (A) is incorrect because an increase in the number of competitors most likely results in price competition and adecrease in sales revenue and gross margin.

Answer (B) is incorrect because a decrease in the number of suppliers most likely results in less price competition on thesupply side, with a consequent increase in costs and decrease in gross margin.

Answer (C) is correct. An overstatement of year-end inventory results in an increase in the gross margin (sales - cost ofsales). Overstating ending inventory understates cost of sales.

Answer (D) is incorrect because an understatement of accounts receivable understates sales and the gross margin.

[434] Gleim #: 3.6.142 -- Source: CIA 594 IV-2

Answer (A) is correct. Cost of goods sold equals beginning inventory, plus purchases, minus ending inventory. Hence,cost of goods sold is 440,000 (140,000 + 530,000 – 230,000).

Answer (B) is incorrect because 620,000 is obtained by reversing the opening and closing inventory figures.

Answer (C) is incorrect because 670,000 omits closing inventory from the calculation.

Answer (D) is incorrect because 900,000 adds, rather than subtracting, closing inventory.

Gleim's CIA Test Prep: Part III: Business Analysis and Information TechnologyAnswer Explanations

(1312 questions)

Copyright 2008 Gleim Publications, Inc. Page 129Printed for Mamdouh Farag

Page 130: P.3 Answer Explanation

[435] Gleim #: 3.6.143 -- Source: CIA 1193 IV-37

Answer (A) is incorrect because the freight was paid earlier in the period and would have been recorded then by a creditto cash and a debit to inventory. Thus, the freight costs will be included in the determination of profit when cost of goodssold is recognized.

Answer (B) is correct. ABC debits the cash received 43,000 [50,000 sales – 2,000 advertising – (.10 × 50,000) salescommission]. The advertising and commission expenses are debited for 2,000 and 5,000, respectively. Finally, 50,000 ofgross revenue is credited.

Answer (C) is incorrect because the 10% commission and the advertising costs are ignored in this answer.

Answer (D) is incorrect because the reimbursable advertising costs are ignored in this answer.

[436] Gleim #: 3.6.144 -- Source: CIA 589 IV-35

Answer (A) is incorrect because 3,500 is the inventory shrinkage at cost.

Answer (B) is correct. The retail inventory method converts ending inventory at retail to cost by applying a cost-retailratio. The advantage is that a physical inventory can be taken at retail without the necessity of counting individual items atcost. Because this method requires detailed inventory records to be kept at retail, lost sales revenue (at retail) caused byinventory shrinkage can be calculated using the following retail data:

Beginning inventory 40,000Purchases 180,000Goods available for sale 220,000Sales (170,000)Estimated ending inventory 50,000Actual ending inventory (45,000)Inventory shrinkage 5,000

Answer (C) is incorrect because 35,000 equals the estimated ending inventory at cost.

Answer (D) is incorrect because 45,000 equals the actual ending inventory at retail.

[437] Gleim #: 3.6.145 -- Source: CIA 592 IV-30

Answer (A) is correct. Under the net method, the payable is initially credited at the discounted amount. Because thepayment was within the discount period and freight was prepaid, the buyer’s remittance to the seller includes the freightcost of 30 and the discounted price of the merchandise [1,000 × (1.0 – .02) = 980], a total of 1,010.

Answer (B) is incorrect because freight-in was debited at the invoice date (debit freight-in and purchases, credit accountspayable). Accounts payable needs to be debited for the entire amount (including freight of 30) owed to the seller.

Answer (C) is incorrect because this entry would have been made at the invoice date if the gross method had been used.

Answer (D) is incorrect because this entry was made at the invoice date.

Gleim's CIA Test Prep: Part III: Business Analysis and Information TechnologyAnswer Explanations

(1312 questions)

Copyright 2008 Gleim Publications, Inc. Page 130Printed for Mamdouh Farag

Page 131: P.3 Answer Explanation

[438] Gleim #: 3.6.146 -- Source: CIA 592 IV-33

Answer (A) is correct. Title and risk of loss passed to the buyer at the destination, and the seller incurred the expense ofdelivery to that point. The goods did not arrive until after year-end, so they should not be included in Year 1 inventory.Freight-in should also not be recorded until Year 2.

Answer (B) is incorrect because no inventory should be included in the Year 1 financial statements.

Answer (C) is incorrect because freight-in should be recorded in Year 2.

Answer (D) is incorrect because no inventory should be included in the Year 1 financial statements, and freight-in shouldbe recorded in Year 2.

[439] Gleim #: 3.6.147 -- Source: CIA 594 IV-18

Answer (A) is incorrect because 62,000 is the ending inventory balance under LIFO.

Answer (B) is incorrect because 70,759 is the ending inventory balance under the weighted average method.

Answer (C) is correct. The first-in-first-out (FIFO) method assumes that the oldest units are used first and the newestunits remain in inventory. Because the company has 12,000 units remaining, ending inventory equals 78,750 [(5,000 × 7)+ (7,000 × 6.25)].

Answer (D) is incorrect because 84,000 computes all 12,000 units of ending inventory at the most recent purchase price of7.

[440] Gleim #: 3.6.148 -- Source: CIA 594 IV-17

Answer (A) is incorrect because the last-in-first-out (LIFO) method assumes that the most recent and costliest units havebeen removed from inventory. If costs rise steadily during the accounting period, this method will result in the lowestinventory balance, the highest cost of goods sold, and the lowest profit. LIFO is not permitted.

Answer (B) is correct. The first-in-first-out (FIFO) method assumes that the oldest and hence least costly units are usedfirst, and the newest and most costly items remain in inventory. This method will result in the highest inventory balance ifcosts rise steadily during the accounting period. Accordingly, FIFO results in the lowest cost of goods sold and the highestprofit.

Answer (C) is incorrect because the weighted-average cost method averages the costs of all inventory items and results ina lower inventory balance and profit than does the FIFO method.

Answer (D) is incorrect because specific identification charges the actual cost of each unit to cost of goods sold eachperiod, leaving as inventory the actual cost of all items still in inventory. Given that the newest and most costly items areremoved from inventory first, the inventory balance and profit will be lower than that obtained using FIFO estimation.

Gleim's CIA Test Prep: Part III: Business Analysis and Information TechnologyAnswer Explanations

(1312 questions)

Copyright 2008 Gleim Publications, Inc. Page 131Printed for Mamdouh Farag

Page 132: P.3 Answer Explanation

[441] Gleim #: 3.6.149 -- Source: CIA 593 IV-35

Answer (A) is incorrect because shrinkage has no effect on the ratio but should be subtracted in arriving at the estimatedending inventory at retail.

Answer (B) is correct. The retail method of inventory estimation applies a cost-retail ratio to the ending inventory atretail to determine ending inventory at cost. For example, a popular method calculates the ratio as goods available for saleat cost divided by goods available at retail, with markups but not markdowns included in the calculation of the retailamount. Normal inventory shrinkage is subtracted from the retail amount of goods available because the goods are notavailable. However, abnormal amounts of theft, etc., are removed from the cost and retail amounts. The reason for thedifference in treatment is that normal but not abnormal inventory losses are anticipated and included in the selling price(retail value). Accordingly, failure to account for normal inventory shrinkage has no effect on the calculation of the cost-retail ratio but overstates ending inventory at retail.

Answer (C) is incorrect because shrinkage has no effect on the ratio but should be subtracted in arriving at the estimatedending inventory at retail.

Answer (D) is incorrect because shrinkage has no effect on the ratio but should be subtracted in arriving at the estimatedending inventory at retail.

[442] Gleim #: 3.6.150 -- Source: Publisher

Answer (A) is incorrect because disclosures should include the accounting policies applied, such as the lower-of-cost-or-NRV method.

Answer (B) is incorrect because disclosures should include the cost formulas used.

Answer (C) is incorrect because disclosures should include the amounts for classifications of inventory appropriate to theentity.

Answer (D) is correct. According to IAS 2, Inventories, disclosures about inventories include, for example, the accountingpolicies applied in measuring inventories, including the cost formulas used; total carrying amount; carrying amount foreach classification appropriate to the entity; carrying amount of items carried at fair value minus costs to sell; amount ofany reversal of writedowns recognized as income; reasons for such a reversal; and carrying amount of inventory pledged assecurity. Thus, the disclosures under IAS 2 include the carrying amount of inventories carried at NRV, not the amount ofobsolete inventory.

[443] Gleim #: 3.6.151 -- Source: CIA 595 IV-13

Answer (A) is incorrect because cost of goods sold is 22,000 under the weighted-average method. Under LIFO, cost ofgoods sold is 23,000 (12,000 + 11,000). The 2,000 most recently purchased units are presumed to have been sold.

Answer (B) is incorrect because the weighted-average unit cost of all items available for sale is 11 [(10,000 + 11,000 +12,000) ÷ 3,000]. Given that 2,000 units were sold, cost of goods sold is 22,000 (2,000 × 11) under this method.

Answer (C) is correct. Under FIFO, the first items purchased are presumed to be the first sold. Given that 3,000 unitswere available and 2,000 units were sold, FIFO cost of goods sold must have been 21,000 [(1,000 × 10) BI + (1,000 × 11)May 1 purchase].

Answer (D) is incorrect because FIFO cost of goods sold is 21,000. Under the weighted-average method, cost of goodssold is 22,000.

Gleim's CIA Test Prep: Part III: Business Analysis and Information TechnologyAnswer Explanations

(1312 questions)

Copyright 2008 Gleim Publications, Inc. Page 132Printed for Mamdouh Farag

Page 133: P.3 Answer Explanation

[444] Gleim #: 3.6.152 -- Source: CIA 595 IV-22

Answer (A) is incorrect because a change to LIFO is not allowed.

Answer (B) is correct. According to IAS 16, Property, Plant, and Equipment, a change in depreciation method resultingfrom a significant change in the expected pattern of economic benefits is accounted for as a change in estimate, that is,prospectively. In a period of falling costs, FIFO results in higher cost of goods sold than the weighted-average method.FIFO includes the higher, earlier costs in cost of goods sold, whereas the weighted-average method averages the later,lower costs with the higher, earlier costs. Thus, a change from FIFO to weighted-average costing reduces cost of goodssold and increases reported profit.

Answer (C) is incorrect because, in the later years of the depreciable life of an asset, accelerated depreciation results inlower depreciation expense than does the straight-line method. A change to straight line increases depreciation expenseand reduces reported profit.

Answer (D) is incorrect because, in the early years of the depreciable life of an asset, straight-line depreciation results inlower depreciation expense than accelerated depreciation. A change to accelerated depreciation increases depreciationexpense and reduces reported profit.

[445] Gleim #: 3.6.153 -- Source: CIA 1195 IV-5

Answer (A) is incorrect because 340,000 omits transportation-in from the calculation.

Answer (B) is correct. Purchase discounts, allowances, and returns are subtractions from purchases because they arereductions of cost. Transportation-in is an addition because it increases cost. Thus, net purchases equals 346,000 (400,000+ 6,000 – 40,000 – 15,000 – 5,000).

Answer (C) is incorrect because 370,000 omits transportation-in and adds, rather than subtracts, purchase allowances.

Answer (D) is incorrect because 376,000 adds, rather than subtracts, purchase allowances.

[446] Gleim #: 3.6.154 -- Source: CIA 1195 IV-18

Answer (A) is incorrect because 186,978 is the value of ending inventory under the weighted-average method.

Answer (B) is incorrect because 197,000 is the ending inventory under the FIFO method.

Answer (C) is correct. Under the weighted-average method, the weighted-average cost per unit is multiplied by thenumber of units sold to determine the cost of goods sold for the period. The total units available for sale equaled 45,500(8,000 + 15,000 + 10,000 + 12,500). The total cost of all units available for sale was 415,000 [(8,000 × 10) + (15,000 × 8)+ (10,000 × 9) + (12,500 × 10)]. Thus, the weighted-average cost per unit of inventory was 9.1209 (415,000 ÷ 45,500),and cost of goods sold was 228,023 (25,000 × 9.1209).

Answer (D) is incorrect because 235,000 is the cost of goods sold under the LIFO method.

Gleim's CIA Test Prep: Part III: Business Analysis and Information TechnologyAnswer Explanations

(1312 questions)

Copyright 2008 Gleim Publications, Inc. Page 133Printed for Mamdouh Farag

Page 134: P.3 Answer Explanation

[447] Gleim #: 3.6.155 -- Source: CIA 1196 IV-8

Answer (A) is incorrect because 20,000 is the difference between sales for the period and cost of goods available for sale.

Answer (B) is incorrect because 40,000 is the amount of gross profit.

Answer (C) is correct. The gross profit margin percentage is given as 20% of sales. Hence, cost of goods sold must havebeen 160,000 [200,000 sales × (1.0 – .2)]. The estimated cost of the inventory at March 31, 2004 is computed as follows:

Inventory, 1/1 30,000Purchases 190,000Cost of goods available for sale, 3/31 220,000Estimated cost of goods sold (160,000)Estimated inventory at 3/31 60,000

Answer (D) is incorrect because 180,000 results from subtracting the gross profit from cost of goods available for sale.

[448] Gleim #: 3.6.156 -- Source: CIA 596 IV-14

Answer (A) is correct. Cost of goods sold equals cost of goods produced (CGP) adjusted for the change in finished goods.CGP equals the sum of raw materials used, direct labor costs, and production overhead, adjusted for the change in work-in-progress. Raw materials used equals 255,000 (105,000 BI + 300,000 purchases – 20,000 returns – 130,000 EI). Thus,CGP equals 1,650,000 (255,000 RM + 600,000 DL + 750,000 OH + 220,000 BWIP – 175,000 EWIP), and CGS equals1,480,000 (1,650,000 CGP + 90,000 BFG – 260,000 EFG).

Answer (B) is incorrect because 1,500,000 results from not subtracting purchase returns when calculating raw materialsused.

Answer (C) is incorrect because 1,610,000 fails to deduct the ending balance of raw materials when calculating rawmaterials used.

Answer (D) is incorrect because 1,650,000 is the cost of goods produced.

[449] Gleim #: 3.6.157 -- Source: CIA 596 IV-16

Answer (A) is incorrect because, if the ending inventory of raw materials is understated, cost of goods sold is overstated,and net income is understated.

Answer (B) is correct. If the ending inventory of raw materials is understated, raw materials used is overstated, cost ofgoods produced is overstated, and cost of goods sold is overstated.

Answer (C) is incorrect because working capital is higher when the balances of current assets are higher. If the rawmaterials inventory balance is understated, working capital will also be understated.

Answer (D) is incorrect because, if the ending inventory of raw materials is understated, raw materials used is overstated,and cost of goods produced is overstated.

Gleim's CIA Test Prep: Part III: Business Analysis and Information TechnologyAnswer Explanations

(1312 questions)

Copyright 2008 Gleim Publications, Inc. Page 134Printed for Mamdouh Farag

Page 135: P.3 Answer Explanation

[450] Gleim #: 3.6.158 -- Source: CIA, adapted

Answer (A) is correct. Under first-in, first-out (FIFO) inventory valuation, the 10,000 units in ending inventory areassumed to have been the most recent items purchased. The cost of the most recent 10,000 units purchased is: (5,000 units× 7.50) + (5,000 units × 8) = 77,500.

Answer (B) is incorrect because this solution is the ending inventory balance under the specific identification method ifthe units remaining in inventory at year end were identified as having been purchased on April 1 and July 1: (5,000 units× 9) + (5,000 units × 8) = 85,000.

Answer (C) is incorrect because this solution is the ending inventory balance under the average cost method. The averagecost of all items purchased is used to calculate the ending inventory balance. The average cost of items purchased is: [10(5,000) + 9 (5,000) + 8 (5,000) + 7.50 (5,000)] ÷ 20,000 = 8.625 per unit so 10,000 units are assigned a value of 86,250.

Answer (D) is incorrect because this solution is the ending inventory balance under the last-in, first-out (LIFO) method ofinventory valuation. The most recent items purchased are assumed to be sold first, so the items remaining in inventory areassigned the cost of the earliest purchases: (5,000 units × 10) + (5,000 units × 9) = 95,000.

[451] Gleim #: 3.7.159 -- Source: CIA 595 IV-15

Answer (A) is incorrect because depreciation in year two will be 20,000 under the straight-line method.

Answer (B) is incorrect because, under the SYD method, second-year depreciation will be 26,667 [(4 ÷ 15) × 100,000].

Answer (C) is incorrect because 16,000 is based on the 100% diminishing-balance method.

Answer (D) is correct. The 200% diminishing-balance method uses twice the straight-line rate. Accordingly, first-yeardepreciation expense is 40,000 (100,000 × 20% × 2). In the second year, the depreciation base is reduced by the amountof depreciation already taken in the first year. Thus, depreciation expense in year two is 24,000 [(2 × 20%) × (100,000 –40,000)].

[452] Gleim #: 3.7.160 -- Source: CIA 595 IV-10

Answer (A) is incorrect because the purchase price, freight costs, and installation costs of a productive asset are includedin the asset’s cost.

Answer (B) is correct. Accordingly, items of property, plant, and equipment (PPE) that meet the recognition criterion areinitially measured at cost. The cost includes the purchase price (minus trade discounts and rebates, plus purchase taxes)and the directly attributable costs of bringing the assets to working condition for their intended use. Directly attributablecosts include site preparation, installation, initial delivery and handling, architect and equipment fees, costs of removingthe assets and restoring the site, etc. Accordingly, the cost of land includes the cost of obtaining the land and readying itfor its intended uses, but it is inappropriate to recognize the proceeds related to site preparation immediately in profit orloss. They should be treated as reductions in the price of the land.

Answer (C) is incorrect because subsequent costs are added to the carrying amount of an item of PPE if it is probable that,as a result, future economic benefits will be received, and the costs are reliably measurable.

Answer (D) is incorrect because all costs of construction should be included as a part of the asset’s cost.

Gleim's CIA Test Prep: Part III: Business Analysis and Information TechnologyAnswer Explanations

(1312 questions)

Copyright 2008 Gleim Publications, Inc. Page 135Printed for Mamdouh Farag

Page 136: P.3 Answer Explanation

[453] Gleim #: 3.7.161 -- Source: CIA 1196 IV-10

Answer (A) is incorrect because at the end of the estimated useful life of a depreciable asset, the amount of accumulateddepreciation should equal the depreciable cost (original cost – estimated residual value), regardless of the depreciationmethod used.

Answer (B) is incorrect because at the end of the estimated useful life of a depreciable asset, the amount of accumulateddepreciation should equal the depreciable cost (original cost – estimated residual value), regardless of the depreciationmethod used.

Answer (C) is incorrect because at the end of the estimated useful life of a depreciable asset, the amount of accumulateddepreciation should equal the depreciable cost (original cost – estimated residual value), regardless of the depreciationmethod used.

Answer (D) is correct. At the end of the estimated useful life of a depreciable asset, the amount of accumulateddepreciation should equal the depreciable cost (original cost – estimated residual value), regardless of the depreciationmethod used. Periodic diminishing-balance depreciation is calculated without regard to residual value, but the asset is notdepreciated below its residual value. The SYD method uses a depreciable base equal to cost minus residual value.

[454] Gleim #: 3.7.162 -- Source: CIA 596 IV-22

Answer (A) is incorrect because the revaluation of the land must be recognized.

Answer (B) is incorrect because the revaluation surplus is not transferred through profit or loss.

Answer (C) is correct. An item of PPE is carried at cost minus any accumulated depreciation and impairment losses.Under the allowed alternative treatment, an item of PPE may be carried at a revalued amount equal to fair value at therevaluation date minus any subsequent accumulated depreciation and impairment losses. Land is not depreciated, so itshould be carried at its revalued amount. This amount should be determined by a professional appraisal. The revaluationincreased the carrying amount and is therefore credited directly to equity as revaluation surplus. Accordingly, the entry isto debit land and credit revaluation surplus for 125,000. The entire surplus may be transferred to retained earnings (butnot through profit or loss) when the asset is derecognized. Upon derecognition, the asset should be removed from thebalance sheet, and the resulting loss (revalued carrying amount of 200,000 – 0 net disposal proceeds = 200,000) should beincluded in profit or loss.

Answer (D) is incorrect because the revaluation surplus is not transferred through profit or loss.

[455] Gleim #: 3.7.163 -- Source: CIA 1195 IV-22

Answer (A) is correct. The cash account is debited for the amount of the sale proceeds. The machinery account and therelated accumulated depreciation account are eliminated by a credit and a debit, respectively. Because the sale price wasless than the carrying amount of the asset on the date of sale, a loss on disposal should be recognized in profit or loss. Theterm “expenses” includes losses.

Answer (B) is incorrect because a loss on disposal should be recognized in profit or loss. The term “expenses” includeslosses.

Answer (C) is incorrect because accumulated depreciation should be debited.

Answer (D) is incorrect because an expense and accumulated depreciation should be debited.

Gleim's CIA Test Prep: Part III: Business Analysis and Information TechnologyAnswer Explanations

(1312 questions)

Copyright 2008 Gleim Publications, Inc. Page 136Printed for Mamdouh Farag

Page 137: P.3 Answer Explanation

[456] Gleim #: 3.7.164 -- Source: CIA 1196 IV-29

Answer (A) is incorrect because 10% is the amount expensed in the last year.

Answer (B) is incorrect because 25% is the percentage expensed each year under the straight-line method.

Answer (C) is correct. Under the SYD method, the amount of the depreciable cost that is expensed each year is theremaining useful life at the beginning of that year divided by the sum of the years of useful life. For the third year, theportion expensed is 20% [2 ÷ (1 + 2 + 3 + 4)].

Answer (D) is incorrect because 70% is the total amount expensed in the first 2 years.

[457] Gleim #: 3.7.165 -- Source: CIA 1193 IV-42

Answer (A) is incorrect because 91,000 ignores the carrying amount of items sold and includes the gain in thecomputation.

Answer (B) is incorrect because 100,000 ignores the depreciation expense for the period.

Answer (C) is correct. The carrying amount of the PPE account, net of accumulated depreciation, is increased by the costof purchases and decreased by the carrying amount of items of PPE sold and depreciation. The net PPE decreased by thecarrying amount of items sold, or 20,000 (50,000 cost – 30,000 accumulated depreciation), and by the 10,000 ofdepreciation. If PPE still increased by 80,000, 110,000 (30,000 total decrease + 80,000 increase) of equipment must havebeen purchased.

Answer (D) is incorrect because 119,000 double counts depreciation expense and deducts the gain.

[458] Gleim #: 3.7.166 -- Source: CIA 1193 IV-38

Answer (A) is incorrect because 160,000 excludes the delivery, installation, and trial-run costs.

Answer (B) is incorrect because 184,000 excludes the trial-run cost.

Answer (C) is correct. Under the straight-line method, the annual depreciation expense for an asset equals the asset’samount (cost – residual value) divided by the asset’s estimated useful life. The cost of the asset includes its price and thedirectly attributable costs of bringing it to working condition for intended use. Thus, the depreciation expense is 192,000[(800,000 purchase price + 50,000 delivery cost + 70,000 installation cost + 40,000 trial-run cost) ÷ 5-year estimatedservice life]. Borrowing costs incurred after the asset is prepared for its intended use are expensed even if the allowedalternative treatment of such costs is followed, and the asset otherwise satisfies the criteria for capitalization of suchexpenses.

Answer (D) is incorrect because 204,000 includes the borrowing costs.

[459] Gleim #: 3.7.167 -- Source: CIA 1196 IV-32

Answer (A) is incorrect because 1,200,000 is the purchase price of the quarry.

Answer (B) is incorrect because 900,000 results from using straight-line depletion.

Answer (C) is correct. The units-of-production method allocates cost based on output. The net amount reported as anasset for the quarry using this method is 840,000 [(350,000 cubic yards ÷ 500,000 total cubic yards) × 1,200,000].

Answer (D) is incorrect because 360,000 equals {1,200,000 – [(350,000 ÷ 500,000) × 1,200,000]}. The 350,000 cubicyards is the volume remaining in the quarry’s pit at year-end Year 5, not the volume depleted.

Gleim's CIA Test Prep: Part III: Business Analysis and Information TechnologyAnswer Explanations

(1312 questions)

Copyright 2008 Gleim Publications, Inc. Page 137Printed for Mamdouh Farag

Page 138: P.3 Answer Explanation

[460] Gleim #: 3.7.168 -- Source: CIA 592 IV-28

Answer (A) is incorrect because the cost should be capitalized.

Answer (B) is incorrect because the same account should be used.

Answer (C) is incorrect because allocation is not an accepted procedure.

Answer (D) is correct. Subsequent costs are added to the carrying amount of an item of PPE if it is probable that, as aresult, future economic benefits will be received, and the costs are reliably measurable. An extended useful life, improvedoutput quantity or quality, and reduced operating costs future economic benefits.

[461] Gleim #: 3.7.169 -- Source: CIA 1196 IV-18

Answer (A) is incorrect because 45,100 equals the carrying amount of the old equipment after being depreciated for 4years plus the cash paid for the new equipment.

Answer (B) is incorrect because 48,700 equals the carrying amount of the old equipment after being depreciated for 3.5years plus cash paid for the new equipment.

Answer (C) is correct. The cost of what is received in an exchange of similar or dissimilar items of PPE is recorded atfair value unless the exchange lacks economic substance or the fair value of what is given or received is not reliablymeasurable. Hence, the entity will record the new equipment at the fair value of the old asset plus cash paid, or 51,000(16,000 + 35,000).

Answer (D) is incorrect because 52,500 is the list price of the new equipment.

[462] Gleim #: 3.7.170 -- Source: CIA 591 IV-43

Answer (A) is incorrect because an excess of fair value over the carrying amount of a plant asset is not reflected inconventional financial statements if the fair value is not reliably measurable.

Answer (B) is incorrect because an excess of fair value over the carrying amount of a plant asset is not reflected inconventional financial statements if the fair value is not reliably measurable.

Answer (C) is incorrect because an excess of fair value over the carrying amount of a plant asset is not reflected inconventional financial statements if the fair value is not reliably measurable.

Answer (D) is correct. An item of property, plant, and equipment may be carried at a revalued amount equal to fair valueminus any subsequent accumulated depreciation and impairment losses. An increase in the carrying amount after arevaluation is credited directly to equity as revaluation surplus. However, revaluation is not allowed when the fair value isnot reliably measurable. Thus, the measurement of property, plant, and equipment subsequent to initial recognition as anasset is at cost minus any accumulated depreciation and impairment losses. The plant asset should be reported at itscarrying amount of 270,000 [300,000 – (300,000 ÷ 10 years)].

Gleim's CIA Test Prep: Part III: Business Analysis and Information TechnologyAnswer Explanations

(1312 questions)

Copyright 2008 Gleim Publications, Inc. Page 138Printed for Mamdouh Farag

Page 139: P.3 Answer Explanation

[463] Gleim #: 3.7.171 -- Source: CIA 594 IV-30

Answer (A) is incorrect because directly attributable costs of bringing the asset to working condition for its intended useare included in its cost.

Answer (B) is correct. An asset classified under property, plant, and equipment is measured initially at cost. This amountincludes the purchase price and any directly attributable costs of bringing the asset to working condition for its intendeduse. Directly attributable costs include costs of, for example, site preparation, initial delivery and handling, installation,professional fees (e.g., those of architects and engineers), and dismantling and removing the asset and restoring the site.The purchase price is determined by adding any import fees and nonrefundable purchase taxes and subtracting any tradediscounts and rebates.

Answer (C) is incorrect because subsequent expenditures should be added to the carrying amount if it is probable that theywill result in future economic benefits exceeding the originally assessed standard of performance.

Answer (D) is incorrect because the costs of a self-constructed asset are determined in the same way as those of anacquired asset. Hence, all costs of construction of a building are included in its initially recorded cost because they aredirectly attributable costs of bringing the asset to its working condition for its intended use.

[464] Gleim #: 3.7.172 -- Source: CIA 597 IV-34

Answer (A) is incorrect because 35,000 gain equals selling price, minus carrying amount, plus residual value.

Answer (B) is correct. The accumulated depreciation was 400,000 {[(1,000,000 – 100,000) ÷ 9 years] × 4 years}, so thecarrying amount was 600,000 (1,000,000 – 400,000). Thus, the loss was 65,000 (600,000 – 535,000 sales price).

Answer (C) is incorrect because 365,000 loss equals cost, minus selling price, minus residual value.

Answer (D) is incorrect because 465,000 loss equals cost minus selling price.

[465] Gleim #: 3.7.173 -- Source: CIA 597 IV-33

Answer (A) is correct. The selling price minus the carrying amount of the machine equals the gain or loss. The carryingamount equals cost minus accumulated depreciation, or 205,000 (700,000 – 495,000). Thus, the selling price was 280,000(205,000 + 75,000 gain).

Answer (B) is incorrect because 240,000 equals carrying amount, plus gain, minus residual value.

Answer (C) is incorrect because 205,000 is the carrying amount.

Answer (D) is incorrect because 115,000 equals the gain plus the residual value.

[466] Gleim #: 3.7.174 -- Source: CIA 1190 IV-46

Answer (A) is correct. The balance outstanding during the year was 75,132 (275,132 – 200,000). At an interest rate of10%, the entity should accrue 7,513.20 (10% × 75,132) of interest for the year.

Answer (B) is incorrect because 8,289 is the balance due in 3 years minus the balance outstanding during the year dividedby 3 [(100,000 – 75,132) ÷ 3].

Answer (C) is incorrect because 10,000 equals 10% of the face amount due.

Answer (D) is incorrect because 27,513 equals the interest rate times the cost of the asset (275,132 × 10%).

Gleim's CIA Test Prep: Part III: Business Analysis and Information TechnologyAnswer Explanations

(1312 questions)

Copyright 2008 Gleim Publications, Inc. Page 139Printed for Mamdouh Farag

Page 140: P.3 Answer Explanation

[467] Gleim #: 3.7.175 -- Source: CIA 1193 IV-29

Answer (A) is incorrect because the charge should be to an asset account rather than an expense account.

Answer (B) is correct. The purchase of office equipment represents the acquisition of an asset. An increase in an asset isrecorded by a debit. The purchase on account increases liabilities. An increase in a liability is recorded by a credit.

Answer (C) is incorrect because an increase in accounts payable is recorded by a credit. The purchase of equipment resultsin an asset that is recorded by a debit to an asset account.

Answer (D) is incorrect because an increase in a liability is recorded by a credit. An increase in an asset is recorded by adebit.

[468] Gleim #: 3.7.176 -- Source: CIA 592 IV-27

Answer (A) is incorrect because 200% diminishing-balance depreciation is higher in Year 1.

Answer (B) is incorrect because 200% diminishing-balance depreciation is higher in Year 1 and lower in Year 2.

Answer (C) is correct. 200% diminishing-balance is an accelerated depreciation method that determines periodicdepreciation expense by multiplying the carrying amount at the beginning of each period by a constant rate that is equal totwice the straight-line rate of depreciation. Each year the carrying amount of the asset decreases by the depreciationexpense recognized. Residual value is ignored in determining the carrying amount except as a floor beneath which theasset may not be depreciated. SYD depreciation multiplies a constant depreciable amount (cost – residual value) by theSYD fraction. The SYD fraction’s numerator is the number of years of remaining useful life (n). The formula to computethe denominator in the SYD method is (n(n+1)) ÷ 2. For a 5-year estimated useful life, the denominator of the fraction is15 [5(5 + 1) ÷ 2].

200% DB: Year 1 = 15,000(.4) = 6,000 Year 2 = 9,000(.4) = 3,600

SYD: Year 1 = 15,000(5 ÷ 15) = 5,000 Year 2 = 15,000(4 ÷ 15) = 4,000

Answer (D) is incorrect because 200% diminishing-balance depreciation is lower in year 2.

[469] Gleim #: 3.7.177 -- Source: CIA 590 IV-45

Answer (A) is incorrect because 40,000 assumes the purchase price allocation is based on carrying amounts instead of fairvalues.

Answer (B) is correct. An item of PPE is initially measured at its cost. Accordingly, the cost, which equals the 120,000purchase price, should be allocated between the land and the building based on their respective fair values. The cost of theland should be recorded at 48,000 {120,000 × [60,000 ÷ (60,000 + 90,000)]}.

Answer (C) is incorrect because 50,000 is the seller’s carrying amount of the land.

Answer (D) is incorrect because 60,000 is the estimated fair value of the land.

Gleim's CIA Test Prep: Part III: Business Analysis and Information TechnologyAnswer Explanations

(1312 questions)

Copyright 2008 Gleim Publications, Inc. Page 140Printed for Mamdouh Farag

Page 141: P.3 Answer Explanation

[470] Gleim #: 3.7.178 -- Source: CMA 1292 2-5

Answer (A) is incorrect because 74,000 assumes that the depreciable amount is the invoice price minus residual value.

Answer (B) is correct. Items of property, plant, and equipment (PPE) that meet the recognition criteria are initiallymeasured at cost. The cost includes the purchase price (minus trade discounts and rebates, plus purchase taxes) and thedirectly attributable costs of bringing the assets to working condition for their intended use. Hence, the depreciableamount is 210,000 (200,000 invoice price + 2,000 delivery expense + 4,500 site preparation + 3,500 electrical work).Under the 200% DB method, residual value is ignored at the beginning. Thus, the full 210,000 will be subject todepreciation. Given a 5-year life, the annual straight-line rate is 20%, and the 200% DB rate will be 40%. Depreciationfor the first year is therefore 84,000 (40% × 210,000).

Answer (C) is incorrect because the depreciable amount of the machine was 210,000, not the 200,000 invoice price.

Answer (D) is incorrect because 80,800 assumes a depreciable amount of 202,000, but the site preparation and electricalcosts are part of that cost.

[471] Gleim #: 3.7.179 -- Source: CIA 1194 IV-19

Answer (A) is incorrect because 57,500 is the depreciation expense for year one.

Answer (B) is incorrect because 75,000 is the expense for year one calculated without subtracting the residual value.

Answer (C) is correct. Depreciation expense equals cost minus residual value, times the quotient of estimated hours ofuse in year two divided by the total estimated hours of use. Thus, depreciation expense is 80,500 [(300,000 – 70,000) ×(700 hours ÷ 2,000 hours)].

Answer (D) is incorrect because 105,000 results from not subtracting the residual value.

[472] Gleim #: 3.7.180 -- Source: CIA 595 IV-9

Answer (A) is incorrect because depreciation must be taken up to the date of disposition and all amounts relating to theretired asset should be eliminated.

Answer (B) is incorrect because the gain should be recorded as a credit.

Answer (C) is incorrect because the PPE account should be credited, and accumulated depreciation should be debited.

Answer (D) is correct. The journal entry to record the sale of an item of PPE for cash in excess of its carrying amountshould debit the cash account to record the sale proceeds received. Accumulated depreciation should be eliminated bydebiting an amount equal to depreciation accumulated up to the start of the current accounting period plus anydepreciation that has accumulated between the start of the current period and the date of disposal. Finally, the PPEaccount should be credited to eliminate the original cost of the asset. The gain should be recorded as a credit andrecognized as income on the income statement.

Gleim's CIA Test Prep: Part III: Business Analysis and Information TechnologyAnswer Explanations

(1312 questions)

Copyright 2008 Gleim Publications, Inc. Page 141Printed for Mamdouh Farag

Page 142: P.3 Answer Explanation

[473] Gleim #: 3.7.181 -- Source: CIA 595 IV-17

Answer (A) is incorrect because 20,000 omits depreciation for the first half of the fifth year.

Answer (B) is correct. The gain on the sale is the difference between the sale proceeds and the undepreciated cost of theasset. Depreciation must be taken up to the time of sale. Assuming that residual value is 0, annual depreciation is 10,000(100,000 ÷ 10 years). Thus, the gain is 25,000 {80,000 – [100,000 – (4.5 years × 10,000)]}.

Answer (C) is incorrect because 30,000 results if 5 full years of depreciation are taken.

Answer (D) is incorrect because 35,000 results from subtracting accumulated depreciation from the sale proceeds.

[474] Gleim #: 3.7.182 -- Source: CIA 596 IV-27

Answer (A) is incorrect because 8,750 is the result of depreciating the remaining carrying amount over 20 years ratherthan the remaining 17 years.

Answer (B) is correct. In Year 4, the carrying amount at the start of the period will be amortized over the revisedestimated years of useful life. The depreciation recognized during Year 1 through Year 3 was 75,000 [3 years × (250,000÷ 10)]. Thus, the carrying amount at the beginning of Year 4 was 175,000, and year four depreciation based on the revisedestimated useful life is 10,294 [175,000 ÷ (20 – 3)].

Answer (C) is incorrect because 12,500 results from accounting for the change in estimate retroactively.

Answer (D) is incorrect because 14,706 results from depreciating the original carrying amount over the revised estimate ofremaining useful life.

[475] Gleim #: 3.8.183 -- Source: CIA 595 IV-25

Answer (A) is correct. IAS 38, Intangible Assets, defines an intangible asset as “an identifiable nonmonetary assetwithout physical substance.” Inventory is a tangible asset. Thus, goods on consignment are not intangible assets.

Answer (B) is incorrect because patents are intangible assets.

Answer (C) is incorrect because copyrights are intangible assets.

Answer (D) is incorrect because trademarks are intangible assets.

[476] Gleim #: 3.8.184 -- Source: CIA 593 IV-32

Answer (A) is incorrect because start-up costs are expensed when incurred.

Answer (B) is incorrect because start-up costs are expensed when incurred.

Answer (C) is incorrect because start-up costs are expensed when incurred.

Answer (D) is correct. Expenditures on start-up activities are expensed when incurred unless they are included in the costof an item of property, plant, and equipment. They include the costs of establishing a new legal entity, such as legal andsecretarial costs; pre-opening costs of an entity’s new business facility; and the pre-operating costs of new operations,products, or processes.

Gleim's CIA Test Prep: Part III: Business Analysis and Information TechnologyAnswer Explanations

(1312 questions)

Copyright 2008 Gleim Publications, Inc. Page 142Printed for Mamdouh Farag

Page 143: P.3 Answer Explanation

[477] Gleim #: 3.8.185 -- Source: Publisher

Answer (A) is incorrect because an intangible asset is amortizable if its useful life is finite. If the pattern of consumptionof the economic benefits of such an intangible asset is not reliably determinable, the straight-line amortization method isapplied.

Answer (B) is correct. A recognized intangible asset is amortized over its useful life if that useful life is finite, that is,unless the useful life is determined to be indefinite. The useful life of an intangible asset is indefinite if no foreseeablelimit exists on the period over which it will contribute, directly or indirectly, to the reporting entity’s cash flows.

Answer (C) is incorrect because, if the precise length of the useful life is not known, an intangible asset with a finiteuseful life is amortized over the best estimate of its useful life.

Answer (D) is incorrect because a recognized intangible asset is not amortized if its useful life is indefinite.

[478] Gleim #: 3.8.186 -- Source: CIA 595 IV-71

Answer (A) is incorrect because a copyright provides legal protection for tangible expressions of ideas, e.g., novels, songs,and software.

Answer (B) is incorrect because laws establishing minimum warranty standards do not limit the similarity of productofferings.

Answer (C) is incorrect because anti-merger laws can affect the ability of one entity to acquire another entity producingsimilar products if the result will be to lessen competition, but they do not affect the design of new products.

Answer (D) is correct. A patent is the exclusive legal right to use or sell an invention, such as a device or process. Apatent may be given to any new and useful process, machine, manufacture, or composition of matter, and any infringementof a patent is a basis for a lawsuit. Thus, patent laws require that entities not design new products that are illegally similarto those of other entities that enjoy patent protection.

[479] Gleim #: 3.8.187 -- Source: CIA 593 IV-43

Answer (A) is incorrect because cost in excess of fair value of the identifiable assets and liabilities is reported asgoodwill.

Answer (B) is correct. Goodwill is recognized as an asset equal to the excess of the cost over the acquirer’s interest in thefair value of the identifiable assets and liabilities acquired. The cost of 4,000,000 (40 × 100,000) is in excess of the3,550,000 (5,000,000 + 550,000 – 2,000,000) fair value of the identifiable assets and liabilities by 450,000. This excess isgoodwill.

Answer (C) is incorrect because 550,000 is the excess of fair value over the carrying amount of the identifiable assets andliabilities on the seller’s books.

Answer (D) is incorrect because the purchase price of 4,000,000 exceeds the seller’s 3,000,000 carrying amount by1,000,000.

Gleim's CIA Test Prep: Part III: Business Analysis and Information TechnologyAnswer Explanations

(1312 questions)

Copyright 2008 Gleim Publications, Inc. Page 143Printed for Mamdouh Farag

Page 144: P.3 Answer Explanation

[480] Gleim #: 3.8.188 -- Source: Publisher

Answer (A) is correct. Evaluation of process alternatives is an activity typically classified as research. Research is“original and planned investigation undertaken with the prospect of gaining new scientific or technical knowledge andunderstanding” (IAS 38).

Answer (B) is incorrect because construction of a pilot plant is a development activity.

Answer (C) is incorrect because construction of a preproduction prototype is a development activity.

Answer (D) is incorrect because the design of tools involving new technology is an activity typically classified asdevelopment.

[481] Gleim #: 3.8.189 -- Source: Publisher

Answer (A) is incorrect because 175,000 assumes no amortization.

Answer (B) is incorrect because 50,000 is the excess of the estimated discounted future net cash inflows over the fairvalue minus cost to sell.

Answer (C) is incorrect because 25,000 is the excess of the carrying amount over the fair value minus cost to sell.

Answer (D) is correct. The entity should determine whether the carrying amount exceeds the recoverable amount. Therecoverable amount is the greater of fair value minus cost to sell (275,000) or value in use (the present value of expectedcash flows from the asset, or 325,000). Thus, Chertco should recognize no impairment loss because the estimateddiscounted future net cash inflows (325,000) exceed the carrying amount {500,000 – [4 years × (500,000 ÷ 10 years)] =300,000}.

[482] Gleim #: 3.8.190 -- Source: CIA 1185 IV-20

Answer (A) is incorrect because an internally generated intangible asset may arise from the development (but not theresearch) phase of asset generation. An asset is recognized if the enterprise demonstrates the technical feasibility ofcompleting the asset, the intent to complete and use or sell the asset, the ability to use or sell it, how it will generateprobable future economic benefits, the availability of resources to complete development and to use or sell the asset, andthe ability to make a reliable measurement of development expenditures for the asset.

Answer (B) is incorrect because items of equipment that qualify for asset recognition are initially measured at cost. Thedepreciable amounts of these assets are then allocated as expense (unless included in the carrying amount of other assets)over their useful lives.

Answer (C) is incorrect because goodwill arising from a business combination that meets the definition of an acquisitionis recognized as an asset. It is not amortized. However, impairment of goodwill is recognized as a loss.

Answer (D) is correct. Research expenditures should be expensed when incurred. No intangible asset resulting fromresearch should be recognized. The enterprise cannot demonstrate at this stage that future economic benefits are probable.

Gleim's CIA Test Prep: Part III: Business Analysis and Information TechnologyAnswer Explanations

(1312 questions)

Copyright 2008 Gleim Publications, Inc. Page 144Printed for Mamdouh Farag

Page 145: P.3 Answer Explanation

[483] Gleim #: 4.1.1 -- Source: CIA 597 IV-21

Answer (A) is correct. A defined contribution plan provides benefits in exchange for services, provides an account foreach participant, and specifies how contributions are to be determined. Postemployment benefits depend only oncontributions, returns on investment, and allocated forfeitures of other participants’ benefits. Thus, employees have thebenefit of gain and the risk of loss.

Answer (B) is incorrect because the employees bear the risk of the plan’s investment performance.

Answer (C) is incorrect because, under a defined benefit plan, the postemployment benefits received by employees aredefined by the plan’s formula.

Answer (D) is incorrect because equal contributions are not required for a defined contribution plan.

[484] Gleim #: 4.1.2 -- Source: CIA 595 IV-16

Answer (A) is incorrect because the defined postemployment benefit includes both vested and nonvested benefits and iscalculated at future levels.

Answer (B) is incorrect because the defined postemployment benefit includes both vested and nonvested benefits and iscalculated at future levels.

Answer (C) is incorrect because the defined postemployment benefit includes both vested and nonvested benefits and iscalculated at future levels.

Answer (D) is correct. The measurement of a postemployment benefit obligation includes estimates of future salaryincreases, the benefits defined in the plan, the benefits arising from any constructive obligation beyond the terms of theplan, and estimates of future changes in state benefits that affect the level of plan benefits. The possibility that nonvestedprojected benefits will not vest is a factor in the measurement of the DBO, but it does not affect the existence of theobligation.

[485] Gleim #: 4.1.3 -- Source: CIA 1189 IV-44

Answer (A) is incorrect because past service cost relates to benefits for employee service provided prior to the adoption oramendment of a defined benefit plan.

Answer (B) is incorrect because a defined benefit plan provides a defined benefit based on one or more factors, such aslevel of compensation, years of service, or age.

Answer (C) is correct. Vested benefits are those earned postemployment benefits owed to an employee that are notcontingent upon the employee’s continued service. Whether benefits have vested affects the measurement of theemployer’s defined benefit obligation but not its existence. Moreover, vesting affects the accounting for past service cost.Past service cost is amortized as an expense over the average period until the benefits are vested.

Answer (D) is incorrect because, under IAS 19, the employer does not recognize an additional minimum liability.

Gleim's CIA Test Prep: Part III: Business Analysis and Information TechnologyAnswer Explanations

(1312 questions)

Copyright 2008 Gleim Publications, Inc. Page 145Printed for Mamdouh Farag

Page 146: P.3 Answer Explanation

[486] Gleim #: 4.1.4 -- Source: Publisher

Answer (A) is correct. The amount of the defined benefit liability recognized equals the present value of the definedbenefit obligation at the balance sheet date, plus (minus) unrecognized actuarial gains (losses), minus unrecognized pastservice cost, minus the fair value of plan assets at the balance sheet date. If this amount is negative, it represents an asset.However, the maximum that may be recognized for such an asset is the sum of unrecognized actuarial losses,unrecognized past service cost, and the present value of future refunds from the plan or reductions in future contributions.Moreover, the application of this section should not result in a gain being recognized solely because of an actuarial loss orpast service cost in the current period or in a loss being recognized solely because of an actuarial gain in the currentperiod. Thus, if the excess of the DBO over the fair value of plan assets is constant, net unrecognized actuarial gains willincrease the liability. Net unrecognized actuarial losses and unrecognized past service cost decrease the liability.

Answer (B) is incorrect because net unrecognized actuarial losses decrease the liability.

Answer (C) is incorrect because net unrecognized actuarial losses and unrecognized past service cost decrease theliability.

Answer (D) is incorrect because net unrecognized actuarial gains increase the liability.

[487] Gleim #: 4.1.5 -- Source: Publisher

Answer (A) is incorrect because 120,000 includes only the current service cost component.

Answer (B) is incorrect because 135,000 excludes the amortization of the actuarial loss.

Answer (C) is incorrect because 140,000 excludes the past service cost.

Answer (D) is correct. Components of the expense are current service cost, interest cost, the expected return on planassets, past service cost (recognition in full of vested amounts and amortization of nonvested amounts). Current servicecost, interest cost, the amortization of actuarial loss, and the past service cost increase the expense. The expected returnon plan assets decreases the expense.

Current service cost 120,000Return on plan assets (30,000)Interest cost 40,000Amortization of actuarial loss 10,000Past service cost 5,000Expense 145,000

[488] Gleim #: 4.1.6 -- Source: Publisher

Answer (A) is incorrect because 50,000 is the periodic amortization of nonvested benefits.

Answer (B) is incorrect because 200,000 is the amount of the nonvested benefits.

Answer (C) is correct. Past service cost is the increase in the present value of the DBO related to prior employee servicethat arises in the current period from the introduction of, or an amendment to, postemployment benefits. Accordingly,400,000 should be recognized immediately to reflect the vested benefits and amortization of the nonvested benefits equals50,000 (200,000 ÷ 4), a total of 450,000.

Answer (D) is incorrect because 600,000 includes nonvested benefits not yet required to be amortized.

Gleim's CIA Test Prep: Part III: Business Analysis and Information TechnologyAnswer Explanations

(1312 questions)

Copyright 2008 Gleim Publications, Inc. Page 146Printed for Mamdouh Farag

Page 147: P.3 Answer Explanation

[489] Gleim #: 4.1.7 -- Source: Publisher

Answer (A) is incorrect because 20,000 is the result of using the full 200,000 liability loss without regard to the corridoramount and assumes an amortization period of 10 years instead of 20.

Answer (B) is incorrect because 3,750 is the result of using 125,000 (1,250,000 plan assets × 10%) as the corridor amountinstead of 150,000.

Answer (C) is correct. At a minimum, amortization of the cumulative unrecognized net actuarial gains and losses must beincluded as income or expense if, as of the close of the prior period, that unrecognized gain or loss exceeds 10% of thegreater of the present fair value of the DBO or the fair value of plan assets. At year-end, the present value of the DBO was200,000 greater than estimated (a 200,000 liability loss). Given that no other gain or loss has occurred, the unrecognizedactuarial net loss to be amortized beginning next year is 200,000. The corridor amount is 150,000 (10% of the greater of1,500,000 present value of the DBO or 1,250,000 fair value of plan assets). The amount outside the corridor is 50,000(200,000 – 150,000), and the amount to be amortized is therefore 2,500 (50,000 ÷ 20 years of average remaining workinglife).

Answer (D) is incorrect because 50,000 of the liability loss must be amortized over the average remaining working lifebeginning the year following the loss.

[490] Gleim #: 4.1.8 -- Source: Publisher

Answer (A) is incorrect because 50,000 equals 10% of the beginning fair value.

Answer (B) is incorrect because 60,000 equals 12% of the beginning fair value.

Answer (C) is incorrect because 75,000 is a nonsense number.

Answer (D) is correct. The actual return on plan assets is equal to the difference between the fair value of plan assets atthe beginning and the end of the year adjusted for contributions and benefits paid. Thus, the actual return is 100,000.

FV, end of year 620,000Benefits paid 100,000Contributions (120,000)FV, beginning of year (500,000)Actual return 100,000

[491] Gleim #: 4.1.9 -- Source: Publisher

Answer (A) is incorrect because 410,000 omits the remaining unrecognized actuarial gain.

Answer (B) is correct. A curtailment arises from a reduction in covered employees or amendment of the plan to reducebenefits for future service. A settlement is a transaction that eliminates the DBO for part or all of plan benefits. When acurtailment or settlement occurs, the gain or loss recognized encompasses the change in the present value of the DBO, thechange in the fair value of plan assets, and previously unrecognized related actuarial gains and losses and past servicecost. Thus, the present value of the DBO after the curtailment was 9,000,000 [10,000,000 – (10,000,000 × 10%)]. The fairvalue of plan assets was given as 8,500,000. The unrecognized actuarial gain was reduced to 450,000. The unrecognizedpast service cost was reduced to 90,000. Accordingly, the defined benefit liability immediately after the discontinuancewas 860,000 (9,000,000 – 8,500,000 + 450,000 – 90,000).

Answer (C) is incorrect because 900,000 is 10% of the present value of the remaining DBO.

Answer (D) is incorrect because 950,000 omits past service cost from the calculation.

Gleim's CIA Test Prep: Part III: Business Analysis and Information TechnologyAnswer Explanations

(1312 questions)

Copyright 2008 Gleim Publications, Inc. Page 147Printed for Mamdouh Farag

Page 148: P.3 Answer Explanation

[492] Gleim #: 4.1.10 -- Source: CIA 1195 IV-26

Answer (A) is correct. Under a defined contribution plan, the employer recognizes an expense and a liability for thecontribution payable in exchange for an employee’s services performed during the period. The amount is determined aftersubtracting any contribution already made. However, if the contribution made exceeds the amount due, the excess istreated as a prepaid expense.

Answer (B) is incorrect because an asset is reported only if the contribution is in excess of the amount required.

Answer (C) is incorrect because the employer reports a liability only if the contribution is less than the required amount.

Answer (D) is incorrect because the employer reports a liability only if the contribution is less than the required amount.

[493] Gleim #: 4.1.11 -- Source: IIA, adapted

Answer (A) is correct. Under a defined contribution plan, the company reports an asset on the balance sheet only if thecontribution to the pension trust is greater than the defined, required contribution.

Answer (B) is incorrect because an asset is reported only if the contribution is in excess of the required contribution. If theactual contribution is equal to that required, no asset is reported.

Answer (C) is incorrect because the company would report a liability on the balance sheet only if the contribution was lessthan the required amount, not greater than the required amount.

Answer (D) is incorrect because the company would not report a liability on the balance sheet if it contributed therequired amount to the pension trust.

[494] Gleim #: 4.1.12 -- Source: CIA 1191 IV-45

Answer (A) is incorrect because the IASs provide for no additional minimum liability.

Answer (B) is incorrect because plan assets are assets held by a separate legal entity (fund) to be used to settle theemployee benefit obligation.

Answer (C) is incorrect because the defined benefit obligation (DBO) consists of future amounts required to settle theobligation arising from services provided by employees in the current and prior periods.

Answer (D) is correct. Past service cost is the increase in the present value of the DBO related to prior employee servicethat arises in the current period from the introduction of, or an amendment to, postemployment benefits. Past service costis expensed on a straight-line basis over the average period until vesting. To the extent it is vested upon introduction of, oramendment to, a plan, past service cost is immediately recognized.

[495] Gleim #: 4.2.13 -- Source: Publisher

Answer (A) is incorrect because the intrinsic-value method is allowed only in rare cases.

Answer (B) is correct. Entities must account for share-based employee compensation plans in accordance with the fair-value-based method except in the rare cases in which the fair value of the equity instruments is not reliably measurable atthe measurement date (the grant date for transactions with employees and those providing similar services). In these rarecases, IFRS 2 requires entities to account for these plans in accordance with the intrinsic-value-based method.

Answer (C) is incorrect because the intrinsic-value method is allowed only in rare cases.

Answer (D) is incorrect because the intrinsic-value method is allowed only in rare cases.

Gleim's CIA Test Prep: Part III: Business Analysis and Information TechnologyAnswer Explanations

(1312 questions)

Copyright 2008 Gleim Publications, Inc. Page 148Printed for Mamdouh Farag

Page 149: P.3 Answer Explanation

[496] Gleim #: 4.2.14 -- Source: Publisher

Answer (A) is correct. Under the fair-value-based method prescribed by IFRS 2, compensation expense is measured atthe grant date. This expense is based on the fair value of the award at that date and recognized over the vesting period, theperiod over which the vesting conditions are expected to be satisfied.

Answer (B) is incorrect because the earliest date on which both the number of shares to be issued and the option price areknown, the date on which the options are exercised by employees, and the date the entity forgoes alternative use of theshares to be sold under option may coincide with, but do not define, the measurement date.

Answer (C) is incorrect because the earliest date on which both the number of shares to be issued and the option price areknown, the date on which the options are exercised by employees, and the date the entity forgoes alternative use of theshares to be sold under option may coincide with, but do not define, the measurement date.

Answer (D) is incorrect because the earliest date on which both the number of shares to be issued and the option price areknown, the date on which the options are exercised by employees, and the date the entity forgoes alternative use of theshares to be sold under option may coincide with, but do not define, the measurement date.

[497] Gleim #: 4.2.15 -- Source: CIA 1195 III-58

Answer (A) is correct. The measurement date is January 2, Year 1. At that date, the intrinsic value of the options is30,000 [1,000 shares × (50 market price – 20 option price)]. This 30,000 will be recorded as both compensation expenseand options outstanding. The net effect on equity is 0. When the options are exercised, the 20,000 (1,000 shares × 20exercise price) cash received and the 30,000 options outstanding will be allocated to share capital as 10,000 ordinary(common) stock and 40,000 additional paid-in capital. The net effect on equity will be a 20,000 increase.

Answer (B) is incorrect because 30,000 is the amount of compensation expense.

Answer (C) is incorrect because 50,000 is the increase in the equity without regard to the compensation expense.

Answer (D) is incorrect because 70,000 results from calculating the increase in equity using the share price on theexercise date.

[498] Gleim #: 4.2.16 -- Source: Publisher

Answer (A) is incorrect because the Year 4 expense is 3,000 [20,000 total expected – 15,000 recognized in Years 1-3 –(100 × 20) not vested].

Answer (B) is incorrect because no retrospective adjustment is made. The Year 3 entry would have been 5,000 based on athen-expected total expense of 20,000.

Answer (C) is correct. The fair value of each share option is determined at the measurement date, which is the grant datefor transactions with employees and others providing similar services. Thus, the fair value of each share option was set atits market price of 20 on January 1, Year 1. The periodic expense varies only with the expected number of equityinstruments expected to vest. Because the events causing 100 options not to vest occurred unexpectedly in Year 4, theentity presumably expected at each balance sheet date for the first three years of the vesting period that all options wouldvest. Total expected expense was therefore 20,000, and the proportional expense recognized in each of the first three yearswas 5,000 [(1,000 options × 20) ÷ 4 years].

Answer (D) is incorrect because 20,000 would have been recognized at January 1, Year 1, if the options had vestedimmediately.

Gleim's CIA Test Prep: Part III: Business Analysis and Information TechnologyAnswer Explanations

(1312 questions)

Copyright 2008 Gleim Publications, Inc. Page 149Printed for Mamdouh Farag

Page 150: P.3 Answer Explanation

[499] Gleim #: 4.3.17 -- Source: CIA 1195 IV-28

Answer (A) is correct. When a lease agreement transfers the risks and rewards of ownership to the lessee, it is treated asa finance lease because the transaction is in essence an installment purchase. Accordingly, the lessee records a depreciableasset and a liability. Moreover, IAS 38, Intangible Assets, specifically does not apply to leases that are within the scope ofIAS 17, Leases. A finance lease is therefore presented as an item of property, plant, and equipment, that is, as a tangibleasset.

Answer (B) is incorrect because the recorded asset is a tangible asset.

Answer (C) is incorrect because, if a lease transfers substantially all of the risks and rewards of ownership, it is a financelease.

Answer (D) is incorrect because, if a lease transfers substantially all of the risks and rewards of ownership, it is a financelease. Furthermore, the recorded asset is a tangible asset.

[500] Gleim #: 4.3.18 -- Source: CIA 596 IV-32

Answer (A) is incorrect because the lessee obtains use of the asset.

Answer (B) is incorrect because the lessee uses the lease as a source of financing under a finance lease, not an operatinglease.

Answer (C) is incorrect because the lessee makes payments to the lessor.

Answer (D) is correct. A lease is a rental or sub-purchase arrangement between a lessor (the owner or seller of theproperty) and a lessee (the renter or purchaser). The issue in all leases is whether the risks and rewards of ownership havebeen transferred from the lessor to the lessee; if so, the lease should be accounted for as a sale-purchase, i.e., a financelease. If the risks and rewards of ownership have not transferred, the lease is a rental arrangement and is called anoperating lease. In effect, the lessor provides financing for an installment purchase, and the lessee’s payments includeboth principal and interest components.

[501] Gleim #: 4.3.19 -- Source: CIA 595 IV-27

Answer (A) is incorrect because, under a finance lease, the lessor recognizes a net receivable equal to the net investmentin the lease: gross investment (minimum lease payments from the lessor’s perspective plus unguaranteed residual value)discounted at the interest rate implicit in the lease.

Answer (B) is correct. When a transaction meets the criteria of a finance lease, the lessor removes the leased item fromthe books and records lease payments receivable regardless of whether the lessor is a manufacturer or dealer. The lesseerecords and depreciates the leased item under a finance lease.

Answer (C) is incorrect because the lessee records depreciation on the leased asset under a finance lease. This process isseparate from the accounting for the lease obligation.

Answer (D) is incorrect because, in essence, the leased asset is being purchased when a lease meets the criteria forcapitalization. Hence, the lease agreement represents a form of financing.

Gleim's CIA Test Prep: Part III: Business Analysis and Information TechnologyAnswer Explanations

(1312 questions)

Copyright 2008 Gleim Publications, Inc. Page 150Printed for Mamdouh Farag

Page 151: P.3 Answer Explanation

[502] Gleim #: 4.3.20 -- Source: CIA 596 IV-31

Answer (A) is incorrect because Leases C and D are also finance leases.

Answer (B) is incorrect because B is the only operating lease in the set. If it were usable by the lessee (not the lessor)without major modification, it would normally be classified as a finance lease.

Answer (C) is correct. A lease should be classified as a finance lease by a lessee if it transfers substantially all of therisks and rewards of ownership. A lease is classified at its inception. It normally is classified as a finance lease if, forexample, (1) the lease provides for the transfer of ownership of the leased asset by the end of the lease term; (2) the leasecontains a bargain purchase option, i.e., the lessee has the option to purchase at a price expected to be sufficiently belowthe fair value of the exercise date that, at the lease’s inception, exercise is reasonably certain; (3) the lease term is for themajor part of the economic life of the leased asset; (4) the present value of the minimum lease payments is at leastsubstantially all of the fair value of the leased asset at the inception of the lease; or (5) the leased asset is such that it canbe used only by the lessee without major modification. Lease A is a finance lease because the terms of the lease include abargain purchase option. Lease C passes the economic life test, and lease D passes the recovery of investment test.

Answer (D) is incorrect because Lease A contains a bargain purchase option, so it qualifies as a finance lease.

[503] Gleim #: 4.3.21 -- Source: CIA 1191 IV-44

Answer (A) is correct. The lessee records a finance lease as an asset and a liability at the inception of the lease at the fairvalue of the leased property (not to exceed the present value of the minimum lease payments). The lessee’s minimumlease payments include required payments (excluding contingent rent and costs for services and taxes to be paid by andreimbursed to the lessor) during the lease term and the amount of a bargain purchase option. If no bargain purchase optionexists, the minimum lease payments equal the sum of the minimum payments payable over the lease term and anyamounts guaranteed by the lessee or by a party related to the lessee.

Answer (B) is incorrect because the guaranteed residual must be discounted to present value.

Answer (C) is incorrect because the guaranteed residual value must be discounted to present value.

Answer (D) is incorrect because a guaranteed residual value is part of the lease contract.

[504] Gleim #: 4.3.22 -- Source: H.F. Bush

Answer (A) is incorrect because the guaranteed residual value, not the estimated residual value, must be subtracted fromthe amount initially recorded.

Answer (B) is correct. Depreciation should be consistent with the accounting policy for owned assets. Absent areasonable certainty that the lessee will own the asset at the end of the lease term, it should be fully depreciated over theshorter of the useful life or the lease term. The lease does not transfer ownership or contain a bargain purchase option.Accordingly, the period of amortization should be the lease term. In accordance with the straight-line method used by KWfor owned assets, the depreciable base for this finance lease is equal to the 240,000 initially recorded minus the 50,000guaranteed residual value allocated equally over the 4-year lease term. Consequently, annual depreciation expense is47,500 [(240,000 – 50,000) ÷ 4 years].

Answer (C) is incorrect because the guaranteed residual value, not the estimated residual value, must be subtracted fromthe initially recorded amount, and the term of the lease, not the estimated economic life, is used as the denominator in thedepreciation calculation.

Answer (D) is incorrect because the term of the lease, not the estimated economic life, must be used as the denominator inthe depreciation calculation.

Gleim's CIA Test Prep: Part III: Business Analysis and Information TechnologyAnswer Explanations

(1312 questions)

Copyright 2008 Gleim Publications, Inc. Page 151Printed for Mamdouh Farag

Page 152: P.3 Answer Explanation

[505] Gleim #: 4.3.23 -- Source: CMA 1293 2-27

Answer (A) is incorrect because Lease A does not qualify as a finance lease.

Answer (B) is incorrect because rental expense is 15,000.

Answer (C) is incorrect because the actual cash outlay for rent, 15,000, is charged to expense.

Answer (D) is correct. Lease A is an operating lease with a 15,000 annual rental expense with annual executory costs(e.g., maintenance, insurance, and taxes) of 800 to be paid by the lessee. An operating lease does not transfer the risks andrewards of ownership to the lessee. Lease A is nothing more than a rental arrangement. Circumstances in which the risksand rewards of ownership are normally deemed to be transferred include the following: the lease transfers title to thelessee, the lease has a bargain purchase option, the lease term is for the major part of the useful life of the leased asset,the present value of the minimum lease payments is at least substantially all of the asset’s fair value, or the asset is usableonly by the lessee without major modification.

[506] Gleim #: 4.3.24 -- Source: CMA 1293 2-28

Answer (A) is incorrect because the initial asset amount cannot exceed the fair value of the leased asset. Moreover,10,960 includes the present value of the executory costs.

Answer (B) is incorrect because 10,200 is the fair value of the leased asset.

Answer (C) is incorrect because the lease meets the criteria of a finance lease.

Answer (D) is correct. A finance lease is one in which the risks and rewards of ownership are transferred to the lessee.For accounting purposes, the lessee treats a finance lease as similar to the purchase of an asset capitalized at the fair valueof the leased asset or, if lower, the present value of the minimum lease payments. The lessee’s minimum lease paymentsinclude the required payments, excluding contingent rent and executory costs (e.g., taxes and insurance), plus anyamounts guaranteed by the lessee or a related party. If a bargain purchase option exists, however, minimum leasepayments equal the required payments plus the amount of the option. If the present value of the minimum lease payments(calculated without guaranteed amounts or a bargain purchase option) is substantially all of the asset’s fair value, the leasenormally is accounted for as a finance lease. Given that the executory costs associated with the lease are to be paid by thelessor, a portion of the lease rental price is for those costs, not for the asset. Consequently, the annual minimum leasepayment equals the annual payment minus the executory costs, or 3,500 (4,000 yearly rental – 500). The present value ofthe minimum lease payments is therefore 9,590 (2.74 × 3,500), which is substantially all of the fair value of the asset.Thus, the lease should be capitalized. The appropriate amount of the initial asset value is the present value of theminimum lease payments calculated above.

[507] Gleim #: 4.3.25 -- Source: J.O. Hall

Answer (A) is correct. For a finance lease, the lessor should record the gross investment in the lease at the undiscountedsum of the minimum lease payments (the total of the lessee’s required payments, excluding contingent rent and costs forservices and taxes to be paid by and reimbursed to the lessor, and any guaranteed residual value) and any unguaranteedresidual value. The gross investment is the same regardless of whether any residual value is guaranteed. The five periodicpayments of 20,000 equal 100,000. The expected residual value including guaranteed and unguaranteed portions equals10,000. The gross investment should be 110,000 (100,000 + 10,000).

Answer (B) is incorrect because 100,000 does not include the residual value in the gross investment.

Answer (C) is incorrect because the annual lease payments should be recorded at their undiscounted value.

Answer (D) is incorrect because the residual value is added to, not subtracted from, the undiscounted lease payments.

Gleim's CIA Test Prep: Part III: Business Analysis and Information TechnologyAnswer Explanations

(1312 questions)

Copyright 2008 Gleim Publications, Inc. Page 152Printed for Mamdouh Farag

Page 153: P.3 Answer Explanation

[508] Gleim #: 4.3.26 -- Source: CIA 585 IV-22

Answer (A) is incorrect because it is the lessee’s journal entry.

Answer (B) is incorrect because the sale should be recorded at the present value of the minimum lease payments, and theunearned finance income should be recorded as the difference between the gross lease payments receivable and thepresent value of this gross investment.

Answer (C) is incorrect because the lease should reflect both cost of goods sold and sales, not the netted gross profit onthe lease.

Answer (D) is correct. For this finance lease, the manufacturer-lessor should recordAs gross investment, the minimum lease payments because there is no residual value1.As net investment, the difference between the gross investment in the lease (a debit to a receivable) and unearnedfinance income (a credit to a liability)

2.

As unearned finance income, the difference between the gross investment and the net investment (gross investmentdiscounted at the interest rate implicit in the lease)

3.

As sales revenue, the fair value of the asset or, if lower, the present value of the minimum lease payments computedat the interest rate implicit in the lease

4.

As cost of goods sold, the cost of the leased asset5.Because the first payment is made at the inception of the lease, the payment structure is that of an annuity due. Salesrevenue is therefore equal to the 10,000 periodic payment times the present value of an annuity due of 1 discounted for 10years at 10% (10,000 × 6.7590 = 67,590).

Given that cash is paid at the beginning of the year, the initial 10,000 cash debit immediately decreases the grossinvestment in the lease (lease payments receivable) from 100,000 to 90,000. The cost of the leased asset (55,000) mustalso be charged to cost of sales and credited to inventory. Finally, at the inception of the lease, unearned finance incomeequals the difference between the gross investment and the sales price (100,000 – 67,590 = 32,410).

[509] Gleim #: 4.3.27 -- Source: CMA 690 3-1

Answer (A) is incorrect because the gross investment is not adjusted for the time value of money or fair value.

Answer (B) is incorrect because the gross investment is not adjusted for the time value of money or fair value.

Answer (C) is correct. For a finance lease, the lessor should record as the gross investment in the lease the amount of theminimum lease payments (payments plus either any bargain purchase option or any residual value guaranteed by thelessee, a party related to the lessee, or by a financially capable party unrelated to the lessor or the lessee) plus anyamounts of unguaranteed residual value. The net investment in the lease is equal to the gross investment, minus unearnedfinance income.

Answer (D) is incorrect because the gross investment is not adjusted for the time value of money or fair value.

[510] Gleim #: 4.3.28 -- Source: CMA 690 3-2

Answer (A) is incorrect because initial direct costs of negotiating and arranging a finance lease are allocated over the leaseterm by a lessor that is not a manufacturer or dealer.

Answer (B) is correct. The initial direct costs of negotiating and arranging a finance lease, e.g., commissions and legalfees, are included in the lease receivable and allocated over the lease term. However, similar costs of a manufacturer- ordealer-lessor incurred in negotiating and arranging a lease are excluded from initial direct costs. Thus, they are expensedwhen the selling profit is recognized (normally the start of the lease term) (IAS 17).

Answer (C) is incorrect because initial direct costs of an operating lease are deferred and expensed over the term of thelease on the same straight-line basis as lease income.

Answer (D) is incorrect because a lessor that is a manufacturer or dealer excludes the costs of negotiating and arranging afinance lease from the net investment. These costs are recognized at the same time as the selling profit.

Gleim's CIA Test Prep: Part III: Business Analysis and Information TechnologyAnswer Explanations

(1312 questions)

Copyright 2008 Gleim Publications, Inc. Page 153Printed for Mamdouh Farag

Page 154: P.3 Answer Explanation

[511] Gleim #: 4.3.29 -- Source: CIA 589 IV-41

Answer (A) is incorrect because a 7-year lease term is less likely to be for the major part of the economic life of the assetthan an 8-year lease term.

Answer (B) is correct. A lease may be classified as either a finance lease or an operating lease by a lessee. A financelease transfers substantially all of the risks and rewards of ownership of the asset to the lessee. A lease is classified at itsinception. It normally is classified as a financial lease if, for example,

The lease provides for the transfer of ownership of the asset by the end of the lease term.1.The lease contains a bargain purchase option; i.e., the lessee has the option to purchase at a price expected to besufficiently below the fair value of the exercise date that, at the lease’s inception, exercise is reasonably certain.

2.

The lease term is for the major part of the economic life of the asset.3.The present value of the minimum lease payments is at least substantially all of the fair value of the leased asset atthe inception of the lease.

4.

The leased assets are such that they can be used only by the lessee without major modification.5.

Thus, a lease term that is 80% (8 ÷ 10) of the economic life of the asset will most likely result in classifying the lease as afinance lease, assuming that 80% is a “major part” of the economic life of the asset.

Answer (C) is incorrect because a present value of the minimum lease payments of only 60,000 is not substantially all ofthe fair value of the leased asset.

Answer (D) is incorrect because a present value of the minimum lease payments of only 70,000 is not substantially all ofthe fair value of the leased asset.

[512] Gleim #: 4.3.30 -- Source: CIA 592 IV-52

Answer (A) is incorrect because finance leases do not normally provide for maintenance service.

Answer (B) is incorrect because finance leases are not normally cancelable.

Answer (C) is correct. A lease may be classified as either a finance lease or an operating lease by a lessee. A financelease transfers substantially all of the risks and rewards of ownership of the asset to the lessee. A lease is classified at itsinception. It normally is classified as a finance lease if, for example,

The lease provides for the transfer of ownership of the asset by the end of the lease term.1.The lease contains a bargain purchase option; i.e., the lessee has the option to purchase at a price expected to besufficiently below the fair value of the exercise date that, at the lease’s inception, exercise is reasonably certain.

2.

The lease term is for the major part of the economic life of the asset.3.The present value of the minimum lease payments is at least substantially all of the fair value of the leased asset atthe inception of the lease.

4.

The leased assets are such that they can be used only by the lessee without major modification.5.

Thus, the rental payments tend to approximate the cost of the leased property plus a return on the investment.

Answer (D) is incorrect because finance leases are used by entities in various industries.

Gleim's CIA Test Prep: Part III: Business Analysis and Information TechnologyAnswer Explanations

(1312 questions)

Copyright 2008 Gleim Publications, Inc. Page 154Printed for Mamdouh Farag

Page 155: P.3 Answer Explanation

[513] Gleim #: 4.3.31 -- Source: CIA 595 IV-26

Answer (A) is incorrect because 64% is the debt-to-assets ratio if the lease is a finance lease.

Answer (B) is incorrect because 100% is the debt-to-equity ratio if the lease is an operating lease. Operating leases aretreated as rental agreements, and neither the asset nor the obligation for future payments is recorded on the balance sheet.The balance of debt remains at 100, so the debt-to-equity ratio is 100% (100 ÷ 100).

Answer (C) is incorrect because 50% is the debt-to-assets ratio if the lease is an operating lease.

Answer (D) is correct. If the lease is a finance lease, both the asset and the obligation for future payments are recorded.The entity would have current assets of 100, debt of 175 (100 + 75), fixed assets of 175 (100 + 75), and equity of 100.Hence, the debt-to-assets ratio would be 64% (175 ÷ 275).

[514] Gleim #: 4.4.32 -- Source: CIA 596 IV-75

Answer (A) is incorrect because this statement describes the deferred method of accounting for deferred income taxes.

Answer (B) is correct. A DTA or DTL is measured at the rates expected to apply when it is realized or settled, based ontax laws and rates enacted or substantively enacted as of the balance sheet date. If different rates apply to different taxableprofit levels, a DTA or DTL is measured based on the average rates expected to apply in the periods when the TDs areexpected to reverse. The tax rate or tax base may vary with the manner of recovery or settlement. For example, one taxrate may apply if an asset is sold immediately and another may apply if it is to be recovered through continued use.

Answer (C) is incorrect because this statement describes the net-of-tax method, which recognizes that future taxability anddeductibility are important factors in the valuation of individual assets and liabilities.

Answer (D) is incorrect because this statement describes the nonallocation or flow-through approach, which does notsupport the calculation and reporting of deferred income tax.

[515] Gleim #: 4.4.33 -- Source: CMA 696 2-9

Answer (A) is incorrect because using an accelerated depreciation method for determining taxable profit (tax loss) resultsin a deferred tax liability.

Answer (B) is incorrect because recognizing installment income on the financial statements but not the tax return resultsin a taxable temporary difference.

Answer (C) is correct. A deferred tax asset records the deferred tax consequences attributable to deductible temporarydifferences and carryforwards. Advance rental receipts accounted for on the accrual basis for determining accountingprofit and on a cash basis for determining taxable profit (tax loss) would give rise to a deferred tax asset. The financialstatements would show no income and no related tax expense because the rental payments apply to future periods. The taxreturn, however, would show the rent as income when the cash was received, and a tax would be due in the year ofreceipt. Because the tax is paid prior to recording accounting profits, it represents an asset that will be recognized as anexpense when income is finally recorded.

Answer (D) is incorrect because recognizing prepaid expenses earlier on the tax return than on the financial statements (asituation akin to the accelerated depreciation of fixed assets) gives rise to a deferred tax liability.

Gleim's CIA Test Prep: Part III: Business Analysis and Information TechnologyAnswer Explanations

(1312 questions)

Copyright 2008 Gleim Publications, Inc. Page 155Printed for Mamdouh Farag

Page 156: P.3 Answer Explanation

[516] Gleim #: 4.4.34 -- Source: Publisher

Answer (A) is incorrect because the taxable amount is 70,000 and the deductible amount is 40,000.

Answer (B) is correct. The difference between the carrying amount of an asset or liability and its tax base is a temporarydifference (TD). A taxable (deductible) TD results in taxable (deductible) amounts in the future when the carrying amountof the asset or liability is recovered or settled. The tax base is the amount attributed for tax purposes to an asset orliability. The tax base of an asset is the amount deductible against future taxable economic benefits when the asset’scarrying amount is recovered. The tax base of a liability is the portion of the carrying amount that will not be deductibleagainst future taxable economic benefits for tax purposes. The tax base of revenue received in advance (a liability) is theportion of the carrying amount taxable in the future. Thus, the 70,000 temporary difference (150,000 carrying amount –80,000 tax base) related to the depreciable assets is classified as a taxable amount. When income, such as rental income,is taxable before being recognized in accounting profit, future sacrifices to provide the rental service or refund amountspaid will result in future tax deductible amounts when the liability is settled. Thus, the 40,000 temporary difference(40,000 carrying amount – 0 tax base) related to the deferred rental revenue is classified as a deductible amount.

Answer (C) is incorrect because the taxable amount is 70,000 and the deductible amount is 40,000.

Answer (D) is incorrect because the taxable amount is 70,000 and the deductible amount is 40,000.

[517] Gleim #: 4.4.35 -- Source: Publisher

Answer (A) is incorrect because the tax base is 1,000.

Answer (B) is incorrect because the tax base is 1,000.

Answer (C) is correct. The difference between the carrying amount of an asset or liability and its tax base is a temporarydifference (TD). A taxable (deductible) TD results in taxable (deductible) amounts in the future when the carrying amountof the asset or liability is recovered or settled. The tax base is the amount attributed for tax purposes to an asset orliability. The tax base of an asset is the amount deductible against future taxable economic benefits when the asset’scarrying amount is recovered. The tax base of a liability is the portion of the carrying amount that will not be deductibleagainst future taxable economic benefits for tax purposes. The tax base of revenue received in advance (a liability) is theportion of the carrying amount taxable in the future. For unearned interest revenue for which the related interest revenuewas taxed on a cash basis, the tax base equals zero (1,000 carrying amount – 1,000 not taxable in the future).

Answer (D) is incorrect because the tax base is 1,000.

[518] Gleim #: 4.4.36 -- Source: CIA 1194 IV-69

Answer (A) is incorrect because the deferred tax liability will increase and then decrease.

Answer (B) is incorrect because the deferred tax liability will increase and then decrease.

Answer (C) is correct. The cumulative deferred tax increases, peaks, and then decreases to zero over the life of the asset.In the early years, the asset is depreciated more quickly for tax purposes than for financial reporting purposes. Thistemporary difference reverses in later years. Hence, in the early years, actual taxes payable will be less than tax expensereported in the financial statements, and a deferred tax liability will be recognized. By the end of the asset’s useful life,cumulative actual taxes paid will equal cumulative reported tax expense, so the deferred tax balance will be zero.

Answer (D) is incorrect because the deferred tax liability will increase and then decrease.

Gleim's CIA Test Prep: Part III: Business Analysis and Information TechnologyAnswer Explanations

(1312 questions)

Copyright 2008 Gleim Publications, Inc. Page 156Printed for Mamdouh Farag

Page 157: P.3 Answer Explanation

[519] Gleim #: 4.4.37 -- Source: CIA 1194 IV-70

Answer (A) is correct. Cash flows are not affected by the basis of accounting used to prepare the financial statements.Accordingly, whether the financial statements are prepared based on the tax basis, the cash basis, or accounting principlesgenerally accepted in a given country, cash flows should be the same. For example, the cash inflow or outflow resultingfrom using an accelerated depreciation method to determine actual tax expense or benefit (the amount paid to or refundedby the taxing authority) is completely unaffected by the depreciation method used in the financial statements. However, ifcash flows are derived indirectly by adjusting profit or loss reported in the financial statements, different adjustments arenecessary to arrive at the same cash flow amounts if different bases of accounting are used in the preparation of thefinancial statements.

Answer (B) is incorrect because cash flows are not affected by the method of depreciation used for reporting purposes.

Answer (C) is incorrect because cash flows are not affected by the method of depreciation used for reporting purposes.

Answer (D) is incorrect because cash flows are not affected by the method of depreciation used for reporting purposes.

[520] Gleim #: 4.4.38 -- Source: Publisher

Answer (A) is incorrect because 15% is the tax rate for the first 50,000 of taxable profit.

Answer (B) is incorrect because 25% is the tax rate for taxable profit over 50,000 but less than 100,000.

Answer (C) is correct. In measuring a deferred tax liability or asset, the objective is to use the enacted or substantivelyenacted tax rate(s) expected to apply to taxable profit in the periods in which the deferred tax liability or asset is expectedto be settled or realized. If different rates apply to different taxable profit levels, a DTA or DTL is measured based on theaverage rates expected to apply in the periods when the TDs are expected to reverse. Accordingly, the applicable tax rateis 27.5%.

Taxable TaxProfit Rate50,000 × 15% = 7,50050,000 × 25% = 12,500

100,000 × 35% = 35,000200,000 55,000

55,000 ÷ 200,000 = 27.5%

Answer (D) is incorrect because 35% is the tax rate for taxable profit over 100,000.

[521] Gleim #: 4.4.39 -- Source: Publisher

Answer (A) is incorrect because deferred tax expense or income reflects changes in deferred tax assets and deferred taxliabilities.

Answer (B) is correct. Deferred tax expense or income is the sum of the changes in the deferred tax assets and deferredtax liabilities. For example, this amount reflects changes relating to the origination or reversal of temporary differences,changes in tax rates, and imposition of new taxes.

Answer (C) is incorrect because deferred tax expense or income reflects changes in deferred tax assets and deferred taxliabilities.

Answer (D) is incorrect because the total tax liability includes the current and deferred tax expense or income for the year.

Gleim's CIA Test Prep: Part III: Business Analysis and Information TechnologyAnswer Explanations

(1312 questions)

Copyright 2008 Gleim Publications, Inc. Page 157Printed for Mamdouh Farag

Page 158: P.3 Answer Explanation

[522] Gleim #: 4.4.40 -- Source: Publisher

Answer (A) is correct. At year-end Year 1, Health entities should report a 120,000 warranty liability in its balance sheet.The warranty liability is equal to the 150,000 warranty expense minus the 30,000 warranty cost actually incurred in Year1. Because warranty costs are not deductible until paid, the tax base of the warranty liability is 0. The result is a 120,000temporary difference (120,000 carrying amount – 0 tax base). When the liability is settled through the actual incurrence ofwarranty costs, the amounts will be deductible. Thus, the temporary difference should be classified as a deductibletemporary difference.

Answer (B) is incorrect because 150,000 equals the warranty expense, not the payable.

Answer (C) is incorrect because warranty costs will result in a deductible amount.

Answer (D) is incorrect because the warranty costs will result in a deductible amount, and the 30,000 actual warrantycosts is currently deductible.

[523] Gleim #: 4.4.41 -- Source: IIA, adapted

Answer (A) is incorrect because it is temporary differences that result in taxable or deductible amounts in some futureyear(s), when the reported amounts of assets are recovered and the reported amounts of liabilities are settled.

Answer (B) is incorrect because temporary differences have deferred tax consequences while the permanent differences donot. Permanent differences affect only the period in which they occur.

Answer (C) is correct. Permanent differences have no deferred tax consequences because they affect only the period inwhich they occur. Permanent differences include (1) items that enter into pre-tax financial income but never into taxableincome and (2) items that enter into taxable income but never into pre-tax financial income. In contrast, temporarydifferences result in taxable or deductible amounts in some future year(s), when the reported amount of assets arerecovered and the reported amount of liabilities are settled. Temporary differences therefore do have deferred taxconsequences while permanent differences do not.

Answer (D) is incorrect because permanent differences, not temporary differences, include items that enter into pre-taxfinancial income but never into taxable income.

[524] Gleim #: 4.4.42 -- Source: Publisher

Answer (A) is incorrect because 87,500 ignores the difference for Year 1.

Answer (B) is incorrect because 125,000 ignores the difference for Year 3.

Answer (C) is correct. In its financial statements issued through December 31, Year 3, Company M has reported1,750,000 (300,000 + 600,000 + 850,000) of income from long-term contracts. In its tax returns for the same period, it hasreported 1,100,000 (400,000 + 700,000) of income from the same sources. The result is a taxable temporary difference.Thus, Company M expects to have future taxable amounts of 650,000 and should recognize a deferred tax liability of162,500 (25% applicable tax rate × 650,000).

Answer (D) is incorrect because 37,500 ignores the differences for Year 1 and Year 2.

Gleim's CIA Test Prep: Part III: Business Analysis and Information TechnologyAnswer Explanations

(1312 questions)

Copyright 2008 Gleim Publications, Inc. Page 158Printed for Mamdouh Farag

Page 159: P.3 Answer Explanation

[525] Gleim #: 4.4.43 -- Source: Publisher

Answer (A) is incorrect because a deferred tax liability (DTL) is recognized for most taxable TDs. However, no DTL isrecognized when it arises from goodwill if the amortization of the goodwill is not deductible for tax purposes. The reasonis that the recognition of a DTL would increase goodwill. In a business combination that is an acquisition, goodwill equalsthe excess of the acquisition cost over the acquirer’s interest in the net fair value of the identifiable net assets, liabilities,and contingent liabilities recognized.

Answer (B) is incorrect because neither a deferred tax liability nor a deferred tax asset is recognized in thesecircumstances.

Answer (C) is correct. A deferred tax asset is recognized for most deductible TDs and for the carryforward of unused taxlosses and credits, but only to the extent it is probable that taxable profit will be available to permit the use of thoseamounts.

Answer (D) is incorrect because no DTA is recognized when it arises from an excess over cost (negative goodwill) treatedas deferred income. The reason is that the recognition of a DTA would increase an excess over cost. In a businesscombination that is an acquisition, an excess over cost equals the excess of the acquirer’s interest in the net fair value ofthe identifiable assets, liabilities, and contingent liabilities recognized over the acquisition cost.

[526] Gleim #: 4.4.44 -- Source: Publisher

Answer (A) is incorrect because a taxable temporary difference usually results in a deferred tax liability in the currentperiod.

Answer (B) is incorrect because a taxable temporary difference usually results in a deferred tax liability in the currentperiod.

Answer (C) is incorrect because a taxable temporary difference usually results in a deferred tax liability in the currentperiod.

Answer (D) is correct. A deferred tax liability is recognized for the deferred tax consequences attributable to most taxabletemporary differences. A deferred tax asset is recognized for the deferred tax consequences attributable to most deductibletemporary differences, but only to the extent it is probable that taxable profit will be available to permit the use of thoseamounts. At the end of Year 2, the cumulative effect of the difference in accounting for financial reporting and taxpurposes is a 300,000 (800,000 – 500,000) temporary difference. Because this 300,000 temporary difference will result infuture taxable amounts, it is a taxable temporary difference for which a deferred tax liability is recognized.

[527] Gleim #: 4.4.45 -- Source: CMA 686 3-7

Answer (A) is correct. Tax expense (tax income) equals the sum of current tax expense (current tax income) and deferredtax expense (deferred tax income). Current tax expense equals taxes paid or payable, and deferred tax expense equals thechange in the entity’s deferred tax assets and liabilities. Because 100,000 of interest income and 80,000 of rental incomeare not subject to tax in Year 1, taxable profit is 3,820,000, and current tax expense is 1,528,000. Deferred tax expenseequals the deferred tax liability (40% × 80,000 = 32,000) at year-end, assuming no deferred tax asset or liability balancesexisted at the beginning of the year. Hence, tax expense is 1,560,000 (1,528,000 + 32,000).

Answer (B) is incorrect because 1,528,000 does not include tax on the rent income.

Answer (C) is incorrect because 1,600,000 includes tax on the tax-exempt interest.

Answer (D) is incorrect because 1,568,000 includes tax on the tax-exempt interest but does not include tax on the rentincome.

Gleim's CIA Test Prep: Part III: Business Analysis and Information TechnologyAnswer Explanations

(1312 questions)

Copyright 2008 Gleim Publications, Inc. Page 159Printed for Mamdouh Farag

Page 160: P.3 Answer Explanation

[528] Gleim #: 4.4.46 -- Source: CMA 686 3-9

Answer (A) is incorrect because 1,960,000 does not include the deferred tax liability from the rent.

Answer (B) is correct. The 5,000,000 of pre-tax profit is reduced by the 100,000 of interest income not subject to tax.However, the rental income accrued in the previous year is taxable in Year 2. Thus, the profit subject to tax is 4,980,000.Given a 40% rate, the current tax expense is 1,992,000.

Answer (C) is incorrect because 2,000,000 includes the tax-exempt interest but does not include the deferred tax liabilityfrom the rent.

Answer (D) is incorrect because 2,032,000 includes the tax-exempt interest.

[529] Gleim #: 4.4.47 -- Source: CMA 686 3-10

Answer (A) is incorrect because the non-taxable interest income does not have deferred tax consequences. The rentalincome created a deferred tax liability in Year 1 that reversed in Year 2. Therefore, no deferred tax amount existed at theend of Year 2.

Answer (B) is incorrect because the non-taxable interest income does not have deferred tax consequences. The rentalincome created a deferred tax liability in Year 1 that reversed in Year 2. Therefore, no deferred tax amount existed at theend of Year 2.

Answer (C) is incorrect because the non-taxable interest income does not have deferred tax consequences. The rentalincome created a deferred tax liability in Year 1 that reversed in Year 2. Therefore, no deferred tax amount existed at theend of Year 2.

Answer (D) is correct. The correct answer is 0. Deferred tax assets arise from deductible temporary differences and thecarryforward of unused tax losses and credits. Deferred tax liabilities arise from taxable temporary differences. Theinterest income not subject to tax resulted in neither future taxable nor future deductible amounts and therefore nodeferred tax item was recognized for it. The only item resulting in a temporary difference was the rental income. Adeferred tax liability would have been created at the end of Year 1. Given that the difference reversed in Year 2, nodeferred amount existed at the end of Year 2.

[530] Gleim #: 4.5.48 -- Source: CIA 597 IV-29

Answer (A) is incorrect because 260,000 is the actual cost of servicing the warranty in Year 2.

Answer (B) is incorrect because 290,000 equals the sum of 2% of Year 2 sales and 3% of Year 1 sales.

Answer (C) is correct. The warranty provision must be matched with revenue in the year of sale. Thus, the provisionrelated to Year 2 sales must be recognized in Year 2 even if actual expenditures will not occur until Year 3. The provisionrelated to Year 2 sales equals 350,000 [7,000,000 × (2% for the year of sale + 3% for the year after the year of sale)].

Answer (D) is incorrect because 370,000 equals the actual cost of servicing the warranty in Year 1 and Year 2.

Gleim's CIA Test Prep: Part III: Business Analysis and Information TechnologyAnswer Explanations

(1312 questions)

Copyright 2008 Gleim Publications, Inc. Page 160Printed for Mamdouh Farag

Page 161: P.3 Answer Explanation

[531] Gleim #: 4.5.49 -- Source: CIA 590 IV-34

Answer (A) is correct. A current liability is an obligation that is expected to be settled within the normal operating cycleor is due to be settled within 12 months of the balance sheet date. Any other liability is noncurrent. Some currentliabilities are included in the working capital employed in the normal operating cycle, e.g., trade payables and accruedemployee operating costs. Current liabilities not settled within the normal operating cycle include the current part ofinterest-bearing debt, dividends, income taxes, and bank overdrafts. Thus, the bonds payable should be classified ascurrent because they are due to be settled within 12 months. Under the Standards, the classification of the sinking-fundassets is irrelevant to the classification of the bond payable.

Answer (B) is incorrect because the bonds should be classified as a current liability.

Answer (C) is incorrect because offsetting assets and liabilities is rarely acceptable.

Answer (D) is incorrect because the bonds are a liability and should not be put in an ambiguous category such as deferredcredits.

[532] Gleim #: 4.5.50 -- Source: CIA 591 IV-36

Answer (A) is correct. Financial liabilities are current if they are due within 12 months even if (1) the original term wasfor more than 12 months and (2) an agreement to refinance on a long-term basis was completed after the balance sheetdate and before the issuance of the financial statements. Thus, both notes are current. The amount excluded from currentliabilities is 0.

Answer (B) is incorrect because the 15% note is also included in current liabilities.

Answer (C) is incorrect because the 17% note is also included in current liabilities.

Answer (D) is incorrect because both liabilities should be treated as current at the balance sheet date.

[533] Gleim #: 4.5.51 -- Source: CIA 1193 IV-39

Answer (A) is incorrect because the cash basis calls for recognizing warranty expense as labor and materials are expendedto satisfy the warranty.

Answer (B) is correct. A provision is a liability of uncertain timing and amount. A liability is a present obligation arisingfrom past events, the settlement of which is expected to result in an outflow of resources embodying economic benefits.Whether a past event results in a present obligation is usually clear. Thus, it is clear from the circumstances that theentity’s sale of goods without warranty is an obligating event that resulted in a present obligation for the issuance ofwarranty costs. Recognition of provisions is appropriate when the entity has a legal or constructive present obligationresulting from a past event (called an obligating event), it is probable that an outflow of economic benefits will benecessary to settle the obligation, and its amount can be reliably estimated. Assuming that the amount of warranty costscan be reliably estimated (although they are uncertain in timing and amount compared with a trade payable, for example)and that the outflow is probable (in these circumstances, “more likely than not”), the manufacturer’s contractual presentobligation should result in recognition of a provision.

Answer (C) is incorrect because the sales warranty method is appropriate for situations when a warranty is sold separatelyfrom the product.

Answer (D) is incorrect because the method of accounting for warranties for tax purposes is the cash basis. The cash basisis unacceptable for accounting purposes because it violates the matching principle.

Gleim's CIA Test Prep: Part III: Business Analysis and Information TechnologyAnswer Explanations

(1312 questions)

Copyright 2008 Gleim Publications, Inc. Page 161Printed for Mamdouh Farag

Page 162: P.3 Answer Explanation

[534] Gleim #: 4.5.52 -- Source: CIA 1192 IV-31

Answer (A) is correct. A current liability is an obligation that is expected to be settled within the normal operating cycleor is due to be settled within 12 months of the balance sheet date. Any other liability is noncurrent. Some currentliabilities are included in the working capital employed in the normal operation cycle, e.g., trade payables and accruedemployee operating costs. Current liabilities not settled within the normal operating cycle include the current part ofinterest-bearing debt, dividends, income taxes, and bank overdrafts. Given that the bonds payable and interest payable aredue within 12 months, they should be classified and current.

Answer (B) is incorrect because the interest payable should be classified as a current liability. It is due within a year afterthe balance sheet date.

Answer (C) is incorrect because the balance of bonds payable should be classified as a current liability. The bonds are duewithin a year after the balance sheet date.

Answer (D) is incorrect because both the balance of bonds payable and interest payable should be classified as currentliabilities.

[535] Gleim #: 4.5.53 -- Source: CIA 1191 IV-41

Answer (A) is incorrect because recognition occurs at the time of the absences if the benefits are not accumulating.

Answer (B) is correct. Short-term employee benefits expected to be paid as a result of service rendered during the periodordinarily should be recognized as an expense and a liability (accrued expense). For short-term compensated absences, thetiming of recognition depends on whether the benefits accumulate. If the benefits for compensated absences accumulate,the expected cost of short-term compensated absences is recognized when services are rendered that increase theemployees’ entitlement to future compensated absences. The obligation is recognized whether it is vesting (the employeeis entitled to a cash payment for an unused entitlement upon leaving the entity) or not vesting. The amount should not bediscounted. It equals the additional amount expected to be paid as a result of the unused accumulated entitlement at thebalance sheet date. Hence, the entity should debit expense and credit liability for 100,000 because the entitlementaccumulates and the employees have rendered services during the period that increase their future entitlement.

Answer (C) is incorrect because a liability rather than an asset is recognized.

Answer (D) is incorrect because the expense is recognized in the income statement.

[536] Gleim #: 4.5.54 -- Source: CIA 1190 IV-37

Answer (A) is correct. An expense and a provision should be accrued for the coupons still outstanding that are expectedto be redeemed. Of the 4 million coupons distributed, 40%, or 1.6 million, are estimated to be redeemable. Of those, 1million have already been redeemed, and 600,000 more are expected to be redeemed. The promotion requires 20 couponsto receive one toy, so 30,000 (600,000 ÷ 20) more toys will be required. Each toy costs 3.00, creating a provision of90,000 (30,000 × 3.00).

Answer (B) is incorrect because the debit should be to an expense.

Answer (C) is incorrect because, although an expense should be accrued, the amount is incorrect.

Answer (D) is incorrect because the debit should be to an expense, and the amount is incorrect.

Gleim's CIA Test Prep: Part III: Business Analysis and Information TechnologyAnswer Explanations

(1312 questions)

Copyright 2008 Gleim Publications, Inc. Page 162Printed for Mamdouh Farag

Page 163: P.3 Answer Explanation

[537] Gleim #: 4.5.55 -- Source: CIA 1192 IV-29

Answer (A) is incorrect because an accrued revenue is revenue that has met the recognition criteria but has not beenreceived.

Answer (B) is correct. Income, which includes revenue and gains, is recognized in the income statement when an increasein future economic benefits related to an increase in an asset or a decrease in a liability can be reliably measured. Revenueis recognized (reported as revenue) in the period in which the recognition criteria are met; therefore, when it is received inadvance, the amount applicable to future periods is deferred. This deferral reflects the uncertainty of the reliablemeasurement of the future economic benefits. The uncertainty arises because the entity still must satisfy an obligation toperform in the future before it is entitled to the future economic benefits. The amount received in advance is considered aliability because it represents a present obligation arising from a past event. Accordingly, deferred or unearned revenue isan amount that has been received but that has not met the recognition criteria for revenue.

Answer (C) is incorrect because the revenue will be recognized in future periods when forthcoming issues of the magazineare published and distributed to the subscribers.

Answer (D) is incorrect because there is no such thing as a precollected receivable. Precollected revenue is deferredrevenue, which is an amount received that has not met the recognition criteria (classified as a liability). A subscriptionreceivable (an asset) would arise from accrued revenue, which is revenue not yet received.

[538] Gleim #: 4.5.56 -- Source: CIA 592 IV-38

Answer (A) is incorrect because deposits meet the definition of liabilities, not revenue. Revenue is income that arises inthe ordinary activities of the entity. Income is an increase in economic benefits in the form of inflows or enhancements ofassets or decreases of liabilities that result in an increase in equity (excluding transactions with owners).

Answer (B) is incorrect because deposits meet the definition of liabilities, not revenue. Revenue is income that arises inthe ordinary activities of the entity. Income is an increase in economic benefits in the form of inflows or enhancements ofassets or decreases of liabilities that result in an increase in equity (excluding transactions with owners).

Answer (C) is incorrect because deposits are liabilities, not equity items. The equity of an entity is the residual interest inthe assets of an entity that remains after deducting its liabilities.

Answer (D) is correct. Liabilities are present obligations arising from past events, the settlement of which is expected toresult in an outflow of resources embodying economic benefits. Customers’ deposits must be returned or credited to theiraccounts. The deposits should therefore be recorded as liabilities.

[539] Gleim #: 4.5.57 -- Source: CIA 590 IV-28

Answer (A) is incorrect because an accrued revenue has met the recognition criteria but has not yet been received. Thejournal entry described indicates that collection has been made.

Answer (B) is incorrect because the entry concerns a revenue rather than an expense transaction.

Answer (C) is incorrect because the entry concerns a revenue rather than an expense transaction.

Answer (D) is correct. A deferred revenue is a revenue item that has been received but has not met the recognitioncriteria. The journal entry described in the question is an adjusting entry to transfer an amount from the revenue accountto a liability (deferred revenue) account. The initial collection of cash in advance from the tenant was apparently recordedby a credit to revenue. An adjusting entry is therefore required at year-end to transfer any remaining amount that does notqualify for revenue recognition.

Gleim's CIA Test Prep: Part III: Business Analysis and Information TechnologyAnswer Explanations

(1312 questions)

Copyright 2008 Gleim Publications, Inc. Page 163Printed for Mamdouh Farag

Page 164: P.3 Answer Explanation

[540] Gleim #: 4.5.58 -- Source: CIA 594 IV-15

Answer (A) is incorrect because the double payment of a liability does not affect expenses of the period so it does notaffect profit and equity.

Answer (B) is incorrect because assets will be reduced.

Answer (C) is correct. When a liability is paid, an entry debiting accounts payable and crediting cash is made. If an entityerroneously pays a liability twice, the accounts payable and cash accounts will be understated by the amount of theliability. Hence, assets and liabilities will be understated.

Answer (D) is incorrect because both assets and liabilities will be understated, whereas profit and equity will beunaffected.

[541] Gleim #: 4.5.59 -- Source: CIA 593 IV-36

Answer (A) is incorrect because 130,000 ignores the errors related to prepaid rent and prepaid advertising expense.

Answer (B) is correct. The computation is as follows:

Error Effect on Expense Effect on ProfitFailure to accrue

interest expense Understate 50,000 Overstate 50,000Failure to record

depreciation Understate 80,000 Overstate 80,000Failure to amortize

prepaid rent expense Understate 100,000 Overstate 100,000Failure to recognize

prepaid advertising Overstate 60,000 Understate 60,000

Totals Understate 170,000 Overstate 170,000

Answer (C) is incorrect because 230,000 ignores the error related to prepaid advertising expense.

Answer (D) is incorrect because 290,000 treats the error related to prepaid advertising expense as an understatement ofexpenses.

[542] Gleim #: 4.6.60 -- Source: CIA 597 IV-36

Answer (A) is correct. A provision is a liability of uncertain timing or amount. Recognition of provisions is appropriatewhen the entity has a legal or constructive present obligation resulting from a past event (called an obligating event), it isprobable that an outflow of economic benefits will be necessary to settle the obligation, and its amount can be reliablyestimated. Consequently, the company must recognize a loss and a liability for 2.5 million.

Answer (B) is incorrect because such an adjustment is appropriate for fundamental errors and changes in accountingpolicies (under the benchmark treatments).

Answer (C) is incorrect because an appropriation of retained earnings is permissible although not required, but the entitymust still recognize a loss and a provision. Moreover, no part of the appropriation may be transferred to income, and noloss may be charged to an appropriation of retained earnings.

Answer (D) is incorrect because, if the loss is probable and can be reliably estimated, it should be recognized by a chargeto income.

Gleim's CIA Test Prep: Part III: Business Analysis and Information TechnologyAnswer Explanations

(1312 questions)

Copyright 2008 Gleim Publications, Inc. Page 164Printed for Mamdouh Farag

Page 165: P.3 Answer Explanation

[543] Gleim #: 4.6.61 -- Source: CIA 1192 IV-45

Answer (A) is incorrect because the criteria for recognition of a provision have been met, including a reliable estimate ofthe range of the obligation.

Answer (B) is incorrect because the loss is probable.

Answer (C) is incorrect because the loss is not deferred; it is accrued.

Answer (D) is correct. A provision is a liability of uncertain timing or amount. Recognition of provisions is appropriatewhen the entity has a legal or constructive present obligation resulting from a past event (called an obligating event), it isprobable that an outflow of economic benefits will be necessary to settle the obligation, and its amount can be reliablyestimated. Thus, a provision should be recognized. Moreover, the amount recognized should be the best estimate of theexpenditure required to settle the obligation. However, the amount within the reliable estimate of the range of theobligation that will be recognized will vary from country to country.

[544] Gleim #: 4.6.62 -- Source: CIA 596 IV-25

Answer (A) is incorrect because no obligating event has occurred. The entity could avoid the future expenditure by itsfuture actions.

Answer (B) is incorrect because no obligating event has occurred. The entity could avoid the future expenditure by itsfuture actions.

Answer (C) is incorrect because no obligating event has occurred. The entity could avoid the future expenditure by itsfuture actions.

Answer (D) is correct. When premiums are offered to customers, for example, upon redemption of coupons, the entity canusually establish that it has a legal present obligation resulting from a past event and that an outflow of economic benefitsis probable. Furthermore, if the entity has prior experience with such offers or information about the experience of similarentities, a reliable estimate of the obligation should be feasible.

[545] Gleim #: 4.6.63 -- Source: CIA 595 IV-20

Answer (A) is incorrect because the time period in which the obligating event occurred is relevant. If it arose after the dateof the financial statements, a provision may not be recognized in those statements.

Answer (B) is incorrect because a provision is not recognized unless it is probable that an outflow of resources embodyingeconomic benefits will be required to settle a present obligation arising from a past event.

Answer (C) is incorrect because a provision is not recognized unless it is probable that an outflow of resources embodyingeconomic benefits will be required to settle a present obligation arising from a past event. Moreover, the amount of theobligation should be capable of reliable estimation.

Answer (D) is correct. The number of parties involved in the litigation is irrelevant. For example, the same accountingtreatment is applied whether a claim is brought by an individual or in a class action suit.

Gleim's CIA Test Prep: Part III: Business Analysis and Information TechnologyAnswer Explanations

(1312 questions)

Copyright 2008 Gleim Publications, Inc. Page 165Printed for Mamdouh Farag

Page 166: P.3 Answer Explanation

[546] Gleim #: 4.6.64 -- Source: CIA 589 IV-33

Answer (A) is incorrect because a journal entry is made when the outflow in settlement is probable and can be reliablyestimated.

Answer (B) is correct. In the very rare case in which a reliable estimate of an obligation that otherwise qualifies fortreatment as a provision cannot be determined, no liability is recognized. Instead, the existing liability is disclosed as acontingent liability (unless the possibility of any outflow in settlement is remote).

Answer (C) is incorrect because a disclosure must be made of a contingent liability.

Answer (D) is incorrect because a journal entry is made when the outflow in settlement is probable and can be reliablyestimated.

[547] Gleim #: 4.6.65 -- Source: Publisher

Answer (A) is incorrect because the contingent liability is required to be neither recognized nor disclosed.

Answer (B) is incorrect because the contingent liability is required to be neither recognized nor disclosed.

Answer (C) is correct. A contingent liability includes a present obligation for which an outflow of resources embodyingeconomic benefits is not probable. A contingent liability is not recognized but is disclosed unless the possibility of theoutflow is remote.

Answer (D) is incorrect because the contingent liability is required to be neither recognized nor disclosed.

[548] Gleim #: 4.6.66 -- Source: IIA, adapted

Answer (A) is incorrect because there is no uncertainty regarding the amount of rent. Rent expense can be accrued as salesoccur.

Answer (B) is incorrect because a service was received and the company owes an amount. The amount is not contingenton a future event. The company can accrue the amount that it expected the invoice to show.

Answer (C) is incorrect because as of the date of the interim financial statements, the income tax is payable becauseearnings have occurred. There is no uncertainty regarding the amount or the timing of the payment as of the date of theinterim financial statements.

Answer (D) is correct. This is a guarantee. The liability is contingent on the lessor’s not receiving the full residual valuefrom a third party.

Gleim's CIA Test Prep: Part III: Business Analysis and Information TechnologyAnswer Explanations

(1312 questions)

Copyright 2008 Gleim Publications, Inc. Page 166Printed for Mamdouh Farag

Page 167: P.3 Answer Explanation

[549] Gleim #: 4.7.67 -- Source: CIA 597 IV-30

Answer (A) is correct. Interest expense equals the carrying amount of the liability at the beginning of the period times theeffective interest rate. The carrying amount of the liability equals the face amount of the bond minus the discount. As thediscount is amortized over the life of the bond, the carrying amount increases. Consequently, the interest expenseincreases over the term of the bond.

Answer (B) is incorrect because interest expense will increase over the term of the bonds.

Answer (C) is incorrect because interest expense exceeds the cash interest payment when bonds are issued at a discount.The reason is that the effective rate is higher than the nominal rate. The excess of interest expense over the cash paymentis the amount of discount amortized each period.

Answer (D) is incorrect because interest expense exceeds the cash interest payment when bonds are issued at a discount.The reason is that the effective rate is higher than the nominal rate. The excess of interest expense over the cash paymentis the amount of discount amortized each period.

[550] Gleim #: 4.7.68 -- Source: CIA 595 IV-19

Answer (A) is incorrect because 990,000 is the result if 1 month of accrued interest is deducted from, rather than addedto, the amount received.

Answer (B) is incorrect because the purchasers must pay for the accrued interest from the last interest date to the issuedate. They will receive 6 months’ interest on July 1 despite holding the bonds for 5 months.

Answer (C) is correct. The amount the issuing entity receives on 2/1/Yr 1 is the face amount of the issue plus 1 month ofaccrued interest, or 1,010,000 {1,000,000 + [(1,000,000 × 12%) ÷ 12]}.

Answer (D) is incorrect because 1,020,000 results from adding 2 months of accrued interest to the face amount.

[551] Gleim #: 4.7.69 -- Source: CIA 1196 IV-19

Answer (A) is correct. The entry is to debit interest expense, debit bond premium, and credit cash paid. Thus, theamortization of a premium on bonds payable reduces the interest expense, thereby increasing profit or loss.

Answer (B) is incorrect because the amortization of a premium on bonds payable reduces interest expense.

Answer (C) is incorrect because interest income is not affected by the amortization of a premium on bonds payable.

Answer (D) is incorrect because interest income is not affected by the amortization of a premium on bonds payable.

Gleim's CIA Test Prep: Part III: Business Analysis and Information TechnologyAnswer Explanations

(1312 questions)

Copyright 2008 Gleim Publications, Inc. Page 167Printed for Mamdouh Farag

Page 168: P.3 Answer Explanation

[552] Gleim #: 4.7.70 -- Source: CIA 1186 IV-30

Answer (A) is incorrect because the bond premium is 15,000 (500,000 × .03), and interest payable should be credited.

Answer (B) is incorrect because interest payable should be 20,000 [500,000 × .12 × (4 ÷ 12)].

Answer (C) is correct. The face amount of the 500 bonds is equal to 500,000 (500 × 1,000). The cash proceeds excludinginterest from the issuance of the bonds are 515,000 (500,000 × 103%). The 15,000 premium is the difference between thecash issuance proceeds and the face amount of the bonds. Because the bonds were issued between interest payment dates,the issuer is also entitled to receive accrued interest for the 4 months between the prior interest date and the issuance date.The accrued interest is 20,000 [500 bonds × 1,000 face value × 12% stated rate × (4 ÷ 12)]. The issuing company willtherefore receive 535,000 in cash (515,000 + 20,000). The resulting journal entry includes a 535,000 debit to cash, a500,000 credit to bonds payable, a 15,000 credit to premium, and a 20,000 credit to either interest payable or interestexpense.

Answer (D) is incorrect because the premium on bonds payable should not include interest payable.

[553] Gleim #: 4.8.71 -- Source: CIA 597 IV-2

Answer (A) is incorrect because 2,000 ignores the 15,000 increase in share premium.

Answer (B) is correct. Assets equals liabilities plus equity. Given an increase of 180,000 in assets, the sum of liabilitiesand equity must also have increased by 180,000. Because liabilities, share capital, and share premium increased by155,000 (50,000 + 90,000 + 15,000), retained earnings must have increased by 25,000 (180,000 – 155,000). Given profitof 42,000, dividends declared must have been 17,000 (42,000 – 25,000).

Answer (C) is incorrect because 33,000 equals the increase in assets minus the sum of the increases in share capitalaccounts and profit.

Answer (D) is incorrect because 67,000 equals profit plus the increase in net assets other than from owner contributions.

[554] Gleim #: 4.8.72 -- Source: CIA 591 IV-37

Answer (A) is incorrect because the ordinary (common) shares account balance is not affected when treasury shares areacquired.

Answer (B) is incorrect because share premium is not affected when treasury shares are acquired and accounted for by thecost method.

Answer (C) is incorrect because retained earnings is not affected by treasury share acquisitions when the cost method isused.

Answer (D) is correct. Using the cost method, the journal entry to record the acquisition of the treasury shares includes adebit to treasury shares for 60,000. The balance of the treasury shares account is classified as a contra equity item. Thus,the acquisition of the treasury shares reduces total equity by 60,000 (2,000 shares × 30 = 60,000).

Gleim's CIA Test Prep: Part III: Business Analysis and Information TechnologyAnswer Explanations

(1312 questions)

Copyright 2008 Gleim Publications, Inc. Page 168Printed for Mamdouh Farag

Page 169: P.3 Answer Explanation

[555] Gleim #: 4.8.73 -- Source: CIA 1188 IV-36

Answer (A) is incorrect because the 30,000 excess of cash paid over the carrying amount of the redeemed shares should bedebited to retained earnings.

Answer (B) is incorrect because the 30,000 excess of cash paid over the carrying amount of the redeemed shares should bedebited to retained earnings.

Answer (C) is incorrect because the premium on the preference (preferred) shares must be debited for only 20,000.Moreover, retained earnings must also be debited for the difference of 30,000.

Answer (D) is correct. The exercise of the call provision resulted in the redemption of the 10,000 preference (preferred)shares issued and outstanding at the call price of 550,000 (10,000 shares × 55 call price per share). To eliminate thecarrying amount of the preference (preferred) shares and recognize the cash paid in this transaction, the required journalentry is to debit preference (preferred) shares for 500,000, debit share premium: preference (preferred) for 20,000, andcredit cash for 550,000. The difference of 30,000 (550,000 cash – 520,000 carrying amount of the preference (preferred)shares) is charged to equity (debit retained earnings). No loss is reported because the recognition of a gain or loss ontransactions involving an entity’s own shares is prohibited.

[556] Gleim #: 4.8.74 -- Source: Publisher

Answer (A) is incorrect because an appropriation of retained earnings for self-insurance is permissible.

Answer (B) is correct. Accrual of an expense prior to the occurrence of the event for which an entity is self-insuredshould not be permitted. This rule holds because the fair value of the property diminishes only if the event actually occurs.But an appropriation of retained earnings is acceptable to disclose the self-insurance policy if, when a fire loss occurs, theentry appropriating retained earnings is reversed, and the loss is charged against income of the period of loss and notagainst retained earnings.

Answer (C) is incorrect because fire losses may never be charged against the appropriation of retained earnings.

Answer (D) is incorrect because the procedure is acceptable only if the appropriation is shown within the equity section ofthe balance sheet.

[557] Gleim #: 4.8.75 -- Source: CIA 580 IV-8

Answer (A) is incorrect because share dividends have no effect on total equity or on the carrying amount of an individualshareholder’s investment.

Answer (B) is incorrect because share splits have no effect on total equity or on the carrying amount of an individualshareholder’s investment.

Answer (C) is incorrect because dividends, whether of shares, cash, or property, are usually payable on a date differentfrom the declaration date.

Answer (D) is correct. The purpose of a share dividend is to provide evidence to the shareholders of their interest inaccumulated earnings without distribution of cash or other property. Share dividends are typically accounted for by atransfer from retained earnings at fair value.

Gleim's CIA Test Prep: Part III: Business Analysis and Information TechnologyAnswer Explanations

(1312 questions)

Copyright 2008 Gleim Publications, Inc. Page 169Printed for Mamdouh Farag

Page 170: P.3 Answer Explanation

[558] Gleim #: 4.8.76 -- Source: CMA 695 1-11

Answer (A) is incorrect because 350,000 is the ordinary (common) shares dividend.

Answer (B) is incorrect because 380,000 omits the 30,000 of cumulative dividends for Year 1.

Answer (C) is incorrect because 206,000 is based on a flat rate of 1 per share.

Answer (D) is correct. If an entity has cumulative preference (preferred) shares, all preference (preferred) dividends forthe current and any unpaid prior years must be paid before any dividends can be paid on ordinary (common) shares. Thetotal preference (preferred) dividends that must be paid equal 60,000 (6,000 shares × 100 par × 5% × 2 years), and theordinary (common) dividend is 350,000 (1,750,000 × 20%), for a total of 410,000.

[559] Gleim #: 4.8.77 -- Source: Publisher

Answer (A) is incorrect because 3,000 is the excess of the fair value of 2,000 rights over the sale price of 1,000 rights.

Answer (B) is correct. The recipient of share purchase warrants should allocate the carrying amount of the shares ownedbetween those shares and the rights based on their relative fair values. Thus, the amounts to be allocated to the ordinary(common) shares and warrants are 47,250 ({(49 × 1,000) ÷ [(49 × 1,000) + (3.50 × 2,000)]} × 54,000) and 6,750 (54,000– 47,250), respectively. The realized gain is therefore 625 [4,000 – (6,750 × 50%)].

Answer (C) is incorrect because 500 equals the excess of the sale price of 1,000 rights over their fair value.

Answer (D) is incorrect because Starr should recognize a realized gain for the excess of the price over the carryingamount.

[560] Gleim #: 4.8.78 -- Source: CIA 1194 IV-50

Answer (A) is incorrect because the split results in a greater number of shares outstanding and a lower EPS than resultsfrom the share dividend.

Answer (B) is incorrect because the split results in a greater number of shares outstanding and a lower EPS than resultsfrom the share dividend.

Answer (C) is correct. The share split will double the number of shares outstanding to 2,000. The 10% share dividendwill increase the number of outstanding shares to 1,100. The higher number of shares in the split will result in a lowerearnings per share than will result from the share dividend.

Answer (D) is incorrect because the split results in a greater number of shares outstanding and a lower EPS than resultsfrom the share dividend.

[561] Gleim #: 4.8.79 -- Source: CIA 1194 IV-51

Answer (A) is incorrect because par value per share does not change following a share dividend.

Answer (B) is incorrect because par value per share decreases following a share split.

Answer (C) is incorrect because par value per share does not change following a share dividend.

Answer (D) is correct. A share split results in a lower par value per share because the total number of shares increasesbut the total par value of outstanding share does not change.

Gleim's CIA Test Prep: Part III: Business Analysis and Information TechnologyAnswer Explanations

(1312 questions)

Copyright 2008 Gleim Publications, Inc. Page 170Printed for Mamdouh Farag

Page 171: P.3 Answer Explanation

[562] Gleim #: 4.8.80 -- Source: CIA 595 IV-24

Answer (A) is incorrect because goodwill must be recorded.

Answer (B) is incorrect because 100,000 is the excess of the acquisition cost over the carrying amount.

Answer (C) is correct. Goodwill equals the excess of the acquisition cost over the acquirer’s interest in the net fair valueof the identifiable assets, liabilities, and contingent liabilities recognized. Consequently, goodwill is 200,000 (1,000,000 –800,000).

Answer (D) is incorrect because 300,000 equals goodwill plus the excess of the carrying amount over fair value.

[563] Gleim #: 4.8.81 -- Source: CIA 1194 IV-21

Answer (A) is incorrect because the lower-of-cost-or-market method is not generally used for equity investments.

Answer (B) is incorrect because the cost basis is used when the investor cannot exercise significant influence over theinvestee (it has less than 20% of the voting power of the investee) and market prices are not readily available.

Answer (C) is incorrect because consolidated reporting is ordinarily required only when the investor controls the investee.

Answer (D) is correct. If an investor can exercise significant influence over an investee, the investment should beaccounted for by the equity method. When a corporation owns 20% or more of the voting power of the investee, the abilityto exercise significant influence is presumed.

[564] Gleim #: 4.8.82 -- Source: CIA 596 IV-30

Answer (A) is incorrect because goodwill is positive, not negative.

Answer (B) is incorrect because $50 is based on carrying amounts.

Answer (C) is correct. Goodwill is the excess of the cost over the net fair value of the acquired identifiable assets,liabilities, and contingent liabilities recognized. This net fair value equals the sum of cash, receivables, inventory, andPPE, minus liabilities. Hence, the net fair value acquired is $700, and goodwill is $200 ($900 cost – $700).

Answer (D) is incorrect because the goodwill recorded is $200.

[565] Gleim #: 4.8.83 -- Source: CIA 1191 IV-46

Answer (A) is correct. In a consolidated balance sheet, reciprocal balances, such as receivables and payables, between aparent and a consolidated subsidiary should be eliminated in their entirety regardless of the portion of the subsidiary’sshares held by the parent. Thus, all effects of the 1,000,000 loan should be eliminated in the preparation of the year-endconsolidated balance sheet.

Answer (B) is incorrect because the interest must be eliminated.

Answer (C) is incorrect because all aspects of the transaction must be eliminated.

Answer (D) is incorrect because all aspects of the transaction must be eliminated.

Gleim's CIA Test Prep: Part III: Business Analysis and Information TechnologyAnswer Explanations

(1312 questions)

Copyright 2008 Gleim Publications, Inc. Page 171Printed for Mamdouh Farag

Page 172: P.3 Answer Explanation

[566] Gleim #: 4.8.84 -- Source: CIA 593 IV-37

Answer (A) is incorrect because the interest expense is a decreasing amount each period. It is computed by applying aconstant rate to a decreasing carrying amount.

Answer (B) is incorrect because the interest expense is a decreasing amount each period. It is computed by applying aconstant rate to a decreasing carrying amount.

Answer (C) is correct. When the effective interest rate method is used, interest expense equals the effective rate (aconstant rate) times the carrying amount at the beginning of the period. The carrying amount is the par value plus thebalance of the unamortized premium. The difference between interest expense and the nominal interest is the premiumamortization for the period. Thus, interest expense is a decreasing amount each period because a constant rate is appliedto a decreasing carrying amount.

Answer (D) is incorrect because the interest expense is a decreasing amount each period. It is computed by applying aconstant rate to a decreasing carrying amount.

[567] Gleim #: 4.8.85 -- Source: CIA 590 IV-41

Answer (A) is incorrect because 40,000 is the beginning liability balance.

Answer (B) is incorrect because 50,000 equals actual expenditures minus the beginning liability.

Answer (C) is incorrect because 90,000 equals current-year expenditures.

Answer (D) is correct. Recognition of provisions is appropriate when the entity has a legal or constructive presentobligation resulting from a past event (called an obligating event), it is probable that an outflow of economic benefits willbe necessary to settle the obligation, and its amount can be reliably estimated. Accordingly, a provision should berecognized for warranty expense in the period in which the matching revenue is recorded. Thus, the debit to warrantyexpense and the credit to provision for warranty expense in year 2 is 100,000 (100 × 1,000 units).

[568] Gleim #: 4.8.86 -- Source: CIA 594 IV-22

Answer (A) is incorrect because the exact payee does not have to be known.

Answer (B) is incorrect because the exact date payable does not have to be known.

Answer (C) is incorrect because the resource outflow must be probable.

Answer (D) is correct. A provision is a liability of uncertain timing and amount. Recognition of provisions is appropriatewhen the entity has a legal or constructive present obligation resulting from a past event (called an obligating event), it isprobable that an outflow of economic benefits will be necessary to settle the obligation, and its amount can be reliablyestimated.

Gleim's CIA Test Prep: Part III: Business Analysis and Information TechnologyAnswer Explanations

(1312 questions)

Copyright 2008 Gleim Publications, Inc. Page 172Printed for Mamdouh Farag

Page 173: P.3 Answer Explanation

[569] Gleim #: 4.8.87 -- Source: CIA 593 IV-33

Answer (A) is correct. Recognition of provisions is appropriate when the entity has a legal or constructive presentobligation resulting from a past event (called an obligating event), it is probable that an outflow of economic benefits willbe necessary to settle the obligation, and its amount can be reliably estimated. Thus, none of the circumstances in thequestion meet the recognition criteria for a provision. Hence, each is treated as a contingent liability, which requiresdisclosure unless the possibility of the outflow of resources is remote.

Answer (B) is incorrect because items I and II but not III should be disclosed.

Answer (C) is incorrect because items I and II but not III should be disclosed.

Answer (D) is incorrect because items I and II but not III should be disclosed.

[570] Gleim #: 4.8.88 -- Source: CIA 593 IV-34

Answer (A) is correct. Using the cost method, the journal entry to record the purchase of the treasury shares is

Treasury shares 17,000Cash 17,000

The journal entry to record the sale is Cash 18,000

Treasury shares 17,000Share premium from treasury shares 1,000

Consequently, the net effect is to increase equity by 1,000.

Answer (B) is incorrect because the purchase of treasury shares reduces equity by the cost of the shares, and the sale oftreasury shares increases equity by the amount received.

Answer (C) is incorrect because the purchase of treasury shares reduces equity by the cost of the shares, and the sale oftreasury shares increases equity by the amount received.

Answer (D) is incorrect because the purchase of treasury shares reduces equity by the cost of the shares, and the sale oftreasury shares increases equity by the amount received.

[571] Gleim #: 4.8.89 -- Source: CIA 1186 IV-29

Answer (A) is incorrect because post-balance-sheet refinancing does not affect the classification as current of a liabilitydue within 12 months.

Answer (B) is correct. A liability is classified as current if (1) it is expected to be settled with the normal operating cycleof the entity, (2) it is held primarily for trading, (3) it is due within 12 months of the balance sheet date, and (4) the entityhas no unconditional right to defer payment for at least 12 months after the balance sheet date. All other liabilities arenoncurrent (IAS 1).

Answer (C) is incorrect because liabilities due to be settled within 12 months should not be classified as noncurrent evenif the original term exceeded 12 months, the entity intends to refinance on a long-term basis, and its intent is supported byan agreement to refinance that was consummated before the financial statements are authorized for issue.

Answer (D) is incorrect because management’s intent is not sufficient for exclusion.

Gleim's CIA Test Prep: Part III: Business Analysis and Information TechnologyAnswer Explanations

(1312 questions)

Copyright 2008 Gleim Publications, Inc. Page 173Printed for Mamdouh Farag

Page 174: P.3 Answer Explanation

[572] Gleim #: 4.8.90 -- Source: CMA 688 3-25

Answer (A) is incorrect because the account balance is found by subtracting the actual expenditures from the totalexpense.

Answer (B) is incorrect because the account balance is found by subtracting the actual expenditures from the totalexpense.

Answer (C) is incorrect because the account balance is found by subtracting the actual expenditures from the totalexpense.

Answer (D) is correct. If the warranty expense is 10% of sales, the total expense for the 2 years is 70,000 (10% ×700,000). Of that 70,000, 12,000 was paid in Year 1 and 30,000 in Year 2. The 42,000 of payments leaves an unpaidbalance of 28,000 (70,000 – 42,000).

[573] Gleim #: 4.8.91 -- Source: CMA 689 3-11

Answer (A) is correct. A transaction is typically measured at the fair value of the consideration given up unless the fairvalue of the consideration received is more clearly evident. No information is given about the value of the services, so themarket price of the treasury shares must be used. This price was 21.50 on March 26, the date of the agreement to trade theshares for services. Thus, the services to be received should have been measured on that date based on the price of theshares, or 21.50 per share. Because no gain or loss is recognized on transactions in treasury shares, the entry is to debitservices for 21,500 (21.50 × 10,000), debit retained earnings or share premium from treasury share transactions for 3,500[(25 – 21.50) × 10,000], and credit treasury shares for 25,000 (25 × 10,000).

Answer (B) is incorrect because the services should be measured at the fair value on the date of the exchange.

Answer (C) is incorrect because the services should be measured at the fair value on the date of the exchange.

Answer (D) is incorrect because the services should be measured at the fair value on the date of the exchange.

[574] Gleim #: 4.8.92 -- Source: CMA 689 3-13

Answer (A) is incorrect because share premium should be credited when using the cost method, not retained earnings.

Answer (B) is incorrect because the repurchase price was 27 per share.

Answer (C) is incorrect because share premium should be credited for only 6,000.

Answer (D) is correct. Under the cost method, treasury shares are carried at cost. In this case, cost is 27,000 (27 × 1,000shares). The journal entry to record a sale at 33 per share is

Cash 33,000Treasury shares 27,000Share premium 6,000

Gleim's CIA Test Prep: Part III: Business Analysis and Information TechnologyAnswer Explanations

(1312 questions)

Copyright 2008 Gleim Publications, Inc. Page 174Printed for Mamdouh Farag

Page 175: P.3 Answer Explanation

[575] Gleim #: 4.8.93 -- Source: CIA 596 IV-24

Answer (A) is incorrect because the two methods of amortization result in the same total interest expense over the term ofthe bonds.

Answer (B) is correct. Under the effective-interest method, interest expense for each period equals the effective interestrate times the carrying value of the bond issue. As the discount is amortized, the carrying value rises and interest expenseincreases.

Answer (C) is incorrect because annual interest expense would decrease if a premium were being amortized.

Answer (D) is incorrect because the straight-line method results in constant annual interest expense.

[576] Gleim #: 4.8.94 -- Source: CIA 596 IV-23

Answer (A) is incorrect because the entry to bonds payable is based on the face, or maturity, amount of the bond issued.The difference between the amount received on issuance and the face amount is recorded as a premium or discount onbonds payable.

Answer (B) is incorrect because the discount should be recognized.

Answer (C) is correct. The entity received 480,000 cash on the issuance of the bond. Its face amount is 500,000, theamount to be paid at maturity. Hence, the credit to bonds payable is 500,000. The 20,000 difference is recorded as adiscount on bonds payable (a debit) and is amortized over the life of the issue.

Answer (D) is incorrect because the debit to cash is 480,000, a 20,000 discount should be debited, and the credit to bondspayable is 500,000.

[577] Gleim #: 4.8.95 -- Source: CIA 596 IV-21

Answer (A) is incorrect because 25,000 is the expected amount of warranty claims for the first year of second-year sales.

Answer (B) is incorrect because 65,000 is the actual amount of claims in the second year.

Answer (C) is incorrect because 85,000 is the expected amount of warranty claims in the second year.

Answer (D) is correct. Under the accrual method, the total estimated warranty costs are charged to operating expense inthe year of sale. The total estimated warranty cost per unit is 1,000 (250 + 750). In year two, 100 units were sold, so thewarranty expense recognized is 100,000.

[578] Gleim #: 4.8.96 -- Source: CIA 1191 IV-59

Answer (A) is incorrect because a share split is merely an accounting action that increases (or occasionally decreases) thenumber of shares outstanding. It does not generate additional capital.

Answer (B) is incorrect because conversion of convertible bonds to ordinary (common) shares simply replaces debt withoutstanding ordinary shares.

Answer (C) is correct. Warrants are options that permit the holder to buy shares for a stated price. Their exercise resultsin inflows and the issuance of shares.

Answer (D) is incorrect because options purchased and exercised through option exchanges are transactions betweenindividual investors not affecting the entity whose shares are involved.

Gleim's CIA Test Prep: Part III: Business Analysis and Information TechnologyAnswer Explanations

(1312 questions)

Copyright 2008 Gleim Publications, Inc. Page 175Printed for Mamdouh Farag

Page 176: P.3 Answer Explanation

[579] Gleim #: 4.8.97 -- Source: CMA 689 1-9

Answer (A) is incorrect because the purchase of treasury shares increases the debt-to-equity ratio. Equity but not debt isreduced by the acquisition.

Answer (B) is incorrect because the purchase of treasury shares decreases assets. Cash is paid out and shareholders’equity is decreased.

Answer (C) is incorrect because treasury shares are recorded as a reduction of equity, not as a marketable security. Thetheory of this treatment is that an entity cannot own itself.

Answer (D) is correct. An entity has many reasons to repurchase its own shares. These include meeting the needs createdby potential mergers or pension and profit-sharing plans. Also, management may want to buy out a dissident shareholder.Sometimes, an entity has excess cash and can find no better investment than its own shares. Moreover, management maybelieve the shares are selling for a low price for no apparent reason. Thus, a purchase may not only be a good investmentbut may also support the market price of the shares.

[580] Gleim #: 4.8.98 -- Source: CIA 595 IV-40

Answer (A) is incorrect because the face amount is not a distinguishing feature of convertible bonds.

Answer (B) is incorrect because the face amount is not a distinguishing feature of convertible bonds.

Answer (C) is incorrect because convertible bonds have lower, not higher, coupon rates.

Answer (D) is correct. Convertible bonds are convertible at the holder’s option into the issuer’s ordinary (common)shares at a specified price. They have a lower coupon rate than nonconvertible bonds because they offer investors a chancefor capital gains.

[581] Gleim #: 4.8.99 -- Source: CMA 692 2-21

Answer (A) is incorrect because contingent assets are not recognized.

Answer (B) is incorrect because contingent assets are not recognized.

Answer (C) is incorrect because contingent assets are not recognized. If disclosure would be misleading, the disclosureshould not be made.

Answer (D) is correct. A contingent asset is a possible asset arising from past events and the existence of which will beconfirmed only by uncertain future events not wholly within the entity’s control. An example is a potential recovery on alegal claim with an uncertain outcome. A contingent asset is not recognized but should be disclosed if an inflow ofeconomic benefits is probable. Disclosures include a description of the contingent asset and an estimate of its financialeffects. A contingent asset is not recognized because the income may not be realized. However, if realization is virtuallycertain, the asset is not contingent and may be recognized.

Gleim's CIA Test Prep: Part III: Business Analysis and Information TechnologyAnswer Explanations

(1312 questions)

Copyright 2008 Gleim Publications, Inc. Page 176Printed for Mamdouh Farag

Page 177: P.3 Answer Explanation

[582] Gleim #: 4.8.100 -- Source: Publisher

Answer (A) is incorrect because the IASs require that the debt and equity components of a compound instrument beseparately presented on the issuer’s balance sheet.

Answer (B) is correct. The initial total carrying amount of convertible debt or of debt instruments issued with detachableshare purchase warrants should be allocated between the debt instruments and the equity feature, and these debt andequity components should be separately accounted for. The total assigned initially to the instrument as a whole equals thefair value of the consideration received. However, the IFRSs do not stipulate a method for making the allocation. Oneapproach is to assign to the less easily measurable component a residual amount after determining the amount of the morereadily measurable component. For example, the future payments on the financial liability might be discounted at themarket rate for a similar instrument without an equity feature. This amount would then be subtracted from the amount ofthe compound instrument as a whole to determine the carrying amount of the equity feature. This method is appropriatewhen the fair value of the equity feature is not determinable. Thus, the present value of the principal repaymentdiscounted at the market rate of 9% is 650,000 (1,000,000 × .650), the present value of the interest payments discountedat 9% is 272,300 [(.07 × 1,000,000) × 3.890], and their total is 922,300. The residual amount assignable to the equityfeature is therefore 77,700 [(1,000 par × 1,000 bonds) proceeds – 922,300 assigned to the debt component], which is alsothe bond discount given that the convertible bonds were issued at par.

Answer (C) is incorrect because the cash flows from the bonds should be discounted at the market rate.

Answer (D) is incorrect because the proceeds equaled 1,000,000, the cash flows from the bonds should be discounted atthe market rate, and the equity feature should be separately presented.

[583] Gleim #: 4.8.101 -- Source: IIA, adapted

Answer (A) is incorrect because the balance of accumulated depreciation would be higher in the financial statements fortax purposes, since higher depreciation expense would be reported under accelerated depreciation than under straight-linedepreciation.

Answer (B) is incorrect because depreciation expense is a non-cash charge. The cash balance is unaffected by thedepreciation method used.

Answer (C) is correct. Under accelerated depreciation, depreciation expense is higher and net income is lower. Retainedearnings would therefore be lower for tax-reporting purposes than for general purpose financial reporting based onstraight-line depreciation.

Answer (D) is incorrect because the historic cost of fixed assets is recorded in the gross fixed assets account. The historiccost of the assets is unaffected by the depreciation method used.

[584] Gleim #: 4.8.102 -- Source: IIA, adapted

Answer (A) is correct. Interest expense equals the carrying value of the liability at the beginning of the period times theeffective interest rate. The carrying value of the liability equals the face value of the of the bond minus the discount. Asthe discount is amortized over the life of the bond, the carrying value increases. Consequently, the interest expenseincreases over the life of the bond.

Answer (B) is incorrect because interest expense increases from one period to the next, due to an increased carrying valueas the discount is amortized over the life of the bond.

Answer (C) is incorrect because interest expense exceeds the cash interest payment. The excess is the amount of discountamortized each period.

Answer (D) is incorrect because interest expense exceeds the cash interest payment. The excess is the amount of discountamortized each period.

Gleim's CIA Test Prep: Part III: Business Analysis and Information TechnologyAnswer Explanations

(1312 questions)

Copyright 2008 Gleim Publications, Inc. Page 177Printed for Mamdouh Farag

Page 178: P.3 Answer Explanation

[585] Gleim #: 4.9.103 -- Source: CIA 1195 IV-24

Answer (A) is incorrect because items are not classified as extraordinary under IFRSs.

Answer (B) is incorrect because changes in accounting estimates are not errors.

Answer (C) is incorrect because catch-up adjustments to prior reported amounts are retroactive. Changes in accountingestimates are accounted for prospectively.

Answer (D) is correct. A change in accounting estimate adjusts the carrying amount of an asset or liability or theconsumption of an asset. It results from reassessing the status and expected benefits and obligations related to assets andliabilities. It is based on new information and is not an error correction.

[586] Gleim #: 4.9.104 -- Source: CIA 597 IV-20

Answer (A) is incorrect because 1,000 assumes no change in estimate.

Answer (B) is correct. The machine’s net carrying amount at January 1, Year 5, is 6,000 (10,000 cost – 4,000accumulated depreciation for 4 years). A change in accounting estimate is applied prospectively. Thus, depreciationexpense is 2,000 per year for the next 3 years.

Answer (C) is incorrect because 3,000 assumes a 2-year remaining useful life.

Answer (D) is incorrect because 6,000 is the carrying amount at January 1, Year 5.

[587] Gleim #: 4.9.105 -- Source: CIA 597 IV-22

Answer (A) is correct. Accounting estimates, e.g., service lives, residual values, warranty costs, uncollectible accounts,and inventory obsolescence, are a necessary part of preparing financial statements. However, they inevitably change asnew events occur and as additional experience and information are obtained. When altered conditions require a change inestimate, it is accounted for prospectively. Thus, a change in the estimate of the service lives of depreciable assets is achange in accounting estimate.

Answer (B) is incorrect because a change from diminishing-balance depreciation to straight-line depreciation is a changein accounting policy.

Answer (C) is incorrect because changing an accounting method due to a change in an IFRS is a change in accountingpolicy.

Answer (D) is incorrect because LIFO inventory valuation is not permitted under IFRSs.

Gleim's CIA Test Prep: Part III: Business Analysis and Information TechnologyAnswer Explanations

(1312 questions)

Copyright 2008 Gleim Publications, Inc. Page 178Printed for Mamdouh Farag

Page 179: P.3 Answer Explanation

[588] Gleim #: 4.9.106 -- Source: CIA 596 IV-28

Answer (A) is incorrect because the purchase price for an acquired building can be ascertained with certainty. No estimateis required.

Answer (B) is incorrect because the price of a marketable security can be calculated with certainty. No estimate isrequired.

Answer (C) is correct. Changes in estimates used in accounting are necessary consequences of periodic presentations offinancial statements. Preparing financial statements requires estimating the effects of future events. Examples of items forwhich estimates are necessary are uncollectible receivables, inventory obsolescence, service lives and residual values ofdepreciable assets, warranty costs, periods benefited by a deferred cost, and recoverable mineral reserves.

Answer (D) is incorrect because the physical quantity of inventory as of the balance sheet date can be measured. Althoughsome estimation of the correct amount may be required, the estimates will not depend on future conditions and events buton current conditions and measurement methods.

[589] Gleim #: 4.9.107 -- Source: CIA 1195 IV-23

Answer (A) is correct. To correct the prior-period material error, the entity must, in the single-period statements issuedfor the current period, adjust the affected opening balances. Hence, the entity must debit equipment for its cost and creditaccumulated depreciation for the depreciation expense appropriate for the first year of the estimated useful life. Retainedearnings must be credited because the error understated profit or overstated loss in the prior period.

Answer (B) is incorrect because this entry is the reverse of the correct entry.

Answer (C) is incorrect because retained earnings should be credited.

Answer (D) is incorrect because accumulated depreciation and retained earnings should be credited.

[590] Gleim #: 4.9.108 -- Source: CIA 1188 IV-45

Answer (A) is correct. A voluntary change in accounting policy is applied retrospectively unless it is impracticable todetermine period-specific effects or the cumulative effect. Retrospective application means adjusting the opening balancesof equity for the first period presented and restating other comparative amounts.

Answer (B) is incorrect because reporting the cumulative effect in current profit or loss is not allowed.

Answer (C) is incorrect because retrospective application is required to the extent practicable.

Answer (D) is incorrect because retrospective application means adjusting the opening balances of equity for the firstperiod presented and restating other comparative amounts.

Gleim's CIA Test Prep: Part III: Business Analysis and Information TechnologyAnswer Explanations

(1312 questions)

Copyright 2008 Gleim Publications, Inc. Page 179Printed for Mamdouh Farag

Page 180: P.3 Answer Explanation

[591] Gleim #: 4.9.109 -- Source: CMA 681 3-24

Answer (A) is incorrect because the residual value of equipment is a change in estimate that is accounted for on aprospective basis (in the future).

Answer (B) is correct. A change in depreciation methods is reported as a change in accounting policy. A voluntary changein accounting policy is applied retrospectively unless it is impracticable. Retrospective application means adjusting theopening balances of equity for the first period presented and restating other comparative amounts.

Answer (C) is incorrect because, if distinguishing between a change in estimate and a change in accounting policy isdifficult, the change is accounted for as a change in estimate and properly disclosed.

Answer (D) is incorrect because a provision for warranty costs is a change in estimate that is accounted for on aprospective basis (in the future).

[592] Gleim #: 4.9.110 -- Source: CIA 595 IV-21

Answer (A) is incorrect because changes in accounting policies must be disclosed in the financial statements for theperiod.

Answer (B) is correct. Under IAS 8, Accounting Policies, Changes in Estimates and Accounting Errors, a voluntarychange in accounting policy should be applied retrospectively unless any resulting adjustment that relates to prior periodsis not reasonably determinable. Thus, if it is not impracticable to apply the new policy retroactively, the policy should beapplied to comparative information as far back as practicable. The entity should determine the cumulative effect on theopening and closing balance sheets for the earliest period for which it is practicable to do so.

Answer (C) is incorrect because the new policy should be retrospectively applied if practicable.

Answer (D) is incorrect because retrospective application is required to the extent practicable.

[593] Gleim #: 4.9.111 -- Source: CIA 1196 IV-2

Answer (A) is incorrect because cost of goods sold will be overestimated.

Answer (B) is incorrect because cost of goods sold will be overestimated and profit will be underestimated.

Answer (C) is correct. Cost of goods sold equals beginning inventory, plus purchases, minus ending inventory. If endinginventory is underestimated, cost of goods sold will be overestimated for the period. If cost of goods sold is overestimated,profit for the period will be underestimated.

Answer (D) is incorrect because profit will be underestimated.

Gleim's CIA Test Prep: Part III: Business Analysis and Information TechnologyAnswer Explanations

(1312 questions)

Copyright 2008 Gleim Publications, Inc. Page 180Printed for Mamdouh Farag

Page 181: P.3 Answer Explanation

[594] Gleim #: 4.9.112 -- Source: CIA 1196 IV-3

Answer (A) is incorrect because the failure to record an accrued expense will result in an overstatement of profit and anoverstatement of working capital and will have no effect on cash.

Answer (B) is incorrect because the failure to record an accrued expense will result in an overstatement of profit.

Answer (C) is incorrect because the failure to record an accrued expense will result in an overstatement of workingcapital.

Answer (D) is correct. An accrued expense is an expense that has been incurred but not paid. The appropriate adjustingentry to record an accrued expense will increase an expense account and increase a liability account. The failure to recordan accrued expense will result in an understatement of expenses leading to an overstatement of profit. The failure torecord the increase in a liability account will result in an understatement of current liabilities leading to an overstatementof working capital. There will be no effect on cash.

[595] Gleim #: 4.9.113 -- Source: CIA 1196 IV-31

Answer (A) is incorrect because understatement of accrued wages is a self-correcting error. Future wage expense will beoverstated, future cost of goods sold will be overstated, and future expenses will be understated, respectively.

Answer (B) is correct. A failure to record depreciation must be corrected because the effects of the error do notautomatically reverse in future periods. Expenses are understated in the year of the error, but no correspondingoverstatement of expenses occurs in later years.

Answer (C) is incorrect because overstatement of inventory and the consequent understatement of cost of goods sold is aself-correcting error. Future wage expense will be overstated, future cost of goods sold will be overstated, and futureexpenses will be understated, respectively.

Answer (D) is incorrect because understatement of prepaid expenses (overstatement of expenses) is a self-correcting error.Future wage expense will be overstated, future cost of goods sold will be overstated, and future expenses will beunderstated, respectively.

[596] Gleim #: 4.9.114 -- Source: CIA 591 IV-45

Answer (A) is incorrect because the net effect of these errors was a 65,000 overstatement.

Answer (B) is correct. The effect of the understatement of the 2003 year-end inventory (beginning inventory for 2004)was to overstate 2004 profit by 40,000. The reason is that beginning inventory is a component of cost of sales. Theoverstatement of the December 31, 2004, inventory overstated 2004 net income by 15,000 because the amounts in endinginventory are excluded from cost of sales. The understatement of 2003 depreciation expense (a nominal account) has noeffect on 2004 net income. Finally, the failure to accrue 10,000 of expenses for 2004 overstated 2004 net income. The neteffect of these errors was a 65,000 (40,000 + 15,000 + 0 + 10,000) overstatement.

Answer (C) is incorrect because the net effect of these errors was a 65,000 overstatement.

Answer (D) is incorrect because the net effect of these errors was a 65,000 overstatement.

Gleim's CIA Test Prep: Part III: Business Analysis and Information TechnologyAnswer Explanations

(1312 questions)

Copyright 2008 Gleim Publications, Inc. Page 181Printed for Mamdouh Farag

Page 182: P.3 Answer Explanation

[597] Gleim #: 4.9.115 -- Source: CIA 591 IV-46

Answer (A) is incorrect because the net effect of the errors was a 32,000 overstatement.

Answer (B) is correct. The 2003 inventory error reversed in 2004 (excluding tax considerations) and therefore had noeffect on reported retained earnings at December 31, 2004. The 15,000 inventory error at year-end 2004 and the failure toaccrue 10,000 of expenses for 2004 both overstated retained earnings as well as 2004 profit. The omission of 7,000 ofdepreciation overstated 2003 net income and 2003 and 2004 retained earnings. Hence, the net effect of the errors onDecember 31, 2004, retained earnings was a 32,000 (0 + 15,000 + 7,000 + 10,000) overstatement.

Answer (C) is incorrect because the net effect of the errors was a 32,000 overstatement.

Answer (D) is incorrect because the net effect of the errors was a 32,000 overstatement.

[598] Gleim #: 4.9.116 -- Source: CMA 1293 2-10

Answer (A) is incorrect because 64,000 is the total accrued wages payable, not the amount of the adjustment.

Answer (B) is incorrect because 51,000 was the correct wage accrual for Year 2.

Answer (C) is correct. Failing to record accrued wages is a self-correcting error. Expenses are understated in one year andoverstated in the next, resulting in the correction of the error over the 2-year period. The Year 1 error overstated Year 1earnings and understated Year 2 earnings by 56,000. Consequently, no correction is necessary for the Year 1 error. TheYear 2 error overstated Year 2 profit before taxes and understated Year 3 profit before taxes by 51,000. The Year 3 erroroverstated Year 3 profit before taxes by 64,000. Thus, the net effect in Year 3 of the Year 2 and Year 3 errors is a 13,000(64,000 – 51,000) overstatement. The correcting entry is to debit expense for 13,000, debit retained earnings for 51,000,and credit wages payable for 64,000.

Answer (D) is incorrect because retained earnings should be debited because of the overstatement of Year 2 income.

[599] Gleim #: 4.9.117 -- Source: CIA 1194 IV-22

Answer (A) is incorrect because errors are excluded from the determination of current period profit or loss.

Answer (B) is incorrect because prior-period financial statements need not be reissued. However, if comparativestatements are presented, the prior-period statements must be restated if it is practicable to do so.

Answer (C) is correct. All material prior-period errors must be corrected retrospectively in the first set of financialstatements issued after their discovery. This may be done by restating the comparative amounts for the prior periods whenthe error occurred. If the error occurred prior to the first period presented, the opening balances for the first periodpresented are restated. A material error is one that could, individually or collectively, affect the decisions of users of thefinancial statements. However, comparative information should be restated only if practicable.

Answer (D) is incorrect because errors are adjustments of the current year’s opening balance of retained earnings andother relevant balances.

Gleim's CIA Test Prep: Part III: Business Analysis and Information TechnologyAnswer Explanations

(1312 questions)

Copyright 2008 Gleim Publications, Inc. Page 182Printed for Mamdouh Farag

Page 183: P.3 Answer Explanation

[600] Gleim #: 4.9.118 -- Source: CIA 1194 IV-40

Answer (A) is correct. A change in an accounting estimate is accounted for prospectively and is shown on the incomestatement only in the relevant account. The effect of the change is included in the same income statement classification asthe previous estimate. However, prior-period statements and opening balances are not adjusted.

Answer (B) is incorrect because changes in an accounting estimate are not extraordinary, that is, clearly distinct from theordinary activities of the entity.

Answer (C) is incorrect because changes in accounting estimates are shown only in the relevant accounts. The effect of thechange is included in the same income statement classification as the previous estimate.

Answer (D) is incorrect because a change in an accounting estimate is only shown in the relevant account. The effect ofthe change is included in the same income statement classification as the previous estimate. No cumulative effect of thechange is recognized because estimates are accounted for prospectively.

[601] Gleim #: 4.9.119 -- Source: CIA 1195 IV-1

Answer (A) is incorrect because the errors offset each other, so cost of goods sold is not understated or overstated for theperiod.

Answer (B) is incorrect because the errors offset each other, so cost of goods sold is not understated or overstated for theperiod.

Answer (C) is incorrect because cost of goods sold is unaffected by the error, so profit is also unaffected.

Answer (D) is correct. The effects of the errors on cost of goods sold are offsetting. Purchases, which increase cost ofgoods sold, and ending inventory, which decreases cost of goods sold, are understated by the same amount. Neither cost ofgoods sold nor profit is affected.

[602] Gleim #: 4.9.120 -- Source: CIA 1195 IV-25

Answer (A) is incorrect because changes in accounting estimates should be reported.

Answer (B) is correct. Changes in accounting estimates arise as new events occur, as more experience is obtained, or asadditional evidence is acquired. A change should be reported in the period in which it occurs, as well as prospectively iffuture periods are affected. Retroactive reporting is impracticable because it would result in continual adjustments of prioryears’ financial statements.

Answer (C) is incorrect because changes in accounting estimates arise from changes in current, not prior, circumstances.

Answer (D) is incorrect because changes in accounting estimates arise from changes in current, not prior, circumstances.

[603] Gleim #: 4.10.121 -- Source: CPA 595 F-32

Answer (A) is incorrect because a loss resulted when the spot rate increased.

Answer (B) is incorrect because 500 results from using the rates at 12/31/Yr 1 and 3/20/Yr 2.

Answer (C) is incorrect because 1,000 results from using the rates at 9/22/Yr 1 and 3/20/Yr 2.

Answer (D) is correct. A receivable or payable denominated in a foreign currency is adjusted to its current exchange rateat each balance sheet date. The resulting gain or loss should ordinarily be included in determining profit or loss. It is thedifference between the spot rate on the date the transaction originates (or the rate applied in the prior year) and the closingrate. Thus, the Year 1 transaction loss for Yumi Corp. is $1,500 [10,000 units × (0.55 – 0.70)].

Gleim's CIA Test Prep: Part III: Business Analysis and Information TechnologyAnswer Explanations

(1312 questions)

Copyright 2008 Gleim Publications, Inc. Page 183Printed for Mamdouh Farag

Page 184: P.3 Answer Explanation

[604] Gleim #: 4.10.122 -- Source: Publisher

Answer (A) is incorrect because assets and liabilities for each balance sheet presented are translated at the closing rate asof its date.

Answer (B) is incorrect because, if the functional currency is that of a hyperinflational economy, translation to a differentpresentation currency is at the closing rate for the most recent balance sheet for all amounts.

Answer (C) is correct. An exchange difference arises when a given amount of one currency is translated into anothercurrency at different exchange rates. Exchange differences are recognized in a separate component of equity when theyarise from translation of functional currency amounts into the presentation amounts. Such exchange differences are notrecognized in profit or loss because they have little, if any, direct effect on operational cash flows.

Answer (D) is incorrect because income and expenses for each income statement presented are translated at the exchangerates at the transaction dates (but an average rate may be used unless rates fluctuate greatly).

[605] Gleim #: 4.10.123 -- Source: Publisher

Answer (A) is incorrect because the FCUs are reported at the spot rate on the date of the balance sheet.

Answer (B) is incorrect because the FCUs are reported at the spot rate on the balance sheet date, and the land is valued atthe date of purchase.

Answer (C) is correct. At the balance sheet date, monetary items are reported at the closing rate, which is often the spotrate on the balance sheet date. Nonmonetary items measured at historical cost are reported at the rate on the transactiondate. Thus, the FCUs are reported at 12,000 (1.20 × 10,000) euros, and the land is reported at 25,000 euros.

Answer (D) is incorrect because the land is reported based upon the historical cost on the purchase date.

[606] Gleim #: 4.10.124 -- Source: IIA, adapted

Answer (A) is correct. Because the return to the U.S. company is adversely affected and the return to the Europeancompany is unaffected, the return to the U.S. company will definitely be lower than the return to the European company.

Answer (B) is incorrect because the return to the U.S. company is adversely affected by the exchange rate movement.

Answer (C) is incorrect because the return to the U.S. company is directly affected by the exchange rate movement, whilethe return to the European company is not.

Answer (D) is incorrect because the return to the U.S. company is adversely affected and the return to the Europeancompany is unaffected, the return to the U.S. company will definitely be lower than the return to the European company.

Gleim's CIA Test Prep: Part III: Business Analysis and Information TechnologyAnswer Explanations

(1312 questions)

Copyright 2008 Gleim Publications, Inc. Page 184Printed for Mamdouh Farag

Page 185: P.3 Answer Explanation

[607] Gleim #: 4.11.125 -- Source: Publisher

Answer (A) is incorrect because jointly controlled assets do not require the establishment of an organization or financialstructure separate from the venturers.

Answer (B) is incorrect because the venturers may establish a separate entity in which each has an interest and thatoperates in the same manner as other entities if they choose to have a jointly controlled entity.

Answer (C) is correct. A joint venture “is a contractual arrangement whereby two or more parties undertake an economicactivity that is subject to joint control.” When a jointly controlled entity becomes a subsidiary of the venturer,consolidation is required.

Answer (D) is incorrect because an investor is a party to the venture who lacks joint control.

[608] Gleim #: 4.12.126 -- Source: Publisher

Answer (A) is incorrect because the uniting-of-interests method is no longer an acceptable method of accounting forbusiness combinations.

Answer (B) is correct. IFRS 3, Business Combinations, requires that all business combinations within its scope beaccounted for using the purchase method.

Answer (C) is incorrect because accounting for a business combination as part purchase and part uniting of interests is notallowed.

Answer (D) is incorrect because a business combination cannot be accounted for as a joint venture. A joint venturerequires some form of separate accounting by each venturer.

[609] Gleim #: 4.12.127 -- Source: CPA 1192 T-33

Answer (A) is incorrect because depreciation will decrease, and goodwill will be recognized but not amortized.

Answer (B) is incorrect because depreciation will decrease, and goodwill will be recognized but not amortized.

Answer (C) is correct. A business combination is accounted for as a purchase regardless of the form of considerationgiven. Under purchase accounting, assets acquired and liabilities assumed should be recorded at their fair values. Thedifferences between fair values and carrying amounts will affect profit or loss when related expenses are incurred. Theeffect of recording the building at fair value in the consolidated balance sheet instead of its higher carrying amount onCompany S’s books will be to decrease future depreciation. If the building is to be used, fair value is its currentreplacement cost for similar capacity unless expected use indicates a lower value to the acquirer. If the building is to besold, it should be reported at fair value minus cost to sell. The excess of the cost over fair value of the net assets acquiredwill be recognized as goodwill, but, under IFRS 3, this amount will be tested for impairment but not amortized.

Answer (D) is incorrect because depreciation will decrease, and goodwill is not amortized.

Gleim's CIA Test Prep: Part III: Business Analysis and Information TechnologyAnswer Explanations

(1312 questions)

Copyright 2008 Gleim Publications, Inc. Page 185Printed for Mamdouh Farag

Page 186: P.3 Answer Explanation

[610] Gleim #: 4.12.128 -- Source: CPA 593 II-8

Answer (A) is correct. A business combination is accounted for as a purchase regardless of the form of the considerationgiven. Under purchase accounting, assets acquired and liabilities assumed should be recorded at their fair values. Anyexcess of cost over the fair value of the net assets acquired is recorded as goodwill. After adjusting the net plant andequipment, and given that other items are stated at fair value, the fair value of the net assets acquired is 560,000 [80,000current assets + (760,000 + 120,000) plant and equipment – 400,000 liabilities]. Hence, goodwill is 40,000 (600,000 cost– 560,000).

Answer (B) is incorrect because 80,000 is the amount of current assets.

Answer (C) is incorrect because 120,000 is the amount plant and equipment is undervalued.

Answer (D) is incorrect because 160,000 is the difference between the 600,000 cost and the 440,000 carrying amount ofthe net assets.

[611] Gleim #: 4.12.129 -- Source: CPA 590 T-33

Answer (A) is incorrect because a deferred credit is never recognized for the excess over cost.

Answer (B) is incorrect because the allocated portion is reported as a gain, not as a deferred credit, and the values of theassets are not reduced below fair value.

Answer (C) is correct. In a business combination, any excess over cost must be recognized immediately in profit or lossas a gain. Before concluding that an excess over cost has arisen, however, IFRS 3 requires the reassessment of theidentification and measurement of the acquiree’s assets, liabilities, and cost of the combination.

Answer (D) is incorrect because the amounts assigned to certain acquired assets (most financial assets, assets to bedisposed of by sale, etc.) are not reduced.

[612] Gleim #: 4.12.130 -- Source: Publisher

Answer (A) is incorrect because a tax-free reorganization may or may not be a combination, and it may or may not resultin a parent-subsidiary relationship.

Answer (B) is incorrect because vertical and horizontal combinations may also be accomplished by a merger or aconsolidation, in which case the combining entities become one. A vertical combination combines a supplier or a customerfirm with the acquirer. A horizontal combination combines two firms in the same line of business.

Answer (C) is incorrect because vertical and horizontal combinations may also be accomplished by a merger or aconsolidation, in which case the combining entities become one. A vertical combination combines a supplier or a customerfirm with the acquirer. A horizontal combination combines two firms in the same line of business.

Answer (D) is correct. A parent-subsidiary relationship arises from an effective investment in the stock of anotherenterprise in excess of 50%. The financial statements for the two companies ordinarily should be presented on aconsolidated basis. To the extent the corporation is not wholly-owned, a minority interest is presented.

Gleim's CIA Test Prep: Part III: Business Analysis and Information TechnologyAnswer Explanations

(1312 questions)

Copyright 2008 Gleim Publications, Inc. Page 186Printed for Mamdouh Farag

Page 187: P.3 Answer Explanation

[613] Gleim #: 4.13.131 -- Source: CIA 591 IV-34

Answer (A) is incorrect because, when the equity method is used, investment income (loss) is recognized for theinvestee’s share of the profits or losses of the associate. Dividends received from the investee are recorded as a reductionof the investment account.

Answer (B) is correct. When the equity method is used, the investment is initially recorded at cost on the entity’s books.The carrying amount is subsequently adjusted to recognize the profits or losses of the associate after the date ofacquisition. Dividends received from an associate reduce the carrying amount.

Answer (C) is incorrect because the investment account is credited.

Answer (D) is incorrect because the investment account is credited.

[614] Gleim #: 4.13.132 -- Source: CIA 595 IV-21

Answer (A) is incorrect because changes in accounting policies must be disclosed in the financial statements for theperiod.

Answer (B) is incorrect because restrospective application is indicated unless it is impracticable.

Answer (C) is incorrect because restrospective application is indicated unless it is impracticable.

Answer (D) is correct. A change in accounting policy should be applied retrospectively unless it is impracticable todetermine period-specific effects or the cumulative effect. Retrospective application means adjusting the opening balancesof equity (e.g., retained earnings) for the first period presented and restating other comparative amounts.

[615] Gleim #: 5.1.1 -- Source: CIA 597 IV-48

Answer (A) is incorrect because there is no prescribed order of shareholder voting.

Answer (B) is incorrect because preemptive rights concern only equity ownership. Thus, they do not apply tononconvertible debt.

Answer (C) is correct. Preemptive rights protect ordinary (common) shareholders, proportional ownership interests fromdilution in value. A secondary purpose is to maintain the shareholders’ control of the company. Accordingly, thepreemptive right, whether granted by statute or by the corporate charter, grants ordinary (common) shareholders the powerto acquire on a pro rata basis any additional ordinary (common) shares sold by the firm. Preemptive rights also apply todebt convertible into ordinary (common) shares.

Answer (D) is incorrect because a proxy fight is an attempt to gain control of a company by persuading shareholders togrant their voting rights to others.

Gleim's CIA Test Prep: Part III: Business Analysis and Information TechnologyAnswer Explanations

(1312 questions)

Copyright 2008 Gleim Publications, Inc. Page 187Printed for Mamdouh Farag

Page 188: P.3 Answer Explanation

[616] Gleim #: 5.1.2 -- Source: CIA 597 IV-49

Answer (A) is incorrect because the participation privilege is unrelated to monitoring privileges.

Answer (B) is incorrect because preference (preferred) shareholders do not have voting rights except in circumstances inwhich the firm has not paid the preference (preferred) share dividends for a specified period.

Answer (C) is incorrect because a conversion feature, not a participation feature, allows conversion to ordinary (common)stock.

Answer (D) is correct. Participating preference (preferred) shareholders are entitled to share in the earnings of the firm.They participate in earnings distributions under set terms and conditions. Hence, after the stated preference (preferred)dividend is paid, and ordinary (common) shareholders receive an equal dividend, any remaining dividends are allocated toall shareholders in proportion to the par values of their shares.

[617] Gleim #: 5.1.3 -- Source: CIA 596 IV-46

Answer (A) is incorrect because preference (preferred) shares do not have a maturity date.

Answer (B) is incorrect because preference (preferred) shares have fixed periodic dividend payments.

Answer (C) is incorrect because preference (preferred) shares do not have a maturity date but do have fixed periodicdividend payments.

Answer (D) is correct. Like ordinary (common) shares (but unlike bonds), preference (preferred) shares have no maturitydate, although certain preference shares (transient preference shares) must be redeemed within a short time (e.g., 5 to 10years). Like bonds [but unlike ordinary (common) shares], preference (preferred) shares have a fixed periodic payment.The fixed payment is in the form of a stated dividend in the case of the preference shares and interest payments in the caseof bonds. However, preference (preferred) dividends, unlike interest, do not become an obligation unless declared.

[618] Gleim #: 5.1.4 -- Source: CIA 1195 IV-47

Answer (A) is incorrect because preference (preferred) shares have priority over ordinary (common) shares with regard toearnings, so dividends must be paid on preference (preferred) shares before they can be paid on ordinary (common)shares.

Answer (B) is incorrect because preference (preferred) shares have priority over ordinary (common) shares with regard toassets. In the event of liquidation, for example, because of bankruptcy, the claims of preference (preferred) shareholdersmust be satisfied in full before the ordinary (common) shareholders receive anything.

Answer (C) is correct. Preference (preferred) shares do not usually have voting rights. Preference (preferred)shareholders are usually given the right to vote for directors only if the company has not paid the preference (preferred)dividend for a specified period of time, such as 10 quarters. Such a provision is an incentive for management to paypreference (preferred) dividends.

Answer (D) is incorrect because cumulative preference (preferred) shares have the right to receive any dividends not paidin prior periods before ordinary (common) share dividends are paid.

Gleim's CIA Test Prep: Part III: Business Analysis and Information TechnologyAnswer Explanations

(1312 questions)

Copyright 2008 Gleim Publications, Inc. Page 188Printed for Mamdouh Farag

Page 189: P.3 Answer Explanation

[619] Gleim #: 5.1.5 -- Source: CIA 596 IV-57

Answer (A) is correct. If interest rates have increased, the prices of outstanding bonds must decrease so that their yieldswill reach the market rate. Hence, the bonds will be selling at a discount (below face value) in the open market. Retiring aportion of the outstanding bonds by buying them in the open market at the discounted price will be cheaper than calling aportion of the bonds at face value.

Answer (B) is incorrect because, if interest rates have remained constant, the bonds will still be selling at face value in theopen market. The cost of buying bonds on the open market will be the same as the cost of calling a portion of the bonds atface value.

Answer (C) is incorrect because, if interest rates have increased, buying bonds in the open market will be the cheaperstrategy.

Answer (D) is incorrect because, if interest rates have decreased, the bonds will be selling at a premium in the openmarket. Retiring a portion of the bonds by purchasing them on the open market will be more costly than calling a portionof the bonds at face value.

[620] Gleim #: 5.1.6 -- Source: CIA 1195 IV-46

Answer (A) is incorrect because bonds issued with warrants and convertible bonds have lower coupon rates thanconventional bonds.

Answer (B) is incorrect because bonds issued with warrants and convertible bonds have lower coupon rates thanconventional bonds.

Answer (C) is incorrect because convertible bonds do not remain outstanding.

Answer (D) is correct. Warrants are usually detachable. They are options to purchase equity securities and should beseparately accounted for. A capital gain results if the share price rises above the option price. The bonds remainoutstanding if the warrants are exercised. Convertible bonds must be surrendered when the conversion privilege isexercised. Under IFRSs, the equity and debt features of convertible bonds are separately accounted for.

[621] Gleim #: 5.1.7 -- Source: CIA 1196 IV-20

Answer (A) is incorrect because deep discount bonds pay interest significantly below the market rate, whereas zero-coupon bonds do not pay interest.

Answer (B) is correct. Zero-coupon bonds sell at a deep discount and increase in value each year until maturity. Thesebonds do not pay interest.

Answer (C) is incorrect because commodity-backed bonds are redeemable in measures of a commodity.

Answer (D) is incorrect because junk bonds are high-interest-rate, high-risk, unsecured bonds.

Gleim's CIA Test Prep: Part III: Business Analysis and Information TechnologyAnswer Explanations

(1312 questions)

Copyright 2008 Gleim Publications, Inc. Page 189Printed for Mamdouh Farag

Page 190: P.3 Answer Explanation

[622] Gleim #: 5.1.8 -- Source: CIA 595 IV-39

Answer (A) is incorrect because income bonds pay interest to the holder only if the interest is earned. The interest paid isnot inflation adjusted.

Answer (B) is incorrect because convertible bonds can be redeemed for the ordinary (common) shares of the issuer at theoption of the holder. Interest payments are not inflation adjusted.

Answer (C) is incorrect because mortgage bonds are secured by fixed assets of the issuer. Thus, they provide greatersecurity to bondholders, but the interest payments are not inflation adjusted.

Answer (D) is correct. The interest payments on indexed or purchasing power bonds are based on an inflation index, suchas the consumer price index. Thus, interest paid to bondholders rises automatically when the inflation rate rises.

[623] Gleim #: 5.1.9 -- Source: CIA 1191 IV-50

Answer (A) is correct. A mortgage bond is secured with specific fixed assets, usually real property. Thus, under the rightsenumerated in the bond indenture, creditors will be able to receive payments from liquidation of the property in case ofdefault. In a bankruptcy proceeding, these amounts are paid before any transfers are made to other creditors, includingthose preferences. Hence, mortgage bonds are less risky than the others listed.

Answer (B) is incorrect because a debenture is long-term debt that is not secured (collateralized) by specific property.Subordinated debentures have a claim on the debtor’s assets that may be satisfied only after senior debt has been paid infull. Debentures of either kind are therefore more risky than mortgage bonds.

Answer (C) is incorrect because an income bond pays interest only if the debtor earns it. Such bonds are also more riskythan secured debt.

Answer (D) is incorrect because unsecured debt is riskier than a mortgage bond.

[624] Gleim #: 5.1.10 -- Source: CIA 589 IV-49

Answer (A) is correct. The general principle is that risk and return are directly correlated. Corporate first mortgage bondsare less risky than income bonds or shares because they are secured by specific property. In the event of default, thebondholders can have the property sold to satisfy their claims. Holders of first mortgages have rights paramount to those ofany other parties, such as holders of second mortgages. Income bonds pay interest only in the event the corporation earnsincome. Thus, holders of income bonds have less risk than shareholders because meeting the condition makes payment ofinterest mandatory. Preference (preferred) shareholders receive dividends only if they are declared, and the directorsusually have complete discretion in this matter. Also, shareholders have claims junior to those of debtholders if theenterprise is liquidated.

Answer (B) is incorrect because the proper listing is mortgage bonds, subordinated debentures, and income bonds.Debentures are unsecured debt instruments. Their holders have enforceable claims against the issuer even if no income isearned or dividends declared.

Answer (C) is incorrect because the proper listing is first mortgage bonds, second mortgage bonds, and ordinary (common)shares. The second mortgage bonds are secured, albeit junior, claims.

Answer (D) is incorrect because the proper listing is debentures, preference (preferred) shares, and ordinary (common)shares. Holders of ordinary (common) shares cannot receive dividends unless the holders of preference (preferred) sharesreceive the stipulated periodic percentage return, in addition to any arrearages if the preference (preferred) shares arecumulative.

Gleim's CIA Test Prep: Part III: Business Analysis and Information TechnologyAnswer Explanations

(1312 questions)

Copyright 2008 Gleim Publications, Inc. Page 190Printed for Mamdouh Farag

Page 191: P.3 Answer Explanation

[625] Gleim #: 5.1.11 -- Source: CIA 1196 IV-45

Answer (A) is correct. For any given quoted nominal rate, the least frequent compounding is associated with the lowesteffective annual percentage cost. Annual compounding is less frequent than semiannual, quarterly, or monthly. The termof the loan is not relevant to the calculation of the effective annual percentage cost of financing.

Answer (B) is incorrect because, the more frequent the interest compounding, the more costly the loan. Semiannual,quarterly, and monthly compounding are all more frequent than annual compounding.

Answer (C) is incorrect because, the more frequent the interest compounding, the more costly the loan. Semiannual,quarterly, and monthly compounding are all more frequent than annual compounding.

Answer (D) is incorrect because, the more frequent the interest compounding, the more costly the loan. Semiannual,quarterly, and monthly compounding are all more frequent than annual compounding.

[626] Gleim #: 5.1.12 -- Source: Publisher

Answer (A) is incorrect because the call feature reduces the value of the bond by the value of the call option.

Answer (B) is correct. A callable bond is not as valuable to an investor as a straight bond. Thus, the 50 call option issubtracted from the 1,000 value of a straight bond to arrive at a 950 value for the callable bond.

Answer (C) is incorrect because the call feature reduces the value of the bond by the value of the call option.

Answer (D) is incorrect because the call feature reduces the value of the bond by the value of the call option.

[627] Gleim #: 5.1.13 -- Source: Publisher

Answer (A) is incorrect because 57 is the result of deducting the call premium from the exercise price.

Answer (B) is incorrect because 60 is the result of failing to consider the impact of the call premium.

Answer (C) is incorrect because the full call premium must be added to the exercise price.

Answer (D) is correct. Because the call premium is 3, the stock price must be at least 63 (60 exercise price + 3 callpremium).

[628] Gleim #: 5.1.14 -- Source: Publisher

Answer (A) is incorrect because the portfolio has the same value at the maturity date regardless of the price of the stock.

Answer (B) is incorrect because the portfolio has the same value at the maturity date regardless of the price of the stock.

Answer (C) is incorrect because the portfolio has the same value at the maturity date regardless of the price of the stock.

Answer (D) is correct. If the stock price at the maturity date is 30, AA Company will have a share of stock worth 30 and aput option worth 10 (40 exercise price – 30 stock price). The call option will be worthless. Hence, the net payoff is 40 (30+ 10). If the stock price at the maturity date is 45, the share of stock will be worth 45, the put will be worthless, and theloss on the call will be 5 (45 – 40). Thus, the net payoff will be 40 (45 – 5). Consequently, the difference in the net payoffon the portfolio because of a difference in the stock price at the maturity date is 0 (40 – 40).

Gleim's CIA Test Prep: Part III: Business Analysis and Information TechnologyAnswer Explanations

(1312 questions)

Copyright 2008 Gleim Publications, Inc. Page 191Printed for Mamdouh Farag

Page 192: P.3 Answer Explanation

[629] Gleim #: 5.1.15 -- Source: CIA, adapted

Answer (A) is incorrect because income bonds only pay interest if interest is earned.

Answer (B) is incorrect because debentures are unsecured bonds.

Answer (C) is incorrect because subordinated debentures are subordinated to other debt.

Answer (D) is correct. First-mortgage bonds are backed by fixed assets.

[630] Gleim #: 5.2.16 -- Source: CIA 1195 IV-53

Answer (A) is correct. The first term of the formula represents the periodic cost of the trade discount, calculated as thecost per unit of trade credit (discount %) divided by the funds made available by not taking the discount (100 – discount%). The second term represents the number of times per year this cost is incurred. The multiple of these terms is theapproximate annual percentage cost of not taking the trade discount. A precise formula would incorporate the effects ofcompounding when calculating the annual cost.

Answer (B) is incorrect because the denominator of the first term should represent the funds made available by not takingthe discount (100 – discount %).

Answer (C) is incorrect because the first term is the reciprocal of the correct term.

Answer (D) is incorrect because the second term is the reciprocal of the correct term.

[631] Gleim #: 5.2.17 -- Source: CIA 597 IV-51

Answer (A) is incorrect because the lower the discount percentage, the lower the opportunity cost of forgoing the discountand using the trade credit financing.

Answer (B) is incorrect because percentage financing cost is unaffected by the purchase price of the items.

Answer (C) is incorrect because percentage financing cost is unaffected by the purchase price of the items.

Answer (D) is correct. If the discount period is longer, the days of extra credit obtained by forgoing the discount arefewer. Assuming other factors are constant, the result is that the cost of trade credit, that is, the cost of not taking thediscount, is greater.

[632] Gleim #: 5.2.18 -- Source: CIA 1192 IV-54

Answer (A) is incorrect because 51,000 less is based on a 30-day borrowing period.

Answer (B) is correct. The interest cost of borrowing 4,900,000 (5,000,000 × 98%) to take advantage of the discount is24,500 [4,900,000 × 12% × (15 ÷ 360)], and the total cost will be 4,924,500. The total cost if the discount is not takenwill be 5,000,000, a difference of 75,500.

Answer (C) is incorrect because 100,000 less does not consider the interest paid.

Answer (D) is incorrect because 24,500 more reflects interest paid but ignores the discounted price.

Gleim's CIA Test Prep: Part III: Business Analysis and Information TechnologyAnswer Explanations

(1312 questions)

Copyright 2008 Gleim Publications, Inc. Page 192Printed for Mamdouh Farag

Page 193: P.3 Answer Explanation

[633] Gleim #: 5.2.19 -- Source: CIA 1196 IV-52

Answer (A) is incorrect because lower compensating balances and regular interest are less costly.

Answer (B) is incorrect because lower compensating balances and regular interest are less costly.

Answer (C) is incorrect because lower compensating balances and regular interest are less costly.

Answer (D) is correct. The most costly combination of characteristics is a higher compensating balance and discountinterest. The higher the compensating balance, the higher the portion of the loan funds that must be left on deposit withthe lender. Hence, the interest paid is charged on a smaller amount of funds available to be used by the borrower, and theeffective cost is higher. Also, discount interest is deducted from the loan funds in advance, resulting in a further increasein the effective financing cost.

[634] Gleim #: 5.2.20 -- Source: CIA 596 IV-41

Answer (A) is incorrect because a sight draft calls for immediate payment upon delivery of the shipping documents to, andthe acceptance of the draft by, the buyer.

Answer (B) is incorrect because an open account is a credit arrangement involving only the signing of an invoice by thebuyer.

Answer (C) is correct. A time draft (trade acceptance) is a form of commercial draft because it is drawn by a seller on thebuyer; that is, it calls for the buyer to pay a specified amount. The draft and the shipping documents related to the goodsare then sent to the buyer’s bank, which transmits the draft to the buyer. The buyer accepts the draft by signing it. A timedraft, however, is similar to a promissory note because it is payable at a specific time in the future rather than uponacceptance by the buyer, which is characteristic of a sight draft. If a seller is reluctant to ship goods because of concernabout the buyer’s ability to pay a time draft, the seller’s bank may, for a fee, guarantee payment. This banker’s acceptanceis an assumption of the obligation to pay at the due date.

Answer (D) is incorrect because the description is of a conditional sales contract except that the seller, not the bank,retains title to the goods until the buyer has completed payment.

[635] Gleim #: 5.2.21 -- Source: CIA 1191 IV-56

Answer (A) is incorrect because commercial paper is a type of unsecured, short-term promissory note issued by large firmsto other firms, insurance companies, mutual funds, etc.

Answer (B) is correct. A document of title is usually issued by a bailee covering goods in the bailee’s possession or care(UCC 1-201). It represents ownership of the goods and is ordinarily needed to obtain the goods from the bailee. The twomajor types of documents of title are bills of lading (issued by carriers) and warehouse receipts. A warehouse receipt isissued by a person engaged in the business of storing goods for hire. Security for short-term inventory financing can bearranged if the debtor places its inventory under the control of the lender or its agent (e.g., a public warehouse), and thelender holds the warehouse receipts.

Answer (C) is incorrect because a revolving credit agreement is a formal line of credit, usually with a bank, that largefirms often use.

Answer (D) is incorrect because accounts payable, or trade credit, is the most common source of unsecured short-termfinancing.

Gleim's CIA Test Prep: Part III: Business Analysis and Information TechnologyAnswer Explanations

(1312 questions)

Copyright 2008 Gleim Publications, Inc. Page 193Printed for Mamdouh Farag

Page 194: P.3 Answer Explanation

[636] Gleim #: 5.2.22 -- Source: CIA 590 IV-50

Answer (A) is correct. A factor purchases a company’s accounts receivable and assumes the risk of collection. The sellerreceives money immediately to reinvest in new inventories. The financing cost is usually high: about 2 points or moreabove prime, plus a fee for collection. Factoring has been traditional in the textile industry for years, and recentlycompanies in many industries have found it an efficient means of operation. A company that uses a factor can eliminate itscredit department, accounts receivable staff, and bad debts. These reductions in costs can more than offset the fee chargedby the factor, which can often operate more efficiently than its clients because of the specialized nature of its service.

Answer (B) is incorrect because factoring is a source of short-term funds through sale of receivables.

Answer (C) is incorrect because factoring is a source of short-term funds through sale of receivables.

Answer (D) is incorrect because factoring is a source of short-term funds through sale of receivables.

[637] Gleim #: 5.2.23 -- Source: CMA 1286 1-35

Answer (A) is incorrect because a chattel mortgage is a loan secured by personal property (movable property such asequipment or livestock). Also, a floating lien is secured by property, such as inventory, the composition of which may beconstantly changing. Factoring is a form of financing in which receivables serve as security.

Answer (B) is incorrect because a chattel mortgage is a loan secured by personal property (movable property such asequipment or livestock). Also, a floating lien is secured by property, such as inventory, the composition of which may beconstantly changing. Factoring is a form of financing in which receivables serve as security.

Answer (C) is incorrect because a chattel mortgage is a loan secured by personal property (movable property such asequipment or livestock). Also, a floating lien is secured by property, such as inventory, the composition of which may beconstantly changing. Factoring is a form of financing in which receivables serve as security.

Answer (D) is correct. An unsecured loan is a loan made by a bank based on credit information about the borrower andthe ability of the borrower to repay the obligation. The loan is not secured by collateral, but is made on the signature of theborrower. Unsecured credit is not backed by collateral. Revolving credit, bankers’ acceptances, lines of credit, andcommercial paper are all unsecured means of borrowing. A chattel mortgage is a loan secured by personal property(movable property such as equipment or livestock). A floating lien is also secured by property, such as inventory, thecomposition of which may be constantly changing. Factoring is a form of financing in which receivables serve as security.

[638] Gleim #: 5.2.24 -- Source: CMA 689 1-13

Answer (A) is incorrect because an agency security is issued by a corporation or agency created by a government.Examples are government securities issued by the bodies that finance mortgages, such as the Federal National MortgageAssociation (Fannie Mae) in the U.S.

Answer (B) is incorrect because bankers’ acceptances are drafts drawn on deposits at a bank. The acceptance by the bankguarantees payment at maturity. They are normally used to finance a specific transaction.

Answer (C) is correct. Commercial paper is the term for the short-term (typically less than 9 months), unsecured, largedenomination (often over 100,000) promissory notes issued by large, creditworthy companies to other companies andinstitutional investors. In many instances, the maturity date is only a few days after issuance.

Answer (D) is incorrect because a repurchase agreement involves a secured loan to a government securities dealer. Itallows the buyer to retain interest income although the seller-dealer can repurchase after a specified time.

Gleim's CIA Test Prep: Part III: Business Analysis and Information TechnologyAnswer Explanations

(1312 questions)

Copyright 2008 Gleim Publications, Inc. Page 194Printed for Mamdouh Farag

Page 195: P.3 Answer Explanation

[639] Gleim #: 5.2.25 -- Source: CIA 1195 IV-52

Answer (A) is incorrect because 16.67% does not adjust for the discount interest arrangement.

Answer (B) is incorrect because 17.65% does not adjust for the compensating balance requirement.

Answer (C) is correct. Discount interest is subtracted before the loan proceeds are paid to the borrower. A compensatingbalance is an amount that the borrower must keep on deposit with the lender. The effective annual interest rate isincreased by both the discount interest arrangement and by the compensating balance requirement. The effective rateequals the nominal rate dividend by one minus the sum of the nominal rate and the compensating balance percentage, or20% [.15 ÷ (1.0 – .15 – .10)].

Answer (D) is incorrect because 25.00% is the sum of the nominal rate and the compensating balance percentage.

[640] Gleim #: 5.3.26 -- Source: CIA 597 IV-42

Answer (A) is incorrect because the cost of capital is a composite, or weighted average, of all financing sources in theirusual proportions. The cost of capital should also be calculated on an after-tax basis.

Answer (B) is incorrect because the cost of capital is a composite, or weighted average, of all financing sources in theirusual proportions. The cost of capital should also be calculated on an after-tax basis.

Answer (C) is incorrect because the cost of capital is a composite, or weighted average, of all financing sources in theirusual proportions. The cost of capital should also be calculated on an after-tax basis.

Answer (D) is correct. A weighted average of the costs of all financing sources should be used, with the weightsdetermined by the usual financing proportions. The terms of any financing raised at the time of initiating a particularproject do not represent the cost of capital for the firm. When a firm achieves its optimal capital structure, the weighted-average cost of capital is minimized. The cost of capital is a composite, or weighted average, of all financing sources intheir usual proportions. The cost of capital should also be calculated on an after-tax basis.

[641] Gleim #: 5.3.27 -- Source: CIA 597 IV-50

Answer (A) is incorrect because an increase in the DFL increases the riskiness of the firm’s shares. Thus, beta rises. Betais a measure of the volatility of a firm’s share price relative to the average share.

Answer (B) is incorrect because systematic risk, also known as market risk, is unrelated to the DFL. Systematic risk is notspecific to a company. It is the risk associated with a company’s shares that cannot be diversified because it arises fromfactors that affect all shares.

Answer (C) is incorrect because systematic risk, also known as market risk, is unrelated to the DFL. Systematic risk is notspecific to a company. It is the risk associated with a company’s shares that cannot be diversified because it arises fromfactors that affect all shares.

Answer (D) is correct. The DFL equals the percentage change in EPS [earnings available to ordinary (common)shareholders] divided by the percentage change in net operating profit or loss. When the DFL rises, fixed interest chargesand the riskiness of the firm rise. As a result, the variability of returns will increase. In other words, the standarddeviation of returns of the company rises.

Gleim's CIA Test Prep: Part III: Business Analysis and Information TechnologyAnswer Explanations

(1312 questions)

Copyright 2008 Gleim Publications, Inc. Page 195Printed for Mamdouh Farag

Page 196: P.3 Answer Explanation

[642] Gleim #: 5.3.28 -- Source: CIA 597 IV-52

Answer (A) is incorrect because 1.25 ignores variable costs.

Answer (B) is correct. The DOL may be calculated as the contribution margin (sales – variable cost) divided by theexcess of the contribution margin over fixed costs. The contribution margin is 150,000 [25,000 units × (10 unit price – 4unit variable cost)]. Hence, the DOL in the second year is 1.50 [150,000 CM ÷ (150,000 – 50,000 FC)].

Answer (C) is incorrect because 2.0 uses the first-year sales level and ignores variable costs.

Answer (D) is incorrect because 6.0 uses the first-year sales level.

[643] Gleim #: 5.3.29 -- Source: CIA 597 IV-53

Answer (A) is correct. The higher the dividend-payout ratio, the sooner retained earnings are exhausted and the companymust seek external financing. Assuming the same investments are undertaken, the result is a higher marginal cost ofcapital because lower-cost capital sources will be used up earlier.

Answer (B) is incorrect because the marginal cost of capital is higher.

Answer (C) is incorrect because the existence of investment opportunities is unrelated to the dividend payout.

Answer (D) is incorrect because the existence of investment opportunities is unrelated to the dividend payout.

[644] Gleim #: 5.3.30 -- Source: CIA 1196 IV-49

Answer (A) is correct. Given that company X is more highly leveraged, it has greater fixed financing charges thancompany Y. Interest payments are fixed financing charges, but ordinary (common) share dividends are not. As a result,company X will be more risky and therefore will have a more volatile net income stream than company Y, if other factorsare constant.

Answer (B) is incorrect because the level of fixed financing charges does not affect operating income variability.

Answer (C) is incorrect because the level of fixed financing charges does not affect operating income variability.

Answer (D) is incorrect because company X has greater, not less, financial leverage than company Y. Greater use of debtfinancing means that a company has greater financial leverage.

Gleim's CIA Test Prep: Part III: Business Analysis and Information TechnologyAnswer Explanations

(1312 questions)

Copyright 2008 Gleim Publications, Inc. Page 196Printed for Mamdouh Farag

Page 197: P.3 Answer Explanation

[645] Gleim #: 5.3.31 -- Source: CIA 1196 IV-25

Answer (A) is correct. The dividend growth model is used to calculate the cost of equity. The simplified formula is

R is the required rate of return, D is the next dividend, P is the share’s price, and G is the growth rate in earnings pershare. The equation is also used to determine the share price.

Thus, when investors have a lower required return on equity, the denominator is smaller, which translates into a highermarket value.

Answer (B) is incorrect because, if investors expect lower dividend growth, the market value of ordinary (common) shareswill decrease.

Answer (C) is incorrect because the expected holding periods of investors are not related to the market value of theordinary (common) shares.

Answer (D) is incorrect because the expected holding periods of investors are not related to the market value of theordinary (common) shares.

[646] Gleim #: 5.3.32 -- Source: CIA 593 IV-49

Answer (A) is incorrect because a higher interest rate raises the required return of investors, which results in a lowershare price.

Answer (B) is correct. The dividend growth model is used to calculate the price of a share.

If: P0 = current price D1 = next dividend RS = required rate of return G = EPS growth rate

Assuming that D1 and G remain constant, an increase in R S resulting from an increase in the nominal interest rate willcause P0 to decrease. A higher interest rate raises the required return of investors, which results in a lower share price.

Answer (C) is incorrect because a higher interest rate raises the required return of investors, which results in a lowershare price.

Answer (D) is incorrect because a higher interest rate raises the required return of investors, which results in a lowershare price.

Gleim's CIA Test Prep: Part III: Business Analysis and Information TechnologyAnswer Explanations

(1312 questions)

Copyright 2008 Gleim Publications, Inc. Page 197Printed for Mamdouh Farag

Page 198: P.3 Answer Explanation

[647] Gleim #: 5.3.33 -- Source: CIA 596 IV-48

Answer (A) is incorrect because IOS schedules do not require that all projects have the same investment cost. The steps ofthe schedule can be of varying lengths.

Answer (B) is incorrect because IOS schedules cannot be drawn for mutually exclusive projects.

Answer (C) is incorrect because IOS schedules do not require that all projects have the same NPV. The NPV of eachproject depends on the investment cost and on the present value of the expected cash flows. Both costs and cash flows canvary for projects on an IOS.

Answer (D) is correct. An IOS schedule is drawn for a set of independent projects. The decision to be made is whether toaccept or reject each project without regard to other investment opportunities. Thus, the cash flows of one independentproject are not influenced by those of another. Independence should be distinguished from mutual exclusivity. Projects aremutually exclusive if acceptance of one requires rejection of the other.

[648] Gleim #: 5.3.34 -- Source: CIA 596 IV-49

Answer (A) is incorrect because both A and B should be undertaken.

Answer (B) is correct. The intersection of the IOS and MCC schedules determines the cost of capital and the optimalcapital budget. The company should begin with the project having the highest return and continue accepting projects aslong as the IRR exceeds the MCC. The highest ranked project is A, with a 50 million cost and a 14% IRR. The MCC isonly 6% over this range of financing. The next highest ranked project is B, with a 75 million cost and a 12% IRR. When125 million has been invested, the marginal cost of the next unit of capital is 10%, so Project B is also acceptable,bringing the optimal capital budget to 125 million. Project C is not acceptable because it has an 8% return. The MCC is10% for the first 50 million invested in this project and 12% for the remaining 75 million.

Answer (C) is incorrect because A is acceptable, but C is not.

Answer (D) is incorrect because C is not acceptable. It offers an IRR less than the marginal cost of financing the project.

[649] Gleim #: 5.3.35 -- Source: CIA 594 IV-48

Answer (A) is incorrect because the marginal cost of debt financing is the interest rate on new debt minus the firm’smarginal tax rate multiplied by the interest rate. Moreover, the marginal or incremental cost of debt to the firm is based onthe cost of newly issued debt, not on the cost of outstanding debt.

Answer (B) is incorrect because the marginal cost of debt financing is the interest rate on new debt minus the firm’smarginal tax rate multiplied by the interest rate. Moreover, the marginal or incremental cost of debt to the firm is based onthe cost of newly issued debt, not on the cost of outstanding debt.

Answer (C) is correct. The marginal cost of debt (as a rate) must equal the cost of new debt (as a rate) minus the taxsavings (as a rate). Hence, marginal cost equals the cost of new debt times one minus the marginal tax rate, or k d(1 – T).This expression equals k d – kdT. The marginal cost of debt financing is the interest rate on new debt minus the firm’smarginal tax rate multiplied by the interest rate. Moreover, the marginal or incremental cost of debt to the firm is based onthe cost of newly issued debt, not on the cost of outstanding debt.

Answer (D) is incorrect because the marginal cost of debt financing is the interest rate on new debt minus the firm’smarginal tax rate multiplied by the interest rate. Moreover, the marginal or incremental cost of debt to the firm is based onthe cost of newly issued debt, not on the cost of outstanding debt.

Gleim's CIA Test Prep: Part III: Business Analysis and Information TechnologyAnswer Explanations

(1312 questions)

Copyright 2008 Gleim Publications, Inc. Page 198Printed for Mamdouh Farag

Page 199: P.3 Answer Explanation

[650] Gleim #: 5.3.36 -- Source: CIA 594 IV-80

Answer (A) is incorrect because the end-of-period expected dividend and the net proceeds of the issue should be includedin calculating the dividend yield.

Answer (B) is incorrect because the end-of-period expected dividend should be included in calculating the dividend yield.

Answer (C) is incorrect because the net proceeds, not market price, should be included in calculating the dividend yield.

Answer (D) is correct. The cost of new equity is calculated by adding the expected dividend yield, based on the netproceeds of the new issue, to the expected dividend growth rate. The expected dividend at the end of the period equals thedividend at time zero times one plus the expected dividend growth rate. Net proceeds received by the firm when issuingone ordinary (common) share equals the market price of a share times one minus the flotation cost percentage. Flotationcosts include items such as underwriting fees, printing, and advertising. The calculation of the cost of new equity is asfollows:

[651] Gleim #: 5.3.37 -- Source: CIA 1191 IV-58

Answer (A) is incorrect because EPS is not a function of investor confidence and is not maximized by concurrentproportional increases in both debt and equity. EPS are usually higher if debt is used instead of equity to raise capital, atleast initially.

Answer (B) is correct. Earnings per share will ordinarily be higher if debt is used to raise capital instead of equity,provided that the firm is not over-leveraged. The reason is that the cost of debt is lower than the cost of equity becauseinterest is tax deductible. However, the prospect of higher EPS is accompanied by greater risk to the firm resulting fromrequired interest costs, creditors’ liens on the firm’s assets, and the possibility of a proportionately lower EPS if salesvolume fails to meet projections.

Answer (C) is incorrect because equity capital is initially more costly than debt.

Answer (D) is incorrect because using only current cash flow to raise capital is usually too conservative an approach for agrowth-oriented firm. Management is expected to be willing to take acceptable risks to be competitive and attain anacceptable rate of growth.

[652] Gleim #: 5.3.38 -- Source: CIA 596 IV-51

Answer (A) is incorrect because financing costs do not depend on rates of return on investment.

Answer (B) is incorrect because financing costs do not depend on rates of return on investment.

Answer (C) is incorrect because as additional funds are raised, an increase in the cost of a source of financing, not adecrease, will result in an increase in the MCC.

Answer (D) is correct. The MCC is a weighted average of the costs of the different financing sources. If the cost of anysource of financing increases, the MCC curve will rise. The MCC curve is upward sloping because the lowest costfinancing sources are assumed to be used first. Thus, as cumulative debt increases, the cost of debt also increases.

Gleim's CIA Test Prep: Part III: Business Analysis and Information TechnologyAnswer Explanations

(1312 questions)

Copyright 2008 Gleim Publications, Inc. Page 199Printed for Mamdouh Farag

Page 200: P.3 Answer Explanation

[653] Gleim #: 5.3.39 -- Source: CMA 1296 1-10

Answer (A) is incorrect because 27.0% is based on weights of 25,000 and 50,000.

Answer (B) is correct. If the company pays Web Master within 10 days, it will save 500 (2% × 25,000). Thus, thecompany is effectively paying 500 to retain 24,500 (25,000 – 500) for 20 days (30 – 10). The annualized interest rate onthis borrowing is 36.7346% [(500 ÷ 24,500) × (360 days ÷ 20 days)]. Similarly, the company is, in effect, paying Softidee2,500 (5% × 50,000) to hold 47,500 (50,000 – 2,500) for 80 days (90 – 10). The annualized rate on this borrowing is23.6842% [(2,500 ÷ 47,500) × (360 days/80 days)]. The average amount borrowed from Web Master is 16,333.33 [1month × 24,500 × (20 days ÷ 30 days)], and the average amount borrowed from Softidee is 126,666.67 [3 months ×47,500 × (80 days ÷ 90 days)]. Thus, the weighted average of these two rates based on average borrowings is 25.2%{[36.7346% × 16,333.33) + (23.6842% × 126,666.67)] ÷ (16,333.33 + 126,666.67)}. This calculation, however,understates the true cost of not taking the discount because it does not consider the effects of compounding.

Answer (C) is incorrect because 28.0% is based on weights of 24,500 and 47,500.

Answer (D) is incorrect because 30.2% is an unweighted average of the two interest rates.

[654] Gleim #: 5.3.40 -- Source: CMA 1296 1-11

Answer (A) is incorrect because the company should continue the current practice unless alternative short-term financingis available at a lower rate.

Answer (B) is incorrect because the weighted-average cost of capital is usually a concern in capital budgeting and is not asimportant in the decision process as the marginal cost of capital. Furthermore, trade credit is just one element in the firm’sfinancing structure. An optimal mix of financing sources may require that trade credit be obtained at less than theweighted-average cost of capital.

Answer (C) is incorrect because the company should maintain its current practice if the cost of alternative long-termfinancing is higher.

Answer (D) is correct. The company is currently paying an annual rate of 25.2% (as determined below) to obtain tradecredit and pay at the end of the credit period. This policy should be continued if trade credit is the only source offinancing, or if other sources are available only at a higher rate. The annual rate is determined as follows: If the companypays Web Master within 10 days, it will save 500 (2% × 25,000). Thus, the company is effectively paying 500 to retain24,500 (25,000 – 500) for 20 days (30 – 10). The annualized interest rate on this borrowing is 36.7346% [(500 ÷ 24,500)× (360 days ÷ 20 days)]. Similarly, the company is, in effect, paying Softidee 2,500 (5% × 50,000) to hold 47,500 (50,000– 2,500) for 80 days (90 – 10). The annualized rate on this borrowing is 23.6842% [(2,500 ÷ 47,500) × (360 days ÷ 80days)]. The average amount borrowed from Web Master is 16,333.33 [1 month × 24,500 × (20 days ÷ 30 days)], and theaverage amount borrowed from Softidee is 126,666.67 [3 months × 47,500 × (80 days ÷ 90 days)]. Thus, the weightedaverage of these two rates based on average borrowings is 25.2% {[36.7346% × 16,333.33) + (23.6842% × 126,666.67)]÷ (16,333.33 + 126,666.67)}.

[655] Gleim #: 5.3.41 -- Source: CIA 1187 IV-66

Answer (A) is incorrect because the coefficient of variation compares risk with expected return (standard deviation ÷expected return).

Answer (B) is correct. The required rate of return on equity capital in the Capital Asset Pricing Model is the risk-freerate, plus the product of the market risk premium times the beta coefficient. The market risk premium is the amount abovethe risk-free rate that will induce investment in the market. The beta coefficient of an individual share is the correlationbetween the volatility (price variation) of the stock market and that of the price of the individual share. For example, if anindividual share goes up 15% and the market only 10%, beta is 1.5.

Answer (C) is incorrect because standard deviation measures dispersion (risk) of project returns.

Answer (D) is incorrect because expected return does not describe risk.

Gleim's CIA Test Prep: Part III: Business Analysis and Information TechnologyAnswer Explanations

(1312 questions)

Copyright 2008 Gleim Publications, Inc. Page 200Printed for Mamdouh Farag

Page 201: P.3 Answer Explanation

[656] Gleim #: 5.3.42 -- Source: CIA, adapted

Answer (A) is correct. The market risk premium (R M – RF) is the amount above the risk-free rate required to includeaverage investors to enter the market. The risk premium is the portion of expected return attributed to the increased risk.

Answer (B) is incorrect because the coefficient of variation represents the standard deviation of an investment’s returnsdivided by the mean returns.

Answer (C) is incorrect because the standard error represents a measure of variability in the investment’s returns.

Answer (D) is incorrect because the beta coefficient represents the sensitivity of the investment’s returns to the marketreturns.

[657] Gleim #: 5.3.43 -- Source: CIA, adapted

Answer (A) is incorrect because this answer ignores equity.

Answer (B) is correct. The optimal capital structure minimizes the weighted average cost of capital and therebymaximizes the value of the firm’s stock. Both debt and equity are factors in a company’s capital structure.

Answer (C) is incorrect because the decision does not directly involve assets.

Answer (D) is incorrect because the decision involves equity, but does not focus on the type of equity used.

[658] Gleim #: 5.3.44 -- Source: CIA, adapted

Answer (A) is correct. In business terminology, a high degree of operating leverage, other things held constant, meansthat a relatively small change in sales will result in a large change in operating income. Therefore, if a high percentage ofa firm’s total cost is fixed, the firm is said to have a high degree of operating leverage.

Answer (B) is incorrect because the opposite is true.

Answer (C) is incorrect because the firm has a high degree of operating leverage.

Answer (D) is incorrect because the firm has a high degree of operating leverage.

[659] Gleim #: 5.3.45 -- Source: CIA, adapted

Answer (A) is correct. The higher the dividend payout ratio, the sooner retained earnings are exhausted and the companymust seek more costly, outside equity financing. This drives the marginal cost of capital.

Answer (B) is incorrect because the debt ratio is computed by dividing total debts by total assets. The dividend payoutratio has no impact on the debt ratio.

Answer (C) is incorrect because the investment opportunities available to the company are not determined by the level ofdividend payout.

Answer (D) is incorrect because the opposite is true. The price to earnings ratio is computed by dividing price per shareby earnings per share, so a company with a higher dividend payout ratio would have a lower price to earnings ratio.

Gleim's CIA Test Prep: Part III: Business Analysis and Information TechnologyAnswer Explanations

(1312 questions)

Copyright 2008 Gleim Publications, Inc. Page 201Printed for Mamdouh Farag

Page 202: P.3 Answer Explanation

[660] Gleim #: 5.3.46 -- Source: CIA, adapted

Answer (A) is incorrect because securities prices are a good estimate of future cash flows under this theory.

Answer (B) is correct. The efficient markets theory proposes that the market is continuously adjusting to new informationand acting to correct pricing errors.

Answer (C) is incorrect because securities prices are the best benchmark under this theory.

Answer (D) is incorrect because securities prices equal their fair value as perceived by investors.

[661] Gleim #: 5.3.47 -- Source: CIA, adapted

Answer (A) is correct. If all earnings are paid out as dividends, then there is no earnings retention. All sales growth mustbe financed from spontaneous or external sources.

Answer (B) is incorrect because the funds-needed line only passes through the origin in the special case where allearnings are paid out as dividends.

Answer (C) is incorrect because the funds-needed line is a graph of the relationship between sales growth rates andadditional financing needs. It is not drawn for just one point, or one level of sales growth.

Answer (D) is incorrect because, while the sales growth rate would be zero at the point where the funds-needed linepassed through the origin, funds needed may be non-zero when sales growth is zero.

[662] Gleim #: 5.3.48 -- Source: CIA, adapted

Answer (A) is correct. Net cash flow represents the numerator in the formula used to derive the value of the firm.Therefore, the economic value of the firm will increase as net cash flow increases.

Answer (B) is incorrect because an increase in systematic (or market) risk will increase the overall cost of capital andthereby increase K, the discount rate. As a result, the value of the firm will fall.

Answer (C) is incorrect because an increase in unsystematic (or firm-specific) risk is diversifiable and will have no affecton the value of the firm.

Answer (D) is incorrect because an increase in the discount rate will reduce the value of the firm.

[663] Gleim #: 5.3.49 -- Source: CIA 595 IV-35

Answer (A) is incorrect because 1.03 results if depreciation expense is omitted from the calculation of profit beforeinterest and tax.

Answer (B) is correct. The degree of financial leverage for B may be calculated as profit before interest and taxes (PBIT),divided by PBIT minus interest. PBIT is 200 (95 profit + 10 interest + 95 tax expense). Thus, the DFL is 1.05 [200 ÷ (200– 10)].

Answer (C) is incorrect because 1.12 results if profit is used instead of PBIT.

Answer (D) is incorrect because 1.25 is the degree of financial leverage for A.

Gleim's CIA Test Prep: Part III: Business Analysis and Information TechnologyAnswer Explanations

(1312 questions)

Copyright 2008 Gleim Publications, Inc. Page 202Printed for Mamdouh Farag

Page 203: P.3 Answer Explanation

[664] Gleim #: 5.3.50 -- Source: CIA 1195 IV-54

Answer (A) is incorrect because, if earnings retention is low (that is, if dividend payout is high), less internal capital isavailable, so more external capital is needed.

Answer (B) is incorrect because higher sales require more assets and more capital.

Answer (C) is correct. The capital intensity ratio is the amount of assets required per dollar of sales. External financialrequirements are lower if the capital intensity ratio is low because sales can grow rapidly without much outside capital,other things held constant.

Answer (D) is incorrect because entities with lower profit margins have lower retained earnings and therefore requiremore external capital.

[665] Gleim #: 5.4.51 -- Source: CMA 691 1-9

Answer (A) is incorrect because net income includes noncash elements, e.g., goodwill amortization and depreciation.

Answer (B) is incorrect because net income includes noncash elements, e.g., goodwill amortization and depreciation.

Answer (C) is incorrect because collection percentages must be considered, and net income includes noncash elements.

Answer (D) is correct. The most direct way of preparing a cash budget requires incorporation of sales projections andcredit terms, collection percentages, estimated purchases and payment terms, and other cash receipts and disbursements.In other words, preparation of the cash budget requires consideration of both inflows and outflows.

[666] Gleim #: 5.4.52 -- Source: CMA Sample Q1-6

Answer (A) is incorrect because 59,125 is the annual lockbox cost.

Answer (B) is correct. If payments are collected 2 days earlier, the company can earn 120,000 (20,000 × 50 payments perday × 2 days × .06) at a cost of 59,125 [50,000 + (50 payments × 365 days × .50)], a gain of 60,875.

Answer (C) is incorrect because 50,000 is the annual fixed fee.

Answer (D) is incorrect because 120,000 is the annual savings without regard to costs.

[667] Gleim #: 5.4.53 -- Source: Publisher

Answer (A) is correct. Checks are currently tied up for 11 days (6 for mailing, 3 for processing, and 2 for clearing). If thatperiod were reduced to 3 days, the company’s cash balance would increase by 1,200,000 (8 days × 150,000 per day).

Answer (B) is incorrect because the decrease is 8 days, not 5.

Answer (C) is incorrect because 600,000 represents only a 4-day savings.

Answer (D) is incorrect because the lockbox system will result in an additional 8 days of savings, not 3.

Gleim's CIA Test Prep: Part III: Business Analysis and Information TechnologyAnswer Explanations

(1312 questions)

Copyright 2008 Gleim Publications, Inc. Page 203Printed for Mamdouh Farag

Page 204: P.3 Answer Explanation

[668] Gleim #: 5.4.54 -- Source: CMA 694 1-24

Answer (A) is correct. The float period is the time between when a check is written and when it clears the payor’schecking account. Check float results in an interest-free loan to the payor because of the delay between payment by checkand its deduction from the bank account. If checks written require one more day to clear than checks received, the net floatequals one day’s receipts. The company will have free use of the money for one day. In this case, the amount is 10,000.

Answer (B) is incorrect because the company enjoys one day’s net float because its checks clear more slowly than itsdeposits.

Answer (C) is incorrect because the net float is positive. The company can write checks (up to 10,000) even when it hasno money because the checks do not clear until a day after deposits clear.

Answer (D) is incorrect because the net float represents the difference between when deposits clear and whendisbursements clear.

[669] Gleim #: 5.4.55 -- Source: CMA 696 1-11

Answer (A) is correct. The 50,000 compensating balance requirement is partially satisfied by the company’s practice ofmaintaining a 25,000 balance for transaction purposes. Thus, only 25,000 of the loan will not be available for current use,leaving 225,000 of the loan usable. At 6% interest, the 250,000 loan would require an interest payment of 15,000 peryear. This is partially offset by the 2% interest earned on the 25,000 incremental balance, or 500. Subtracting the 500interest earned from the 15,000 of expense results in net interest expense of 14,500 for the use of 225,000 in funds.Dividing 14,500 by 225,000 produces an effective interest rate of 6.44%.

Answer (B) is incorrect because 7.00% fails to consider that the 25,000 currently being maintained counts toward thecompensating balance requirement.

Answer (C) is incorrect because 5.8% fails to consider the compensating balance requirement.

Answer (D) is incorrect because 6.66% fails to consider the interest earned on the incremental balance being carried.

[670] Gleim #: 5.4.56 -- Source: CMA 691 1-12

Answer (A) is incorrect because negotiable CDs do have a secondary market (i.e., they are negotiable).

Answer (B) is incorrect because negotiable CDs are regulated.

Answer (C) is incorrect because negotiable CDs are typically issued in a large denomination.

Answer (D) is correct. A certificate of deposit (CD) is a form of savings deposit that cannot be withdrawn before maturitywithout incurring a high penalty. A negotiable CD can be traded. CDs usually have a fairly high rate of return comparedwith other savings instruments because they are for fixed, usually long-term periods. However, their yield is less than thatof commercial paper and bankers’ acceptances because they are less risky.

Gleim's CIA Test Prep: Part III: Business Analysis and Information TechnologyAnswer Explanations

(1312 questions)

Copyright 2008 Gleim Publications, Inc. Page 204Printed for Mamdouh Farag

Page 205: P.3 Answer Explanation

[671] Gleim #: 5.4.57 -- Source: Publisher

Answer (A) is incorrect because (24,000) ignores the additional interest revenue from investing the increased funds.

Answer (B) is incorrect because 12,000 is based on 2 days of accelerated inflows rather than 3.

Answer (C) is correct. Because collections will be accelerated by 3 days at a rate of 300,000 per day, the company willhave an additional 900,000 to invest. At a rate of 6%, the interest earned will be 54,000 per year. However, the bank willcharge 24,000 (12 months × 2,000 per month) for its services. Thus, the firm will increase its income by 30,000 (54,000 –24,000).

Answer (D) is incorrect because 54,000 ignores the 24,000 bank service charge.

[672] Gleim #: 5.4.58 -- Source: CMA 696 1-12

Answer (A) is incorrect because 3,000 miscalculates the annual service charge.

Answer (B) is incorrect because 12,000 fails to deduct the annual service charge from the interest earned.

Answer (C) is incorrect because 0 results from figuring the interest earned for only one day, not two.

Answer (D) is correct. If collection time is 2 days, and average daily receipts are 100,000, the average cash balance willincrease by 200,000 if the bank’s system is adopted. At a 6% interest rate, 200,000 will generate 12,000 of interestrevenue annually. The 500 monthly charge by the bank will result in an annual expense of 6,000. Thus, the net annualbenefit is 6,000 (12,000 – 6,000).

[673] Gleim #: 5.4.59 -- Source: CMA 697 1-20

Answer (A) is incorrect because the relevant cost of the minimum premium account deposit is based on the 2,000incremental deposit, not the full 2,500.

Answer (B) is incorrect because the savings on the premium account is 16.

Answer (C) is incorrect because the savings on the premium account is 16.

Answer (D) is correct. The standard account will cost 10 per month plus 8 in check charges (.10 x 80 checks), for a totalof 18 per month or 216 per year. The premium account has no check charges, but it will require the depositor to maintaina balance of 2,000 more than desired. At a 10% cost of capital, the incremental 2,000 minimum deposit will cost 200 peryear. Thus, the premium account should be selected because it is cheaper by 16 per year.

[674] Gleim #: 5.4.60 -- Source: CMA 697 1-13

Answer (A) is incorrect because the annual cost is 126,000.

Answer (B) is incorrect because the annual cost is 125,000.

Answer (C) is incorrect because the annual cost is 136,080.

Answer (D) is correct. Multiplying 700 checks by 360 days results in a total of 252,000 checks per year. Accordingly,under (A), total annual cost is 126,000 (.50 × 252,000), which is less desirable than the 125,000 flat fee in (B). Given thatthe annual collections equal 453,600,000 (1,800 × 700 × 360), (C) is also less desirable because the annual fee would be136,080 (.03% × 453,600,000). The best option is therefore to maintain a compensating balance of 1,750,000 when thecost of funds is 7%, resulting in a total cost of 122,500 (.07 × 1,750,000).

Gleim's CIA Test Prep: Part III: Business Analysis and Information TechnologyAnswer Explanations

(1312 questions)

Copyright 2008 Gleim Publications, Inc. Page 205Printed for Mamdouh Farag

Page 206: P.3 Answer Explanation

[675] Gleim #: 5.4.61 -- Source: Publisher

Answer (A) is incorrect because Bank 1 will increase the firm’s income by only 18,000.

Answer (B) is incorrect because Bank 2 will increase the firm’s income by only 12,000.

Answer (C) is incorrect because Bank 3 will increase the firm’s income by only 12,000.

Answer (D) is correct. Because collections made using Bank 4’s lockbox service will be accelerated by 2 days at a rate of300,000 per day, the firm will have an additional 600,000 to invest. At a rate of 6%, the interest earned will be 36,000 peryear. However, the bank will charge 12,000 (12 months × 1,000 per month) for its services. Thus, the firm will gain24,000 (36,000 – 12,000).

[676] Gleim #: 5.4.62 -- Source: CMA 1294 1-17

Answer (A) is incorrect because the 25 transfer fee is covered by the interest on 62,500 for 2 days.

Answer (B) is incorrect because 125,000 is required if collections are accelerated by only one day.

Answer (C) is incorrect because the interest on 173 for 2 days is less than .07.

Answer (D) is correct. Given a 25 fee and an interest rate of 0.02% per day for 2 days, the breakeven amount is 62,500[25 transfer fee ÷ (2 × .02% interest rate)]. Thus, the interest earned on a transfer of any amount greater than 62,500would exceed the 25 fee.

[677] Gleim #: 5.4.63 -- Source: CMA 1296 1-5

Answer (A) is incorrect because concentration banking, a lockbox system, and the use of a local post office box aretechniques used to accelerate cash receipts.

Answer (B) is correct. Payment by draft, a three-party instrument in which the drawer orders the drawee to pay money tothe payee, is a means of slowing cash outflows. A check is the most common type of draft. Check float arises from thedelay between an expenditure and the clearing of the check through the banking system.

Answer (C) is incorrect because EDI is the communication of electronic documents directly from a computer in one entityto a computer in another entity. Thus, EDI expedites cash payments. The payee receives the money almostinstantaneously.

Answer (D) is incorrect because concentration banking, a lockbox system, and the use of a local post office box aretechniques used to accelerate cash receipts.

[678] Gleim #: 5.4.64 -- Source: CMA 688 1-13

Answer (A) is correct. Banks sometimes require a borrower to keep a certain percentage of the face amount of a loan in anoninterest-bearing checking account. This requirement raises the effective rate of interest paid by the borrower. Thisgreater rate compensates a bank for services provided and results in greater profitability for the financial institution.Funds kept as a compensating balance can often be withdrawn if a certain average balance is maintained.

Answer (B) is incorrect because, in financial accounting, a valuation allowance is used to reflect losses on marketablesecurities.

Answer (C) is incorrect because safety stock is held for such purposes.

Answer (D) is incorrect because interest deducted in advance is discount interest.

Gleim's CIA Test Prep: Part III: Business Analysis and Information TechnologyAnswer Explanations

(1312 questions)

Copyright 2008 Gleim Publications, Inc. Page 206Printed for Mamdouh Farag

Page 207: P.3 Answer Explanation

[679] Gleim #: 5.4.65 -- Source: CMA 697 1-19

Answer (A) is incorrect because the borrower has access to less, not more, than the face amount of the loan.

Answer (B) is incorrect because the effective rate is higher than the contract rate as a result of the compensating balancerequirement.

Answer (C) is incorrect because 8.40% is 120% of the contract rate.

Answer (D) is correct. Interest on the loan is 7,000 (7% × 100,000). Given that the borrower has to maintain a 20%compensating balance, only 80,000 [100,000 – (20% × 100,000)] is available for use. Thus, the company is paying 7,000for the use of 80,000 in funds at an effective cost of 8.75% (7,000 ÷ 80,000).

[680] Gleim #: 5.4.66 -- Source: CMA 696 1-10

Answer (A) is incorrect because 1,000 results from using 24% in the denominator.

Answer (B) is incorrect because 2,000 results from using 6% in the denominator.

Answer (C) is correct. The EOQ for inventory is a function of ordering cost per order, inventory demand, and carryingcost. In the cash model, the fixed cost per sale of securities is equivalent to the ordering cost, the demand for cash issimilar to the demand for inventory, and the interest rate is effectively the cost of carrying a dollar of cash for the period.Substituting in the formula yields an optimal cash balance of about 6,928. Thus, the average cash balance is 3,464 (6,928÷ 2).

Answer (D) is incorrect because 6,928 is the optimal cash balance.

[681] Gleim #: 5.4.67 -- Source: Publisher

Answer (A) is incorrect because the Miller-Orr cash management model is not deterministic, but simply provides a rangein which marketable securities will not be transacted.

Answer (B) is correct. The Baumol cash management model is essentially the inventory EOQ model in that it minimizesthe total of fixed trading costs and cash carrying costs. It is a deterministic model in the same way that the EOQ model isdeterministic.

Answer (C) is incorrect because Altman’s Z-Score is a means of forecasting a firm’s future solvency by adding theweighted amounts of five financial ratios.

Answer (D) is incorrect because an arbitrage pricing model is a means of valuing an asset by means of using several riskfactors.

Gleim's CIA Test Prep: Part III: Business Analysis and Information TechnologyAnswer Explanations

(1312 questions)

Copyright 2008 Gleim Publications, Inc. Page 207Printed for Mamdouh Farag

Page 208: P.3 Answer Explanation

[682] Gleim #: 5.4.68 -- Source: CMA 694 1-25

Answer (A) is incorrect because U.S. Treasury bills are short-term marketable securities.

Answer (B) is incorrect because Eurodollars are short-term marketable securities.

Answer (C) is incorrect because commercial paper is a short-term marketable security.

Answer (D) is correct. Marketable securities are near-cash items used primarily for short-term investment. Examplesinclude U.S. Treasury bills, Eurodollars, commercial paper, money-market mutual funds with portfolios of short-termsecurities, bankers’ acceptances, floating rate preference (preferred) stock, and negotiable CDs of U.S. banks. Aconvertible bond is not a short-term investment because its maturity date is usually more than one year in the future andits price can be influenced substantially by changes in interest rates or by changes in the investee’s stock price.

[683] Gleim #: 5.4.69 -- Source: CIA, adapted

Answer (A) is incorrect because the cash balance maintained for making routine payments and collections is called thetransactions balance.

Answer (B) is correct. The compensating cash balance is the money left in a checking account in the bank in order tocompensate the bank for services it provides.

Answer (C) is incorrect because the cash balance maintained as a reserve for unforeseen cash flow fluctuations is calledthe precautionary balance.

Answer (D) is incorrect because it is the speculative cash balance that is maintained in order to enable the firm to takeadvantage of any bargain purchase opportunities that may arise.

[684] Gleim #: 5.4.70 -- Source: CMA 697 1-15

Answer (A) is incorrect because interest must be paid monthly when the credit line is used in April, May, and June.

Answer (B) is correct. The sum of the beginning balance and inflows exceeds the outflows for the first 2 months. At theend of March, however, the company must use $2,000,000 of its line of credit ($2,000,000 beginning balance +$6,000,000 inflows – $10,000,000 outflows). Thus, interest for April is $20,000 (1% × $2,000,000). The net cash outflowfor April (ignoring short-term borrowings) is $1,000,000 of an additional $1,000,000 of the line of credit. However, the$20,000 of interest for April must also be paid, so the amount of the line of credit used in May is $3,020,000 ($2,000,000+ $1,000,000 + $20,000). Interest for May is therefore $30,200 (1% × $3,020,000). Given the net cash inflow for May of$2,000,000 (again ignoring short-term borrowings) and the borrowing of $30,200 to pay the interest for May, the amountof the line of credit used in June is $1,050,200. Interest in June is $10,502 (1% × $1,050,200), and total interest is$60,702 ($20,000 + $30,200 + $10,502). Consequently, the closest answer is $61,000.

Answer (C) is incorrect because the company would repay the credit line at the end of months with a positive cash flow.

Answer (D) is incorrect because the company would repay the credit line at the end of months with a positive cash flow.

Gleim's CIA Test Prep: Part III: Business Analysis and Information TechnologyAnswer Explanations

(1312 questions)

Copyright 2008 Gleim Publications, Inc. Page 208Printed for Mamdouh Farag

Page 209: P.3 Answer Explanation

[685] Gleim #: 5.4.71 -- Source: CMA 692 1-27

Answer (A) is incorrect because the disbursements for the month are greater than the purchases for the month.

Answer (B) is incorrect because 152,000 excludes operating expenses, payroll, and interest.

Answer (C) is incorrect because 200,000 is based on a miscalculation of operating expenses and payroll.

Answer (D) is correct. The cash disbursements for the month of April include 40% of April purchases, 30% of Marchpurchases, 30% of February purchases, April payroll (10% of April sales), operating expenses (20% of May sales), and aninterest payment of 20,000.

40% × 140,000 = 56,00030% × 160,000 = 48,00030% × 160,000 = 48,00010% × 300,000 = 30,00020% × 260,000 = 52,000Interest = 20,000 Total disbursements = 254,000

[686] Gleim #: 5.4.72 -- Source: CMA 1295 1-8

Answer (A) is incorrect because $138,000 equals the sum of February credit sales and March cash sales.

Answer (B) is incorrect because $122,000 equals 50% of January credit sales, 30% of February credit sales, 20% ofMarch credit sales, and 100% of March cash sales.

Answer (C) is correct. Cash inflows for March would consist of 50% of March credit sales ($90 × 50% = $45), plus 30%of February credit sales (120 × 30% = $36), plus 20% of January credit sales ($100 × 20% = $20), plus cash sales forMarch of $18. Consequently, total collections equal $119,000.

Answer (D) is incorrect because $108,000 is the total sales for March, not the total cash collections for March.

[687] Gleim #: 5.5.73 -- Source: CMA 1295 1-12

Answer (A) is incorrect because most companies are not in business to earn high returns on liquid assets (i.e., they areheld to facilitate operations).

Answer (B) is incorrect because the holding of cash and cash-like assets is not a major factor in controlling taxes.

Answer (C) is incorrect because investments in government bonds do not have sufficient liquidity to serve as short-termassets.

Answer (D) is correct. Cash and short-term investments are crucial to a firm’s continuing success. Sufficient liquiditymust be available to meet payments as they come due. At the same time, liquid assets are subject to significant controlrisk. Therefore, liquidity and safety are the primary concerns of the treasurer when dealing with highly liquid assets. Cashand short-term investments are held because of their ability to facilitate routine operations of the company. These assetsare not held for purposes of achieving investment returns.

Gleim's CIA Test Prep: Part III: Business Analysis and Information TechnologyAnswer Explanations

(1312 questions)

Copyright 2008 Gleim Publications, Inc. Page 209Printed for Mamdouh Farag

Page 210: P.3 Answer Explanation

[688] Gleim #: 5.5.74 -- Source: CIA 1195 IV-65

Answer (A) is incorrect because mortgages are long-term, capital market securities.

Answer (B) is incorrect because preference (preferred) shares are long-term, capital market securities.

Answer (C) is correct. Funds are borrowed or lent for short periods (less than one year) in money markets. Examples ofinstruments traded in money markets are U.S. Treasury bills, bankers’ acceptances, commercial paper, negotiablecertificates of deposit, money market mutual funds, Eurodollar market time deposits, and consumer credit loans. Capitalmarkets trade shares and long-term debt.

Answer (D) is incorrect because U.S. Treasury bonds are long-term, capital market securities.

[689] Gleim #: 5.6.75 -- Source: CMA 697 1-14

Answer (A) is incorrect because the cost of funds is an obvious element in the analysis of any investment.

Answer (B) is correct. All factors should be considered that differ between the two policies. Factors that do not differ,such as the current bad debt experience, are not relevant. Company R must estimate the expected bad debt losses undereach new policy.

Answer (C) is incorrect because the impact on the current customer base of extending terms to only certain customers isrelevant. The current customers may demand the same terms.

Answer (D) is incorrect because existing loan agreements may require Company R to maintain certain ratios at statedlevels. Thus, Company R’s ability to increase receivables and possible bad debt losses may be limited.

[690] Gleim #: 5.6.76 -- Source: CMA 697 1-8

Answer (A) is incorrect because the length of the extra credit period is 20 days, not 30 days.

Answer (B) is incorrect because the length of the extra credit period is 20 days, not 30 days.

Answer (C) is incorrect because 36.0% calculates the interest rate based on the full invoice price.

Answer (D) is correct. On a 1,000 invoice, the company could save 20 by paying within the discount period. Thus, animmediate payment of 980 would save the company 20, and the interest rate charged for holding 980 an additional 20 days(30 – 10) is 2.04% (20/980). Because the number of 20-day periods in a year is 18 (360/20), the annual rate is 36.7% (18× 2.04%).

[691] Gleim #: 5.6.77 -- Source: CMA 1295 1-4

Answer (A) is correct. The average collection period measures the number of days between the date of sale and the dateof collection. It should be related to a firm’s credit terms. For example, a firm that allows terms of 2/15, net 30, shouldhave an average collection period of somewhere between 15 and 30 days.

Answer (B) is incorrect because it describes the concept of float.

Answer (C) is incorrect because the average collection period includes the total time before a payment is received,including the periods both before and after the end of the normal credit period.

Answer (D) is incorrect because it describes the normal credit period.

Gleim's CIA Test Prep: Part III: Business Analysis and Information TechnologyAnswer Explanations

(1312 questions)

Copyright 2008 Gleim Publications, Inc. Page 210Printed for Mamdouh Farag

Page 211: P.3 Answer Explanation

[692] Gleim #: 5.6.78 -- Source: CMA 1292 1-20

Answer (A) is incorrect because 36,000 is the minimum savings required.

Answer (B) is incorrect because 36,000 is the minimum savings required.

Answer (C) is incorrect because 36,000 is the minimum savings required.

Answer (D) is correct. Given sales of 27,000,000, the average amount of daily sales must be 75,000 (27,000,000 ÷ 360days). The increased accounts receivable balance is therefore 450,000 (6 days × 75,000). With an additional 450,000 ofcapital invested in receivables, the company’s interest cost will increase by 36,000 per year (8% × 450,000). Thus, thecompany must save at least 36,000 per year to justify the change in procedures.

[693] Gleim #: 5.6.79 -- Source: CMA 1294 1-22

Answer (A) is incorrect because 4,000 is only one day’s sales.

Answer (B) is correct. If sales are 4,000 per day, and customers pay in 30 days, 30 days of sales are outstanding, or120,000. Whether customers pay by credit card or cash, collection requires 30 days.

Answer (C) is incorrect because invoices are outstanding for 30 days, not 12 days.

Answer (D) is incorrect because 54,000 is based on the 45% of collections via credit card.

[694] Gleim #: 5.6.80 -- Source: Publisher

Answer (A) is incorrect because the age of payables should be deducted from the sum of the other items.

Answer (B) is incorrect because the payables are not added to the inventory period. They are deducted.

Answer (C) is correct. The cash flow cycle begins when the firm pays for merchandise it has purchased and ends whenthe firm receives cash from the sale of the merchandise. Inventory is held for an average of 60 days prior to sale, but theaverage age of accounts payable is 30 days. Consequently, the average time between outlay and sale is 30 days.Receivables are collected an average of 45 days after sale, so the length of the cash flow cycle is 75 days (30 + 45).

Answer (D) is incorrect because 105 days equals the sum of the inventory cycle and the receivables cycle.

[695] Gleim #: 5.6.81 -- Source: CMA 1295 1-6

Answer (A) is incorrect because 60% of the sales will be paid for within the 10-day discount period.

Answer (B) is incorrect because 990,000 is based on a sales total of 1,500,000 for 30 days rather than 1,350,000.

Answer (C) is incorrect because 900,000 is based on a sales total of 1,500,000 for 30 days rather than 1,350,000.

Answer (D) is correct. The firm has daily sales of 45,000 consisting of 150 units at 300 each. For 30 days, sales total1,350,000. Forty percent of these sales, or 540,000, will be uncollected because customers do not take their discounts. Theremaining 60%, or 810,000, will be paid within the discount period. However, by the end of 30 days, only 2/3 of the810,000 will be collected because the sales from days 21 through 30 are still within the discount period. Therefore, anadditional 270,000 (810,000 – 540,000) will still be uncollected after the 30th day, but will be subject to a discount. Intotal, the average receivable balance is 810,000, consisting of 540,000 on which no discount will be taken and 270,000that will be paid within the discount period.

Gleim's CIA Test Prep: Part III: Business Analysis and Information TechnologyAnswer Explanations

(1312 questions)

Copyright 2008 Gleim Publications, Inc. Page 211Printed for Mamdouh Farag

Page 212: P.3 Answer Explanation

[696] Gleim #: 5.6.82 -- Source: CMA 1289 1-15

Answer (A) is correct. An increase in discounts taken accompanied by declines in receivables balances and doubtfulaccounts all indicate that collections on the increased sales have been accelerated. Accordingly, the average collectionperiod must have declined. The average collection period is a ratio calculated by dividing the number of days in a year(365) by the receivable turnover. Thus, the higher the turnover, the shorter the average collection period. The turnoverincreases when either sales (the numerator) increase, or receivables (the denominator) decrease. Accomplishing bothhigher sales and a lower receivables increases the turnover and results in a shorter collection period.

Answer (B) is incorrect because a decrease in the percentage discount offered provides no incentive for early payment.

Answer (C) is incorrect because accounts receivable turnover (sales ÷ average receivables) has increased.

Answer (D) is incorrect because no information is given relative to working capital elements other than receivables. Bothreceivables and cash are elements of working capital, so an acceleration of customer payments will have no effect onworking capital.

[697] Gleim #: 5.6.83 -- Source: CMA 1294 1-24

Answer (A) is incorrect because the decrease will be 3,333,334.

Answer (B) is incorrect because the decrease will be 3,333,334.

Answer (C) is correct. If sales are 50 million, 70% of which are on credit, total credit sales will be 35 million. Thereceivables turnover equals 4.8 times per year (360 days ÷ 75-day collection period). Receivables turnover equals netcredit sales divided by average receivables. Accordingly, average receivables equal 7,291,667 (35,000,000 ÷ 4.8). Underthe new policy, sales will be 47.5 million (95% × 50,000,000), and credit sales will be 28.5 million (60% × 47,500,000).The collection period will be reduced to 50 days, resulting in a turnover of 7.2 times per year (360 ÷ 50). The averagereceivables balance will therefore be 3,958,333 (28,500,000 ÷ 7.2), a reduction of 3,333,334 (7,291,667 – 3,958,333).

Answer (D) is incorrect because receivables will decrease.

[698] Gleim #: 5.6.84 -- Source: CMA 1296 1-18

Answer (A) is incorrect because Alternative I cannot be correct. Neither of the multiplicands is a dollar figure, so theproduct could not be the dollar balance of receivables.

Answer (B) is incorrect because Alternative I cannot be correct. Neither of the multiplicands is a dollar figure, so theproduct could not be the dollar balance of receivables.

Answer (C) is correct. A firm’s average gross receivables balance can be calculated by multiplying average daily sales bythe average collection period (days’ sales outstanding). Alternatively, annual credit sales can be divided by the accounts-receivable turnover (net credit sales/average accounts receivable) to obtain the average balance in receivables.

Answer (D) is incorrect because Alternative III cannot be correct. It contains average gross receivables, the amount beingcalculated.

Gleim's CIA Test Prep: Part III: Business Analysis and Information TechnologyAnswer Explanations

(1312 questions)

Copyright 2008 Gleim Publications, Inc. Page 212Printed for Mamdouh Farag

Page 213: P.3 Answer Explanation

[699] Gleim #: 5.6.85 -- Source: Publisher

Answer (A) is incorrect because the prime rate must be greater than 7.37% to make the company’s terms preferable tothose of a bank.

Answer (B) is incorrect because the prime rate must be greater than 7.37% to make the company’s terms preferable tothose of a bank.

Answer (C) is incorrect because the prime rate must be greater than 7.37% to make the company’s terms preferable tothose of a bank.

Answer (D) is correct. Terms of 1/10, net 60 mean that a buyer can save 1% of the purchase price by paying 50 daysearly. In essence, not taking the discount results in the buyer’s borrowing 99% of the invoice price for 50 days at a totalinterest charge of 1% of the invoice price. Because a year has 7.3 50-day periods (365 ÷ 50), the credit terms 1/10, net 60yield an effective annualized interest charge of approximately 7.37% [(1% ÷ 99%) × 7.3]. If the prime rate were higherthan 7.37%, the buyer would prefer to borrow from the vendor (i.e., not pay within the discount period) rather than from abank. Consequently, an 8% prime rate could cause the vendor’s receivables to increase.

[700] Gleim #: 5.6.86 -- Source: CIA 592 IV-43

Answer (A) is correct. The accounts receivable turnover equals net credit sales divided by average trade receivables (net).In Year Six, the accounts receivable turned over 19 times {285,000 ÷ [(16,000 ending A/R + 14,000 beginning A/R) ÷2]}.

Answer (B) is incorrect because 16 times was derived by dividing average sales of Years Five and Six [(285,000 +200,000) ÷ 2] by average receivables (15,000), which produces no meaningful ratio.

Answer (C) is incorrect because 10 times was derived by dividing cost of goods sold (150,000) for Year Six by averageaccounts receivable (15,000), which produces no meaningful ratio.

Answer (D) is incorrect because 6 times is the inventory turnover ratio which equals the cost of goods sold for Year Six(150,000) divided by average inventory (25,000).

[701] Gleim #: 5.6.87 -- Source: CIA 594 IV-35

Answer (A) is incorrect because 10 days assumes all customers take the discount.

Answer (B) is incorrect because 15 days assumes half of the customers pay on day 30 but ignores the remaining half of thecustomers who pay on day 10.

Answer (C) is correct. The average collection period is the average time it takes to receive payment from customers.because one-half of the customers will pay on day 10 and half will pay on day 30, the average collection period is 20 days[.5(10 days) + .5(30 days)].

Answer (D) is incorrect because 30 days assumes all customers pay on day 30.

Gleim's CIA Test Prep: Part III: Business Analysis and Information TechnologyAnswer Explanations

(1312 questions)

Copyright 2008 Gleim Publications, Inc. Page 213Printed for Mamdouh Farag

Page 214: P.3 Answer Explanation

[702] Gleim #: 5.6.88 -- Source: CIA 594 IV-36

Answer (A) is incorrect because 684.93 is based on annual credit sales of 10,000.

Answer (B) is incorrect because 1,808.22 equals the credit sales per day.

Answer (C) is incorrect because 27,123.30 is based on a 15-day average collection period.

Answer (D) is correct. The expected average accounts receivable balance equals the average collection period times thecredit sales per day. Thus, the average accounts receivable balance is 45,205.48 {[(10,000 units sold on credit × 66 price)÷ 365 days] × 25 days}. The foregoing calculation assumes that receivables are recorded at their gross amounts.

[703] Gleim #: 5.7.89 -- Source: Publisher

Answer (A) is incorrect because market values is an essential element of the valuation.

Answer (B) is incorrect because incremental cash flow is an essential element of the valuation.

Answer (C) is correct. If the net incremental cash flows to the acquiring firm’s shareholders are to be valued, the discountrate used should be the cost of equity capital. Moreover, this rate should reflect the risk associated with the use of fundsrather than their source. The rate therefore should not be the cost of capital of the acquiring firm but rather the cost ofequity of the combined firm after the combination. This calculation requires a new estimate of beta to be used in theCapital Asset Pricing Model.

Answer (D) is incorrect because transaction costs is an essential element of the valuation.

[704] Gleim #: 5.7.90 -- Source: Publisher

Answer (A) is incorrect because the cost of equity, not the cost of debt, should be the discount rate.

Answer (B) is incorrect because the discount rate should be that of the combined firm, not the acquiring firm.

Answer (C) is incorrect because the discount rate should be that of the combined firm, not the acquiring firm.

Answer (D) is correct. If the net incremental cash flows to the acquiring firm’s shareholders are to be calculated, thediscount rate used should be the cost of equity capital. Moreover, this rate should reflect the risk associated with the useof funds rather than their source. The rate therefore should not be the cost of capital of the acquiring firm but rather thecost of equity of the acquired firm after the combination. This calculation requires a new estimate of beta to be used in theCapital Asset Pricing Model.

[705] Gleim #: 5.8.91 -- Source: CIA 590 IV-57

Answer (A) is incorrect because a call option is the right to purchase shares at a given price within a specified period.

Answer (B) is correct. A put option is the right to sell stock at a given price within a certain period. If the market pricefalls, the put option may allow the sale of stock at a price above market, and the profit of the option holder will be thedifference between the price stated in the put option and the market price, minus the cost of the option, commissions, andtaxes. The company that issues the stock has nothing to do with put (and call) options.

Answer (C) is incorrect because selling a put option could force the company to purchase additional stock if the option isexercised.

Answer (D) is incorrect because a warrant gives the holder a right to purchase stock from the issuer at a given price (it isusually distributed along with debt).

Gleim's CIA Test Prep: Part III: Business Analysis and Information TechnologyAnswer Explanations

(1312 questions)

Copyright 2008 Gleim Publications, Inc. Page 214Printed for Mamdouh Farag

Page 215: P.3 Answer Explanation

[706] Gleim #: 5.8.92 -- Source: Publisher

Answer (A) is incorrect because an out-of-the-money option may not have expired.

Answer (B) is correct. When the value of the asset underlying a call option is less than the exercise price of the option,the option is “out-of-money.”

Answer (C) is incorrect because the option does exist; it is just not worth exercising.

Answer (D) is incorrect because call options do not change into put options.

[707] Gleim #: 5.8.93 -- Source: CIA 1196 IV-58

Answer (A) is incorrect because a call option is the right to purchase an ordinary (common) share at a set price for a settime period. If the underlying share has a lower market value, the call option is less, not more, valuable.

Answer (B) is correct. The lower the exercise price, the more valuable the call option. The exercise price is the price atwhich the call holder has the right to purchase the underlying share.

Answer (C) is incorrect because a call option is less, not more, valuable given less time to maturity. When the option hasless time to maturity, the chance that the share price will rise is smaller.

Answer (D) is incorrect because a call option is less, not more, valuable if the price of the underlying share is lessvariable. Less variability means a lower probability of a price increase.

[708] Gleim #: 5.8.94 -- Source: Publisher

Answer (A) is incorrect because a covered option is one that is written against stock held in the option writer’s portfolio.

Answer (B) is incorrect because an unsecured option is a nonsense term.

Answer (C) is correct. A naked or uncovered option is a call option that does not have the backing of stock. Thus, theoption writer will have to purchase the underlying stock if the call option is exercised.

Answer (D) is incorrect because a put option is an option that gives the owner the right to sell the underlying asset for afixed price.

[709] Gleim #: 5.8.95 -- Source: Publisher

Answer (A) is incorrect because a European option is exercisable only at the expiration date.

Answer (B) is incorrect because a foreign option is a nonsense term.

Answer (C) is incorrect because although an option can be exercised in the future, it is not called a future option.

Answer (D) is correct. An American option is a contractual arrangement that gives the owner the right to buy or sell anasset at a fixed price at any moment in time before or on a specified date.

Gleim's CIA Test Prep: Part III: Business Analysis and Information TechnologyAnswer Explanations

(1312 questions)

Copyright 2008 Gleim Publications, Inc. Page 215Printed for Mamdouh Farag

Page 216: P.3 Answer Explanation

[710] Gleim #: 5.8.96 -- Source: Publisher

Answer (A) is incorrect because hedging decreases risk by using offsetting commitments that avoid the impact of adverseprice movements.

Answer (B) is incorrect because speculation involves the assumption of risk in the hope of gaining from price movements.

Answer (C) is correct. Derivatives, including options and futures, are contracts between the parties who contract. Unlikestocks and bonds, they are not claims on business assets. A futures contract is entered into as either a speculation or ahedge. Speculation involves the assumption of risk in the hope of gaining from price movements. Hedging is the processof using offsetting commitments to minimize or avoid the impact of adverse price movements.

Answer (D) is incorrect because speculating increases risk while hedging offsets risk.

[711] Gleim #: 5.8.97 -- Source: Publisher

Answer (A) is incorrect because a short hedge is a futures contract that is sold to protect against price declines. Theautomobile company wishes to protect itself against price increases.

Answer (B) is correct. A change in prices can be minimized or avoided by hedging. Hedging is the process of usingoffsetting commitments to minimize or avoid the impact of adverse price movements. The automobile company desires tostabilize the price of steel so that its cost to the company will not rise and cut into profits. Accordingly, the automobilecompany uses the futures market to create a long hedge, which is a futures contract that is purchased to protect againstprice increases.

Answer (C) is incorrect because the automobile company needs to purchase futures in order to protect itself from loss, notsell futures. Selling futures protects against price declines.

Answer (D) is incorrect because it is the definition of a short hedge, which is used for avoiding price declines. Theautomobile company wants to protect itself against price increases.

[712] Gleim #: 5.8.98 -- Source: Publisher

Answer (A) is incorrect because the value of the forward contract will increase when interest rates decrease.

Answer (B) is correct. Interest rate futures contracts involve risk-free bonds. When interest rates decrease over the periodof a forward contract, the value of the bonds and the forward contract increase.

Answer (C) is incorrect because the value of the forward contract will not remain constant if interest rates decline.

Answer (D) is incorrect because any decline in interest rates increases the value of the bonds.

Gleim's CIA Test Prep: Part III: Business Analysis and Information TechnologyAnswer Explanations

(1312 questions)

Copyright 2008 Gleim Publications, Inc. Page 216Printed for Mamdouh Farag

Page 217: P.3 Answer Explanation

[713] Gleim #: 5.8.99 -- Source: Publisher

Answer (A) is incorrect because it involves a net investment equal to the fair value of the stock.

Answer (B) is incorrect because insuring the CEO’s life is a transaction based on identifiable events, not underlyings.

Answer (C) is incorrect because cosigning a subsidiary’s note is a transaction based on identifiable events, notunderlyings.

Answer (D) is correct. A derivative is a financial instrument commonly used in hedging activities. Its value changes withthe change in the underlying (a specified interest rate, security price, foreign currency exchange rate, price index,commodity price, etc.). It requires little or no initial net investment compared with contracts having similar responses tochanging market conditions, and it is settled in the future. The purchase of the forward contract as a hedge of a forecastedneed to purchase wheat meets the criteria above. The forward contract’s value will change with changes in the underlying(the commodity price), requires little or no initial net investment (the purchase is in three months), and settlement is inthe future.

[714] Gleim #: 5.8.100 -- Source: CIA 04 Model Exam I-58

Answer (A) is incorrect because insurance is a contract in which the insurer undertakes to guarantee the insured againstloss from specified contingencies or perils up to a specified amount.

Answer (B) is correct. Hedging is the use of offsetting commitments to minimize the effect of adverse future pricemovements. Thus, a financial manager may limit many risk exposures by trading in futures markets.

Answer (C) is incorrect because short-selling is the sale of commodities or stocks that are not owned in anticipation of aprice decline.

Answer (D) is incorrect because factoring is the sale of accounts receivable.

[715] Gleim #: 5.8.101 -- Source: CMA 1294 1-16

Answer (A) is incorrect because margin trading involves buying securities by borrowing from a broker.

Answer (B) is correct. Short-selling is accomplished by borrowing securities from a broker and selling those securities.At a later time, the loan is repaid by buying securities on the open market and returning them to the broker. The sellerspeculates that the stock’s market price will decline.

Answer (C) is incorrect because the investor does not own the shares sold in a short-sale.

Answer (D) is incorrect because the short-seller is betting that the stock will decrease in price.

[716] Gleim #: 5.8.102 -- Source: CMA 1291 1-13

Answer (A) is incorrect because working capital management is short-term asset management.

Answer (B) is incorrect because return maximization is more aggressive than maturity matching. It entails using thelowest cost forms of financing.

Answer (C) is incorrect because financial leverage is the relationship between debt and equity financing.

Answer (D) is correct. Maturity matching, or equalizing the life of an asset and the debt instrument used to finance thatasset, is a hedging approach. The basic concept is that the company has the entire life of the asset to recover the amountinvested before having to pay the lender.

Gleim's CIA Test Prep: Part III: Business Analysis and Information TechnologyAnswer Explanations

(1312 questions)

Copyright 2008 Gleim Publications, Inc. Page 217Printed for Mamdouh Farag

Page 218: P.3 Answer Explanation

[717] Gleim #: 5.8.103 -- Source: Publisher

Answer (A) is incorrect because the price of a future contract is determined on the day of commitment, not some time inthe future.

Answer (B) is incorrect because performance is deferred in a future contract, and the price of the product is notnecessarily its present price. The price can be any price determined on the day of commitment.

Answer (C) is correct. A forward contract is an executory contract in which the parties involved agree to the terms of apurchase and a sale, but performance is deferred. Accordingly, a forward contract involves a commitment today topurchase a product on a specific future date at a price determined today.

Answer (D) is incorrect because a forward contract is a firm commitment to purchase a product. It is not based on acontingency. Also, a forward contract does not involve an exercise price (exercise price is in an option contract).

[718] Gleim #: 5.8.104 -- Source: Publisher

Answer (A) is correct. C is the current value of a call option with time t in years until expiration, S is the current stockprice, N (d i ) is the cumulative probability that a i deviation less than d i will occur in a standardized normal distribution[N (di) is an area to the left of d under the curve for the standard normal distribution], E is the call’s exercise price, e is aconstant (approximately 2.7183), and r is the annualized continuous risk-free rate of return. Thus, the value of the call is

C = (50 × .65) – (48 × .9512 × .58)= 32.50 – (48 × .9512 × .58)= 32.50 – 26.48= 6.02

Answer (B) is incorrect because 4.66 results from omitting the term e (-rt) from the equation.

Answer (C) is incorrect because 4.02 equals the estimated call price minus the difference between the current stock priceand the exercise price.

Answer (D) is incorrect because 2.00 is the difference between the current stock price and the exercise price.

[719] Gleim #: 5.9.105 -- Source: CIA 597 IV-24

Answer (A) is incorrect because the ratio may indicate undercapitalization.

Answer (B) is incorrect because fluctuations in inventory do not affect fixed-assets turnover.

Answer (C) is correct. The fixed-assets turnover ratio equals net sales divided by net fixed assets. A high ratio indicateseither that the entity is undercapitalized, that is, it cannot afford to buy enough fixed assets, or that it uses fixed assetsefficiently.

Answer (D) is incorrect because the fixed-assets turnover ratio is not a profitability indicator. It measures the efficiency ofasset management.

Gleim's CIA Test Prep: Part III: Business Analysis and Information TechnologyAnswer Explanations

(1312 questions)

Copyright 2008 Gleim Publications, Inc. Page 218Printed for Mamdouh Farag

Page 219: P.3 Answer Explanation

[720] Gleim #: 5.9.106 -- Source: CIA 597 IV-31

Answer (A) is correct. The times-interest-earned ratio equals profit or loss before taxes and interest divided by interest. Itmeasures the extent to which operating profit can decline before the entity is unable to meet its annual interest cost. Thus,it is a measure of debt-paying capacity (solvency).

Answer (B) is incorrect because liquidity ratios, e.g., the current ratio, indicate the relationship of current assets to currentliabilities.

Answer (C) is incorrect because asset management ratios indicate how effectively the entity is using its assets.

Answer (D) is incorrect because profitability ratios measure operating results.

[721] Gleim #: 5.9.107 -- Source: CIA 597 IV-27

Answer (A) is incorrect because the inventory conversion period (days of inventory) is the average time required toconvert materials into finished goods and then to sell them. This process typically occurs before the receivables collectionperiod, and the amount of time in one period does not necessarily bear any relationship to the other.

Answer (B) is incorrect because the cash conversion cycle equals the inventory conversion period, plus the receivablescollection period, minus the payables deferral period (average time between resource purchases and payment of cash forthem). It estimates the time between when the entity makes payments and when it receives cash inflows.

Answer (C) is correct. The day’s sales outstanding (days of receivables) may be stated as the accounts receivable balancedivided by average credit sales per day or as days in the year divided by the receivables turnover. It is the average timerequired to convert the entity’s receivables into cash. Thus, it is also called the receivables collection period.

Answer (D) is incorrect because the inventory divided by the sales per day is the inventory conversion period (days ofinventory).

[722] Gleim #: 5.9.108 -- Source: CIA 597 IV-35

Answer (A) is incorrect because inflation badly distorts balance sheets, depreciation charges, inventory costs, and profits.

Answer (B) is incorrect because inflation affects any financial ratio analysis involving comparisons of prior-period withcurrent-period monetary amounts.

Answer (C) is correct. Inflation is the diminution over time of the purchasing power of money. Because balance sheetamounts are expressed in terms of money, historical cost amounts for different periods are measured in units representingdifferent levels of purchasing power. Profit or loss is also distorted because of inflation’s impact on depreciation expenseand inventory costs. Inflation therefore impairs the comparability of financial statement items, whether for the same entityover time or for entities of differing ages.

Answer (D) is incorrect because inflation affects any financial ratio analysis involving comparisons of prior-period withcurrent-period monetary amounts.

Gleim's CIA Test Prep: Part III: Business Analysis and Information TechnologyAnswer Explanations

(1312 questions)

Copyright 2008 Gleim Publications, Inc. Page 219Printed for Mamdouh Farag

Page 220: P.3 Answer Explanation

[723] Gleim #: 5.9.109 -- Source: CIA 597 IV-37

Answer (A) is incorrect because the market-to-carrying-amount ratio and the total-debt-to-total-assets ratio do not provideany information about profit available to shareholders.

Answer (B) is incorrect because the price-to-earnings ratio, EPS, and the profit margin do not provide information aboutthe carrying amount of ordinary (common) equity.

Answer (C) is incorrect because the price-to-earnings ratio and the return-on-assets ratio do not provide information aboutthe carrying amount of ordinary (common) equity.

Answer (D) is correct. The profit margin equals the profit available to ordinary (common) shareholders divided by sales,the total assets turnover equals sales divided by total assets, and the product of these two ratios is the return on assets.This result is the basic Du Pont equation. In the extended Du Pont equation, the return on assets is multiplied by theleverage factor, also called the equity multiplier [total assets ÷ ordinary (common) equity at carrying amount]. Theextended Du Pont equation gives the return on ordinary (common) equity. This result is obtained because the total assetsand sales factors cancel in the multiplication of the three ratios.

[724] Gleim #: 5.9.110 -- Source: CIA 597 IV-38

Answer (A) is incorrect because the question gave no information about market share.

Answer (B) is correct. The use of financial leverage has a multiplier effect on the return on assets. The extended Du Pontformula illustrates this point by showing that the return on equity equals the return on assets times the leverage factor,also called the equity multiplier [total assets ÷ ordinary (common) equity]. Thus, greater use of debt increases the equitymultiplier and the return on equity. In this example, the equity multiplier is 1.92 (15.2% ROE ÷ 7.9% ROA), and theindustry average is 1.40 (12.9% ROE ÷ 9.2% ROA). The higher equity multiplier indicates that the entity uses more debtthan the industry average.

Answer (C) is incorrect because this comparison is with an industry average, not over time.

Answer (D) is incorrect because share valuation is a response to many factors. The higher-than-average return on equitydoes not mean that the entity has a more favorable market-to-carrying-amount ratio.

[725] Gleim #: 5.9.111 -- Source: CIA 1196 IV-53

Answer (A) is incorrect because the current ratio will be lower.

Answer (B) is correct. An entity with a higher dividend-payout ratio is distributing more of its earnings as dividends toordinary (common) shareholders. It will have less cash and less total assets than a comparable entity with a lower payoutratio. The debt-to-assets ratio will be higher because total assets are lower, and the current ratio will be lower becausecash is lower.

Answer (C) is incorrect because the debt-to-assets ratio will be higher and the current ratio will be lower.

Answer (D) is incorrect because the debt-to-assets ratio will be higher.

Gleim's CIA Test Prep: Part III: Business Analysis and Information TechnologyAnswer Explanations

(1312 questions)

Copyright 2008 Gleim Publications, Inc. Page 220Printed for Mamdouh Farag

Page 221: P.3 Answer Explanation

[726] Gleim #: 5.9.112 -- Source: CIA 596 IV-36

Answer (A) is incorrect because 19.6% is the ratio of dividends paid to the December 31 carrying amount of ordinary(common) equity.

Answer (B) is incorrect because 28.6% is the ratio of dividends paid to the sum of beginning retained earnings and profit.

Answer (C) is incorrect because 40.0% is the ratio of dividends paid to the December 31 retained earnings.

Answer (D) is correct. The dividend-payout ratio is the ratio of dividends paid to profit for the period. Hence, it equals50.0% (100 dividends ÷ 200 profit).

[727] Gleim #: 5.9.113 -- Source: CIA 596 IV-38

Answer (A) is correct. The return on assets is the ratio of profit to total assets. It equals 21.1% (200 profit ÷ 950 totalassets).

Answer (B) is incorrect because 39.2% is the ratio of profit to ordinary (common) equity.

Answer (C) is incorrect because 42.1% is the ratio of profit before tax to total assets.

Answer (D) is incorrect because 45.3% is the ratio of profit before interest and tax to total assets.

[728] Gleim #: 5.9.114 -- Source: CIA 596 IV-40

Answer (A) is correct. The profit margin is the ratio of profit to sales. It equals 6.67% (200 profit ÷ 3,000 sales).

Answer (B) is incorrect because 13.33% is the ratio of profit before tax to sales.

Answer (C) is incorrect because 14.33% is the ratio of profit before interest and taxes to sales.

Answer (D) is incorrect because 46.67% is the ratio of gross profit to sales.

[729] Gleim #: 5.9.115 -- Source: CIA 1195 IV-32

Answer (A) is incorrect because 2.5 equals EPS divided by dividends per share.

Answer (B) is correct. The P-E ratio equals the share price divided by EPS. If the dividends per share equaled 2 and thedividend-payout ratio was 40%, EPS must have been 5 (2 ÷ .4). Accordingly, the P-E ratio is 4 (20 share price ÷ 5 EPS).

Answer (C) is incorrect because 10 equals share price divided by dividends per share.

Answer (D) is incorrect because 50 equals price per share divided by the dividend-payout percentage.

Gleim's CIA Test Prep: Part III: Business Analysis and Information TechnologyAnswer Explanations

(1312 questions)

Copyright 2008 Gleim Publications, Inc. Page 221Printed for Mamdouh Farag

Page 222: P.3 Answer Explanation

[730] Gleim #: 5.9.116 -- Source: CIA 1193 IV-46

Answer (A) is incorrect because 1.42 excludes the income taxes payable from the current liabilities.

Answer (B) is incorrect because 1.08 includes prepaid expenses in the quick assets and excludes income taxes payable inthe current liabilities.

Answer (C) is correct. The acid test (quick) ratio equals quick assets (cash, financial assets held for trading, and accountsreceivable) divided by current liabilities. Quick assets total 206,500 (27,500 cash + 64,000 financial assets held fortrading + 115,000 net accounts receivable). Given current liabilities of 213,500 (67,000 accounts payable + 54,000 currentnotes payable + 70,000 income taxes payable + 22,500 other current liabilities), the quick ratio is 0.967 (206,500 ÷213,500).

Answer (D) is incorrect because 0.82 includes deferred income taxes payable in the current liabilities.

[731] Gleim #: 5.9.117 -- Source: CIA 1192 IV-52

Answer (A) is incorrect because a decrease of 1,000,000 results from omitting inventories.

Answer (B) is incorrect because the difference between all assets and all liabilities stayed the same.

Answer (C) is correct. Net working capital equals current assets (cash, accounts receivable, inventories for this entity)minus current liabilities (accounts payable, notes payable, accrued wages). From January 1 to June 30, the net workingcapital increased by 1,000,000 {[(4 + 4 + 10) – (3 + 3 + 2)] – [(3 + 5 + 8) – (2 + 4 + 1)]}.

Answer (D) is incorrect because an increase of 2,000,000 results from omitting accrued wages.

[732] Gleim #: 5.9.118 -- Source: CMA 1296 3-27

Answer (A) is incorrect because it reflects an assumption that the subunit manager does not influence the resource base(denominator of the ROI calculation).

Answer (B) is correct. ROI equals income divided by invested capital. The denominator may be defined in various ways,e.g., total assets available, assets employed, working capital plus other assets, and equity. If equity (total assets – totalliabilities) is chosen, a portion of long-term liabilities must be allocated to the investment center to determine themanager’s resource base. One problem with this definition of the resource base is that, although it has the advantage ofemphasizing return to owners, it reflects decisions at different levels of the entity: short-term liabilities incurred by theresponsibility center (operating decisions) and long-term liabilities controlled at the corporate level (long-term financingdecisions).

Answer (C) is incorrect because working capital plus other assets reflects the assumption that the manager controls short-term credit. However, no corporate-level decision to allocate long-term liabilities to subunits is necessary.

Answer (D) is incorrect because it reflects an assumption that the subunit manager does not influence the resource base(denominator of the ROI calculation).

Gleim's CIA Test Prep: Part III: Business Analysis and Information TechnologyAnswer Explanations

(1312 questions)

Copyright 2008 Gleim Publications, Inc. Page 222Printed for Mamdouh Farag

Page 223: P.3 Answer Explanation

[733] Gleim #: 5.9.119 -- Source: CIA 596 IV-53

Answer (A) is incorrect because 11 days results from subtracting the receivables collection period.

Answer (B) is incorrect because 41 days results from subtracting the receivables collection period and adding the payablesdeferral period.

Answer (C) is correct. The cash conversion cycle is the length of time between paying for purchases and receiving cashfrom the sale of finished goods. It equals the inventory conversion period, plus the receivables collection period, minus thepayables deferral period, or 75 days (58 days + 32 days – 15 days).

Answer (D) is incorrect because 90 days omits the payables deferral period.

[734] Gleim #: 5.9.120 -- Source: Publisher

Answer (A) is incorrect because EPS data must be reported on the face of the income statement for income fromcontinuing operations and net income but not for the effect of an accounting change.

Answer (B) is correct. EPS data for profit or loss from continuing operations and profit or loss must be reported on theface of the income statement. EPS data for a discontinued operation may be disclosed on the face of the income statementor in a note.

Answer (C) is incorrect because EPS data must be reported on the face of the income statement for income fromcontinuing operations and net income but not for the effect of an accounting change.

Answer (D) is incorrect because EPS data must be reported on the face of the income statement for income fromcontinuing operations and net income but not for the effect of an accounting change.

[735] Gleim #: 5.9.121 -- Source: CPA 1186 T-32

Answer (A) is incorrect because nonconvertible instruments do not entitle holders to obtain ordinary (common) shares.However, share call options allow holders to purchase shares under specified conditions.

Answer (B) is incorrect because nonconvertible instruments do not entitle holders to obtain ordinary (common) shares.However, share call options allow holders to purchase shares under specified conditions.

Answer (C) is correct. Potential ordinary (common) shares are contracts that may entitle holders to obtain ordinary(common) shares. They include options, warrants, convertible preference (preferred) shares, convertible debt, andcontingently issuable shares. Unlike an option, nonconvertible preference (preferred) shares are never potential ordinary(common) shares.

Answer (D) is incorrect because nonconvertible instruments do not entitle holders to obtain ordinary (common) shares.However, share call options allow holders to purchase shares under specified conditions.

Gleim's CIA Test Prep: Part III: Business Analysis and Information TechnologyAnswer Explanations

(1312 questions)

Copyright 2008 Gleim Publications, Inc. Page 223Printed for Mamdouh Farag

Page 224: P.3 Answer Explanation

[736] Gleim #: 5.9.122 -- Source: CIA 1196 IV-34

Answer (A) is incorrect because 12 equals current assets minus current liabilities.

Answer (B) is correct. Total assets equal total liabilities and equity. Hence, if total assets equal 100, total liabilities andequity must equal 100, and current liabilities must equal 30 (100 – 40 – 30). Because the quick ratio equals the quickassets (cash + accounts receivable) divided by current liabilities, the quick assets must equal 36 (30 × 1.2 quick ratio),and the accounts receivable balance is 26 (36 – 10 cash).

Answer (C) is incorrect because 36 equals the quick assets.

Answer (D) is incorrect because 66 equals the sum of the quick assets and current liabilities.

[737] Gleim #: 5.9.123 -- Source: CIA 1196 IV-35

Answer (A) is incorrect because the sum of cash, accounts receivable, and inventory is less than 100.

Answer (B) is incorrect because 16 is the result of neglecting to subtract the equity balance when calculating the currentliability balance.

Answer (C) is correct. Total assets (given as 100) equals the sum of cash (given as 10), accounts receivable (26),inventory, and fixed assets. Inventory can be determined because it is included in current, but not quick, assets, and thecurrent and quick ratios are known. Current assets equal 42 (1.4 current ratio × 30 current liabilities), and the quick assetsequal 36 (1.2 quick ratio × 30 current liabilities). Thus, inventory, which is the only difference in this question betweencurrent and quick assets, equals 6 (42 – 36). Fixed assets must then equal 58 (100 total assets – 10 cash – 26 accountsreceivable – 6 inventory).

Answer (D) is incorrect because 64 assumes that inventory is 0.

[738] Gleim #: 5.9.124 -- Source: Publisher

Answer (A) is incorrect because the effect is to reduce loss per share.

Answer (B) is correct. When preference (preferred) shares are cumulative, the dividend, whether earned or not, isdeducted from profit or loss from continuing operations and profit or loss, or added to any loss for the year, in computingearnings or loss. When preference (preferred) shares are noncumulative, an adjustment is made for dividends declared. Ifthe dividend is cumulative only if earned, no adjustment is necessary except to the extent of available income; that is, thepreference (preferred) dividends accumulate only to the extent of profit or loss.

Answer (C) is incorrect because preference (preferred) dividends are an adjustment when they accumulate.

Answer (D) is incorrect because preference (preferred) dividends are an adjustment when they accumulate.

Gleim's CIA Test Prep: Part III: Business Analysis and Information TechnologyAnswer Explanations

(1312 questions)

Copyright 2008 Gleim Publications, Inc. Page 224Printed for Mamdouh Farag

Page 225: P.3 Answer Explanation

[739] Gleim #: 5.9.125 -- Source: CPA 1190 I-52

Answer (A) is incorrect because 1.10 assumes no preference (preferred) dividends were declared.

Answer (B) is correct. BEPS is equal to the amount of profit or loss attributable to ordinary (common) equity holdersdivided by the weighted-average number of ordinary (common) shares outstanding during the year. To calculate thenumerator, dividends on cumulative preference (preferred) shares must be subtracted from profit or loss whether or notthe dividends were declared. Earnings per share for Year 2 thus amounted to 0.90.

330,000 – 60,000= 0.90

300,000

Answer (C) is incorrect because 0.85 assumes the ordinary (common) but not the preference (preferred) dividends weresubtracted from the numerator.

Answer (D) is incorrect because 0.65 assumes all dividends are subtracted from the numerator.

[740] Gleim #: 5.9.126 -- Source: CMA 1294 2-15

Answer (A) is incorrect because 2.90 is based on the shares outstanding at year-end.

Answer (B) is correct. BEPS equals profit or loss attributable to ordinary (common) equity holders divided by theweighted-average number of ordinary (common) shares outstanding. The latter is calculated as follows:

24,000 × (2 ÷ 12) = 4,00029,400 × (4 ÷ 12) = 9,80036,000 × (5 ÷ 12) = 15,00035,040 × (1 ÷ 12) = 2,920

31,720

Accordingly, BEPS is 3.20 (101,504 profit ÷ 31,720 shares).

Answer (C) is incorrect because 3.26 is based on an unweighted average of the four levels of shares outstanding duringthe year.

Answer (D) is incorrect because 3.45 is based on the shares outstanding March through June.

[741] Gleim #: 5.9.127 -- Source: CMA 693 2-18

Answer (A) is correct. A public entity or any entity that discloses EPS must report EPS information on the face of theincome statement for both profit or loss from continuing operations and profit or loss. In addition, EPS data for anydiscontinued operation must be presented on the face of the income statement or in a note. The entity must present BEPSand DEPS with equal prominence.

Answer (B) is incorrect because certain EPS amounts must be presented on the face of the income statement.

Answer (C) is incorrect because EPS also must be presented for profit or loss from continuing operations and anydiscontinued operations.

Answer (D) is incorrect because BEPS and DEPS are to be presented on the face of the income statement with equalprominence.

Gleim's CIA Test Prep: Part III: Business Analysis and Information TechnologyAnswer Explanations

(1312 questions)

Copyright 2008 Gleim Publications, Inc. Page 225Printed for Mamdouh Farag

Page 226: P.3 Answer Explanation

[742] Gleim #: 5.9.128 -- Source: CPA 577 T-27

Answer (A) is incorrect because, when the exercise price exceeds the average market price, the result is antidilutive.

Answer (B) is incorrect because, when the exercise price exceeds the average market price, the result is antidilutive.

Answer (C) is incorrect because, if an option is not in the money, it is antidilutive.

Answer (D) is correct. Options and warrants [instruments that give the holders the right to purchase ordinary (common)shares of the entity] issued by the reporting entity are assumed to be exercised at the beginning of the period or at time ofissuance, if later. The proceeds are assumed to be from an issuance at the average market price for the period. Thedifference between (1) the shares issued and (2) the shares that would have been issued at the average market price is anissue for no consideration. If the options are in the money (exercise price is less than average market price), they aredilutive because (1) exceeds (2), and the excess will be added to the BEPS denominator. However, when the exerciseprice exceeds the average market price, the result is antidilutive.

[743] Gleim #: 5.9.129 -- Source: CIA 1195 III-67

Answer (A) is incorrect because ROI is not certain to increase if investment increases.

Answer (B) is incorrect because ROI is not certain to increase if revenue, costs, and investment decrease.

Answer (C) is incorrect because ROI is not certain to increase if costs and investment increase.

Answer (D) is correct. An increase in revenue and a decrease in costs will increase the ROI numerator. A decrease ininvestment will decrease the denominator. The ROI must increase in this situation.

[744] Gleim #: 5.9.130 -- Source: CIA 594 IV-13

Answer (A) is incorrect because 10.00 excludes retained earnings from the numerator.

Answer (B) is incorrect because 15.00 excludes ordinary (common) shares from the numerator.

Answer (C) is incorrect because 21.63 is based on average equity.

Answer (D) is correct. Book value per share, based on balance sheet amounts, measures the per share amount that wouldbe received if the entity were liquidated. The ratio is calculated as ordinary (common) equity divided by the number ofoutstanding shares.

Gleim's CIA Test Prep: Part III: Business Analysis and Information TechnologyAnswer Explanations

(1312 questions)

Copyright 2008 Gleim Publications, Inc. Page 226Printed for Mamdouh Farag

Page 227: P.3 Answer Explanation

[745] Gleim #: 5.9.131 -- Source: CIA 595 IV-33

Answer (A) is incorrect because 0 days equals accounts receivable divided by sales.

Answer (B) is incorrect because 3 days is the result if accounts payable is used rather than accounts receivable.

Answer (C) is correct. Days’ sales of receivables (days’ sales outstanding) equals the days in the year divided by thereceivables turnover ratio (net credit sales ÷ average accounts receivable). Assuming sales are net credit sales and thatending accounts receivable equals average accounts receivable, the turnover ratio is 58 (5,800 ÷ 100), and the days’ salesof receivables equals 6 days (365 ÷ 58) (rounded).

Answer (D) is incorrect because 7 days is the result if average cost of goods sold per day is used rather than sales.

[746] Gleim #: 5.9.132 -- Source: CIA, adapted

Answer (A) is incorrect because a higher profit margin would reduce the additional financing needed, as stated in thequestion. The result would be a downward, not an upward, shift in the funds-needed line.

Answer (B) is incorrect because a higher profit margin would cause a downward shift with a lower slope.

Answer (C) is correct. A higher profit margin would reduce the additional financing needed, shifting the funds neededline down.

Answer (D) is incorrect because the line would become less, not more, steep if the firm had a higher profit margin.

[747] Gleim #: 5.9.133 -- Source: CMA 1291 1-5

Answer (A) is correct. A purchase of treasury share involves a decrease in assets (usually cash) and a correspondingdecrease in shareholders’ equity. Thus, equity is reduced and the debt-to-equity ratio and financial leverage increase.

Answer (B) is incorrect because equity (assets – liabilities) declines. A treasury share purchase is equivalent to a specialdividend because assets are paid out to one or more shareholders.

Answer (C) is incorrect because a firm’s interest coverage ratio is unaffected. Earnings, interest expense, and taxes willall be the same regardless of the transaction.

Answer (D) is incorrect because the purchase of treasury share is antidilutive; the same earnings will be spread over fewershares. Some firms purchase treasury share for this reason.

Gleim's CIA Test Prep: Part III: Business Analysis and Information TechnologyAnswer Explanations

(1312 questions)

Copyright 2008 Gleim Publications, Inc. Page 227Printed for Mamdouh Farag

Page 228: P.3 Answer Explanation

[748] Gleim #: 5.9.134 -- Source: CIA 1193 IV-36

Answer (A) is correct. Working capital is the excess of total current assets (CA) over total current liabilities (CL). Thus,working capital at the end of January equals 80,500 computed as follows:

CA* CL*Beginning working capital 70,000Performed services on account 30,000 I NPurchased supplies on account -0- I IConsumed supplies (4,000) D NPurchased office equipment (2,000) D NPaid short-term bank loan -0- D DPaid salaries (10,000) D NAccrued salaries (3,500) N I Working capital, end of January 80,500

* N = no effect; I = increase; D = decrease

Answer (B) is incorrect because 78,500 ignores the consumed supplies, the cash purchase of office equipment, and theaccrued salaries. However, it incorrectly considers the supplies purchased on account and the repayment of the short-termbank loan.

Answer (C) is incorrect because 50,500 does not include the services performed on account.

Answer (D) is incorrect because 47,500 omits the services performed on account and accrued salaries but includes therepayment of short-term loan.

[749] Gleim #: 5.9.135 -- Source: CIA 593 IV-39

Answer (A) is incorrect because a horizontal analysis indicates the proportionate change over a period of time and isuseful in trend analysis of an individual entity.

Answer (B) is correct. Vertical analysis is the expression of each item on a financial statement in a given period inrelation to a base figure. On the income statement, each item is stated as a percentage of sales. Thus, the percentages forthe entity in question can be compared with industry norms.

Answer (C) is incorrect because activity ratio analysis includes the preparation of turnover ratios such as those forreceivables, inventory, and total assets.

Answer (D) is incorrect because the defensive-interval ratio is part of a liquidity analysis.

Gleim's CIA Test Prep: Part III: Business Analysis and Information TechnologyAnswer Explanations

(1312 questions)

Copyright 2008 Gleim Publications, Inc. Page 228Printed for Mamdouh Farag

Page 229: P.3 Answer Explanation

[750] Gleim #: 5.9.136 -- Source: CIA 594 IV-12

Answer (A) is incorrect because 3.375 times results from including in the numerator deductions for taxes and interest.

Answer (B) is incorrect because 6.75 times results from including in the numerator a deduction for interest.

Answer (C) is correct. The TIE ratio is a leverage ratio. It emphasizes the ability to pay interest expense. The ratio equalsprofit before interest and taxes divided by interest.

Answer (D) is incorrect because 9.5 times results from failing to deduct the administrative expenses from the numerator.

[751] Gleim #: 5.9.137 -- Source: CIA 594 IV-14

Answer (A) is correct. Rate of return on equity, a profitability ratio, measures the rate of return on investment. The ratioequals profit divided by average equity.

Answer (B) is incorrect because 58.06% excludes ordinary (common) shares from the denominator.

Answer (C) is incorrect because 67.50% excludes retained earnings from the denominator.

Answer (D) is incorrect because 71.68% excludes interest expense and tax expense from the numerator.

Gleim's CIA Test Prep: Part III: Business Analysis and Information TechnologyAnswer Explanations

(1312 questions)

Copyright 2008 Gleim Publications, Inc. Page 229Printed for Mamdouh Farag

Page 230: P.3 Answer Explanation

[752] Gleim #: 5.9.138 -- Source: CIA 1195 IV-30

Answer (A) is correct. BEPS equals profit minus preference (preferred) dividends, divided by ordinary (common) sharesoutstanding. Thus, BEPS equals 1.73 {[150,000 – (2 × 10,000 preference shares)] ÷ 75,000}.

Answer (B) is incorrect because 1.87 assumes the preference (preferred) dividend is 1.

Answer (C) is incorrect because 2.00 does not subtract preference (preferred) dividends from profit.

Answer (D) is incorrect because 2.27 adds the preference (preferred) dividends to profit.

[753] Gleim #: 5.9.139 -- Source: CIA 592 IV-40

Answer (A) is correct. The current ratio equals current assets divided by current liabilities. At the end of Year Six, it was4.5 (54,000 ÷ 12,000).

Answer (B) is incorrect because 2.4 was derived by dividing current liabilities (12,000) by the amount of cash (5,000),which produces no meaningful ratio.

Answer (C) is incorrect because 2.0 was derived by dividing the total of cash, financial assets held for trading, and netreceivables (the quick assets) by total current liabilities. This computation determines the acid-test or quick ratio [(5,000+ 3,000 + 16,000) ÷ 12,000].

Answer (D) is incorrect because 1.5 [12,000 ÷ (5,000 + 3,000)] was derived by dividing total current liabilities by the sumof cash and financial assets held for trading, which produces no meaningful ratio.

[754] Gleim #: 5.9.140 -- Source: CIA 592 IV-41

Answer (A) is incorrect because 2.4 was derived by dividing total current liabilities (12,000) by the amount of cash(5,000), which produces no meaningful ratio.

Answer (B) is incorrect because 2.18 was derived by dividing quick assets (5,000 + 3,000 + 16,000) by accounts payable(11,000). The denominator should include all current liabilities other than accounts payable.

Answer (C) is correct. Liquidity ratios measure the ability to meet short-term obligations. A commonly used liquidityratio is the acid-test or quick ratio, which equals the sum of the quick assets (net accounts receivable, financial assets heldfor trading, and cash) divided by current liabilities. This ratio at the end of Year Six is 2.0 [(5,000 + 3,000 + 16,000) ÷12,000].

Answer (D) is incorrect because 1.5 was derived by dividing total current liabilities (12,000) by the sum of cash (5,000)and financial assets held for trading (3,000), which produces no meaningful ratio.

Gleim's CIA Test Prep: Part III: Business Analysis and Information TechnologyAnswer Explanations

(1312 questions)

Copyright 2008 Gleim Publications, Inc. Page 230Printed for Mamdouh Farag

Page 231: P.3 Answer Explanation

[755] Gleim #: 5.9.141 -- Source: CIA 592 IV-42

Answer (A) is incorrect because the quick ratio compares current assets (minus inventory) with current liabilities; it doesnot provide a basis for conclusions about long-term investments.

Answer (B) is incorrect because the quick ratio compares current assets (minus inventory) with current liabilities; it doesnot provide a basis for determining the effects or the existence of inventory write-offs.

Answer (C) is correct. RST’s quick ratio decreased from 2.5 in Year Five [(4,000 cash + 2,000 financial assets held fortrading + 14,000 net A/R) ÷ 8,000] to 2.0 in Year Six [(5,000 + 3,000 + 16,000) ÷ 12,000]. RST has fewer assets that areeasily convertible to cash available to meet current liabilities. Thus, its ability to meet short-term financing needs hasdeclined.

Answer (D) is incorrect because RST is less liquid.

[756] Gleim #: 5.9.142 -- Source: CIA 592 IV-58

Answer (A) is incorrect because 21.88% equals net operating profit divided by total assets.

Answer (B) is incorrect because 35.00% equals net operating profit divided by net plant and equipment.

Answer (C) is correct. Net operating assets equal total assets minus goodwill, or 7,500,000. The return on net operatingassets is therefore 23.33% (1,750,000 net operating profit ÷ 7,500,000 net operating assets).

Answer (D) is incorrect because 12.50% equals profit divided by total assets.

[757] Gleim #: 5.9.143 -- Source: CIA 1193 IV-58

Answer (A) is incorrect because 20% excludes short-term debt.

Answer (B) is incorrect because 40% divides long-term debt by total equity.

Answer (C) is correct. The debt ratio equals total debt divided by total assets, or .50 [(3,000,000,000 total debt andequity – 500,000,000 ordinary (common) shares – 1,000,000,000 retained earnings) ÷ 3,000,000,000 total assets].

Answer (D) is incorrect because 100% divides total debt by total equity.

[758] Gleim #: 5.9.144 -- Source: CIA 1193 IV-59

Answer (A) is correct. The return on equity equals profit (70,000,000) divided by equity (500,000,000 ordinary(common) shares + 1,000,000,000 retained earnings).

Answer (B) is incorrect because 6.67% uses profit before taxes instead of profit.

Answer (C) is incorrect because 13.33% uses profit before interest and taxes instead of profit.

Answer (D) is incorrect because 14% divides profit by ordinary (common) shares.

Gleim's CIA Test Prep: Part III: Business Analysis and Information TechnologyAnswer Explanations

(1312 questions)

Copyright 2008 Gleim Publications, Inc. Page 231Printed for Mamdouh Farag

Page 232: P.3 Answer Explanation

[759] Gleim #: 5.9.145 -- Source: CIA 590 IV-59

Answer (A) is correct. Because total assets will decline without any impact on sales, the total assets turnover ratio (sales÷ total assets) will increase. In addition, a reduced debt level should cause a reduction in annual interest payments, so thetimes-interest-earned ratio [(profit + interest + taxes) ÷ interest] should increase.

Answer (B) is incorrect because the times-interest-earned ratio will increase.

Answer (C) is incorrect because the total assets turnover ratio will increase.

Answer (D) is incorrect because the total assets turnover ratio will increase and the times-interest-earned ratio willincrease.

[760] Gleim #: 5.9.146 -- Source: CIA 1188 IV-60

Answer (A) is correct. The borrowing of funds for 180 days constitutes short-term borrowing. The new machine is a fixedasset. Current liabilities have increased, and current assets have remained constant. Consequently, the current ratio(current assets ÷ current liabilities) has decreased. Total debt and total assets increased by the same absolute amount, andthe debt ratio (total debt ÷ total assets) should have increased, assuming total debt is less than total asset.

Answer (B) is incorrect because the current ratio decreased and the debt ratio increased.

Answer (C) is incorrect because the current ratio decreased and the debt ratio increased.

Answer (D) is incorrect because the current ratio decreased and the debt ratio increased.

[761] Gleim #: 5.9.147 -- Source: CMA 688 1-20

Answer (A) is incorrect because it does not refer to expenses that have to be paid out of the sales dollars. Also, the salesrevenue available to pay liabilities is indeterminable. Thus, sales minus debt service is not a liquidity measure.

Answer (B) is incorrect because fixed assets are included in the numerator of the ratio; hence, this ratio is not a measureof liquidity.

Answer (C) is correct. Liquidity is the degree to which assets can be converted to cash in the short run to meet maturingobligations. The usual measures of liquidity are the current ratio and the quick (acid-test) ratio. The quick ratio is the bestmeasure of short-term liquidity because it uses only the most liquid assets (for example, cash, financial assets held fortrading, and receivables) in the calculation; inventories are not included because they are two steps away from cash (theyhave to be sold, and then the receivable has to be collected).

Answer (D) is incorrect because it shows only the amount of profit available to pay interest expense and not otherobligations. Also, profit is not a measure of liquidity because it includes items that did not result in an increase of liquidassets.

Gleim's CIA Test Prep: Part III: Business Analysis and Information TechnologyAnswer Explanations

(1312 questions)

Copyright 2008 Gleim Publications, Inc. Page 232Printed for Mamdouh Farag

Page 233: P.3 Answer Explanation

[762] Gleim #: 5.9.148 -- Source: CMA 691 1-4

Answer (A) is incorrect because profit margin and return on assets are measures of the management of all assets.

Answer (B) is correct. Working capital equals current assets minus current liabilities. The acid-test ratio equals quickassets (current assets – inventory – prepaid expenses) divided by current liabilities. Inventory (a current asset) turnoverequals cost of goods sold divided by average inventory. The average collection period (number of days’ sales in accountsreceivable, a current asset) equals the number of days in a year divided by the accounts receivable turnover (net creditsales ÷ average receivables). The foregoing are some of the many ratios that can be used to evaluate working capitalmanagement.

Answer (C) is incorrect because times interest earned and the debt-to-equity ratio are measures of capital structuremanagement.

Answer (D) is incorrect because return on equity is a measure of capital structure management.

[763] Gleim #: 5.9.149 -- Source: CMA 1288 1-11

Answer (A) is correct. The times-interest-earned (interest coverage) ratio is computed by dividing the profit available forpaying interest (pretax, pre-interest profit) by the annual interest expense. The first step is to determine the annualinterest expense:

First Mortgage 9.0% × 5,000 = 450Debenture 10.2% × 10,000 = 1,020Subordinated 12.0% × 6,000 = 720 TOTAL EXPENSE 2,190

Dividing the pretax, pre-interest profit of 10,000 by the 2,190 of interest expense produces an interest coverage ratio of4.57.

Answer (B) is incorrect because the coverage of the debt is measured by the times-interest-earned ratio, which is found bydividing the pretax, pre-interest profit of 10,000 by the 2,190 of interest expense, yielding 4.57.

Answer (C) is incorrect because the coverage of the debt is measured by the times-interest-earned ratio, which is found bydividing the pretax, pre-interest profit of 10,000 by the 2,190 of interest expense, yielding 4.57.

Answer (D) is incorrect because the coverage of the debt is measured by the times-interest-earned ratio, which is found bydividing the pretax, pre-interest profit of 10,000 by the 2,190 of interest expense, yielding 4.57.

Gleim's CIA Test Prep: Part III: Business Analysis and Information TechnologyAnswer Explanations

(1312 questions)

Copyright 2008 Gleim Publications, Inc. Page 233Printed for Mamdouh Farag

Page 234: P.3 Answer Explanation

[764] Gleim #: 5.9.150 -- Source: CMA 1291 1-9

Answer (A) is incorrect because increased economic uncertainty makes equity financing more desirable. There is no legalmandate to make regular payments to equity holders.

Answer (B) is incorrect because increased operating leverage (a greater degree of fixed production costs) increases risk.Hence, debt holders will require higher rates of interest. Higher interest costs will reduce the desirability of debt as ameans of financing.

Answer (C) is incorrect because an increase in the price-earnings ratio makes equity financing less costly and moredesirable. Hence, a given level of earnings will support a higher share price. The cost of equity capital declines when theshare price increases.

Answer (D) is correct. Debt financing usually has a lower cost than equity financing because interest payments areusually tax deductible. If tax rates rise, debt becomes even more desirable because the tax shield becomes more valuable.The disadvantages of debt include the increase in fixed payments (interest). Thus, in an unstable economy, debt representsa greater risk to a firm than equity financing.

[765] Gleim #: 5.9.151 -- Source: CIA 587 III-42

Answer (A) is incorrect because increasing the denominator of the ratio decreases turnover.

Answer (B) is correct. Inventory turnover equals cost of sales divided by average inventory. Reducing inventory thereforeincreases turnover. Sales incentives to improve sales should lower inventory levels.

Answer (C) is incorrect because increasing price would probably decrease sales, increase inventory, and decreaseturnover.

Answer (D) is incorrect because a constant inventory level would not affect the turnover rate.

[766] Gleim #: 5.9.152 -- Source: CIA 595 IV-34

Answer (A) is incorrect because four times results if profit before tax rather than profit before interest and tax is used inthe formula.

Answer (B) is correct. The times-interest-earned ratio for A equals profit before interest and tax divided by interestexpense. Profit before interest and tax is 100 (40 profit + 20 interest + 40 tax expense). Hence, interest was earned fivetimes (100 ÷ 20).

Answer (C) is incorrect because 10 times results if depreciation expense is not deducted in calculating profit beforeinterest and tax.

Answer (D) is incorrect because 15 times results if gross margin is used in the numerator rather than profit before interestand tax.

[767] Gleim #: 5.9.153 -- Source: CIA 595 IV-36

Answer (A) is incorrect because 1.67 results if retained earnings is omitted from the numerator.

Answer (B) is incorrect because 2.50 results if the amount of ordinary (common) shares is omitted from the numerator.

Answer (C) is correct. The book value per share for A equals the sum of ordinary (common) shares and retained earnings,divided by the number of shares, or 4.17 [(100 + 150) ÷ 60].

Answer (D) is incorrect because 5.00 is the book value per share for B.

Gleim's CIA Test Prep: Part III: Business Analysis and Information TechnologyAnswer Explanations

(1312 questions)

Copyright 2008 Gleim Publications, Inc. Page 234Printed for Mamdouh Farag

Page 235: P.3 Answer Explanation

[768] Gleim #: 5.9.154 -- Source: CIA 595 IV-79

Answer (A) is incorrect because 65,000 is the simple average of the shares outstanding on January 1 and December 31.

Answer (B) is incorrect because 70,000 is the year-end number of shares outstanding.

Answer (C) is correct. The weighted-average number of shares outstanding for the year (76,000 shares) is used in theBEPS calculation.

Dates Shares Fraction WeightedOutstanding Outstanding of Year Shares

January 1 - April 30 60,000 4 ÷ 12 20,000May 1 - June 30 96,000 2 ÷ 12 16,000

July 1 - September 30 90,000 3 ÷ 12 22,500October 1 - December 31 70,000 3 ÷ 12 17,500

76,000

Answer (D) is incorrect because 79,000 is the simple average of the shares outstanding at various times during the year.

[769] Gleim #: 5.9.155 -- Source: CIA 1195 IV-35

Answer (A) is correct. If the debt-to-total assets ratio is 70% and debt is 1,000,000, total assets must be 1,428,571.4(1,000,000 ÷ .7). Given total asset turnover (sales ÷ total assets) of 3.5, sales must be 5,000,000 (3.5 × 1,428,571.4).

Answer (B) is incorrect because 2,450,000.00 is obtained if total assets is calculated as total debt multiplied by .7.

Answer (C) is incorrect because 408,163.26 results if sales is determined by dividing total assets by 3.5.

Answer (D) is incorrect because 200,000.00 is obtained by calculating total assets as total debt multiplied by .7 and salesas total assets divided by 3.5.

[770] Gleim #: 5.9.156 -- Source: CIA 1194 IV-12

Answer (A) is incorrect because 0.80 is based on total liabilities instead of current liabilities.

Answer (B) is incorrect because 1.29 uses working capital in the numerator.

Answer (C) is correct. The acid-test ratio equals current assets (400) minus inventories (100), divided by currentliabilities (100 + 75 = 175), or 1.71.

Answer (D) is incorrect because 2.29 is the current ratio.

Gleim's CIA Test Prep: Part III: Business Analysis and Information TechnologyAnswer Explanations

(1312 questions)

Copyright 2008 Gleim Publications, Inc. Page 235Printed for Mamdouh Farag

Page 236: P.3 Answer Explanation

[771] Gleim #: 5.9.157 -- Source: CIA 596 IV-34

Answer (A) is correct. The weighted-average number of ordinary (common) shares outstanding must be increased toreflect the shares into which the bonds could be converted. Also, the effect of the bond interest on profit attributable toordinary (common) shareholders [profit after subtracting preference (preferred) dividends] must be eliminated. In thisway, diluted earnings per share is calculated as if the bonds had been converted into ordinary (common) shares as of thestart of the year.

Answer (B) is incorrect because the profit attributable to ordinary (common) shareholders must be increased.

Answer (C) is incorrect because the weighted-average number of ordinary (common) shares outstanding must beincreased, and the profit attributable to ordinary (common) shareholders must be increased.

Answer (D) is incorrect because the weighted-average number of ordinary (common) shares outstanding must beincreased, and the profit attributable to ordinary (common) shareholders must be increased.

[772] Gleim #: 5.9.158 -- Source: CIA 596 IV-37

Answer (A) is incorrect because 160 equals net quick assets minus current liabilities.

Answer (B) is correct. Net working capital is the difference between current assets and current liabilities. It equals 210(100 cash + 200 A/R + 50 inventory – 140 A/P).

Answer (C) is incorrect because 350 equals current assets.

Answer (D) is incorrect because 490 equals current assets plus current liabilities.

[773] Gleim #: 5.9.159 -- Source: CIA, adapted

Answer (A) is incorrect because the market value to book value ratio is the market value of ordinary (common) equity pershare derived by dividing the book value of ordinary (common) equity by the average number of shares outstanding.Neither this ratio nor the total debt to total assets ratio provides any information about net income available toshareholders, which is necessary to calculate the return on equity.

Answer (B) is incorrect because the price to earnings ratio is the ratio of the stock’s market price divided by earnings pershare; the earnings per share is the net income available to shareholders divided by the average number of sharesoutstanding; and the profit margin is profit divided by sales. While all three ratios contain much information about theequity account, none of them provides information about the book value of ordinary (common) equity, which is necessaryto calculate the return on equity.

Answer (C) is incorrect because the price to earnings ratio is the ratio of the stock’s market price divided by earnings pershare, and the return on assets ratio is net income divided by assets. Neither of these two ratios provides informationabout the book value of ordinary (common) equity, which is necessary to calculate the return on equity.

Answer (D) is correct. The simple Du Pont equation combines the following ratios:

(Net income available to Shareholders ÷ Sales) × (Sales ÷ Total assets) × (Total assets ÷ Ordinary (common) equity atBook value) = Return on equity.

The total assets and sales cancel out in multiplication, leaving net income available to shareholders divided by ordinary(common) equity at book value, which equals return on equity.

Gleim's CIA Test Prep: Part III: Business Analysis and Information TechnologyAnswer Explanations

(1312 questions)

Copyright 2008 Gleim Publications, Inc. Page 236Printed for Mamdouh Farag

Page 237: P.3 Answer Explanation

[774] Gleim #: 5.9.160 -- Source: CIA, adapted

Answer (A) is incorrect because inflation also distorts depreciation charges, inventory costs, and profits.

Answer (B) is incorrect because inflation impacts both aspects.

Answer (C) is correct. Inflation has considerable impact on financial ratio analysis. This impact distorts both analysis of asingle firm over time and comparative analysis of different aged firms.

Answer (D) is incorrect because inflation impacts both aspects.

[775] Gleim #: 5.9.161 -- Source: CIA 596 IV-50

Answer (A) is incorrect because 6% applies only to the first 75 million of new financing. Given that the optimal capitalbudget exceeds 75 million, 6% cannot be the discount rate.

Answer (B) is incorrect because 8% is the IRR of Project C.

Answer (C) is correct. The appropriate discount rate (the cost of capital used in capital budgeting) theoretically isdetermined at the intersection of the IOS and MCC schedules. This intersection is at an MCC of 10% and an optimalcapital budget of 125 million. However, if the optimal capital budget is assumed to be 150 million, the company is still inthe second interval of the MCC schedule. The marginal cost of financing in this part of the schedule is 10%.

Answer (D) is incorrect because at an investment level of 150 million, the MCC is 10%.

[776] Gleim #: 5.9.162 -- Source: CIA 1194 IV-3

Answer (A) is correct. The entity has a dividend payout ratio of 25% of profit. Profit before taxes is 85,000 [500,000sales – 250,000 CGS – 25,000 G&A expenses – (10% interest rate × 400,000 LT debt) – (600,000 fixed assets ÷ 6years)]. Hence, after-tax profit is 42,500 [(1.0 – .5) × 85,000], and the dividend payout is 10,625 (25% × 42,500).

Answer (B) is incorrect because 21,250 uses a 50% dividend payout ratio instead of 25%.

Answer (C) is incorrect because 23,125 calculates profit without subtracting the 100,000 depreciation expense.

Answer (D) is incorrect because 42,500 is the after-tax profit.

[777] Gleim #: 5.9.163 -- Source: CIA 1194 IV-6

Answer (A) is incorrect because the ordinary (common) shares account is not affected, but dividends will increase.

Answer (B) is incorrect because the ordinary (common) shares account is not affected, but dividends will increase.

Answer (C) is correct. The balance in the ordinary (common) shares account represents share capital, i.e., capitalcontributed by owners, not earnings retained in the business. It is not affected by a change in tax rate. Dividends willincrease, however, because they are a constant percentage of after-tax earnings.

Answer (D) is incorrect because the ordinary (common) shares account is not affected, but dividends will increase.

Gleim's CIA Test Prep: Part III: Business Analysis and Information TechnologyAnswer Explanations

(1312 questions)

Copyright 2008 Gleim Publications, Inc. Page 237Printed for Mamdouh Farag

Page 238: P.3 Answer Explanation

[778] Gleim #: 5.10.164 -- Source: CMA 693 1-3

Answer (A) is incorrect because the trough of a business cycle is characterized by excess resources and a lack of demand-pull inflation; thus, prices are more apt to be stable or decline during the trough.

Answer (B) is incorrect because investors are unwilling to risk new investments during the trough.

Answer (C) is incorrect because costs ordinarily do not rise during the trough.

Answer (D) is correct. There are four phases of a business cycle: trough, recovery, peak, and recession. The trough ismarked by low levels of economic activity and underuse of resources. Investors are unwilling to risk new investments inproductive capacity.

[779] Gleim #: 5.10.165 -- Source: CMA 693 1-2

Answer (A) is incorrect because, technically, the purchasing power of money is unrelated to the concept of recession;purchasing power relates to inflation. However, demand-pull inflation is unlikely during a recession.

Answer (B) is incorrect because the natural rate of unemployment, which results from the normal workings of the labormarket, remains unchanged. Any increase in unemployment will be cyclical, i.e., the amount caused by inadequateaggregate demand.

Answer (C) is correct. There are four phases of a business cycle: trough, recovery, peak, and recession. During therecessionary phase of a business cycle, economic activities and employment levels contract and society’s resources areunderused. Because of the underuse of resources, potential national income will exceed actual national income.

Answer (D) is incorrect because potential income will be greater than actual national income given the existence of idlecapacity.

[780] Gleim #: 5.10.166 -- Source: CIA 595 IV-55

Answer (A) is incorrect because a falling money supply is associated with falling GDP.

Answer (B) is incorrect because a decline in the issuance of building permits signals lower expected building activity anda falling GDP.

Answer (C) is incorrect because an increase in the timeliness of delivery by vendors indicates slacking business demandand potentially falling GDP.

Answer (D) is correct. An economic indicator is highly correlated with changes in aggregate economic activity. A leadingindicator changes prior to a change in the direction of the business cycle. The leading indicators included in theConference Board’s index are average weekly hours worked by manufacturing workers, unemployment claims, consumergoods orders, stock prices, orders for fixed assets, building permits, timeliness of deliveries, money supply, consumerconfidence, and the spread between the yield on 10-year Treasury bonds and the federal funds rate. An increase in weeklyhours worked by production workers is favorable for economic growth.

Gleim's CIA Test Prep: Part III: Business Analysis and Information TechnologyAnswer Explanations

(1312 questions)

Copyright 2008 Gleim Publications, Inc. Page 238Printed for Mamdouh Farag

Page 239: P.3 Answer Explanation

[781] Gleim #: 5.10.167 -- Source: CIA 597 IV-58

Answer (A) is correct. Economists use a variety of economic indicators to forecast turns in the business cycle. Economicindicators are variables that in the past have had a high correlation with aggregate economic activity. The best known arethe composite indexes calculated by The Conference Board, a private research group with more than 2,700 corporate andother members worldwide. Indicators may lead, lag, or coincide with economic activity. The Conference Board’s laggingindicators include average duration of unemployment in weeks, the change in the index of labor cost per unit of output,the average prime rate charged by banks, the ratio of manufacturing and trade inventories to sales, the commercial andindustrial loans outstanding, the ratio of consumer installment credit outstanding to personal income, and the change inthe CPI for services.

Answer (B) is incorrect because orders for consumer and producer goods are leading indicators.

Answer (C) is incorrect because housing starts are leading indicators.

Answer (D) is incorrect because consumer expectations are leading indicators.

[782] Gleim #: 5.10.168 -- Source: Publisher

Answer (A) is incorrect because these are stages in the business cycle.

Answer (B) is incorrect because these are stages in the business cycle.

Answer (C) is incorrect because these are stages in the business cycle.

Answer (D) is correct. A business cycle has four stages: the trough, recovery, peak, and recession. Acceleration is not oneof the stages.

[783] Gleim #: 5.10.169 -- Source: CIA, adapted

Answer (A) is incorrect because poor product quality is evident during the introduction stage of the product life cycle.

Answer (B) is correct. In the growth stage, sales and profits increase rapidly, cost per customer decreases, customers areearly adopters, new competitors enter an expanding market, new product models and features are introduced, andpromotion spending declines or remains stable. The firm enters new market segments and distribution channels andattempts to build brand loyalty and achieve the maximum share of the market. Thus, prices are set to penetrate the market,distribution channels are extended, and the mass market is targeted through advertising. The strategy is to advance bythese means and by achieving economies of productive scale.

Answer (C) is incorrect because competitors are most numerous and products become less differentiated during thematurity stage of the product life cycle. In this stage, imitators have entered the market and competitors have learnedwhich technologies and features are successful.

Answer (D) is incorrect because the quality of the products becomes more variable and products are less differentiatedduring the decline stage of the product life cycle.

Gleim's CIA Test Prep: Part III: Business Analysis and Information TechnologyAnswer Explanations

(1312 questions)

Copyright 2008 Gleim Publications, Inc. Page 239Printed for Mamdouh Farag

Page 240: P.3 Answer Explanation

[784] Gleim #: 5.10.170 -- Source: CIA, adapted

Answer (A) is incorrect because a decline in the firm’s purchases, resulting in a decline in the firm’s inventory levels, isnot the first symptom. It will occur only when production declines as a result of a drop in sales.

Answer (B) is correct. The sales of most product types and brands eventually decrease permanently. This decline may beslow or rapid. This first symptom of the decline stage of a product’s life cycle triggers such other effects as price cutting,narrowing of the product line, and reduction in promotion budgets.

Answer (C) is incorrect because a decline in production costs may be due to many factors, e.g., new plant technology orthe increased availability of raw materials. Moreover, production costs may decrease in any stage of a product’s life cycleand not specifically in the decline stage.

Answer (D) is incorrect because a change in prices is a marketing decision. It is an action that may be taken in thematurity stage to compete in the market. Moreover, a decrease in the product’s prices is a response to a permanent declinein sales.

[785] Gleim #: 5.10.171 -- Source: CIA, adapted

Answer (A) is correct. The introduction stage is characterized by slow sales growth and lack of profits because of the highexpenses of promotion and selective distribution to generate awareness of the product and encourage customers to try it.Thus, the per-customer cost is high. Competitors are few, basic versions of the product are produced, and higher-incomecustomers (innovators) are usually targeted. Cost-plus prices are charged. They may initially be high to permit costrecovery when unit sales are low. The strategy is to infiltrate the market, plan for financing to cope with losses, buildsupplier relations, increase production and marketing efforts, and plan for competition.

Answer (B) is incorrect because, during the introduction stage, little competition exists. Competitors tend not to enter themarket until they have greater assurance of profits.

Answer (C) is incorrect because no mass market is available during the introduction stage.

Answer (D) is incorrect because, by definition, not many alternatives are available during the introduction stage of aninnovative product.

[786] Gleim #: 5.10.172 -- Source: CIA, adapted

Answer (A) is incorrect because, during the introduction stage, per-unit costs of production are high and little competitionexists. Hence, prices are at their highest.

Answer (B) is incorrect because, during the growth stage, prices will be lower than during the introduction stage, but notas low as during the maturity stage. In the growth stage, costs are dropping and competitors are being added, but costs arenot at their minimum and competitors are not at their maximum.

Answer (C) is correct. During the maturity stage, competition is at its greatest and costs are at their lowest. Moreover,firms are engaged in competitive price-cutting measures, resulting in some of the lowest prices seen during a product’slife cycle.

Answer (D) is incorrect because, during the decline stage, price-cutting predominates as firms struggle to maintain salesvolume in the face of a permanent decrease in demand. However, late in the decline stage, there are few competitors, soprices can be raised. In addition, per-unit costs are on the rise because volume is declining, resulting in higher prices.

Gleim's CIA Test Prep: Part III: Business Analysis and Information TechnologyAnswer Explanations

(1312 questions)

Copyright 2008 Gleim Publications, Inc. Page 240Printed for Mamdouh Farag

Page 241: P.3 Answer Explanation

[787] Gleim #: 5.10.173 -- Source: CIA, adapted

Answer (A) is incorrect because production volume is low during the introduction stage. Although costs are also highduring this period, low volume reduces the opportunities for cost reductions.

Answer (B) is correct. During the growth stage, the opportunity for cost reductions is at its maximum because productionvolume is increasing at a high rate. Thus, fixed costs are being spread over more units of production, and the benefits ofthe learning curve are being realized.

Answer (C) is incorrect because production volume changes little during the maturity stage. The result is less opportunityfor cost reductions.

Answer (D) is incorrect because costs per unit typically rise during the decline stage as production volume declines.

[788] Gleim #: 5.10.174 -- Source: CIA, adapted

Answer (A) is incorrect because few competitors exist during the introduction stage, and quality is sometimes poor.

Answer (B) is incorrect because buyers are less concerned with price and quality during the growth stage than in thematurity stage.

Answer (C) is correct. The maturity stage is the ideal time for advertising lower prices and superior quality because thisis the period during a product’s life when competition is greatest. Due to the availability of many substitutes, a firm hasreasons to set itself apart. Because price and quality are both concerns of customers during the maturity stage, it is anideal time for the firm to differentiate its product by advertising low prices and higher quality.

Answer (D) is incorrect because few competitors exist during the decline stage. Moreover, prices may rise late in thedecline stage for the remaining firms as per-unit costs increase.

[789] Gleim #: 5.10.175 -- Source: CIA 597 III-20

Answer (A) is incorrect because this response illustrates two of the characteristics of organizational decline: increasedcentralization of decision making and lack of long-term planning. The exclusive emphasis on short-term results is likely tobe counterproductive.

Answer (B) is incorrect because another characteristic of organizational decline is nonprioritized downsizing. By itself,downsizing rarely turns a company around.

Answer (C) is incorrect because reducing staff disproportionately in control functions could have disastrous consequences.

Answer (D) is correct. Organizational decline has been found to have the following characteristics: greater centralization,lack of long-term planning, reduced innovation, scapegoating, resistance to change, high turnover of competent leaders,low morale, nonprioritized downsizing, and conflict. Reversing these characteristics is the key to reversing organizationaldecline, for example, by encouraging innovation in all aspects of the organization’s activities and by redeployingpersonnel.

Gleim's CIA Test Prep: Part III: Business Analysis and Information TechnologyAnswer Explanations

(1312 questions)

Copyright 2008 Gleim Publications, Inc. Page 241Printed for Mamdouh Farag

Page 242: P.3 Answer Explanation

[790] Gleim #: 5.10.176 -- Source: CIA 1196 IV-51

Answer (A) is incorrect because, during the formation stage, personal savings, trade credit, and government agencies arethe main sources of financing. Prior to demonstrating initial success, a firm is not likely to attract venture capital financingeasily.

Answer (B) is correct. At the rapid growth stage, if a company is reasonably profitable, it will experience financing needsin excess of funds available either internally or from trade credit or bank credit. Additional debt financing often results inan unreasonable amount of financial leverage at this stage, and public equity financing ordinarily is not yet available.Hence, a rapidly growing firm is most likely to seek and obtain venture capital financing.

Answer (C) is incorrect because, in the growth to maturity stage of development, the firm is able to access formal marketsfor debt and equity. It has a record of success and a better balance between cash inflows and outflows than in the rapidgrowth stage. Formal capital markets provide financing at lower cost than venture capitalists, so venture capital is notlikely to be sought at this stage.

Answer (D) is incorrect because the decline phase is characterized by more than adequate cash flows, relative to availableinvestment opportunities, so venture capital is not likely to be sought at this stage of development.

[791] Gleim #: 5.10.177 -- Source: CIA 592 IV-45

Answer (A) is incorrect because long-run solvency has improved.

Answer (B) is correct. The times-interest-earned ratio increased from 14.67 times in Year Five (44,000 ÷ 3,000) to 23.33times in Year Six (70,000 ÷ 3,000). This increase signifies that RST has more profit available to meet the interestpayments on its debt, and that long-run solvency has improved. Consequently, creditors will view RST’s obligations asless risky.

Answer (C) is incorrect because liquidity is a firm’s short-term solvency, that is, its ability to meet current liabilities withcurrent assets by converting assets to cash. It is not related to the times-interest-earned ratio, which compares profit beforeinterest and taxes with interest expense associated with long-term investments.

Answer (D) is incorrect because the times-interest-earned ratio is not a liquidity measure.

[792] Gleim #: 5.10.178 -- Source: CIA 596 IV-15

Answer (A) is incorrect because 2.04 times results from adding all inventory balances, opening and closing, to obtain thedenominator of the turnover ratio.

Answer (B) is incorrect because 3.54 times results from using the year-end inventory balances only.

Answer (C) is correct. Inventory turnover is the ratio of cost of goods sold to the average inventory balance. The totalaverage inventory is 490,000 [(90,000 BFG + 105,000 BRM + 220,000 BWIP + 260,000 EFG + 130,000 ERM + 175,000EWIP) ÷ 2]. Hence, total inventory turnover is 4.08 times (2,000,000 assumed CGS ÷ 490,000 average total inventory).

Answer (D) is incorrect because 4.82 times results from using the opening inventory balances only.

Gleim's CIA Test Prep: Part III: Business Analysis and Information TechnologyAnswer Explanations

(1312 questions)

Copyright 2008 Gleim Publications, Inc. Page 242Printed for Mamdouh Farag

Page 243: P.3 Answer Explanation

[793] Gleim #: 6.1.1 -- Source: CIA 1192 IV-16

Answer (A) is incorrect because line HEO represents total sales.

Answer (B) is incorrect because line CED represents total expenses.

Answer (C) is correct. A cost-volume-profit chart contains elements (lines, points, axes) that identify variable cost, fixedcost, the breakeven point, total revenue, profit, and volume in units. When the total sales revenue line rises above the totalexpense line, a company will have positive net income.

Answer (D) is incorrect because the loss area, i.e., the area of the chart where total expenses exceed sales, is representedby the area OED.

[794] Gleim #: 6.1.2 -- Source: CIA 1196 III-91

Answer (A) is incorrect because 388,800 units results from not subtracting the variable costs per unit from sales price.

Answer (B) is incorrect because 518,400 units does not reflect the sales commissions in the total variable costs.

Answer (C) is correct. The breakeven point is determined by dividing total fixed costs by the unit contribution margin.The total fixed costs are $9,331,200 ($5,598,720 manufacturing overhead + $3,732,480 general and administrative). Thecontribution margin is $16.00 ($40 sales price – $22 variable production cost – $2 commission). Thus, the breakevenpoint is 583,200 units ($9,331,200 ÷ $16).

Answer (D) is incorrect because 972,000 units includes taxes in the total variable costs, which understates the unitcontribution margin.

[795] Gleim #: 6.1.3 -- Source: CIA 1193 IV-11

Answer (A) is incorrect because 27.5% omits markdowns from the denominator.

Answer (B) is correct. The contribution margin equals revenues minus variable costs. The CMR equals the UCM dividedby the selling price. For example, if variable costs average $10 per unit, the average selling price is $16 (1.60 × $10).However, the 10% markdown implies that the actual average selling price is $14.40 (.90 × $16). The CMR is therefore30.6% [($14.40 – $10.00) ÷ $14.40].

Answer (C) is incorrect because 37.5% ignores markdowns.

Answer (D) is incorrect because 41.7% omits markdowns from the numerator.

Gleim's CIA Test Prep: Part III: Business Analysis and Information TechnologyAnswer Explanations

(1312 questions)

Copyright 2008 Gleim Publications, Inc. Page 243Printed for Mamdouh Farag

Page 244: P.3 Answer Explanation

[796] Gleim #: 6.1.4 -- Source: CIA 596 III-84

Answer (A) is incorrect because $20,160,000 does not adjust after-tax profit to pretax profit.

Answer (B) is correct. The desired after-tax profit is $1,188,000 (the past year’s amount). Given a 40% tax rate, thepretax equivalent is $1,980,000 [$1,188,000 ÷ (1.0 – .40)]. Pretax profit equals dollar sales (unit sales × $40), minus totalfixed costs, minus total variable costs (unit sales × unit variable cost). Hence, the contribution margin (sales – variablecosts) is equated with the sum of fixed costs and the targeted pretax profit. Unit sales (S) equal 540,000, and sales dollarsequal $21,600,000 (540,000 units × $40).

$40S – $9,900,000 – $18S = $1,980,000$22S = $11,880,000

S = 540,000 units

Answer (C) is incorrect because $23,400,000 results from adjusting after-tax profit by dividing by the tax rate rather thanone minus the tax rate.

Answer (D) is incorrect because $26,400,000 results from equating the sum of the desired pretax profit and total fixedcosts with total variable costs instead of the contribution margin.

[797] Gleim #: 6.1.5 -- Source: CIA 596 III-85

Answer (A) is incorrect because 337,500 units results from using the wrong sign for the pretax profit.

Answer (B) is incorrect because 346,875 units results from subtracting, not adding, the incremental marketing costs todetermine total fixed costs.

Answer (C) is incorrect because 425,000 units fails to adjust for the increase in direct materials costs.

Answer (D) is correct. Pretax profit (10% of dollar sales) equals dollar sales (unit sales × $50), minus total fixed costs(increased by $1,575,000 of marketing costs), minus total variable costs (increased by $3 per unit). Unit sales (S)therefore equal 478,125 units.

.10($50S) = $50S – $9,900,000 – $1,575,000 – ($18 + $3)S$5S = $29S – $11,475,000

$24S = $11,475,000S = 478,125 units

Gleim's CIA Test Prep: Part III: Business Analysis and Information TechnologyAnswer Explanations

(1312 questions)

Copyright 2008 Gleim Publications, Inc. Page 244Printed for Mamdouh Farag

Page 245: P.3 Answer Explanation

[798] Gleim #: 6.1.6 -- Source: CIA 593 IV-11

Answer (A) is correct. The breakeven point in sales dollars is equal to the sum of fixed cost plus any desired pretaxprofit, divided by contribution margin ratio [(sales – variable costs) ÷ sales]. Fixed cost was 50,000 (500,000 sales –300,000 VC – 150,000 pretax profit). Given the increase in sales of 10% and decrease in fixed costs of 20%, thebreakeven point in sales is 88,000.

Fixed costs (50,000 × .8) +Desired pretax profit 0 40,000

=Sales [(500,000 × 1.1) – Variable costs 250,000 ÷ 550,000

300,000] ÷ Sales (500,000 × 1.1)

Answer (B) is incorrect because 100,000 ignores the 10% sales price increase.

Answer (C) is incorrect because 110,000 ignores the 20% decrease in fixed costs.

Answer (D) is incorrect because 125,000 ignores the changes in sales price and fixed costs.

[799] Gleim #: 6.1.7 -- Source: CIA 595 III-85

Answer (A) is incorrect because an increase of 8,000 assumes unit variable selling cost is 2.00.

Answer (B) is correct. If the company accepts the special order, its revenue will increase by 76,000 (7.60 × 10,000 units).However, its incremental cost will include only the variable costs because fixed manufacturing and selling costs will beunchanged. The increase in cost from accepting the special order is 60,000 [(3.00 + 1.00 + 0.80 + 1.20) × 10,000 units].Thus, acceptance of the special order will increase profits by 16,000 (76,000 – 60,000).

Answer (C) is incorrect because profit will increase.

Answer (D) is incorrect because profit will increase.

[800] Gleim #: 6.1.8 -- Source: CIA 595 III-86

Answer (A) is incorrect because profit will decrease by 15,000.

Answer (B) is correct. Because total fixed costs are unaffected, the change in profit is the change in the contributionmargin. The contribution margin at the current selling price is 240,000 [(10 – 3 – 1 – 0.80 – 2) × 75,000 units]. Thecontribution margin at the 9.30 selling price is 225,000 [(9.30 – 3 – 1 – 0.80 – 2) × 90,000 units]. Hence, profit will bereduced by 15,000 (240,000 – 225,000) if the selling price is lowered to 9.30.

Answer (C) is incorrect because profit will decrease by 15,000.

Answer (D) is incorrect because profit will decrease by 15,000.

Gleim's CIA Test Prep: Part III: Business Analysis and Information TechnologyAnswer Explanations

(1312 questions)

Copyright 2008 Gleim Publications, Inc. Page 245Printed for Mamdouh Farag

Page 246: P.3 Answer Explanation

[801] Gleim #: 6.1.9 -- Source: CIA 596 IV-43

Answer (A) is incorrect because 18,000 units results from dividing fixed costs by unit price.

Answer (B) is incorrect because 30,000 units equals fixed costs divided by unit variable cost.

Answer (C) is correct. The breakeven volume in sales units equals fixed costs divided by the unit contribution margin(unit price – unit variable cost). Hence, the breakeven point is 45,000 units [450,000 ÷ (25 – 15)].

Answer (D) is incorrect because 60,000 units is calculated by adding 150,000 to the fixed costs.

[802] Gleim #: 6.1.10 -- Source: CIA R98 IV-57

Answer (A) is incorrect because 2,970,000 results from subtracting income tax.

Answer (B) is correct. The total contribution margin equals sales minus variable costs, or 4,950,000 (9,000,000 –1,800,000 – 720,000 – 1,080,000 – 450,000).

Answer (C) is incorrect because 5,400,000 omits variable selling costs from the calculation.

Answer (D) is incorrect because 6,030,000 omits variable overhead from the calculation.

[803] Gleim #: 6.1.11 -- Source: CIA R98 IV-58

Answer (A) is incorrect because 3,429,360 equals pretax operating income (revenue minus total costs).

Answer (B) is incorrect because 4,232,160 equals the gross margin minus fixed administrative expenses.

Answer (C) is incorrect because 4,350,000 equals the gross margin minus the variable selling expenses.

Answer (D) is correct. The gross margin equals revenue minus cost of goods sold (beginning finished goods inventory +cost of goods manufactured – ending finished goods inventory). Cost of goods manufactured equals all manufacturingadjusted for the change in work-in-process. Consequently, the gross margin was 4,800,000 (9,000,000 – 1,800,000 DM –720,000 DL – 1,080,000 VOH – 600,000 FOH), given no beginning or ending finished goods or work-in-processinventories.

[804] Gleim #: 6.1.12 -- Source: CIA R98 IV-59

Answer (A) is incorrect because 36,495 units includes fixed manufacturing overhead and excludes variable sellingexpenses from the UCM calculation. It also excludes fixed selling expenses from the total fixed costs.

Answer (B) is incorrect because 42,240 units excludes variable selling expenses from the UCM calculation.

Answer (C) is correct. The breakeven point in unit sales equals total fixed costs divided by the unit contribution margin(UCM). Total fixed costs are 1,520,640 (600,000 + 567,840 + 352,800), and the UCM is 33 [4,950,000 contributionmargin (sales minus variable costs) ÷ 150,000 units sold]. Hence, the breakeven point in unit sales is 46,080 units(1,520,640 ÷ 33).

Answer (D) is incorrect because 56,320 units results from dividing total fixed costs by the total unit variable cost.

Gleim's CIA Test Prep: Part III: Business Analysis and Information TechnologyAnswer Explanations

(1312 questions)

Copyright 2008 Gleim Publications, Inc. Page 246Printed for Mamdouh Farag

Page 247: P.3 Answer Explanation

[805] Gleim #: 6.1.13 -- Source: CIA R98 IV-60

Answer (A) is incorrect because 6,938,031 includes net income instead of pretax operating income in the calculation.

Answer (B) is incorrect because 8,736,000 omits the unit variable selling expense from the calculation of the total unitvariable cost.

Answer (C) is correct. To earn the same net income in Year 2 as in Year 1, the contribution margin must equal the sumof the Year 1 pretax operating income and the Year 2 fixed costs. The pretax operating income is calculated as 3,429,360(9,000,000 – 4,050,000 variable costs – 1,520,640 fixed costs). Fixed costs in Year 2 will be the same as in Year 1 exceptfor an increase of 29,520 in fixed selling expenses, a total of 1,550,160 (1,520,640 as previously calculated + 29,520).The desired Year 2 sales volume must equal the Year 2 contribution margin of 4,979,520 (3,429,360 desired pretaxoperating income + 1,550,160 fixed costs in Year 2) divided by the contribution margin ratio (CMR). The CMR may bedetermined from the unit price (constant at 9,000,000 ÷ 150,000 units = 60 per unit) and unit variable cost data. Unitdirect materials cost in Year 2 will increase by 10% to 13.20 [1.1 × (1,800,000 ÷ 150,000 units)]. Unit direct labor cost inYear 2 is given as 5.40. Unit variable overhead cost is constant at 7.20 (1,080,000 ÷ 150,000 units). Unit variable sellingexpense is constant at 3.00 (450,000 ÷ 150,000 units). Thus, total unit variable cost is 28.80 (13.20 + 5.40 + 7.20 + 3.00),and the UCM is 31.20 (60 unit selling price – 28.80). The CMR is therefore .52 (31.20 ÷ 60.00), and the desired Year 2sales volume is 9,576,000 (4,979,520 CM ÷ .52).

Answer (D) is incorrect because 10,374,000 uses the variable cost ratio instead of the CMR.

[806] Gleim #: 6.1.14 -- Source: CIA R98 IV-61

Answer (A) is incorrect because 61.80 equals the original unit selling price plus the increase in the unit variable costs.

Answer (B) is correct. The CMR equals unit contribution margin (UCM) divided by unit selling price. The UCM equalsthe unit selling price minus unit variable cost. The original unit selling price was 60 (9,000,000 revenue ÷ 150,000 units),and the original unit variable cost was 27 [(1,800,000 + 720,000 + 1,080,000 + 450,000) ÷ 150,000 units]. The originalCMR was therefore .55 [(60 – 27) ÷ 60]. The new unit variable cost was calculated as 28.80. Accordingly, the unit sellingprice (X) is 64.

(X – 28.80) ÷ X = .55X – 28.80 = .55X

.45X = 28.80X = 28.8 ÷ .45X = 64

Answer (C) is incorrect because 64.50 omits the unit variable selling costs from all calculations.

Answer (D) is incorrect because 72.00 omits the unit variable selling costs from the calculation of the original unitvariable cost.

Gleim's CIA Test Prep: Part III: Business Analysis and Information TechnologyAnswer Explanations

(1312 questions)

Copyright 2008 Gleim Publications, Inc. Page 247Printed for Mamdouh Farag

Page 248: P.3 Answer Explanation

[807] Gleim #: 6.1.15 -- Source: CIA 1189 IV-15

Answer (A) is incorrect because 5.00 covers the purchase price (4.95) but not the variable selling (2.00) andadministrative (1.00) expenses.

Answer (B) is incorrect because 7.00 covers the purchase price and the variable selling expenses (4.95 + 2.00) but not thevariable administrative expenses (1.00).

Answer (C) is correct. In the short run, the fixed costs are sunk costs. They will be incurred whether or not the item isproduced. Accordingly, the retailer will suffer no additional loss if the price charged covers the incremental costs, whichare the variable costs in this case. The lowest feasible price is the one that provides a zero (but not a negative)contribution margin (revenue – variable costs). Thus, the lowest short-run price is the unit variable cost (4.95 + 2.00 +1.00 = 7.95). The lowest price provided in the answer choices that will recover the unit variable cost is 8.00.

Answer (D) is incorrect because 9.50 covers the fixed costs of 1.50 (7,500 ÷ 5,000) and the variable costs.

[808] Gleim #: 6.1.16 -- Source: CIA 1194 IV-65

Answer (A) is incorrect because a tax rate increase would not affect the breakeven point. At the breakeven point, profitsequal zero.

Answer (B) is incorrect because a tax rate increase would not affect the breakeven point. At the breakeven point, profitsequal zero.

Answer (C) is correct. The firm owes no tax if it does not earn a profit. At the breakeven point, there is no profit. Thus, atax increase would have no effect on a firm’s breakeven point, which is calculated by dividing fixed costs by thecontribution margin ratio. The contribution margin equals sales revenues minus total variable costs.

Answer (D) is incorrect because the contribution margin per unit will be lower if the corporate tax rate is higher, for allcompanies operating above the breakeven point.

[809] Gleim #: 6.1.17 -- Source: CIA, adapted

Answer (A) is incorrect because insurance on the corporate headquarters building is not a cost of production and istherefore a period cost.

Answer (B) is correct. Product costs such as material, labor, and overhead attach to the product and are carried in futureperiods if the revenue from the product is recognized in subsequent periods. Period costs are expensed immediatelybecause no direct relationship between cost and revenue exists.

Answer (C) is incorrect because depreciation on salespersons’ vehicles is not a cost of production and is therefore a periodcost.

Answer (D) is incorrect because the salary of a sales manager is not a cost of production and is therefore a period cost.

Gleim's CIA Test Prep: Part III: Business Analysis and Information TechnologyAnswer Explanations

(1312 questions)

Copyright 2008 Gleim Publications, Inc. Page 248Printed for Mamdouh Farag

Page 249: P.3 Answer Explanation

[810] Gleim #: 6.2.18 -- Source: CIA 595 III-87

Answer (A) is incorrect because $750,000 equals absorption costing profit minus ending inventory (15,000 units × $10).

Answer (B) is correct. Under absorption costing, product costs include fixed and variable manufacturing costs. The unitproduct cost under absorption costing is $10 [($600,000 + $400,000) ÷ 100,000 units produced]. All nonmanufacturingcosts are expensed in the period incurred. Thus, operating profit is $900,000.

Revenue (85,000 units × $30) $2,550,000Cost of goods sold (85,000 units × $10) (850,000)Nonmanufacturing costs

($500,000 + $300,000) (800,000)Operating profit $ 900,000

Answer (C) is incorrect because $975,000 treats the variable nonmanufacturing costs as manufacturing costs.

Answer (D) is incorrect because $1,020,000 assumes that all costs are manufacturing costs.

[811] Gleim #: 6.2.19 -- Source: CIA 595 III-88

Answer (A) is incorrect because $750,000 equals variable costing profit minus ending inventory (15,000 units × $6).

Answer (B) is correct. Under variable costing, the product cost includes only variable manufacturing costs. All fixed costsare expensed in the period incurred. Unit product cost under variable costing is $6 ($600,000 ÷ 100,000 units produced).

Revenue (85,000 units × $30) $2,550,000Variable cost of goods sold

(85,000 units × $6) (510,000)Variable nonmanufacturing costs (500,000)Contribution margin $1,540,000Fixed costs (700,000)Operating profit $ 840,000

Answer (C) is incorrect because $915,000 treats all variable costs as manufacturing costs.

Answer (D) is incorrect because $975,000 treats all variable costs and fixed manufacturing costs as product costs.

[812] Gleim #: 6.2.20 -- Source: CIA 1193 IV-20

Answer (A) is incorrect because $18,000 is the result of deducting the central corporate expenses.

Answer (B) is incorrect because $20,000 is the result of excluding other revenue.

Answer (C) is correct. Division A’s total contribution to corporate profits includes everything except the central corporateexpense allocation. Thus, the total contribution is $30,000 ($150,000 sales + $10,000 other revenue – $30,000 directmaterials – $20,000 direct labor – $5,000 variable overhead – $25,000 fixed overhead – $15,000 variable S&A expense –$35,000 fixed S&A expense).

Answer (D) is incorrect because $90,000 is the result of failing to deduct the fixed costs.

Gleim's CIA Test Prep: Part III: Business Analysis and Information TechnologyAnswer Explanations

(1312 questions)

Copyright 2008 Gleim Publications, Inc. Page 249Printed for Mamdouh Farag

Page 250: P.3 Answer Explanation

[813] Gleim #: 6.2.21 -- Source: CIA 1193 IV-21

Answer (A) is incorrect because $150,000 is the result of subtracting fixed costs.

Answer (B) is incorrect because $205,000 is the result of subtracting fixed S&A costs.

Answer (C) is incorrect because $235,000 is the result of subtracting fixed S&A costs but not variable S&A costs.

Answer (D) is correct. The contribution margin equals revenue minus variable costs. Thus, Division B’s contributionmargin is $265,000 ($400,000 sales + $15,000 other revenue – $65,000 direct materials – $40,000 direct labor – $15,000variable overhead – $30,000 variable S&A expense).

[814] Gleim #: 6.2.22 -- Source: CIA 1190 IV-12

Answer (A) is correct. Absorption-costing operating profit will exceed variable-costing operating income becauseproduction exceeds sales, resulting in a deferral of fixed manufacturing overhead in the inventory calculated using theabsorption method. The difference of $200,000 is equal to the fixed manufacturing overhead per unit ($2,200,000 ÷275,000 = $8.00) times the difference between production and sales (275,000 – 250,000 = 25,000, which is the inventorychange in units).

Answer (B) is incorrect because units produced, not units sold, should be used as the denominator to calculate the fixedmanufacturing cost per unit.

Answer (C) is incorrect because fixed selling and administrative costs are not properly inventoriable under absorptioncosting.

Answer (D) is incorrect because variable selling and administrative costs are period costs under both variable- andabsorption-cost systems in the determination of operating profit.

[815] Gleim #: 6.2.23 -- Source: CIA 593 IV-5

Answer (A) is correct. Beginning inventory is 40% complete. Hence, direct materials have already been added. Endinginventory has not reached the 25% stage of completion, so direct materials have not yet been added to these units. Thus,the equivalent units for direct materials calculated on a FIFO basis are equal to the units started and completed in thecurrent period (85,000 units completed – 15,000 units in BWIP = 70,000 units started and completed).

Answer (B) is incorrect because 80,000 total units were transferred in from Department 1.

Answer (C) is incorrect because 85,000 equals the equivalent units for direct materials calculated on a weighted-averagebasis.

Answer (D) is incorrect because 95,000 equals the total units to be account for.

Gleim's CIA Test Prep: Part III: Business Analysis and Information TechnologyAnswer Explanations

(1312 questions)

Copyright 2008 Gleim Publications, Inc. Page 250Printed for Mamdouh Farag

Page 251: P.3 Answer Explanation

[816] Gleim #: 6.2.24 -- Source: CIA R98 IV-66

Answer (A) is incorrect because insurance on the corporate headquarters building is a general and administrative expense.

Answer (B) is correct. Product (inventoriable) costs are incurred to produce units of output and are deferred to futureperiods to the extent output is not sold (kept on hand for sale in future periods). They are expensed in the period theproduct is sold. Hence, product costs are those that can be associated with specific revenues. Examples are directmaterials, direct labor, and factory (not general and administrative) overhead. Property taxes on a factory are included infactory (manufacturing) overhead.

Answer (C) is incorrect because depreciation on a salesperson’s automobile is a general and administrative expense.

Answer (D) is incorrect because the salary of a sales manager is a general and administrative expense.

[817] Gleim #: 6.3.25 -- Source: CIA 595 III-94

Answer (A) is incorrect because $18,000 includes only $8,000 for overhead (based on 100% of direct labor).

Answer (B) is incorrect because $18,400 includes only $8,400 for overhead (based on 100% of direct labor and 20% ofraw materials).

Answer (C) is incorrect because $24,780 is the overhead applied, not total cost.

Answer (D) is correct. The total cost is $34,780. It consists of direct labor ($8,000), raw materials ($2,000), and overhead($24,780). The overhead applied component of total costs is calculated as follows:

$8,000 of direct labor × 100% = $ 8,000$2,000 of raw materials × 20% = 400140 machine hours × $117 = 16,380

$24,780

Accordingly, no answer given is correct.

[818] Gleim #: 6.3.26 -- Source: CIA 1184 IV-21

Answer (A) is incorrect because $3.36 per direct labor hour is based on use of an activity level of 110,000 direct laborhours to determine the fixed overhead rate.

Answer (B) is correct. The predetermined overhead rate is calculated by dividing the total fixed overhead by the activitylevel to arrive at a unit fixed overhead cost that is added to the unit variable overhead cost. The unit variable overheadrate is the same at each activity level. Thus, the predetermined overhead rate is $3.50 [($150,000 FOH ÷ 100,000 hrs.) +($220,000 VOH ÷ 110,000 hrs.)].

Answer (C) is incorrect because $3.70 is the result of assuming that $220,000 of variable overhead will be incurred for100,000 (not 110,000) direct labor hours.

Answer (D) is incorrect because $3.88 (rounded) results from using an activity level of 80,000 direct labor hours todetermine the fixed overhead rate.

Gleim's CIA Test Prep: Part III: Business Analysis and Information TechnologyAnswer Explanations

(1312 questions)

Copyright 2008 Gleim Publications, Inc. Page 251Printed for Mamdouh Farag

Page 252: P.3 Answer Explanation

[819] Gleim #: 6.3.27 -- Source: Publisher

Answer (A) is incorrect because 10,250 would have been produced if overhead had been overapplied by $1,500[($39,500 + $1,500) ÷ $4].

Answer (B) is incorrect because 10,000 is the result of dividing budgeted, not applied, overhead by the application rate.

Answer (C) is incorrect because 9,875 units would have been produced if $39,500 had been the amount of appliedoverhead.

Answer (D) is correct. Given actual overhead of $39,500 and underapplied overhead of $1,500, overhead applied was$38,000 ($39,500 – $1,500). Overhead is applied at the rate of $4 per unit ($40,000 budgeted overhead ÷ 10,000budgeted units). Accordingly, 9,500 units were produced ($38,000 applied overhead ÷ $4 per unit application rate).

[820] Gleim #: 6.4.28 -- Source: CMA 695 3-1

Answer (A) is incorrect because 97,600 units omits the 6,400 EUP added to beginning work-in-process.

Answer (B) is correct. Under FIFO, EUP are based solely on work performed during the current period. The EUP equalsthe sum of the work done on the beginning work-in-process inventory, units started and completed in the current period,and the ending work-in-process inventory. Given that beginning work-in-process was 60% complete as to materials, thecurrent period is charged for 6,400 EUP (16,000 units × 40%). Because 92,000 units were completed during the period,76,000 (92,000 – 16,000 in BWIP) must have been started and completed during the period. They represent 76,000 EUP.Finally, the EUP for ending work-in-process equal 21,600 (24,000 units × 90%). Thus, total EUP for May are 104,000(6,400 + 76,000 + 21,600).

Answer (C) is incorrect because 107,200 units assumes beginning work-in-process was 40% complete.

Answer (D) is incorrect because 108,000 units equals the sum of the physical units in beginning work-in-process and thephysical units completed.

[821] Gleim #: 6.4.29 -- Source: CMA 695 3-2

Answer (A) is incorrect because 85,600 units omits the work done on beginning work-in-process.

Answer (B) is incorrect because 88,800 units omits the work done on ending work-in-process.

Answer (C) is incorrect because 95,200 units assumes the beginning work-in-process was 40% complete as to conversioncosts.

Answer (D) is correct. The beginning inventory was 20% complete as to conversion costs. Hence, 12,800 EUP (16,000units × 80%) were required for completion. EUP for units started and completed equaled 76,000 [100% × (92,000completed units – 16,000 units in BWIP)]. The work done on ending work-in-process totaled 9,600 EUP (24,000 units ×40%). Thus, total EUP for May are 98,400 (12,800 + 76,000 + 9,600).

Gleim's CIA Test Prep: Part III: Business Analysis and Information TechnologyAnswer Explanations

(1312 questions)

Copyright 2008 Gleim Publications, Inc. Page 252Printed for Mamdouh Farag

Page 253: P.3 Answer Explanation

[822] Gleim #: 6.4.30 -- Source: CMA 695 3-3

Answer (A) is incorrect because $4.12 is based on EUP calculated under the weighted-average method.

Answer (B) is correct. Under the FIFO method, EUP for materials equal 104,000 [(16,000 units in BWIP × 40%) +(76,000 units started and completed × 100%) + (24,000 units in EWIP × 90%)]. Consequently, the equivalent unit cost ofmaterials is $4.50 ($468,000 total materials cost in May ÷ 104,000 EUP).

Answer (C) is incorrect because $4.60 is the weighted-average cost per equivalent unit.

Answer (D) is incorrect because $4.80 omits the 6,400 EUP added to beginning work-in-process.

[823] Gleim #: 6.4.31 -- Source: CMA 695 3-4

Answer (A) is incorrect because $5.65 is based on EUP calculated under the weighted-average method.

Answer (B) is correct. Under the FIFO method, EUP for conversion costs equal 98,400 [(16,000 units in BWIP × 80%) +(76,000 units started and completed × 100%) + (24,000 units in EWIP × 40%)]. Conversion costs incurred during thecurrent period equal $574,040 ($182,880 DL + $391,160 FOH). Hence, the equivalent unit cost for conversion costs is$5.83 ($574,040 ÷ 98,400).

Answer (C) is incorrect because $6.00 is the cost per equivalent unit calculated under the weighted-average method.

Answer (D) is incorrect because $6.20 results from combining conversion costs for May with those in beginning work-in-process and dividing by 98,400 EUP.

[824] Gleim #: 6.4.32 -- Source: CMA 695 3-5

Answer (A) is correct. The FIFO costs per equivalent unit for materials and conversion costs are $4.50 and $5.83,respectively. EUP for materials in ending work-in-process equal 21,600 (24,000 × 90%). Thus, total FIFO materials costis $97,200 (21,600 EUP × $4.50). EUP for conversion costs in ending work-in-process equal 9,600 (24,000 × 40%). Totalconversion costs are therefore $55,968 (9,600 EUP × $5.83). Consequently, total work-in-process costs are $153,168($97,200 + $55,968).

Answer (B) is incorrect because $154,800 is based on a FIFO calculation for materials and a weighted-average calculationfor conversion costs.

Answer (C) is incorrect because $155,328 is based on a weighted-average calculation for materials and a FIFO calculationfor conversion costs.

Answer (D) is incorrect because $156,960 is the weighted-average cost of ending work-in-process.

[825] Gleim #: 6.4.33 -- Source: Publisher

Answer (A) is incorrect because job-order costing is used for unique products.

Answer (B) is correct. Operation costing is used when different groups of products are subject to some but not all of thesame processing steps (operations). Every unit passing through a given operation is processed in the same way, andconversion costs are accumulated by operation. Direct materials costs are accumulated by batch.

Answer (C) is incorrect because process costing is used for homogeneous products.

Answer (D) is incorrect because food and beverage products may be unique or homogeneous, and produced in jobs, lots,or batches.

Gleim's CIA Test Prep: Part III: Business Analysis and Information TechnologyAnswer Explanations

(1312 questions)

Copyright 2008 Gleim Publications, Inc. Page 253Printed for Mamdouh Farag

Page 254: P.3 Answer Explanation

[826] Gleim #: 6.4.34 -- Source: CIA 593 IV-4

Answer (A) is correct. Job-order costing is used by organizations whose products or services are readily identified byindividual units or batches. The advertising agency accumulates its costs by client. Job-order costing is the mostappropriate system for this type of nonmanufacturing firm.

Answer (B) is incorrect because operation costing would most likely be employed by a manufacturer producing goods thathave common characteristics plus some individual characteristics. This would not be an appropriate system for anadvertising agency with such a diverse client base.

Answer (C) is incorrect because relevant costing refers to expected future costs that are considered in decision making.

Answer (D) is incorrect because process costing is employed when a company mass produces a homogeneous product in acontinuous fashion through a series of production steps.

[827] Gleim #: 6.4.35 -- Source: CIA 1196 III-89

Answer (A) is correct. Absorption costing inventories all direct manufacturing costs and both variable and fixedmanufacturing overhead (indirect) costs.

Answer (B) is incorrect because activity-based costing develops cost pools for activities and then allocates those costs tocost objects based on the drivers of the activities.

Answer (C) is incorrect because a backflush costing system applies costs based on output.

Answer (D) is incorrect because variable costing excludes fixed manufacturing overhead costs from inventoriable costsand treats them as period costs.

[828] Gleim #: 6.4.36 -- Source: CIA 1195 III-80

Answer (A) is incorrect because 67,500 equivalent units of direct materials were added in accordance with the FIFOmethod.

Answer (B) is incorrect because 80,000 units were transferred.

Answer (C) is correct. The weighted-average method does not distinguish between work done currently and in the priorperiod. Given that materials are added when units are 80% complete, that ending work-in-process is 60% complete, andthat goods are inspected when they are 100% complete, and assuming that equivalent units are calculated for normalspoilage, the total weighted-average equivalent units for direct materials equals 81,500 (80,000 units transferred + 1,500normally spoiled units).

Answer (D) is incorrect because 90,000 units equals the actual physical flow for the month.

Gleim's CIA Test Prep: Part III: Business Analysis and Information TechnologyAnswer Explanations

(1312 questions)

Copyright 2008 Gleim Publications, Inc. Page 254Printed for Mamdouh Farag

Page 255: P.3 Answer Explanation

[829] Gleim #: 6.4.37 -- Source: CIA 1195 III-81

Answer (A) is incorrect because 72,500 ignores spoilage.

Answer (B) is correct. The FIFO method distinguishes between work done in the prior period and work done currently.The total FIFO equivalent units equals the work done currently on beginning work-in-process, plus the work done onending work-in-process, plus all units started and completed currently. Hence, total FIFO equivalent units equals 74,000{(10% × 14,000 units in BWIP) + (60% × 8,500 units in EWIP) + [100% × (81,500 spoiled and transferred – 14,000 unitsin BWIP)]}.

Answer (C) is incorrect because 85,200 includes 90% of the beginning work-in-process.

Answer (D) is incorrect because 86,600 calculates the equivalent units for conversion costs according to the weighted-average method.

[830] Gleim #: 6.4.38 -- Source: CIA 594 III-70

Answer (A) is correct. Job order costing is appropriate when producing products with individual characteristics and/orwhen identifiable groupings are possible. Process costing should be used to assign costs to similar products that are massproduced on a continuous basis. Operations costing is a hybrid of job order and process costing systems. It is used bycompanies that manufacture goods that undergo some similar and some dissimilar processes. Thus, job order costingwould be appropriate for auto repair, operations costing for clothing manufacturing, and process costing for oil refining.

Answer (B) is incorrect because custom printing would not use process costing.

Answer (C) is incorrect because paint manufacturing would not use operations costing.

Answer (D) is incorrect because motion picture production would not use process costing.

[831] Gleim #: 6.4.39 -- Source: CIA 595 III-93

Answer (A) is incorrect because an EOQ system is an inventory control tool, not a cost system.

Answer (B) is incorrect because a job-order system is used when products are differentiated from one customer to thenext. Each job (customer) is a separate cost center.

Answer (C) is correct. A process costing system is used when a company mass produces a standardized product on acontinuous basis. The production department becomes the cost center.

Answer (D) is incorrect because this company is a manufacturer, not a retailer. The retail method converts endinginventory stated at retail to cost.

[832] Gleim #: 6.4.40 -- Source: CIA, adapted

Answer (A) is correct. Cost accumulation is performed by accounting systems that organize data by an appropriatecatalog. Actual costs, rather than predicted costs, are accumulated.

Answer (B) is incorrect because computing depreciation expense would not organize date into categories.

Answer (C) is incorrect because producing financial statements would not organize data into categories.

Answer (D) is incorrect because forecasting material shortages would not organize data into categories.

Gleim's CIA Test Prep: Part III: Business Analysis and Information TechnologyAnswer Explanations

(1312 questions)

Copyright 2008 Gleim Publications, Inc. Page 255Printed for Mamdouh Farag

Page 256: P.3 Answer Explanation

[833] Gleim #: 6.4.41 -- Source: CIA, adapted

Answer (A) is incorrect because process costing does not allocate costs per packing line. (150,000 + 90,000 + 30,000 +15,000 + 3,000 + 66,000 = 354,000 ÷ 600,000 = .59 ÷ 3 = 0.197).

Answer (B) is incorrect because process costing includes all costs. (150,000 + 15,000 = 165,000 ÷ 600,000 = 0.275).

Answer (C) is incorrect because process costing includes all costs. (90,000 + 30,000 + 3,000 + 66,000 = 189,000 ÷600,000 = 0.315).

Answer (D) is correct. Process costing is used to assign costs to products or services. It is applicable to relativelyhomogeneous items that are mass produced on a continuous basis. Process costing is the average cost per unit produced, ortotal cost divided by the number of units. 150,000 + 90,000 + 30,000 + 15,000 + 3,000 + 66,000 = 354,000 ÷ 600,000 =0.59.

[834] Gleim #: 6.5.42 -- Source: Publisher

Answer (A) is correct. As calculated below, the budgeted standard unit cost of a TV board is $120. This amount includes$6 of variable overhead [1.5 DLH × ($1,120,000 total VOH ÷ 280,000 DLH)]. Given a unit price of $150, the unitcontribution margin is therefore $30. Total budgeted contribution is $1,950,000 ($30 UCM × 65,000 budgeted units).

Direct materials $ 80DM handling ($80 × 10%) 8Direct labor ($14 × 1.5 hr.) 21Machine time ($10 × .5 hr.) 5Variable overhead ($4 × 1.5 hr.) 6Budgeted unit cost $120

Answer (B) is incorrect because $2,275,000 excludes the cost of machine time (65,000 units × $5 = $325,000).

Answer (C) is incorrect because $2,340,000 excludes the variable overhead (65,000 units × $6 = $390,000).

Answer (D) is incorrect because $2,470,000 excludes the direct materials handling cost (65,000 units × $8 = $520,000).

Gleim's CIA Test Prep: Part III: Business Analysis and Information TechnologyAnswer Explanations

(1312 questions)

Copyright 2008 Gleim Publications, Inc. Page 256Printed for Mamdouh Farag

Page 257: P.3 Answer Explanation

[835] Gleim #: 6.5.43 -- Source: Publisher

Answer (A) is incorrect because $1,594,000 is the CM for the PC board based on an ABC calculation.

Answer (B) is incorrect because $1,950,000 is the CM for the TV board based on a standard-cost calculation.

Answer (C) is incorrect because $2,037,100 erroneously includes $8 per board for materials handling (65,000 units × $8 =$520,000).

Answer (D) is correct. As calculated below, the budgeted activity-based unit cost of a TV board is $110.66. Given a unitprice of $150, the unit contribution margin is $39.34. Total budgeted contribution is $2,557,100 ($39.34 UCM × 65,000budgeted units).

Direct materials $ 80.00Procurement [25 parts × ($400,000 ÷ 4,000,000 parts)] 2.50Scheduling ($220,000 ÷ 110,000 boards) 2.00Packaging and shipping ($440,000 ÷ 110,000 boards) 4.00Setups [($446,000 ÷ 278,750 setups) × 2 setups] 3.20Waste disposal [($48,000 ÷ 16,000 lb.) × .02] .06Quality control ($560,000 ÷ 160,000 inspections) 3.50General supplies ($66,000 ÷ 110,000 boards) .60Machine insertion [24 parts × ($1,200,000 ÷ 3,000,000 parts)] 9.60Manual insertion ($4,000,000 ÷ 1,000,000 parts) 4.00Soldering ($132,000 ÷ 110,000 boards) 1.20Budgeted ABC unit cost $110.66

[836] Gleim #: 6.5.44 -- Source: CIA 1195 III-93

Answer (A) is incorrect because $5.39 assumes that 80 machine hours are required for the total production of20,000 units.

Answer (B) is incorrect because $5.44 is based on the machining overhead rate ($18).

Answer (C) is correct. Given that manufacturing overhead is applied on the basis of machine hours, the overhead rate is$60 per hour ($1,800,000 ÷ 30,000) or $.96 per unit [(80 machine hours per batch × $60) ÷ 5,000 units per batch].Accordingly, the unit full cost is $6.11 ($5.15 unit price cost + $.96).

Answer (D) is incorrect because $6.95 is based on the direct labor hour manufacturing overhead rate.

[837] Gleim #: 6.5.45 -- Source: CIA 1195 III-94

Answer (A) is incorrect because $6.00 assumes one setup per batch and 80 total machine hours.

Answer (B) is incorrect because $6.08 assumes that only 80 machine hours were used.

Answer (C) is incorrect because $6.21 assumes one setup per batch.

Answer (D) is correct. Materials handling cost per part is $.12 ($720,000 ÷ 6,000,000), cost per setup is $420 ($315,000÷ 750), machining cost per hour is $18 ($540,000 ÷ 30,000), and quality cost per batch is $450 ($225,000 ÷ 500). Hence,total manufacturing overhead applied is $22,920 [(5 parts per unit) × 20,000 units × $.12) + (4 batches × 2 setups perbatch × $420) + (4 batches × 80 machine hours per batch × $18) + (4 batches × $450)]. The total unit cost is $6.296[$5.15 prime cost + ($22,920 ÷ 20,000 units) overhead].

Gleim's CIA Test Prep: Part III: Business Analysis and Information TechnologyAnswer Explanations

(1312 questions)

Copyright 2008 Gleim Publications, Inc. Page 257Printed for Mamdouh Farag

Page 258: P.3 Answer Explanation

[838] Gleim #: 6.5.46 -- Source: CIA 1194 III-44

Answer (A) is correct. Job-order costing systems are designed to accumulate costs for tasks or projects that are uniqueand nonrepetitive. Service organizations are interested in identifying the costs applicable to each customer and/or eachservice call.

Answer (B) is incorrect because activity-based costing systems focus on generating more accurate cost information byidentifying the activities that are cost drivers.

Answer (C) is incorrect because process costing systems are designed for homogeneous products that are mass produced incontinuous production runs.

Answer (D) is incorrect because operations costing systems are designed for batches of homogeneous products; operationscosting is a hybrid of job-order and process costing.

[839] Gleim #: 6.5.47 -- Source: CIA, adapted

Answer (A) is incorrect because this allocation basis is related to batch costs and not to individual unit costs.

Answer (B) is correct. An essential element of activity-based costing (ABC) is driver analysis, which identifies the cause-and-effect relationship between an activity and its consumption of resources and for an activity and the demands made onit by a cost object. There is a direct causal relationship between the number of components in a finished product and theamount of material handling cost incurred

Answer (C) is incorrect because this allocation basis is the traditional basis for allocating overhead costs to the unitsproduced when the production process is labor-intensive.

Answer (D) is incorrect because this is not an allocation basis but rather the result of the allocation process whendetermining product costs.

[840] Gleim #: 6.6.48 -- Source: CIA 596 III-82

Answer (A) is incorrect because $140,000 is 40% of other traceable costs.

Answer (B) is incorrect because $160,000 assumes an allocation base of 55,000 square feet.

Answer (C) is incorrect because $176,000 assumes an allocation base of 50,000 square feet.

Answer (D) is correct. The direct method does not allocate service costs to other service departments. Hence, theallocation base is the square footage in the two production departments. Fabricating’s share is 40% (16,000 ÷ 40,000) ofthe total cost incurred by Building Operations, or $220,000 ($550,000 × 40%).

Gleim's CIA Test Prep: Part III: Business Analysis and Information TechnologyAnswer Explanations

(1312 questions)

Copyright 2008 Gleim Publications, Inc. Page 258Printed for Mamdouh Farag

Page 259: P.3 Answer Explanation

[841] Gleim #: 6.6.49 -- Source: CIA 596 III-83

Answer (A) is incorrect because $657,000 results from allocating Building Operations costs first.

Answer (B) is incorrect because $681,600 results from allocating Building Operations costs to Information Services.

Answer (C) is incorrect because $730,000 allocates the costs of both service departments according to the direct methodrather than the step method.

Answer (D) is correct. The step method of service department cost allocation is a sequential (but not a reciprocal)process. These costs are allocated to other service departments as well as to users. The process usually starts with theservice department that renders the greatest percentage of its services to other service departments, the department thatprovides services to the greatest number of other departments, or the department with the greatest costs of servicesprovided to other service departments. If the $1,200,000 of Information Services costs is allocated first, the allocation baseis 2,000 computer hours (200 + 1,200 + 600). Thus, $120,000 [(200 ÷ 2,000) × $1,200,000] will be allocated to BuildingOperations and $360,000 [(600 ÷ 2,000) × $1,200,000] to Finishing. The total of the Building Operations costs to beallocated to production equals $670,000 ($550,000 + $120,000). The allocation base will be 40,000 square feet becauseno costs are allocated back to Information Services. Accordingly, the total of service costs allocated to Finishing equals$762,000 {$360,000 + [$670,000 × (24,000 ÷ 40,000)]}.

[842] Gleim #: 6.6.50 -- Source: CIA 1190 IV-8

Answer (A) is incorrect because the step-down method may start with the department that renders the highest percentageof its total services to other service (not production) departments.

Answer (B) is correct. The step-down method may start with the department that renders the highest percentage of itstotal services to other service departments. It then progresses in descending order to the service department rendering theleast percentage of its services to the other service departments. An alternative is to begin with the department thatrenders the highest dollar value of services to other service departments. A third possibility is to begin with thedepartment that renders service to the greatest number of other service departments.

Answer (C) is incorrect because beginning with the service department with the greatest output is not customary.

Answer (D) is incorrect because beginning with the service department with the highest costs is not customary.

[843] Gleim #: 6.6.51 -- Source: CIA 1193 IV-3

Answer (A) is incorrect because the direct allocation method ignores any services that are rendered by one servicedepartment to another service department.

Answer (B) is incorrect because the step-down allocation method allows for limited recognition of services rendered byservice departments to other service departments.

Answer (C) is correct. The reciprocal method uses simultaneous equations to allocate each service department’s costs. Itallocates costs by explicitly including the mutual services rendered among all departments. When service departmentsrender services to each other, the use of the direct method or the step-down method would not be theoretically accurate.Accordingly, in such situations, the reciprocal method would result in the most accurate allocation.

Answer (D) is incorrect because the sequential allocation method is another name for the step-down allocation method.

Gleim's CIA Test Prep: Part III: Business Analysis and Information TechnologyAnswer Explanations

(1312 questions)

Copyright 2008 Gleim Publications, Inc. Page 259Printed for Mamdouh Farag

Page 260: P.3 Answer Explanation

[844] Gleim #: 6.6.52 -- Source: CIA 1195 III-86

Answer (A) is incorrect because 144,000 is the allocation based on the direct method.

Answer (B) is correct. The allocation base is 36,000 square feet [6,000 + 6,000 + 24,000]. Plating’s share is one-sixth[6,000 ÷ 36,000] of the total cost of maintenance services, or 120,000 (720,000 ÷ 6).

Answer (C) is incorrect because 115,200 is based on the total square footage of the entire plant.

Answer (D) is incorrect because 90,000 results from allocating overhead costs only.

[845] Gleim #: 6.6.53 -- Source: CIA 1195 III-85

Answer (A) is incorrect because a dual-rate method is a refinement of the direct method permitting the allocation of thevariable and fixed costs in separate pools. Although this method is more precise than the direct method, it does notrecognize the mutual services rendered among all service departments.

Answer (B) is incorrect because the step-down method provides for partial recognition of services rendered by otherservice departments, but it does not recognize all mutual services.

Answer (C) is incorrect because the direct method is not theoretically precise. It does not give any recognition to theservices rendered by other service departments.

Answer (D) is correct. The reciprocal method is the theoretically most defensible allocation method. It recognizes themutual services rendered among all service departments.

[846] Gleim #: 6.6.54 -- Source: CIA 591 IV-1

Answer (A) is incorrect because S2’s overhead allocated to P2 is 4,320.

Answer (B) is incorrect because S2’s overhead allocated to P2 is 4,320.

Answer (C) is correct. Using the direct method, service use by other service departments is ignored. P2 accounted for60% [45% ÷ (30% + 45%)] of the service provided to production departments by S2. Hence, the amount of S2’s overheadallocated to P2 is 4,320 (60% × 7,200).

Answer (D) is incorrect because S2’s overhead allocated to P2 is 4,320.

Gleim's CIA Test Prep: Part III: Business Analysis and Information TechnologyAnswer Explanations

(1312 questions)

Copyright 2008 Gleim Publications, Inc. Page 260Printed for Mamdouh Farag

Page 261: P.3 Answer Explanation

[847] Gleim #: 6.6.55 -- Source: CIA 591 IV-2

Answer (A) is incorrect because P1 had total overhead costs of 12,605.

Answer (B) is incorrect because P1 had total overhead costs of 12,605.

Answer (C) is incorrect because P1 had total overhead costs of 12,605.

Answer (D) is correct. The reciprocal method applies simultaneous equations to allocate each service department’s costsamong the departments providing mutual services before reallocation to other users. Thus, P1 had total overhead costs of12,605 as shown below.

Determine costs:S1 = 4,000 + .25 S2S2 = 7,200 + .30 S1S1 = 4,000 + 1,800 + .075 S1S1 = 6,270

S2 = 7,200 + .30 (6,270)S2 = 9,081

Allocate costs: S1 S2 P1 P2 Overhead costs 4,000 7,200 8,000 10,000Allocate S1 (6,270) 1,881 1,881 2,508Allocate S2 2,270 (9,081) 2,724 4,086 Balances 0 0 12,605 16,594

[848] Gleim #: 6.7.56 -- Source: CMA 1293 3-3

Answer (A) is incorrect because $3,000 is the value of the by-product.

Answer (B) is correct. The NRV at split-off for each of the joint products must be determined. Given that Alfa has a $4selling price and an additional $2 of processing costs, the value at split-off is $2 per pound. The total value at split-off for10,000 pounds is $20,000. Betters has a $10 selling price and an additional $2 of processing costs. Thus, the value atsplit-off is $8 per pound. The total value of 5,000 pounds of Betters is therefore $40,000. The 1,000 pounds of Morefeedhas a split-off value of $3 per pound, or $3,000. Assuming that Morefeed (a by-product) is inventoried (recognized in theaccounts when produced) and treated as a reduction of joint costs, the allocable joint cost is $90,000 ($93,000 – $3,000).(NOTE: Other methods of accounting for by-products are possible.) The total net realizable value of the main products is$60,000 ($20,000 Alfa + $40,000 Betters). The allocation to Alfa is $30,000 [($20,000 ÷ $60,000) × $90,000].

Answer (C) is incorrect because $31,000 fails to adjust the joint processing cost for the value of the by-product.

Answer (D) is incorrect because $60,000 is the amount allocated to Betters.

Gleim's CIA Test Prep: Part III: Business Analysis and Information TechnologyAnswer Explanations

(1312 questions)

Copyright 2008 Gleim Publications, Inc. Page 261Printed for Mamdouh Farag

Page 262: P.3 Answer Explanation

[849] Gleim #: 6.7.57 -- Source: CMA 1293 3-4

Answer (A) is incorrect because $3,000 is the value of the by-product.

Answer (B) is incorrect because $30,000 is based on the net realizable value method.

Answer (C) is incorrect because $31,000 is based on the net realizable value method and fails to adjust the jointprocessing cost for the value of the by-product.

Answer (D) is correct. Joint cost is $93,000, and Morefeed has a split-off value of $3,000. Assuming the latter amount istreated as a reduction in joint cost, the allocable joint cost is $90,000. The total physical quantity (volume) of the two jointproducts is 15,000 pounds (10,000 Alfa + 5,000 Betters). Hence, $60,000 [(10,000 ÷ 15,000) × $90,000] of the net jointcosts should be allocated to Alfa.

[850] Gleim #: 6.7.58 -- Source: CMA 1293 3-5

Answer (A) is incorrect because $39,208 is the amount allocated to Alfa if the 1,000,000 calories attributable to Morefeedis included in the computation.

Answer (B) is incorrect because $39,600 is the allocation to Alfa.

Answer (C) is correct. The net allocable joint cost is $90,000. The caloric value of Alfa is 44,000,000 (4,400 × 10,000pounds), the caloric value of Betters is 56,000,000 (11,200 × 5,000 pounds), and the total is 100,000,000. Of this totalvolume, Alfa makes up 44% and Betters 56%. Thus, $50,400 ($90,000 × 56%) should be allocated to Betters.

Answer (D) is incorrect because $52,080 is the allocation to Betters if the joint cost is not adjusted for the value of the by-product.

[851] Gleim #: 6.7.59 -- Source: CMA 1293 3-6

Answer (A) is incorrect because $36,000 is based on 40%, not 4/9.

Answer (B) is correct. Alfa has a gross sales value of $40,000 (10,000 pounds × $4), Betters has a gross sales value of$50,000 (5,000 pounds × $10), and Morefeed has a split-off value of $3,000. If the value of Morefeed is inventoried andtreated as a reduction in joint cost, the allocable joint cost is $90,000 ($93,000 – $3,000). The total gross sales value ofthe two main products is $90,000 ($40,000 + $50,000). Of this total value, $40,000 should be allocated to Alfa [($40,000÷ $90,000) × $90,000].

Answer (C) is incorrect because $41,333 fails to adjust the joint cost by the value of the by-product.

Answer (D) is incorrect because $50,000 is the joint cost allocated to Betters.

[852] Gleim #: 6.7.60 -- Source: CIA 592 IV-6

Answer (A) is incorrect because a quantity variance is not recorded for scrap that is anticipated. Furthermore, work-in-process inventory is credited only when scrap is unique to a job.

Answer (B) is incorrect because an accounting entry is not needed. The amount is not material.

Answer (C) is correct. Making a memorandum entry at the time of recovery is appropriate. The value of the scrap is thenrecognized at the time of sale. The factory overhead control account is credited because scrap is inevitable to thecompany’s production operations and not attributable to a specific job. This accounting method has the effect of spreadingthe revenue from scrap sales over all jobs or products.

Answer (D) is incorrect because normal scrap is not the basis for recording a variance.

Gleim's CIA Test Prep: Part III: Business Analysis and Information TechnologyAnswer Explanations

(1312 questions)

Copyright 2008 Gleim Publications, Inc. Page 262Printed for Mamdouh Farag

Page 263: P.3 Answer Explanation

[853] Gleim #: 6.7.61 -- Source: CIA 1192 IV-10

Answer (A) is incorrect because 40,000 is the NRV of by-product Z.

Answer (B) is incorrect because 106,250 is the joint cost allocated to Y assuming that X, Y, and Z are joint products andno separable costs need to be incurred.

Answer (C) is correct. By-products are joint products that have minor sales value compared with the sales value of themain product(s). To be salable, the by-product may or may not require additional processing beyond the split-off point.Thus, the incurrence of separable costs beyond the split-off point may or may not be required. Practice with regard torecognizing by-products in the accounts is not uniform. The most cost-effective method for the initial recognition of by-products is to account for their value at the time of sale as a reduction in the joint cost or as a revenue. The alternative isto recognize the net realizable value at the time of production, a method that results in the recording of by-productinventory. Assuming the NRV is recognized as a reduction in the joint cost, the joint cost allocated to Y is determined asfollows:

Joint cost to be allocated to X and Y= Joint cost – NRV of by-product Z= 340,000 – [(5,000 tons × 13) – 25,000]= 300,000

NRVX = 10,000 tons × 50 = 500,000

NRVY = (2,000 tons × 165) – 30,000 = 300,000

Joint cost allocated to Y

300,000= × 300,000

300,000 + 500,000

= 112,500

Answer (D) is incorrect because 120,000 is the joint cost of Y if X, Y, and Z are joint products, Y has separable costs of30,000, and Y has a selling price of 165.

[854] Gleim #: 6.7.62 -- Source: CIA 1194 III-46

Answer (A) is incorrect because waste is input material that is either lost in the production process or has no sales value.

Answer (B) is incorrect because scrap is input material that has a relatively minor sales value at the end of the productionprocess.

Answer (C) is correct. Rejected units that are discarded are classified as spoilage. Spoilage is separated into abnormal ornormal spoilage. Normal spoilage is an inherent result of the normal production process. Abnormal spoilage is spoilagethat is not expected to occur under normal, efficient operating conditions.

Answer (D) is incorrect because rework costs are incurred to make unacceptable units appropriate for sale or use.

Gleim's CIA Test Prep: Part III: Business Analysis and Information TechnologyAnswer Explanations

(1312 questions)

Copyright 2008 Gleim Publications, Inc. Page 263Printed for Mamdouh Farag

Page 264: P.3 Answer Explanation

[855] Gleim #: 6.7.63 -- Source: CIA, adapted

Answer (A) is incorrect because abnormal spoilage is not a function of the costing system; it is a function of theproduction process.

Answer (B) is incorrect because abnormal spoilage may result from any of a variety of conditions or circumstances, whichare generally controllable by first-line supervisors.

Answer (C) is incorrect because abnormal spoilage may result from any of a variety of conditions or circumstances, whichare not necessarily related to standards.

Answer (D) is correct. Abnormal spoilage is not expected under efficient operating conditions. It is not an inherent part ofthe production process. Furthermore, the cost of abnormal waste of materials and labor is expensed as incurred and notincluded in inventory.

[856] Gleim #: 6.8.64 -- Source: Publisher

Answer (A) is incorrect because $294 favorable is the materials quantity variance.

Answer (B) is correct. The materials mix variance equals the actual total quantity used times the difference between thebudgeted weighted-average standard unit cost for the budgeted mix and the budgeted weighted-average standard unit costfor the actual mix. This variance is favorable if the standard weighted-average cost for the actual mix is less than thestandard weighted-average cost for the budgeted mix. The standard mix weighted-average standard unit cost is $.225 perliter ($135 standard total cost ÷ 600 liters).

The standard cost of the actual quantity used was $18,606 (see below). Thus, the actual mix weighted-average standardunit cost was $.220398 ($18,606 ÷ 84,420 liters used), and the mix variance was $388.50 favorable [84,420 liters ×($.220398 – $.225)].

26,600 × $.200 = $ 5,320.0012,880 × .425 = 5,474.0037,800 × .150 = 5,670.00

7,140 × .300 = 2,142.00$18,606.00

Answer (C) is incorrect because $94.50 unfavorable is the materials yield variance.

Answer (D) is incorrect because $219.50 favorable is based on the actual mix of purchases.

[857] Gleim #: 6.8.65 -- Source: Publisher

Answer (A) is incorrect because $294.50 favorable is the materials quantity variance.

Answer (B) is incorrect because $388.50 favorable is the materials mix variance.

Answer (C) is correct. The materials yield variance equals the difference between the actual input and the standard inputallowed for the actual output, times the budgeted weighted-average standard cost per input unit at the standard mix. Thestandard input for the actual output was 84,000 liters (140 batches × 600 liters per batch). The standard mix budgetedweighted-average standard unit cost is $.225 per liter ($135 total cost ÷ 600 liters). Thus, the yield variance is $94.50unfavorable [(84,420 liters used – 84,000 liters allowed) × $.225].

Answer (D) is incorrect because $219.50 favorable is based on the actual mix of purchases.

Gleim's CIA Test Prep: Part III: Business Analysis and Information TechnologyAnswer Explanations

(1312 questions)

Copyright 2008 Gleim Publications, Inc. Page 264Printed for Mamdouh Farag

Page 265: P.3 Answer Explanation

[858] Gleim #: 6.8.66 -- Source: CMA 1294 3-26

Answer (A) is correct. The fixed overhead spending (budget) variance is the difference between actual and budgetedfixed factory overhead. Actual fixed overhead was $540,000. Budgeted fixed overhead was $5 per hour based on acapacity of 100,000 direct labor hours per month, or $500,000. Because these costs are fixed, the budgeted fixed overheadis the same at any level of production. Hence, the variance is $40,000 unfavorable ($540,000 – $500,000).

Answer (B) is incorrect because $70,000 unfavorable is the difference between actual fixed overhead and the product ofthe standard rate and the actual direct labor hours.

Answer (C) is incorrect because $460,000 unfavorable is the volume variance.

Answer (D) is incorrect because $240,000 unfavorable is the difference between actual variable overhead and budgetedfixed overhead.

[859] Gleim #: 6.8.67 -- Source: CMA 1294 3-27

Answer (A) is incorrect because $60,000 favorable is based on 100,000 hours, not the actual hours of 94,000.

Answer (B) is correct. The variable overhead spending variance is the difference between actual variable overhead andthe variable overhead based on the standard rate and the actual activity level. Thus, the variable overhead spendingvariance was $12,000 favorable [$740,000 actual cost – (94,000 actual hours × $8 standard rate)]. Because actual is lessthan standard, the variance was favorable.

Answer (C) is incorrect because $48,000 unfavorable is the variable overhead efficiency variance.

Answer (D) is incorrect because $40,000 unfavorable is the fixed overhead spending variance.

[860] Gleim #: 6.8.68 -- Source: CMA 1294 3-28

Answer (A) is correct. The variable overhead efficiency variance equals the standard price ($8 an hour) times thedifference between the actual hours and the standard hours allowed for the actual output. Thus, the variance is $48,000{[94,000 actual hours – (4 standard hours per unit × 22,000 units produced)] × $8}. The variance is unfavorable becauseactual hours exceeded standard hours.

Answer (B) is incorrect because $60,000 favorable is the variable overhead spending variance calculated based oncapacity, not actual hours.

Answer (C) is incorrect because $96,000 unfavorable is based on the difference between standard hours allowed for theactual output and capacity hours.

Answer (D) is incorrect because $200,000 unfavorable is the excess of actual direct labor costs over actual variableoverhead costs.

Gleim's CIA Test Prep: Part III: Business Analysis and Information TechnologyAnswer Explanations

(1312 questions)

Copyright 2008 Gleim Publications, Inc. Page 265Printed for Mamdouh Farag

Page 266: P.3 Answer Explanation

[861] Gleim #: 6.8.69 -- Source: CIA 592 IV-18

Answer (A) is incorrect because the direct labor rate variance is $100 favorable ($48,500 – $48,600).

Answer (B) is correct. The total flexible budget direct labor variance equals the difference between total actual directlabor cost and standard direct labor cost allowed for the actual output. It combines the direct labor rate and efficiencyvariances. For this company, the variance is $1,900 unfavorable ($48,500 actual wages at actual hours – $46,600 standardwages at standard hours).

Answer (C) is incorrect because the total labor variance is unfavorable.

Answer (D) is incorrect because the direct labor efficiency variance is $2,000 unfavorable.

[862] Gleim #: 6.8.70 -- Source: CIA 593 IV-14

Answer (A) is incorrect because budgeted, not actual, UCM is used to calculate this variance.

Answer (B) is incorrect because the flexible budget volume is the actual volume, resulting in a zero variance.

Answer (C) is incorrect because budgeted, not actual, UCM is used to calculate this variance.

Answer (D) is correct. The sales volume variance is the difference between flexible budget (actual volume) and masterbudget sales quantity, times master budget unit contribution margin (UCM). This amount is calculated for each product inthe sales mix, and the results are added to determine the total sales volume variance. This variance may be further dividedinto quantity and mix variances.

[863] Gleim #: 6.8.71 -- Source: CIA 1193 IV-6

Answer (A) is incorrect because factory overhead should be charged for direct materials, supplies, direct labor, andapplied overhead incurred for rework.

Answer (B) is incorrect because factory overhead should be charged for direct materials, supplies, direct labor, andapplied overhead incurred for rework.

Answer (C) is incorrect because 19,300 excludes the predetermined manufacturing overhead.

Answer (D) is correct. The rework charge for direct materials, indirect materials (supplies), direct labor, and overheadapplied on the basis of direct labor cost is 40,300 [5,000 + 300 + 14,000 + (1.5 × 14,000)]. If an allowance for rework isincluded in a company’s manufacturing overhead budget, rework of defective units is spread over all jobs or batches aspart of the predetermined overhead application rate. Hence, the debit is to overhead control.

[864] Gleim #: 6.8.72 -- Source: CIA R98 IV-67

Answer (A) is incorrect because 10 unfavorable equals the difference between the planned and actual DLH times thedifference between the standard and actual rates.

Answer (B) is incorrect because 240 unfavorable equals the planned DLH times the difference between the standard andactual rates.

Answer (C) is incorrect because 248 unfavorable equals the DLH allowed for the actual output times the differencebetween the standard and actual rates.

Answer (D) is correct. The direct labor rate variance equals actual DLH times the difference between the actual andstandard direct labor rates, or 250 unfavorable {6,250 actual DLH × [(75,250 actual costs ÷ 6,250 actual DLH) – 12.00standard rate]}.

Gleim's CIA Test Prep: Part III: Business Analysis and Information TechnologyAnswer Explanations

(1312 questions)

Copyright 2008 Gleim Publications, Inc. Page 266Printed for Mamdouh Farag

Page 267: P.3 Answer Explanation

[865] Gleim #: 6.8.73 -- Source: CIA R98 IV-68

Answer (A) is correct. The direct labor efficiency variance equals the standard direct labor rate times the differencebetween the actual DLH and the standard DLH allowed for the actual output, or 600 unfavorable {12 × [6,250 actual DLH– (.4 DLH × 15,500 pounds actually produced)]}.

Answer (B) is incorrect because 602 unfavorable is based on the actual direct labor rate.

Answer (C) is incorrect because 2,400 unfavorable equals the standard direct labor rate times the difference between theDLH allowed for the planned output and the DLH allowed for the actual output.

Answer (D) is incorrect because 3,000 unfavorable equals the standard direct labor rate times the difference between theDLH allowed for the planned output and the actual DLH.

[866] Gleim #: 6.8.74 -- Source: CIA 1192 IV-20

Answer (A) is incorrect because 1,125 favorable results from multiplying the standard direct materials that should havebeen used in production [(3 × 7,500) = 22,500 lbs.] times the difference between standard price and actual price [(1.60 –1.55) = .05].

Answer (B) is incorrect because 1,150 favorable results from multiplying the direct materials issued to production (23,000lbs.) times the difference between standard price and actual price [(1.60 – 1.55) = .05].

Answer (C) is incorrect because 1,200 favorable results from multiplying the direct materials that should have been usedfor budgeted production [(3 × 8,000) = 24,000 lbs.] times the difference between standard price and actual price [(1.60 –1.55) = .05].

Answer (D) is correct. The materials price variance measures the difference between what was actually paid for the goodspurchased and the standard amount allowed for the goods purchased. Thus, it equals the difference between actual priceand standard price, multiplied by the actual quantity purchased. This question assumes that price variances are isolated atthe time of purchase. If they are isolated when the materials are used, the variance is the difference between standard andactual price, times the amount used (not amount purchased). Accordingly, the direct materials purchase price variance is1,250 F {[(38,750 TC ÷ 25,000 lbs.) – 1.60 per lb.] × 25,000 lbs.}. The variance is favorable because actual price wasless than standard price.

[867] Gleim #: 6.8.75 -- Source: CIA 1192 IV-21

Answer (A) is incorrect because 775 unfavorable results from multiplying the difference between standard quantity [(3 ×7,500) = 22,500] and actual quantity used (23,000) times the actual price (1.55).

Answer (B) is correct. The efficiency (quantity or usage) variance for direct materials equals standard unit price times thedifference between actual usage and the standard usage for the actual output. Accordingly, the direct materials efficiencyvariance is 800 U {[23,000 lbs. issued – (3 lbs. × 7,500 units)] × 1.60}. The variance is unfavorable because actual usageexceeded standard usage.

Answer (C) is incorrect because 1,600 favorable results from multiplying the difference between standard quantity thatshould have been used for budgeted production [(3 × 8,000) = 24,000] and actual quantity used (23,000) times thestandard price (1.60).

Answer (D) is incorrect because 3,200 favorable results from multiplying the difference between the actual quantitypurchased (25,000) and actual quantity used (23,000) times the standard price (1.60).

Gleim's CIA Test Prep: Part III: Business Analysis and Information TechnologyAnswer Explanations

(1312 questions)

Copyright 2008 Gleim Publications, Inc. Page 267Printed for Mamdouh Farag

Page 268: P.3 Answer Explanation

[868] Gleim #: 6.9.76 -- Source: CMA 1295 3-9

Answer (A) is incorrect because the sales forecast is insufficient for completion of the direct labor budget.

Answer (B) is incorrect because the raw material purchases budget is not needed to prepare a direct labor budget.

Answer (C) is incorrect because the schedule of cash receipts and disbursements cannot be prepared until after the directlabor budget has been completed.

Answer (D) is correct. A master budget typically begins with the preparation of a sales budget. The next step is to preparea production budget. Once the production budget has been completed, the next step is to prepare the direct labor, rawmaterial, and overhead budgets. Thus, the production budget provides the input necessary for the completion of the directlabor budget.

[869] Gleim #: 6.9.77 -- Source: CMA 1294 3-13

Answer (A) is correct. The most difficult items to coordinate in any budget, particularly for a seasonal business, areproduction volume, finished goods inventory, and sales. Budgets usually begin with sales volume and proceed toproduction volume, but the reverse is sometimes used when production is more of an issue than generation of sales.Inventory levels are also important because sales cannot occur without inventory, and the maintenance of high inventorylevels is costly.

Answer (B) is incorrect because direct labor hours and work-in-process are only two components of a production budget.

Answer (C) is incorrect because sales is usually the most important aspect of any budget.

Answer (D) is incorrect because sales is usually the most important aspect of any budget.

[870] Gleim #: 6.9.78 -- Source: CMA 1293 3-19

Answer (A) is incorrect because $45,000 equals cash receipts.

Answer (B) is incorrect because the cash deficit will be $92,500 without borrowing.

Answer (C) is correct. Assuming Raymar maintained a $100,000 cash balance at the end of March, the amount to beborrowed or invested in April is the difference between cash receipts and disbursements. April’s cash collections are$45,000 [($50,000 April sales × 50%) + ($40,000 March sales × 50%)]. Disbursements for accounts payable are $37,500[($40,000 April payables × 75%) + ($30,000 March payables × 25%)]. In addition to the accounts payable disbursements,payroll and other disbursements will require an additional $100,000. Hence, total disbursements are estimated to be$137,500. The net negative cash flow (amount to be borrowed to reach the required minimum cash balance of $100,000)is $92,500 ($137,500 – $45,000). Because the line of credit must be drawn upon in $10,000 increments, the loan must befor $100,000.

Answer (D) is incorrect because a loan of only $90,000 would still leave a negative cash balance of $2,500.

Gleim's CIA Test Prep: Part III: Business Analysis and Information TechnologyAnswer Explanations

(1312 questions)

Copyright 2008 Gleim Publications, Inc. Page 268Printed for Mamdouh Farag

Page 269: P.3 Answer Explanation

[871] Gleim #: 6.9.79 -- Source: CMA 1293 3-20

Answer (A) is incorrect because no funds are available to repay the loan. May receipts are less than May disbursements.

Answer (B) is incorrect because the 1% interest is calculated on a $100,000 loan, not a $90,000 loan.

Answer (C) is incorrect because the 1% interest is calculated on a $100,000 loan, not a $90,000 loan.

Answer (D) is correct. The company will have to borrow $100,000 in April, which means that interest will have to bepaid in May at the rate of 1% per month (12% annual rate). Consequently, interest expense is $1,000 ($100,000 × 1%).May receipts are $75,000 [($100,000 May sales × 50%) + ($50,000 April sales × 50%)]. Disbursements in May are$40,000 [($40,000 May payables × 75%) + ($40,000 April payables × 25%)]. In addition to the May accounts payabledisbursements, payroll and other disbursements are $60,000, bringing total disbursements to $101,000 ($60,000 +$40,000 + $1,000). Thus, disbursements exceed receipts by $26,000 ($101,000 – $75,000). However, cash has abeginning surplus balance of $7,500 ($100,000 April loan – $92,500 negative cash flow for April). As a result, thecompany needs to borrow an additional $18,500 to eliminate its cash deficit. Given the requirement that loans be in$10,000 increments, the May loan must be for $20,000.

[872] Gleim #: 6.9.80 -- Source: CMA 1292 3-11

Answer (A) is incorrect because $650,000 is the cost at a production level of 100,000 units.

Answer (B) is incorrect because $715,000 assumes a variable unit cost of $6.50 with no fixed costs.

Answer (C) is correct. Raw materials unit costs are strictly variable at $2 ($200,000 ÷ 100,000 units). Similarly, directlabor has a variable unit cost of $1 ($100,000 ÷ 100,000 units). The $200,000 of manufacturing overhead for 100,000units is 50%. The variable unit cost is $1. Selling costs are $100,000 fixed and $50,000 variable for production of 100,000units, and the variable unit selling expense is $.50 ($50,000 ÷ 100,000 units). The total unit variable cost is therefore$4.50 ($2 + $1 + $1 + $.50). Fixed costs are $200,000. At a production level of 110,000 units, variable costs are $495,000(110,000 units × $4.50). Hence, total costs are $695,000 ($495,000 + $200,000).

Answer (D) is incorrect because total costs are $695,000 based on a unit variable cost of $4.50 each.

[873] Gleim #: 6.9.81 -- Source: CMA 1292 3-12

Answer (A) is incorrect because $225,000 is the net income before the increase in sales.

Answer (B) is incorrect because net income was originally $1.50 per game. The $270,000 figure simply extrapolates thatamount to sales of 180,000 games.

Answer (C) is correct. Revenue of $2,400,000 reflects a unit selling price of $16 ($2,400,000 ÷ 150,000 games). Thecontribution margin is $975,000, or $6.50 per game ($975,000 ÷ 150,000 games). Thus, unit variable cost is $9.50 ($16 –$6.50). Increasing sales will result in an increased contribution margin of $195,000 (30,000 × $6.50). Assuming noadditional fixed costs, net income will increase to $420,000 ($225,000 originally reported + $195,000).

Answer (D) is incorrect because $510,000 assumes that variable overhead is fixed. Variable overhead is a $3 component($450,000 ÷ 150,000 units) of unit variable cost.

Gleim's CIA Test Prep: Part III: Business Analysis and Information TechnologyAnswer Explanations

(1312 questions)

Copyright 2008 Gleim Publications, Inc. Page 269Printed for Mamdouh Farag

Page 270: P.3 Answer Explanation

[874] Gleim #: 6.9.82 -- Source: CIA 595 III-20

Answer (A) is incorrect because traditional or incremental budgeting takes the previous year’s budgets and adjusts themfor inflation.

Answer (B) is correct. ZBB is a planning process in which each manager must justify his/her department’s full budget foreach period. The purpose is to encourage periodic reexamination of all costs in the hope that some can be reduced oreliminated.

Answer (C) is incorrect because traditional or incremental budgeting assumes all activities are legitimate and worthy ofreceiving budget increases to cover any increased costs.

Answer (D) is incorrect because it is a definition of a capital budget.

[875] Gleim #: 6.9.83 -- Source: CIA 1194 III-55

Answer (A) is correct. Zero-base budgeting treats the budget as if it were being initiated for the first time. Thus, all costsare reexamined each period. Beginning with the smallest budgetary subunits, managers must determine objectives,operations, and costs for all activities. Alternative methods of conducting each activity are considered, different levels ofservice are evaluated for each activity, means of measuring work and performance are determined, and activities areranked in order of importance to the organization.

Answer (B) is incorrect because reviewing the prior period’s budget, actual results for that period, and the expectationsfor the coming period is incremental budgeting.

Answer (C) is incorrect because requiring managers to establish priorities in the event the budget is increased ordecreased by a fixed percentage is priority incremental budgeting.

Answer (D) is incorrect because ascertaining desired outputs and necessary inputs is the input-output procedure for budgetdevelopment.

Gleim's CIA Test Prep: Part III: Business Analysis and Information TechnologyAnswer Explanations

(1312 questions)

Copyright 2008 Gleim Publications, Inc. Page 270Printed for Mamdouh Farag

Page 271: P.3 Answer Explanation

[876] Gleim #: 6.9.84 -- Source: CMA 694 3-7

Answer (A) is incorrect because 1,800 equals projected unit sales.

Answer (B) is incorrect because 1,565 equals units needed for sales minus all inventory amounts.

Answer (C) is incorrect because 1,815 equals finished units needed.

Answer (D) is correct. The finished units needed equal 1,815:

Needed for sales 1,800Needed for ending inventory 100 Total finished units needed 1,900Minus beginning inventory 85 Finished units needed 1,815 The units to be produced equal 1,845:Finished units needed 1,815Needed for ending inventory 40 Total units in process 1,855Minus beginning WIP inventory 10 Units to be produced 1,845

[877] Gleim #: 6.9.85 -- Source: CMA 696 3-14

Answer (A) is incorrect because comparing results using a monthly budget is no easier than using a budget of any otherduration.

Answer (B) is incorrect because a master budget is the overall budget. It will not facilitate comparisons unless it is also aflexible budget.

Answer (C) is incorrect because a rolling (or continuous) budget is revised on a regular (continuous) basis. It will notfacilitate comparisons unless it is also a flexible budget.

Answer (D) is correct. A flexible budget is essentially a series of several budgets prepared for many levels of sales orproduction. At the end of the period, management can compare actual costs or performance with the appropriate budgetedlevel in the flexible budget. New columns can quickly be made by interpolation or extrapolation, if necessary. A flexiblebudget is designed to allow adjustment of the budget to the actual level of activity before comparing the budgeted activitywith actual results.

[878] Gleim #: 6.9.86 -- Source: CIA, adapted

Answer (A) is correct. A flexible budget is a series of budgets prepared for many levels of activity. It is designed to allowadjustment of the budget to the actual level of activity before comparing the budgeted activity with actual results.

Answer (B) is incorrect because this is the definition of an operating budget.

Answer (C) is incorrect because this is the definition of activity-based budgeting.

Answer (D) is incorrect because this is the definition of Kaizen budgeting.

Gleim's CIA Test Prep: Part III: Business Analysis and Information TechnologyAnswer Explanations

(1312 questions)

Copyright 2008 Gleim Publications, Inc. Page 271Printed for Mamdouh Farag

Page 272: P.3 Answer Explanation

[879] Gleim #: 6.9.87 -- Source: CIA, adapted

Answer (A) is incorrect because the master budget does not contain actual results.

Answer (B) is incorrect because the master budget reflects all applicable expected costs, whether controllable byindividual managers or not.

Answer (C) is incorrect because the master budget is not structured to allow determination of manufacturing costvariances. This is accomplished using the flexible budget and actual results.

Answer (D) is correct. The operating budget sequence is part of the master budget process that begins with the salesbudget and culminates in the pro forma income statement.

[880] Gleim #: 6.9.88 -- Source: Publisher

Answer (A) is incorrect because “managing to a budget” can result in suboptimal achievement.

Answer (B) is incorrect because rigid adherence to the budget could prevent a manager from taking an appropriate actionand thus miss a profitable opportunity.

Answer (C) is correct. Top management involvement in support of the budget/control system is absolutely vital for thecontinued success of the operation. The attitude of top management will affect the implementation of the budget andcontrol system because lower level management will recognize and reflect top management’s attitude.

Answer (D) is incorrect because budgeting is not directly related to departmental growth. A budget should promote overallcompany strategies, missions, objectives, and policies.

[881] Gleim #: 6.9.89 -- Source: Publisher

Answer (A) is correct. Budgets provide a means for coordinating the plans of all organizational subunits. Thus, budgetsare a way to promote goal congruence. Although budgets should be consistent with the strategic plans of top management,they should also be based on input from lower-level managers since the latter have detailed knowledge of operatingactivities. Successful budgets are therefore a compromise. In a top-down process, however, budgets are imposed onsubordinates without their participation. This lack of participation may impair the coordination of the goals of subunitswith those of the organization (goal congruence) since lower-level managers will tend not to have understanding of andsupport for the top-down budget.

Answer (B) is incorrect because, in a top-down process, upper-level management imposes a budget on those below, soconsistency with strategic plans is unlikely to be impaired.

Answer (C) is incorrect because, in a top-down process, upper-level management imposes a budget on those below, soconsistency with strategic plans is unlikely to be impaired.

Answer (D) is incorrect because the motivational effect is likely to be very negative.

Gleim's CIA Test Prep: Part III: Business Analysis and Information TechnologyAnswer Explanations

(1312 questions)

Copyright 2008 Gleim Publications, Inc. Page 272Printed for Mamdouh Farag

Page 273: P.3 Answer Explanation

[882] Gleim #: 6.9.90 -- Source: Publisher

Answer (A) is incorrect because disadvantages of participatory budgeting and standard setting include the cost in terms oftime and money.

Answer (B) is incorrect because the quality of participation is affected by the goals, values, beliefs, and expectations ofthose involved.

Answer (C) is incorrect because an advantage of participatory budgeting is that it yields information known to employeesbut not to management.

Answer (D) is correct. Participatory budgeting (grass-roots budgeting) and standard setting use input from lower-leveland middle-level employees. Participation encourages employees to have a sense of ownership of the output of theprocess. The result is an acceptance of, and commitment to, the goals expressed in the budget.

[883] Gleim #: 6.9.91 -- Source: CMA 1293 3-17

Answer (A) is incorrect because flexible budgeting is the process of preparing a series of multiple budgets for varyinglevels of production or sales.

Answer (B) is incorrect because human resource management is the process of managing personnel.

Answer (C) is correct. Managerial performance should ideally be evaluated only on the basis of those factors controllableby the manager. Managers may control revenues, costs, or investments in resources. A well-designed responsibilityaccounting system establishes responsibility centers within the organization. However, controllability is not an absolutebasis for establishment of responsibility. More than one manager may be able to influence a cost, and responsibility maybe assigned on the basis of knowledge about the incurrence of a cost rather than the ability to control it. Management byobjectives (MBO) is a related concept. It is a behavioral, communications-oriented, responsibility approach to employeeself-direction. Under MBO, a manager and his/her subordinates agree upon objectives and the means of attaining them.The plans that result are reflected in responsibility accounting and in the budgeting process.

Answer (D) is incorrect because capital budgeting is a means of evaluating long-term investments.

[884] Gleim #: 6.9.92 -- Source: Publisher

Answer (A) is incorrect because motivation is the desire and the commitment to achieve a specific goal.

Answer (B) is correct. Goal congruence is agreement on the goals of the organization or the segment by both supervisorsand subordinates. Performance is assumed to be optimized when there is an understanding that personal and segmentalgoals be consistent with those of the organization.

Answer (C) is incorrect because autonomy is the extent to which individuals have the authority to make decisions.

Answer (D) is incorrect because managerial effort is the extent of the attempt to accomplish a specific goal.

Gleim's CIA Test Prep: Part III: Business Analysis and Information TechnologyAnswer Explanations

(1312 questions)

Copyright 2008 Gleim Publications, Inc. Page 273Printed for Mamdouh Farag

Page 274: P.3 Answer Explanation

[885] Gleim #: 6.10.93 -- Source: CIA 597 IV-40

Answer (A) is incorrect because $316,920 discounts the cash inflow over a 4-year period.

Answer (B) is incorrect because $23,140 assumes a 16% discount rate.

Answer (C) is correct. The cash inflow occurs 5 years after the cash outflow, and the NPV method uses the firm’s cost ofcapital of 18%. The present value of $1 due at the end of 5 years discounted at 18% is .4371. Thus, the NPV of project Ais $(265,460) [(.4371 × $7,400,000 cash inflow) – $3,500,000 cash outflow].

Answer (D) is incorrect because $(316,920) discounts the cash inflow over a 4-year period and also subtracts the presentvalue of the cash inflow from the cash outflow.

[886] Gleim #: 6.10.94 -- Source: CIA 597 IV-41

Answer (A) is incorrect because 15% results in a positive NPV for project B.

Answer (B) is incorrect because 16% is the approximate internal rate of return for project A.

Answer (C) is incorrect because 18% is the company’s cost of capital.

Answer (D) is correct. The internal rate of return is the discount rate at which the NPV is zero. Consequently, the cashoutflow equals the present value of the inflow at the internal rate of return. The present value of $1 factor for project B’sinternal rate of return is therefore .4020 ($4,000,000 cash outflow ÷ $9,950,000 cash inflow). This factor is closest to thepresent value of $1 for 5 periods at 20%.

[887] Gleim #: 6.10.95 -- Source: CIA 595 IV-37

Answer (A) is incorrect because the present value of the cash inflows is inversely related to the discount rate; that is, if thediscount rate is higher, the present value of the cash inflows is lower. If the investment cost is lower, a higher discountrate (the IRR) will be required to set the net present value equal to zero.

Answer (B) is correct. The IRR is the discount rate at which the net present value is zero. Because the present value of adollar is higher the sooner it is received, projects with later cash flows will have lower net present values for any givendiscount rate than will projects with earlier cash flows, if other factors are constant. Hence, projects with later cash flowswill have a lower IRR.

Answer (C) is incorrect because the larger the cash inflows, the higher the IRR will be. Higher cash inflows have a higherpresent value at any given discount rate. A higher discount rate will be required to set the net present value equal to zero.

Answer (D) is incorrect because projects with shorter payback periods have higher cash inflows early in the life of theproject. Projects with earlier cash inflows have higher IRRs.

[888] Gleim #: 6.10.96 -- Source: CIA 1190 IV-54

Answer (A) is incorrect because the ranking of IRRs is indeterminate.

Answer (B) is incorrect because the ranking of IRRs is indeterminate.

Answer (C) is correct. The IRR is the discount rate at which the net present value is zero. The NPV is the present valueof future cash flows minus the present value of the investment. Because of a possible difference in the scale of the projectsand other factors, a higher NPV does not necessarily result in a higher IRR.

Answer (D) is incorrect because the ranking for payback periods is indeterminate.

Gleim's CIA Test Prep: Part III: Business Analysis and Information TechnologyAnswer Explanations

(1312 questions)

Copyright 2008 Gleim Publications, Inc. Page 274Printed for Mamdouh Farag

Page 275: P.3 Answer Explanation

[889] Gleim #: 6.10.97 -- Source: CIA 594 IV-43

Answer (A) is incorrect because the NPV method ranks B over A.

Answer (B) is correct. For two mutually exclusive projects, the company should undertake the project with the highestpositive net present value. Project B has a higher NPV than Project A. The NPVs of both projects are calculated using thefirm’s 8% cost of capital as follows:

40,000 40,000NPV of Project A = –30,000 + +

(1.08) (1.08)2

= 41,331

700,000 500,000NPV of Project B = –1,000,000 + +

(1.08) (1.08)2

= 76,816

Answer (C) is incorrect because the projects are mutually exclusive.

Answer (D) is incorrect because both projects have positive NPVs and one should be selected.

[890] Gleim #: 6.10.98 -- Source: CIA 594 IV-44

Answer (A) is incorrect because each project has a positive NPV and both should be undertaken.

Answer (B) is incorrect because each project has a positive NPV and both should be undertaken.

Answer (C) is correct. Independent projects are those whose cash flows are not affected by the acceptance ornonacceptance of other projects. The company must decide whether to accept or to reject each of the projects. Becauseboth projects have a positive NPV calculated using the firm’s 8% cost of capital, both should be accepted.

Answer (D) is incorrect because each project has a positive NPV and both should be undertaken.

[891] Gleim #: 6.10.99 -- Source: CMA 695 3-17

Answer (A) is incorrect because capital budgeting involves long-term investment needs, not immediate operating needs.

Answer (B) is incorrect because strategic planning establishes long-term goals in the context of relevant factors in thefirm’s environment.

Answer (C) is incorrect because cash budgeting determines operating cash flows. Capital budgeting evaluates the rate ofreturn on specific investment alternatives.

Answer (D) is correct. Capital budgeting is the process of planning expenditures for long-lived assets. It involveschoosing among investment proposals using a ranking procedure. Evaluations are based on various measures involvingrate of return on investment.

Gleim's CIA Test Prep: Part III: Business Analysis and Information TechnologyAnswer Explanations

(1312 questions)

Copyright 2008 Gleim Publications, Inc. Page 275Printed for Mamdouh Farag

Page 276: P.3 Answer Explanation

[892] Gleim #: 6.10.100 -- Source: CIA, adapted

Answer (A) is incorrect because this answer discounts the cash inflow at the correct discount rate (18%), but for fouryears instead of five, and also subtracts the cash inflow from the cash outflow, instead of vice versa.

Answer (B) is correct. The cash flow at December 31 of year five is five years from the present cash outflow, and the netpresent value method uses the firm’s cost of capital of 18%. The present value factor for 18% for five years is .4371, and7,400,000 multiplied by .4371 equals 3,234,540, which is 265,460 less than the present cash outflow of 3,500,000.

Answer (C) is incorrect because this answer cannot be computed using the table values and dollar amounts given.

Answer (D) is incorrect because this answer discounts the cash inflow at the correct discount rate (18%), but for fouryears instead of five.

[893] Gleim #: 6.10.101 -- Source: CIA, adapted

Answer (A) is incorrect because twenty percent gives the project an NPV of zero.

Answer (B) is incorrect because twenty percent gives the project an NPV of zero.

Answer (C) is correct. Twenty percent is the rate of return that equates the cash inflows with the cash outflows. Thepresent value of 20 percent for five years is .4019, which multiplied by 9,950,000 equals 3,998,905. Therefore, the netpresent value of the project approximates 0 using the 20 percent rate.

Answer (D) is incorrect because twenty percent gives the project an NPV of zero.

[894] Gleim #: 6.11.102 -- Source: CIA 1194 III-56

Answer (A) is incorrect because variable and fixed costs may be relevant or irrelevant.

Answer (B) is incorrect because expected incremental out-of-pocket expenses should be considered, but common costsshould not.

Answer (C) is correct. The analysis of a make-or-buy decision is based on relevant costs. If costs do not vary with theoption chosen, they are irrelevant. Moreover, the decision may be based on nonquantitative factors, for example, thedesire to maintain a relationship with a vendor or to assume control over development of a product.

Answer (D) is incorrect because historical costs are not relevant to cost-benefit analysis because they are sunk costs.

[895] Gleim #: 6.11.103 -- Source: CIA 1194 III-57

Answer (A) is correct. Development cost equals the cost of the outside contractor plus the costs for hardware andsupplies. Computer charges are transfer prices and do not require additional expenditures, given idle capacity. Therelevant cost of supplies is $300 ($500 – $200 cost if the software is purchased). The contractor’s use of an otherwise idleoffice is not relevant. Thus, the relevant cost of development is $25,500 [(1,000 hours × $22 hourly cost of the contractor)+ $3,200 hardware purchases + $300 incremental cost of supplies]. This amount is $3,500 less than the $29,000 cost ofpurchase. A systems analyst’s work on the new software is not relevant. It is part of the regular workload.

Answer (B) is incorrect because the contractor is not paid $25 per hour.

Answer (C) is incorrect because $550 in computer charges and $4,500 in occupancy charges should not be included.

Answer (D) is incorrect because the contractor is not paid $25 per hour, and 40 hours of modification and testing, $550 ofthe computer charges, and the occupancy costs are irrelevant.

Gleim's CIA Test Prep: Part III: Business Analysis and Information TechnologyAnswer Explanations

(1312 questions)

Copyright 2008 Gleim Publications, Inc. Page 276Printed for Mamdouh Farag

Page 277: P.3 Answer Explanation

[896] Gleim #: 6.11.104 -- Source: CIA 596 III-80

Answer (A) is incorrect because $(120,000) considers only the production costs of the good units sold. Moreover, itincludes fixed overhead, a cost that is not affected by the choice of materials.

Answer (B) is incorrect because $120,000 considers only the variable costs of the good units produced.

Answer (C) is correct. If a different direct material is used, incremental revenue will be $1,500,000 {[(12% defect rate –2%) × 300,000 units] × $50}. Incremental cost will be $750,000 ($2.50 × 300,000 units). Thus, the net benefit will be$750,000 ($1,500,000 – $750,000).

Answer (D) is incorrect because $1,425,000 includes only the incremental direct materials cost of the increase in thenumber of good units produced.

[897] Gleim #: 6.11.105 -- Source: CIA 593 IV-19

Answer (A) is incorrect because $8,000 assumes that $3 of fixed overhead is avoidable.

Answer (B) is correct. Differential (incremental) cost is the difference in total cost between two decisions. The relevantcosts do not include unavoidable costs, such as the $3 of fixed overhead. It would cost ABC an additional $20,000 topurchase, rather than manufacture, the components.

Cost to purchase (12,000 × $37) $444,000Minus rental income (40,000)

$404,000Cost to manufacture (12,000 × $32) $384,000Cost differential $ 20,000

Answer (C) is incorrect because $24,000 compares the full cost of manufacturing with cost to purchase.

Answer (D) is incorrect because $44,000 ignores the opportunity cost.

Gleim's CIA Test Prep: Part III: Business Analysis and Information TechnologyAnswer Explanations

(1312 questions)

Copyright 2008 Gleim Publications, Inc. Page 277Printed for Mamdouh Farag

Page 278: P.3 Answer Explanation

[898] Gleim #: 6.11.106 -- Source: CIA 1196 III-98

Answer (A) is incorrect because Product Line 2 has an increased profit of $600.

Answer (B) is incorrect because Product Line 2 has an increased profit of $600.

Answer (C) is incorrect because $1,400 omits the commissions from the calculation.

Answer (D) is correct. Fixed costs should be ignored. Thus, the increase in sales revenue should be multiplied by thecontribution margin ratio for each product line. The incremental promotion cost ($7,000) is subtracted from this amount todetermine the marginal benefit of promoting each product line. Hence, Product Line 3 has an increased profit of $1,120.

Calculation of Contribution Margin

Total Product Product Product Company Line 1 Line 2 Line 3

Sales revenue $2,000,000 $800,000 $700,000 $500,000Variable costs

Commissions $ 40,000 $ 16,000 $ 14,000 $ 10,000Cost of sales 980,000 360,000 420,000 200,000

Total $1,020,000 $376,000 $434,000 $210,000CM $ 980,000 $424,000 $266,000 $290,000CMR 49% 53% 38% 58%Revenue $ 15,000 $ 20,000 $ 14,000CMR .53 .38 .58CM $ 7,950 $ 7,600 $ 8,120Promotion cost 7,000 7,000 7,000Increased profits $ 950 $ 600 $ 1,120

[899] Gleim #: 6.11.107 -- Source: CIA 1196 III-99

Answer (A) is incorrect because an increase of $50,000 assumes the revenue will be lost and all of its costs will beavoided.

Answer (B) is incorrect because a decrease of $94,000 results from treating rent as an avoidable cost.

Answer (C) is correct. The operating income will decrease. Product Line 2 income will be lost, but only the traceablecosts of commissions, cost of sales, and salaries will be avoided. Accordingly, the decrease will be $234,000 [-$700,000 +($14,000 + 420,000 + 32,000)]. The other shared costs will have to be absorbed by the two remaining product lines.

Answer (D) is incorrect because an increase of $416,000 subtracts the costs that will not be avoided if Product Line 2 isdropped from the lost sales revenue.

Gleim's CIA Test Prep: Part III: Business Analysis and Information TechnologyAnswer Explanations

(1312 questions)

Copyright 2008 Gleim Publications, Inc. Page 278Printed for Mamdouh Farag

Page 279: P.3 Answer Explanation

[900] Gleim #: 6.11.108 -- Source: CIA 1196 III-100

Answer (A) is correct. Product Line 1 needs to cover its variable out-of-pocket costs as a minimum on this special-orderproduct; therefore, any selling price greater than the variable cost will contribute towards profits. Thus, the minimumselling price of the special-order product is the variable cost divided by 1 minus the commission rate, or $15 [$14.70 ÷(1.0 – .02)].

Answer (B) is incorrect because $17.30 includes the average cost of salaries (at the new volume level of 24,000 units) as acost that needs to be covered when determining the minimum selling price.

Answer (C) is incorrect because $27.50 is calculated based on a full cost approach.

Answer (D) is incorrect because $30.20 adds all costs and expenses (except cost of sales) and divides them by the originalvolume level of 20,000 units to determine the average operating costs. The new cost of sales is added to the averageoperating costs to determine the minimum selling price.

[901] Gleim #: 6.11.109 -- Source: CIA 592 IV-10

Answer (A) is correct. The necessary assumptions are that all fixed costs and the unit variable costs of direct materials,direct labor, and variable manufacturing overhead are not affected by the special order. Hence, the fixed costs are notrelevant. The unit costs of direct materials and direct labor are given as $9.80 and $4.50, respectively. The unit variablemanufacturing overhead cost is $3.00 [($6,000,000 total manufacturing overhead – $4,500,000 total fixed manufacturingoverhead) ÷ 500,000 units normal volume]. The unit variable selling and administrative cost is $1.00 [$2.50 × (1.0 – .6)].Consequently, the total relevant cost of the special order is $1,830,000 [100,000 units × ($9.80 + $4.50 + $3.00 + $1.00)].

Answer (B) is incorrect because variable manufacturing per unit is determined by using normal volume, not plantcapacity, as the denominator level.

Answer (C) is incorrect because the total relevant cost of the special order is $1,830,000 [100,000 units × ($9.80 +$4.50 + $3.00 + $1.00)].

Answer (D) is incorrect because fixed selling and administrative expenses of $4.20 per unit should not be included.Furthermore, variable manufacturing of $3 per unit, not total manufacturing overhead of $12 per unit, should be used inthe calculation of relevant costs.

[902] Gleim #: 6.11.110 -- Source: CIA 1195 III-68

Answer (A) is incorrect because saving 54,000 by purchasing assumes that fixed manufacturing overhead and the currentspace rental are relevant and that the new space rental is not.

Answer (B) is incorrect because saving 69,000 by purchasing assumes that fixed manufacturing overhead is relevant.

Answer (C) is correct. The relevant costs of making the motor are direct materials, direct labor, variable manufacturingoverhead, rental costs of the special machinery, and rental costs of the new warehouse. The fixed manufacturing overheadand the costs of the current warehouse space are not relevant because they will be incurred regardless of whether themotor is made or bought. Buying the motor will cost 630,000 (21 × 30,000). Making the motor will cost 549,000 {[(8 DM+ 4 DL + 3 VOH) × 30,000] + 54,000 + 45,000}. Hence, making the motor saves 81,000 (630,000 – 549,000).

Answer (D) is incorrect because saving 96,000 by making the motor assumes that the current space rental is relevant andthat the new space rental is not.

Gleim's CIA Test Prep: Part III: Business Analysis and Information TechnologyAnswer Explanations

(1312 questions)

Copyright 2008 Gleim Publications, Inc. Page 279Printed for Mamdouh Farag

Page 280: P.3 Answer Explanation

[903] Gleim #: 6.11.111 -- Source: CIA 1195 III-69

Answer (A) is incorrect because residual income equals net operating income minus an imputed interest charge for theinvestment base.

Answer (B) is incorrect because sunk costs are unavoidable. They cannot be changed regardless of the action taken.

Answer (C) is incorrect because separable costs arise after a joint process. They are incurred beyond the split-off point andcan be directly associated with individual products.

Answer (D) is correct. Opportunity cost is the net benefit that could be obtained from the best alternative use of aresource. Accordingly, it is the benefit forgone by using that resource in a particular way.

[904] Gleim #: 6.11.112 -- Source: CIA 1192 IV-18

Answer (A) is incorrect because 8.35 does not consider the variable selling expenses.

Answer (B) is correct. Because the manufacturer has excess capacity and existing sales will be unaffected, the minimumprice the manufacturer should be willing to accept is anything above the total variable cost of the unit (2.05 + 3.60 + 2.70+ 0.90 = 9.25), an amount that includes the variable manufacturing cost and the variable selling expenses. The fixed costsare not relevant.

Answer (C) is incorrect because 14.00 includes the fixed manufacturing overhead, which will be incurred whether theorder is accepted or not. Additional fixed manufacturing overhead costs will not be incurred because the manufacturer isbelow full capacity. The fixed manufacturing overhead is a sunk cost that is not relevant to this decision.

Answer (D) is incorrect because 14.80 does not consider that the manufacturer is below full capacity and that the customeris placing a one-time order. Under these circumstances, the manufacturer would not use its targeted selling price formula.

[905] Gleim #: 6.11.113 -- Source: CIA, adapted

Answer (A) is incorrect because this answer only takes into account the rework cost and excludes the income that was lostwhen the units were sold as factory seconds: 12 × 1,200 = 14,400.

Answer (B) is correct. Both the cost of rework and the income lost when units were sold as factory seconds rather thanfirst-quality units need to be included in the calculation. The following is the appropriate calculation:[12 + (50 – 45)] × 1,200 = 20,400.

Answer (C) is incorrect because this answer computes what the contribution margin would have been if the reworkeditems had been sold as first-quality units: [50 – (20 + 12)] × 1,200 = 21,600.

Answer (D) is incorrect because this answer incorrectly computes the contribution margin for the reworked units, ignoringthe 20 variable costs per unit. (45 – 12) × 1,200 = 39,600.

[906] Gleim #: 6.11.114 -- Source: CIA, adapted

Answer (A) is incorrect because this is 80 percent of the actual sales, calculated as .80 (2,000,000) = 1,600,000.

Answer (B) is incorrect because this is the actual sales.

Answer (C) is correct. Full capacity sales can be calculated as follows: Actual sales ÷ percent of capacity at which fixedassets were operated = 2,000,000 ÷ .80 = 2,500,000.

Answer (D) is incorrect because this is actual sales divided by the proportion of unused, rather than used, capacity, or2,000,000 ÷ .2 = 10,000,000.

Gleim's CIA Test Prep: Part III: Business Analysis and Information TechnologyAnswer Explanations

(1312 questions)

Copyright 2008 Gleim Publications, Inc. Page 280Printed for Mamdouh Farag

Page 281: P.3 Answer Explanation

[907] Gleim #: 6.11.115 -- Source: CIA, adapted

Answer (A) is incorrect because incremental costs are relevant if they occur in the future.

Answer (B) is incorrect because opportunity costs (benefits foregone) are relevant if they occur in the future.

Answer (C) is incorrect because outlay costs are relevant if they occur in the future.

Answer (D) is correct. Sunk costs are always irrelevant because they occurred in the past.

[908] Gleim #: 6.11.116 -- Source: CIA 597 IV-55

Answer (A) is incorrect because 1,600,000 is 80% of the actual sales.

Answer (B) is incorrect because 2,000,000 equals the actual sales.

Answer (C) is correct. Full capacity sales equals actual sales divided by the percentage of capacity at which PPEs wereoperated. Thus, full capacity sales equals 2,500,000 (2,000,000 ÷ 80%).

Answer (D) is incorrect because 10,000,000 equals actual sales divided by the proportion of unused capacity.

[909] Gleim #: 6.12.117 -- Source: CIA 1195 III-96

Answer (A) is incorrect because a transfer at full cost means that the selling division will not make a profit. In addition,the selling division may be forgoing profits that could be obtained by selling to outside customers. Thus, full-cost transferprices can lead to suboptimal decisions.

Answer (B) is incorrect because a transfer at full cost plus markup results in no incentive for the selling division to controlits costs. Hence, a sustained level of management effort may not be maintained.

Answer (C) is incorrect because a transfer at variable cost plus markup has the same weaknesses as full cost plus markup.

Answer (D) is correct. A market price transfer price promotes goal congruence and a sustained level of managementeffort. It is also consistent with divisional autonomy. A market transfer price is most appropriate when the market iscompetitive, interdivisional dependency is low, and buying in the market involves no marginal costs or benefits.

[910] Gleim #: 6.12.118 -- Source: CIA 1193 IV-19

Answer (A) is correct. A transfer price is the price charged by one segment of an organization for a product or servicesupplied to another segment of the same organization. The three basic criteria that the transfer pricing system in adecentralized company should satisfy are to (1) provide information allowing central management to evaluate divisionswith respect to total company profit and each division’s contribution to profit, (2) stimulate each manager’s efficiencywithout losing each division’s autonomy, and (3) motivate each divisional manager to achieve his/her own profit goal in amanner contributing to the company’s success. Because the $220 transfer price selected is based on the quoted externalprice (market), it is an example of market-based transfer pricing.

Answer (B) is incorrect because the cost-based price would be $210 ($100 + $50 + $40 + $20).

Answer (C) is incorrect because no negotiations took place.

Answer (D) is incorrect because cost plus 20% would be $252 ($210 × 1.20).

Gleim's CIA Test Prep: Part III: Business Analysis and Information TechnologyAnswer Explanations

(1312 questions)

Copyright 2008 Gleim Publications, Inc. Page 281Printed for Mamdouh Farag

Page 282: P.3 Answer Explanation

[911] Gleim #: 6.12.119 -- Source: CIA 589 IV-16

Answer (A) is correct. From the seller’s perspective, the price should reflect at least its incremental cash outflow (outlaycost) plus the contribution from an outside sale (opportunity cost). Because A has idle capacity, the opportunity cost is $0.Thus, the minimum price Division A should charge Division B is $7.00.

Answer (B) is incorrect because $7.00 is the minimum that should be charged.

Answer (C) is incorrect because Division A should not include any fixed costs in its transfer price because Division A hasidle capacity.

Answer (D) is incorrect because, since Division A has idle capacity, the minimum transfer price should recover DivisionA’s variable (outlay) costs.

[912] Gleim #: 6.12.120 -- Source: CIA, adapted

Answer (A) is incorrect because country X has a higher tax rate than the home country and country Y has a lower tax rate.The transfer pricing incentives will be different for the two sales divisions.

Answer (B) is incorrect because country X has a higher tax rate than the home country and country Y has a lower tax rate.The transfer pricing incentives will be different for the two sales divisions.

Answer (C) is correct. Country X has a higher tax rate than the home country, so the incentive is to transfer profits out ofcountry X. The sales division in country X will be less profitable if it is charged a high transfer price, hence pay less tax.

Answer (D) is incorrect because country Y has a lower tax rate than the home country, so the incentive is to transferprofits into country Y by charging that sales division a low transfer price.

[913] Gleim #: 6.13.121 -- Source: CMA 1291 3-10

Answer (A) is incorrect because a service center has no responsibility for developing markets or selling.

Answer (B) is incorrect because a production center is engaged in manufacturing.

Answer (C) is incorrect because a profit center can choose its markets and sources of supply.

Answer (D) is correct. A service center exists primarily and sometimes solely to provide specialized support to other unitswithin the organization. Service centers are usually operated as cost centers.

[914] Gleim #: 6.13.122 -- Source: CIA 595 III-96

Answer (A) is incorrect because return on assets cannot be computed for a cost center. The manager is not responsible forrevenue (return) or the assets available.

Answer (B) is incorrect because return on investments cannot be computed for a cost center. The manager is notresponsible for revenue (return) or the assets available.

Answer (C) is incorrect because the payback method is a means of evaluating alternative investment proposals.

Answer (D) is correct. A cost center is a responsibility center that is responsible for costs only. Of the alternatives given,variance analysis is the only one that can be used in a cost center. Variance analysis involves comparing actual costs withpredicted or standard costs.

Gleim's CIA Test Prep: Part III: Business Analysis and Information TechnologyAnswer Explanations

(1312 questions)

Copyright 2008 Gleim Publications, Inc. Page 282Printed for Mamdouh Farag

Page 283: P.3 Answer Explanation

[915] Gleim #: 6.13.123 -- Source: CMA 694 3-24

Answer (A) is incorrect because a revenue center is responsible only for revenue generation, not for costs or capitalinvestment.

Answer (B) is incorrect because a profit center is responsible for revenues and costs but not for invested capital.

Answer (C) is incorrect because a cost center is evaluated only on the basis of costs incurred. It is not responsible forrevenues or invested capital.

Answer (D) is correct. An investment center is the organizational type most like an independent business because it isresponsible for its own revenues, costs incurred, and capital invested. The other types of centers do not incorporate allthree elements.

[916] Gleim #: 6.13.124 -- Source: CMA 1291 3-7

Answer (A) is incorrect because fixed operating assets are controlled by the division manager and contribute to profits.

Answer (B) is correct. An evaluation of an investment center is based upon the return on the investment base. Theseassets include plant and equipment, inventories, and receivables. Most likely, however, an asset, such as land, that isbeing held by the division as a site for a new plant would not be included in the investment base because it is not currentlybeing used in operations. Total assets in use rather than total assets available is preferable when the investment center hasbeen forced to carry idle assets.

Answer (C) is incorrect because inventories are operating assets that contribute to profits and are controlled by thedivision manager.

Answer (D) is incorrect because the level of accounts payable is an operating decision that should be considered in theevaluation of the division manager.

[917] Gleim #: 6.13.125 -- Source: CMA 697 3-24

Answer (A) is incorrect because this method uses the same asset base.

Answer (B) is correct. Residual income is the excess of the amount of the ROI over a targeted amount equal to animputed interest charge on invested capital. The rate used is ordinarily the weighted-average cost of capital. Theadvantage of using residual income rather than percentage ROI is that it emphasizes maximizing a monetary amountinstead of a percentage. Managers of divisions with a high ROI are encouraged to accept projects with returns exceedingthe cost of capital even if those projects reduce ROI. However, comparisons of investment centers based on residualincome may be misleading because of differences in products, markets, costs, and local conditions.

Answer (C) is incorrect because this method uses the same asset base.

Answer (D) is incorrect because use of the residual income method requires a knowledge of the cost of capital; thus,arguments about the implicit cost of interest may escalate with use of the residual income method.

Gleim's CIA Test Prep: Part III: Business Analysis and Information TechnologyAnswer Explanations

(1312 questions)

Copyright 2008 Gleim Publications, Inc. Page 283Printed for Mamdouh Farag

Page 284: P.3 Answer Explanation

[918] Gleim #: 6.13.126 -- Source: CMA 697 3-29

Answer (A) is incorrect because 34.78% results from subtracting working capital from plant and equipment in calculatingthe net investment.

Answer (B) is incorrect because 22.54% fails to include average working capital in the total for the net investment.

Answer (C) is incorrect because 19.79% results from not subtracting general and administrative expenses in thecalculation of before-tax profit.

Answer (D) is correct. An investment center is responsible for revenues, expenses, and invested capital. Given averageplant and equipment of $1,775 and average working capital of $625, the net investment is $2,400. Before-tax profit is$400 ($4,000 sales – $3,525 cost of goods sold – $75 general expenses). If before-tax ROI equals before-tax profit dividedby net investment, the answer is 16.67% ($400 ÷ $2,400).

[919] Gleim #: 6.13.127 -- Source: CMA 697 3-30

Answer (A) is incorrect because $9,000,000 in costs result in a residual income greater than $2,000,000.

Answer (B) is incorrect because $10,800,000 in costs would result in a residual income greater than $2,000,000.

Answer (C) is correct. Residual income is the excess of the amount of the ROI over a targeted amount equal to animputed interest charge on invested capital. If a manager has $19,000,000 of invested capital ($17,200,000 of plant andequipment + $1,800,000 of working capital), a 15% imputed interest charge equals $2,850,000. Adding $2,000,000 ofresidual income to the imputed interest results in a target profit of $4,850,000. This profit can be achieved if costs are$25,150,000 ($30,000,000 revenue – $4,850,000 profit).

Answer (D) is incorrect because $25,690,000 results from subtracting working capital from plant and equipment indetermining invested capital.

[920] Gleim #: 6.13.128 -- Source: CMA 695 3-20

Answer (A) is incorrect because, although the firm’s return on equity investment was 4%, its return on all funds investedwas 5% ($25,000 pretax operating income ÷ $500,000).

Answer (B) is correct. Return on investment is commonly calculated by dividing pretax income by total assets available.Residual income is the excess of the return on investment over a targeted amount equal to an imputed interest charge oninvested capital. The rate used is ordinarily the weighted-average cost of capital. Some companies measure managerialperformance in terms of the amount of residual income rather than the percentage return on investment. Because REB hasassets of $500,000 and a cost of capital of 6%, it must earn $30,000 on those assets to cover the cost of capital. Given thatoperating income was only $25,000, it had a negative residual income of $5,000.

Answer (C) is incorrect because ROI is commonly based on before-tax income.

Answer (D) is incorrect because $(22,000) equals the difference between profit after taxes and targeted income.

Gleim's CIA Test Prep: Part III: Business Analysis and Information TechnologyAnswer Explanations

(1312 questions)

Copyright 2008 Gleim Publications, Inc. Page 284Printed for Mamdouh Farag

Page 285: P.3 Answer Explanation

[921] Gleim #: 6.13.129 -- Source: Publisher

Answer (A) is incorrect because current cost is an attempt to remedy the theoretical deficiencies of historical cost bypresenting more accurate balance sheet values.

Answer (B) is incorrect because current disposal price is an attempt to remedy the theoretical deficiencies of historicalcost by presenting more accurate balance sheet values.

Answer (C) is correct. Historical cost creates comparability issues because returns on significantly depreciated assets maybe higher than those on newer assets that have been acquired using inflated currency. Thus, otherwise similarly situatedmanagers may report different operating results. Moreover, managers may be reluctant to replace aging assets.

Answer (D) is incorrect because present value is an attempt to remedy the theoretical deficiencies of historical cost bypresenting more accurate balance sheet values.

[922] Gleim #: 6.13.130 -- Source: Publisher

Answer (A) is incorrect because self-interest provides an incentive to maximize the measures used in performanceevaluation.

Answer (B) is correct. Effective management control requires performance measurement and feedback. This processaffects allocation of resources to organizational subunits. It also affects decisions about managers’ compensation,advancement, and future assignments. Furthermore, evaluating their performance serves to motivate managers to optimizethe measures in the performance evaluation model. However, that model may be inconsistent with the organization’smodel for managerial decision making.

Answer (C) is incorrect because a manager evaluated on the basis of annual ROI has an interest in maximizing short-termnet income, not long-term NPV.

Answer (D) is incorrect because the models should be synchronized so that the goals of the organization and the managerare congruent.

[923] Gleim #: 6.13.131 -- Source: CIA 1191 IV-17

Answer (A) is correct. The responsibility accounting report should list only the costs over which the warehousingsupervisor exercises control. The supervisor’s salary should therefore be excluded because it is controlled by thewarehouse supervisor’s superior. Moreover, only the product costs are to be considered. These exclude the shippingclerks’ wages and fringe benefits because they are period costs (shipping is a selling expense). Thus, the only product costunder the control of the warehouse supervisor is the receiving clerks’ wages (75,000) and the related fringe benefits (.3 ×75,000 = 22,500), or a total of 97,500.

Answer (B) is incorrect because the product costs controlled by the supervisor total 97,500.

Answer (C) is incorrect because the product costs controlled by the supervisor total 97,500.

Answer (D) is incorrect because the product costs controlled by the supervisor total 97,500.

Gleim's CIA Test Prep: Part III: Business Analysis and Information TechnologyAnswer Explanations

(1312 questions)

Copyright 2008 Gleim Publications, Inc. Page 285Printed for Mamdouh Farag

Page 286: P.3 Answer Explanation

[924] Gleim #: 6.13.132 -- Source: CIA 595 III-95

Answer (A) is correct. Segment reporting is an aspect of responsibility accounting. It facilitates evaluation of companymanagement and of the quality of the economic investment in particular segments.

Answer (B) is incorrect because interdependence of segments is not affected by reporting methods.

Answer (C) is incorrect because masking the effects of intersegment transfers is a disadvantage of segment reporting.

Answer (D) is incorrect because providing information to competitors is a disadvantage of segment reporting.

[925] Gleim #: 6.13.133 -- Source: CMA 1294 3-20

Answer (A) is correct. Responsibility accounting holds managers responsible only for factors under their control. For thispurpose, operations are organized into responsibility centers. Costs are classified as controllable and noncontrollable,which implies that some revenues and costs can be changed through effective management. If a manager has authority toincur costs, a responsibility accounting system will charge them to the manager’s responsibility center.

Answer (B) is incorrect because functional accounting allocates costs to functions regardless of responsibility.

Answer (C) is incorrect because reciprocal allocation is a means of allocating service department costs.

Answer (D) is incorrect because transfer price accounting is a means of charging one department for products acquiredfrom another department in the same organization.

[926] Gleim #: 6.13.134 -- Source: CIA 1190 IV-21

Answer (A) is correct. Residual income is income of an investment center, minus an imputed interest charge for investedcapital. The theory is that earning an income greater than residual income indicates that expansion is desirable. However,comparisons of investment centers based on residual income may be misleading because of differences in products,markets, costs, and local conditions.

Answer (B) is incorrect because comparisons are valid if adjustments are made for relative investment bases.

Answer (C) is incorrect because use of the same imputed interest rate provides a consistent objective against which eachinvestment can be measured.

Answer (D) is incorrect because common amounts of invested capital would eliminate a major factor causing differencesin residual income.

[927] Gleim #: 6.13.135 -- Source: CIA, adapted

Answer (A) is incorrect because both measures represent the results for a single time period.

Answer (B) is correct. Residual income concentrates on earnings in excess of the minimum desired return. With ROI, asegment may reject a project that exceeds the minimum return if the project will decrease the segment’s overall ROI. Forexample, a project that earns an ROI of 22%, which is greater than the target rate of 20%, might be rejected if thesegment is currently earning 25%, because the project will decrease the segment’s ROI. This would not occur ifperformance was measured using residual income.

Answer (C) is incorrect because the target rate for ROI is the same as the imputed interest rate used in the residualincome calculation.

Answer (D) is incorrect because average investment should be employed in both methods. At any rate, the investmentbase employed for both methods would be the same.

Gleim's CIA Test Prep: Part III: Business Analysis and Information TechnologyAnswer Explanations

(1312 questions)

Copyright 2008 Gleim Publications, Inc. Page 286Printed for Mamdouh Farag

Page 287: P.3 Answer Explanation

[928] Gleim #: 7.1.1 -- Source: CMA 1291 1-22

Answer (A) is correct. A regulatory agency may regulate some aspect of all industries or may regulate a specific industryin accordance with power delegated by the enabling legislation. Agency functions include executive, adjudicatory, andrule-making activities. Such agencies, however, may not impose taxes.

Answer (B) is incorrect because regulatory agencies have the power to issue rules and regulations.

Answer (C) is incorrect because regulatory agencies have the power to investigate violations of statutes and rules.

Answer (D) is incorrect because regulatory agencies have the power to conduct hearings and decide whether violationshave occurred.

[929] Gleim #: 7.1.2 -- Source: CMA 1289 1-29

Answer (A) is incorrect because national law applies in all parts of a country.

Answer (B) is correct. The basic purpose of the securities laws is to provide disclosure of adequate information so thatinvestors can evaluate investments. This is accomplished through reporting requirements concerning the issuance andsubsequent trading of securities. However, the government does not assess the merits of these securities.

Answer (C) is incorrect because the government does not determine the merits of securities. It evaluates whethersufficient information is provided.

Answer (D) is incorrect because the securities laws generally are not intended to influence the investment of capital inmore socially or economically beneficial ways.

[930] Gleim #: 7.1.3 -- Source: CMA 679 3-22

Answer (A) is incorrect because officers, directors and beneficial holders of 10% of the corporation’s stock may not tradeon the basis of inside information.

Answer (B) is incorrect because officers, directors and beneficial holders of 10% of the corporation’s stock may not tradeon the basis of inside information.

Answer (C) is incorrect because officers, directors and beneficial holders of 10% of the corporation’s stock may not tradeon the basis of inside information.

Answer (D) is correct. Such antifraud provisions typically prohibit any person from engaging in manipulative or deceptiveacts in the purchase or sale of any security. It prohibits trading on the basis of inside information and applies to anyonewho has not made a full disclosure of the inside information. It may apply not only to officers, directors, and shareholders,but also to tippees, i.e., those who receive inside information from insiders.

Gleim's CIA Test Prep: Part III: Business Analysis and Information TechnologyAnswer Explanations

(1312 questions)

Copyright 2008 Gleim Publications, Inc. Page 287Printed for Mamdouh Farag

Page 288: P.3 Answer Explanation

[931] Gleim #: 7.1.4 -- Source: Publisher

Answer (A) is incorrect because profit percentages are not set by antitrust laws other than to the extent that pricediscrimination is prohibited.

Answer (B) is incorrect because firms may enter into joint ventures.

Answer (C) is incorrect because patents are available to all inventors, regardless of size.

Answer (D) is correct. Antitrust laws are designed to promote more efficient allocation of resources, greater choice forconsumers, greater business opportunities, fairness in economic behavior, and avoidance of concentrated political powerresulting from economic power. Competition results in greater output and lower prices than other market structures.

[932] Gleim #: 7.1.5 -- Source: CMA 1295 1-23

Answer (A) is incorrect because quantity discounts are not prohibited.

Answer (B) is incorrect because antitrust laws usually do not apply to labor unions.

Answer (C) is incorrect because only mergers that could lead to restraint of trade are outlawed.

Answer (D) is correct. Antitrust law addresses restraints of trade. Some types of arrangements are considered automaticviolations. These violations may include price fixing, division of markets, group boycotts, and resale price maintenance.Agreeing to submit identical bids on a government contract is a form of price fixing.

[933] Gleim #: 7.1.6 -- Source: CMA 687 1-15

Answer (A) is correct. Tie-in sales involve a seller’s requirement that the buyer purchase another distinct product toobtain the first. They are usually prohibited under antitrust law unless a small company is attempting to enter a market orthe tie-in is needed to protect the firm’s goodwill (e.g., by maintaining a certain quality standard). Tie-in salesrequirements are usually illegal if the seller has enough market power to restrict competition.

Answer (B) is incorrect because tie-in sales requirements are usually illegal if the seller has enough market power torestrict competition.

Answer (C) is incorrect because tie-in sales requirements are usually illegal if the seller has enough market power torestrict competition.

Answer (D) is incorrect because tie-in sales requirements are usually illegal if the seller has enough market power torestrict competition.

[934] Gleim #: 7.1.7 -- Source: CFM Sample Q2-11

Answer (A) is incorrect because legislation addresses food labels.

Answer (B) is incorrect because product safety is addressed by legislation.

Answer (C) is correct. Statutes often provide for regulation of the matters listed in (a), (b), and (d), respectively. To date,no legislation allows the award of monetary damages to persons injured by false and misleading advertising. However,laws may prohibit unfair or deceptive acts in commerce, including false or misleading advertising.

Answer (D) is incorrect because legislation addresses truth in lending.

Gleim's CIA Test Prep: Part III: Business Analysis and Information TechnologyAnswer Explanations

(1312 questions)

Copyright 2008 Gleim Publications, Inc. Page 288Printed for Mamdouh Farag

Page 289: P.3 Answer Explanation

[935] Gleim #: 7.1.8 -- Source: CMA 690 1-5

Answer (A) is incorrect because such laws merely require disclosure. They do not regulate interest rates.

Answer (B) is incorrect because truth-in-lending laws have nothing to do with wage garnishment.

Answer (C) is incorrect because such laws do not regulate interest rates.

Answer (D) is correct. Truth-in-lending laws are disclosure statutes. They typically apply to creditors that extendconsumer credit to individual debtors. Examples of disclosures for a closed-end credit transaction, e.g., the typical carloan, include the total finance charge, annual percentage interest rate, amount financed, late charges, security interest heldby the creditor, the number and amounts of payments, due dates, and the total amount of payments.

[936] Gleim #: 7.1.9 -- Source: CMA 1291 1-25

Answer (A) is incorrect because a regulator usually does not levy fines, although it may bring suit for violations, and acourt may levy fines as a result.

Answer (B) is correct. These laws promote voluntary safety standards, develop and enforce mandatory standards, prohibitunsafe products if safety standards will not be sufficient, recall hazardous products, furnish information to consumers, andwork with local governments. Thus, the emphasis is the prevention of problems through standard setting, not thepunishment of wrongdoers.

Answer (C) is incorrect because consumer choice is lessened when the regulator keeps unsafe products off the market.

Answer (D) is incorrect because the requirements of the regulator are mandatory.

[937] Gleim #: 7.1.10 -- Source: CMA 1293 1-4

Answer (A) is correct. Social regulation concerns quality of life issues, e.g., workplace and product safety, environmentaldegradation, and fair employment practices. The abuses addressed are those that are difficult for market forces to remedy.For example, consumers may purchase products on the basis of price and quality but without regard to the environmentalimpact of their production, and unsafe working conditions may be tolerated by individuals who have few opportunities forother employment.

Answer (B) is incorrect because, although consumer and environmental groups may occasionally exercise some lobbyingpower, they are typically underfunded and would have little impact on legislatures in the absence of an obvious need forsocial regulation.

Answer (C) is incorrect because there is great difficulty in measuring both the benefits and costs of most social regulation.

Answer (D) is incorrect because no regulation is desirable, but some regulation is necessary when market forces areineffective.

Gleim's CIA Test Prep: Part III: Business Analysis and Information TechnologyAnswer Explanations

(1312 questions)

Copyright 2008 Gleim Publications, Inc. Page 289Printed for Mamdouh Farag

Page 290: P.3 Answer Explanation

[938] Gleim #: 7.1.11 -- Source: CMA 1291 1-20

Answer (A) is incorrect because pollution would not be totally eliminated. Some firms might find that a low level ofpollution is more cost beneficial than total elimination.

Answer (B) is correct. The setting of effluent standards has been criticized as inefficient. Economists prefer a sliding taxbased on the amount of effluent emitted. This method is preferable because, as effluent discharge increases, the taxincreases, providing an incentive for firms to discover new methods of controlling pollution. Rather than dictatingtechnology, the tax allows firms to seek out the technology that is the most cost effective.

Answer (C) is incorrect because the regulatory body would not be directly involved in technology.

Answer (D) is incorrect because the tax would be an indirect method of economic intervention.

[939] Gleim #: 7.1.12 -- Source: CMA 695 1-22

Answer (A) is incorrect because raising the reserve requirement and the rate at which member banks may borrow from thecentral bank reduces the money supply.

Answer (B) is correct. A central bank affects monetary policy primarily through the purchase and sale of governmentsecurities. A purchase of securities is expansionary because it increases bank reserves and the money supply. However,the sale of government securities by the central bank contracts the money supply by removing resources from the economy.Lowering the reserve requirement (the percentage of deposits that a bank must keep on hand) also expands the moneysupply by increasing the loanable funds held by banks. Similarly, lowering the the rate at which member banks mayborrow from the central bank encourages borrowing and increases the money supply.

Answer (C) is incorrect because raising the rate at which member banks may borrow from the central bank reduces themoney supply.

Answer (D) is incorrect because raising the rate at which member banks may borrow from the central bank and sellinggovernment securities reduce the money supply.

[940] Gleim #: 7.1.13 -- Source: CMA 684 1-20

Answer (A) is correct. A tight monetary policy means that little money is available for borrowing. When supply isreduced, the price increases. Thus, interest rates are increased when the money supply contracts. Because of high interestrates, the cost of investment is increased and investment is discouraged.

Answer (B) is incorrect because lower interest rates would lead to higher business investment.

Answer (C) is incorrect because increased business investment is a desirable effect.

Answer (D) is incorrect because increased business investment is a desirable effect.

[941] Gleim #: 7.1.14 -- Source: IIA, adapted

Answer (A) is correct. Selling government securities is contractional because it takes money out of circulation.

Answer (B) is incorrect because lower reserve requirements would fuel the economy because banks could lend moremoney.

Answer (C) is incorrect because lower discount rates would fuel the economy because borrowing would be encouraged.

Answer (D) is incorrect because this is fiscal policy, not monetary policy.

Gleim's CIA Test Prep: Part III: Business Analysis and Information TechnologyAnswer Explanations

(1312 questions)

Copyright 2008 Gleim Publications, Inc. Page 290Printed for Mamdouh Farag

Page 291: P.3 Answer Explanation

[942] Gleim #: 7.2.15 -- Source: CMA 1287 1-25

Answer (A) is incorrect because prices will be lower and greater quantities will be available with international trade.

Answer (B) is incorrect because variety will be increased by international trade.

Answer (C) is correct. Under the concept of comparative advantage, total world output will be maximized when eachnation specializes in the products in which it has the lowest opportunity costs, that is, a comparative advantage. Whennations specialize in what they produce most efficiently and then exchange with others, more is produced and consumedthan if each nation tries to be self-sufficient. Specialization of labor is beneficial for individuals; the same principleapplies to nations.

Answer (D) is incorrect because competition will be increased when more producers are in the market.

[943] Gleim #: 7.2.16 -- Source: CMA 1287 1-26

Answer (A) is incorrect because investment abroad has proven to be beneficial.

Answer (B) is incorrect because both nations should benefit from international trade. Under the concept of comparativeadvantage, no country will be worse off through international trade.

Answer (C) is incorrect because a country may have other advantages not possessed by countries with low labor costs.

Answer (D) is correct. Imports can raise the standard of living because more goods are available to consumers. Tariffs,quotas, and other trade restraints are undesirable because free trade will maximize the total benefit to consumersworldwide (under the concept of comparative advantage).

[944] Gleim #: 7.2.17 -- Source: CMA 688 1-21

Answer (A) is correct. The U.S. dollar is the most widely used currency in international markets today. It is consideredmuch more stable than any of the third-world currencies. Thus, many third-world countries rely on the U.S. dollar forforeign trade.

Answer (B) is incorrect because the Euro is not as heavily used as the U.S. dollar.

Answer (C) is incorrect because the Japanese yen is not as heavily used as the U.S. dollar.

Answer (D) is incorrect because the Swiss franc is not as heavily used as the U.S. dollar.

[945] Gleim #: 7.2.18 -- Source: CMA 1287 1-24

Answer (A) is correct. If exchange rates are allowed to fluctuate, the value of a particular currency will be determined inaccordance with the supply of and demand for that currency. For example, if the European Union’s exports to Japan aregreater than imports, the euro will be in great demand. Thus, the euro will be driven up in price relative to the Japaneseyen. This increase in price will discourage the Japanese from buying European goods. The decrease in Japanese purchaseswill then result, in principle, in less demand for the euro and a movement back toward equilibrium in the export/importratio.

Answer (B) is incorrect because tight money policies would not affect the balance of payments but would reduce economicactivity generally.

Answer (C) is incorrect because deficits can be overcome by less drastic and counterproductive methods than eliminatingtrade.

Answer (D) is incorrect because demand for particular products is as important as relative costs in creating trade deficits.

Gleim's CIA Test Prep: Part III: Business Analysis and Information TechnologyAnswer Explanations

(1312 questions)

Copyright 2008 Gleim Publications, Inc. Page 291Printed for Mamdouh Farag

Page 292: P.3 Answer Explanation

[946] Gleim #: 7.2.19 -- Source: CMA 1288 1-12

Answer (A) is incorrect because Taiwan does not have an absolute advantage with respect to either product.

Answer (B) is correct. Given two countries and two products, each country will always have a comparative advantagewith respect to one of the products. Each country may or may not have an absolute advantage. In this case, the U.S. has anabsolute advantage with respect to both products because with one unit of resources it can produce more soybeans (12 to6) and more chips (1,800 to 1,500) than Taiwan. Comparatively, the U.S. has an advantage with respect to soybeans.Comparative advantage is measured by determining which product can be manufactured more cheaply (for the loweropportunity cost) in comparison to the other country. For 1 ton of soybeans, the U.S. opportunity cost is 150 units (1,800 ÷12) of chips. The opportunity cost for Taiwan is 250 units (1,500 ÷ 6) of chips.

Answer (C) is incorrect because the U.S. has the comparative advantage in the production of soybeans, and Taiwan hasthe comparative advantage in the production of chips. Taiwan’s opportunity cost for the production of one unit of chips is.004 ton (6 ÷ 1,500) of soybeans. U.S. chip production has an opportunity cost of .0067 ton (12 ÷ 1,800) of soybeans.

Answer (D) is incorrect because the U.S. has the comparative advantage in the production of soybeans, and Taiwan hasthe comparative advantage in the production of chips. Taiwan’s opportunity cost for the production of one unit of chips is.004 ton (6 ÷ 1,500) of soybeans. U.S. chip production has an opportunity cost of .0067 ton (12 ÷ 1,800) of soybeans.

[947] Gleim #: 7.2.20 -- Source: CMA 1288 1-13

Answer (A) is incorrect because both countries will benefit from free trade. Specialization results in the optimal totaloutput.

Answer (B) is incorrect because the U.S. will specialize in the production of soybeans.

Answer (C) is incorrect because Taiwan will produce chips and export them to the U.S.

Answer (D) is correct. Countries should specialize in the production and export of products for which they have acomparative advantage. They should import those products for which they do not have a comparative advantage. Thus,Taiwan will specialize in the production of chips, and the U.S. will specialize in the production of soybeans.

[948] Gleim #: 7.2.21 -- Source: CMA 1288 1-14

Answer (A) is incorrect because 1 ton of soybeans for 250 chips is the opportunity cost for Taiwan.

Answer (B) is correct. Taiwan’s opportunity cost for 1 ton of soybeans is 250 units of chips. The U.S.’s opportunity costfor 1 ton of soybeans is 150 units of chips (1800 ÷ 12). Thus, the lowest price that the U.S. should charge is 150 units ofchips per ton of soybeans. Moreover, Taiwan should not pay more than 250 units of chips for imported soybeans becausethey could be made domestically for that price. Therefore, the trading price must be between 150 and 250 units of chipsper ton of soybeans if trade is to be advantageous.

Answer (C) is incorrect because the lowest price the U.S. should charge is 150 units of chips per ton of soybeans and themaximum price that Taiwan would be willing to pay is 250 units of chips per ton of soybeans.

Answer (D) is incorrect because the lowest price that the U.S. would charge is 150 units of chips per ton of soybeans.Additionally, Taiwan would not be willing to pay more than 250 units of chips per ton of soybeans.

Gleim's CIA Test Prep: Part III: Business Analysis and Information TechnologyAnswer Explanations

(1312 questions)

Copyright 2008 Gleim Publications, Inc. Page 292Printed for Mamdouh Farag

Page 293: P.3 Answer Explanation

[949] Gleim #: 7.2.22 -- Source: CMA 1287 1-28

Answer (A) is incorrect because the euro has appreciated (increased in value) relative to the dollar.

Answer (B) is correct. The dollar has declined in value relative to the euro. If an American had previously wished topurchase a European product that was priced at 10 euros, the price would have been about $10.53. After the dollar’sdecline in value, the price of the item has increased to about $11.11. Hence, imports from Europe should decrease andexports increase.

Answer (C) is incorrect because dollars will buy fewer European products.

Answer (D) is incorrect because U.S. exports should increase.

[950] Gleim #: 7.2.23 -- Source: CMA 1286 1-19

Answer (A) is correct. An overvalued exchange rate is a tax on exports because they will be overvalued in terms of theforeign currency. For example, if the true value of one euro is 5 foreign currency units (FCUs) but the exchange rate is oneeuro to 6 FCUs, the cost of goods priced in euros will include a 20% tax for holders of FCUs. It is also a subsidy toimports because the overly high exchange rate causes the price of foreign goods and services to be undervalued. In thesame example, one euro will buy 6 FCUs’ worth of imports instead of 5, a 20% subsidy.

Answer (B) is incorrect because devaluation taxes imports and subsidizes exports.

Answer (C) is incorrect because both will be affected.

Answer (D) is incorrect because both will be affected.

[951] Gleim #: 7.2.24 -- Source: CMA 688 1-22

Answer (A) is incorrect because governments have only temporary influence, if any, on the setting of exchange rates.

Answer (B) is incorrect because the International Monetary Fund has only temporary influence, if any, on the setting ofexchange rates.

Answer (C) is correct. Although currencies can be supported by various means for short periods, the primary determinantof exchange rates is the supply of and demand for the various currencies. Under current international agreements,exchange rates are allowed to “float.” During periods of extreme fluctuations, however, governments and control banksmay intervene to maintain stability in the market.

Answer (D) is incorrect because exporters and importers have only temporary influence, if any, on the setting of exchangerates.

Gleim's CIA Test Prep: Part III: Business Analysis and Information TechnologyAnswer Explanations

(1312 questions)

Copyright 2008 Gleim Publications, Inc. Page 293Printed for Mamdouh Farag

Page 294: P.3 Answer Explanation

[952] Gleim #: 7.2.25 -- Source: CIA 1195 IV-67

Answer (A) is incorrect because, if incomes in country 1 rise, the result will be a tendency for it to devalue relative to thecurrency of country 2.

Answer (B) is incorrect because, if incomes in country 1 fall, consumers in country 1 will reduce their imports. Theresulting decrease in the supply of currency 1 will result in a tendency for it to appreciate relative to the currency ofcountry 2.

Answer (C) is correct. If incomes in country 1 rise, consumers in country 1 will increase their imports from country 2.The resulting increase in the supply of currency 1 will result in a tendency for it to depreciate relative to the currency ofcountry 2.

Answer (D) is incorrect because, if incomes in country 1 remain constant, the currency of country 1 will not tend toappreciate or depreciate relative to the currency of country 2.

[953] Gleim #: 7.2.26 -- Source: CMA 1285 1-33

Answer (A) is incorrect because purchasing-power parity is achieved through floating exchange rates.

Answer (B) is incorrect because the purchasing-power parity exchange rate is a long-run measure, but the market rate mayreflect short-term or medium-term conditions.

Answer (C) is incorrect because purchasing-power parity does not affect the valuation of currency.

Answer (D) is correct. The purchasing-power parity theorem states that, in the long run, the real price of a good incountry A will equal the price of the same good in country B when the prices are expressed in a common currency andconverted at the current exchange rate (adjustments for tariffs, taxes, or transportation cost may need to be made).

[954] Gleim #: 7.2.27 -- Source: CMA 1287 1-30

Answer (A) is incorrect because the Canadian dollar has depreciated against foreign currencies.

Answer (B) is incorrect because the Canadian trade balance of payments should improve.

Answer (C) is correct. The decline in the value of the Canadian dollar reduces the prices of Canadian goods to foreignersand should increase exports. Also, foreign goods will be higher priced (in Canadian dollars) and imports from foreigncountries should decrease, thus helping the Canadian balance of payments.

Answer (D) is incorrect because Canadian imports will decline. Foreign goods will be higher priced than before.

[955] Gleim #: 7.2.28 -- Source: CMA 694 1-4

Answer (A) is correct. Exchange rates fluctuate depending upon the demand for each country’s currency. If a countryraises its interest rates, its currency will appreciate. The demand for investment at the higher interest rates will shift thedemand curve for the currency to the right. The reverse holds true for a decrease in interest rates.

Answer (B) is incorrect because the currency should increase in relative value when interest rates in the country risesharply. More investors will want to earn the higher rates of interest.

Answer (C) is incorrect because the currency should increase in relative value when interest rates in the country risesharply. More investors will want to earn the higher rates of interest.

Answer (D) is incorrect because the currency should increase in relative value when interest rates in the country risesharply. More investors will want to earn the higher rates of interest.

Gleim's CIA Test Prep: Part III: Business Analysis and Information TechnologyAnswer Explanations

(1312 questions)

Copyright 2008 Gleim Publications, Inc. Page 294Printed for Mamdouh Farag

Page 295: P.3 Answer Explanation

[956] Gleim #: 7.2.29 -- Source: CMA 695 1-24

Answer (A) is incorrect because an increase in imports drives down the value of the nation’s currency.

Answer (B) is incorrect because a high rate of inflation devalues a nation’s currency.

Answer (C) is correct. Assuming that exchange rates are allowed to fluctuate freely, a nation’s currency will appreciate ifthe demand for it is constant or increasing while supply is decreasing. For example, if the nation decreases its importsrelative to exports, less of its currency will be used to buy foreign currencies for import transactions and more of itscurrency will be demanded for export transactions. Thus, the supply of the nation’s currency available in foreign currencymarkets decreases. If the demand for the currency increases or does not change, the result is an increase in (appreciationof) the value of the currency.

Answer (D) is incorrect because lower interest rates relative to those in other countries discourage foreign investment,decreases demand for the nation’s currency, and reduces its value.

[957] Gleim #: 7.2.30 -- Source: CIA 1196 IV-74

Answer (A) is correct. If the real rates of interest are equal, the country with the higher nominal interest rate is expectedto experience a higher rate of inflation. A higher rate of inflation is associated with a devaluing currency, so the currencyof the country with the higher nominal interest rate will likely be selling at a forward discount.

Answer (B) is incorrect because the currency of Country A will be selling at a forward discount.

Answer (C) is incorrect because the spot relationship between the two currencies cannot be determined from theinformation given.

Answer (D) is incorrect because the spot relationship between the two currencies cannot be determined from theinformation given.

[958] Gleim #: 7.2.31 -- Source: CMA 1286 1-18

Answer (A) is incorrect because the combined effect of British inflation and the decline in value of the FCU would causethe FCU costs for British imports to increase.

Answer (B) is incorrect because the combined effect of British inflation and the decline in value of the FCU would causethe FCU costs for British imports to increase.

Answer (C) is incorrect because 5% is the difference between the currency depreciation and the inflation rate.

Answer (D) is correct. Assuming the original exchange rate is 1 pound to 2,000 FCUs and that British inflation is 5%, thecost in FCUs to purchase what once cost 1 pound will now be 2,100 FCUs (2,000 × 1.05). However, if the FCU alsodepreciates by 10%, that is, if the FCU is expected to be worth 90% of its current value against the pound, the exchangerate before inflation will be 1 pound to 2,222 (2,000 ÷ .9) FCUs. At this rate, 2,333 FCUs (2,222 × 1.05) will be requiredto purchase 1.05 pounds. FCU costs will thus increase by over 16.6% (333 ÷ 2,000).

Gleim's CIA Test Prep: Part III: Business Analysis and Information TechnologyAnswer Explanations

(1312 questions)

Copyright 2008 Gleim Publications, Inc. Page 295Printed for Mamdouh Farag

Page 296: P.3 Answer Explanation

[959] Gleim #: 7.2.32 -- Source: CIA 1196 IV-73

Answer (A) is correct. If the exchange rate changes from 1 LCU to 5 FCUs to 1 LCU to 5.5 FCUs, the LCU hasappreciated by 10% [(5.5 – 5) ÷ 5].

Answer (B) is incorrect because the LCU has appreciated in value. It will now purchase more FCUs.

Answer (C) is incorrect because the FCU is depreciating, not appreciating.

Answer (D) is incorrect because the FCU has depreciated by 9.09%.

[960] Gleim #: 7.2.33 -- Source: CMA 1285 1-34

Answer (A) is correct. The importer should buy pounds now. If the euro depreciates against the pound in the next 90days, the gain on the forward exchange contract would offset the loss from having to pay more euros to satisfy the liability.

Answer (B) is incorrect because selling pounds would compound the risk of loss for someone who has incurred a liability.However, it would be an appropriate hedge of a receivable denominated in pounds.

Answer (C) is incorrect because the importer needs pounds, not euros.

Answer (D) is incorrect because, although buying pounds might be equivalent to selling euros for pounds, this is not thebest answer. This choice does not state what is received for the euros.

[961] Gleim #: 7.2.34 -- Source: CMA 1288 1-18

Answer (A) is incorrect because inflation affects exchange rates by diminishing a currency’s purchasing power.

Answer (B) is correct. Because Ruritania has experienced the greater inflation, its currency should depreciate in relationto Canada’s. For example, if Canada trades 100 units of a product to Ruritania for a preinflation unit price of $100 (thedomestic price in Canada), and Ruritania pays with 10,000 units of a product that sells domestically for a unit price of10,000 preinflation RCUs, the exchange rate without regard to inflation is 100 RCUs per $1 (10,000 RCUs ÷ $100).Allowing for the inflation, the 100 units of the Canadian product would sell for $105. The 10,000 units of the Ruritanianproduct would sell for 11,000 RCUs. Thus, the new exchange rate will be 104.76 RCUs per $1 (11,000 RCUs ÷ $105),and the price of the Canadian dollar will increase by 4.76% (rounded to 5%).

Answer (C) is incorrect because the Canadian currency will appreciate relative to Ruritania’s. Canadian inflation waslower.

Answer (D) is incorrect because the Canadian currency will appreciate relative to Ruritania’s. Canadian inflation waslower.

Gleim's CIA Test Prep: Part III: Business Analysis and Information TechnologyAnswer Explanations

(1312 questions)

Copyright 2008 Gleim Publications, Inc. Page 296Printed for Mamdouh Farag

Page 297: P.3 Answer Explanation

[962] Gleim #: 7.2.35 -- Source: CMA 1288 1-17

Answer (A) is incorrect because £737 results from considering only the effect of the 5% decline in the pound.

Answer (B) is incorrect because £721 results from considering only the effect of the 3% U.S. inflation rate.

Answer (C) is correct. Two factors are to be considered: (1) the 3% U.S. inflation rate and (2) the 5% decline in thepound versus the dollar. Assuming that the original exchange rate was $1.20 to £1, £700 was equivalent to $840. Given5% depreciation of the pound, the new exchange rate is $1.14 to £1 ($1.20 × .95). After U.S. inflation of 3% ($840 × 1.03= $865.20), the new price is about £759 ($865.20 ÷ $1.14). Accordingly, the price has increased by about 8.4% [(£759 –£700) ÷ £700].

Answer (D) is incorrect because the effects of both the 3% U.S. inflation rate and the 5% decline in the pound must beconsidered.

[963] Gleim #: 7.2.36 -- Source: CIA 1196 IV-64

Answer (A) is incorrect because a forward market sale of foreign currency is appropriate to hedge a receivabledenominated in a foreign currency.

Answer (B) is incorrect because waiting to buy the currency in 60 days does not eliminate the risk of an adverse exchange-rate movement.

Answer (C) is correct. The company can arrange to purchase the foreign currency today rather than in 60 days by buyingthe currency in the forward market. This hedging transaction will eliminate the exchange-rate risk associated with thetrade payable.

Answer (D) is incorrect because this strategy would be comparable to a future sale of the foreign currency at a rate knowntoday, which would not provide the currency needed to pay the invoice. However, the opposite strategy would be aneffective money market hedge. If the company converted domestic currency to foreign currency in the spot market todayand invested in a foreign bank deposit or treasury bill, it could then use the proceeds from the foreign investment to paythe invoice in 60 days.

[964] Gleim #: 7.2.37 -- Source: Publisher

Answer (A) is incorrect because the world began operating on a gold standard is 1870. In 1944, the system was changed toa modified gold standard.

Answer (B) is correct. The Bretton Woods Agreement abolished the gold standard that had existed since 1870 in favor ofa modified gold standard. Under the modified gold standard, the U.S. dollar was pegged to gold, and other currencieswere then pegged to the dollar.

Answer (C) is incorrect because managing floating exchange rates were not adopted until the Jamaica Agreement of 1976.

Answer (D) is incorrect because floating exchange rates did not occur until 1971.

Gleim's CIA Test Prep: Part III: Business Analysis and Information TechnologyAnswer Explanations

(1312 questions)

Copyright 2008 Gleim Publications, Inc. Page 297Printed for Mamdouh Farag

Page 298: P.3 Answer Explanation

[965] Gleim #: 7.2.38 -- Source: Publisher

Answer (A) is incorrect because they are all causes of higher costs of capital in foreign projects.

Answer (B) is incorrect because they are all causes of higher costs of capital in foreign projects.

Answer (C) is incorrect because they are all causes of higher costs of capital in foreign projects.

Answer (D) is correct. The cost of capital is typically higher for foreign projects for a variety of reasons, includingexchange-rate risk, political risk, tax issues, and limitations on sources of financing that often require a certain percentageof domestic ownership or impose other protectionist measures. Trigger pricing of currency option premiums is not a cause.Currency options are a means of managing exchange-rate risk. If the firm has obligations in a currency that willappreciate, it buys call options. If it has receivables in a currency that will depreciate, it buys put options. The premiumfor a currency option is its price. However, a firm may enter into an arrangement with a bank that makes the premiumconditional. For example, for a put option no premium may be charged unless the value of the currency rises to a specifiedtrigger price.

[966] Gleim #: 7.2.39 -- Source: CMA 688 1-30

Answer (A) is incorrect because, although most exchanges quote the price of gold in U.S. dollars, the dollar’s value is notlinked to that of gold.

Answer (B) is incorrect because floating exchange rates have existed since about 1973. Tying currency values to a goldstandard, in effect, fixes exchange rates.

Answer (C) is correct. Gold has no special role in the modern international monetary system. The present system is basedupon managed floating currency exchange rates. Consequently, gold is treated as a commodity, the price of which dependsupon supply and demand.

Answer (D) is incorrect because the only reserves of the IMF are international currencies.

[967] Gleim #: 7.2.40 -- Source: CIA 594 IV-64

Answer (A) is incorrect because licensing requirements limit exports, e.g., of militarily sensitive technology.

Answer (B) is correct. Tariffs are excise taxes on imported goods imposed either to generate revenue or protect domesticproducers. Thus, consumption taxes on imported goods are tariffs.

Answer (C) is incorrect because unreasonable standards pertaining to product quality and safety are nontariff tradebarriers.

Answer (D) is incorrect because domestic content rules require that a portion of an imported good be made in theimporting country.

Gleim's CIA Test Prep: Part III: Business Analysis and Information TechnologyAnswer Explanations

(1312 questions)

Copyright 2008 Gleim Publications, Inc. Page 298Printed for Mamdouh Farag

Page 299: P.3 Answer Explanation

[968] Gleim #: 7.2.41 -- Source: CIA 594 IV-65

Answer (A) is incorrect because a protective tariff can only increase the domestic price of the imported item.

Answer (B) is correct. A protective tariff adds to the purchase price of imported goods. If an imported good’s sales priceis higher than a comparable, less expensive domestic good, consumers will purchase the domestic good. Thus, the directeffect of imposing a protective tariff on an imported good is lower domestic consumption.

Answer (C) is incorrect because, as the imported item’s domestic price increases, demand for domestic goods willincrease. Thus, domestic production will increase, not decrease.

Answer (D) is incorrect because, as the imported item’s domestic price increases, demand for the item decreases. Lowersales revenues will result.

[969] Gleim #: 7.2.42 -- Source: CIA 1195 IV-68

Answer (A) is incorrect because domestic producers will sell more, and domestic consumers will consume less, as a resultof the imposition of a protective tariff.

Answer (B) is correct. Domestic producers are not subject to the tariff and will therefore have a price advantage overtheir foreign competitors. However, absent such competition, the domestic price of the item will be higher. Domesticproducers will sell more at a higher price, and domestic consumers will consume less following the price increase.

Answer (C) is incorrect because domestic producers will sell more, and domestic consumers will consume less, as a resultof the imposition of a protective tariff.

Answer (D) is incorrect because domestic producers will sell more, and domestic consumers will consume less, as a resultof the imposition of a protective tariff.

[970] Gleim #: 7.2.43 -- Source: CMA 1287 1-27

Answer (A) is incorrect because trade restricts full inflation since incentives to charge lower prices for domestic productsare removed.

Answer (B) is incorrect because domestic industries support protection from imports to reduce competition.

Answer (C) is incorrect because consumption is reduced owing to higher prices and reduced supply.

Answer (D) is correct. Tariffs lead to higher prices on imported products. Similarly, the imposition of quotas leads tohigher prices through an artificial limitation on supply.

[971] Gleim #: 7.2.44 -- Source: CMA 695 1-23

Answer (A) is incorrect because domestic producers of export goods are not benefitted. Indeed, they may be harmed byretaliatory tariffs.

Answer (B) is correct. Despite the advantages of free trade, nations often levy tariffs to discourage the importation ofcertain products. A tariff is a tax on imports intended to protect a domestic producer from foreign competition. Forinstance, a tariff on imported autos benefits domestic auto manufacturers because it is an additional cost imposed ondomestic consumers of such products. The disadvantages of the tariff are that it may protect an inefficient domesticproducer and increase prices paid by domestic consumers.

Answer (C) is incorrect because domestic consumers must pay higher prices for imported goods.

Answer (D) is incorrect because the foreign producers will be forced to bear an additional cost.

Gleim's CIA Test Prep: Part III: Business Analysis and Information TechnologyAnswer Explanations

(1312 questions)

Copyright 2008 Gleim Publications, Inc. Page 299Printed for Mamdouh Farag

Page 300: P.3 Answer Explanation

[972] Gleim #: 7.2.45 -- Source: CIA 1196 IV-78

Answer (A) is incorrect because the inclusion of the tariff increases the domestic price.

Answer (B) is incorrect because foreign sales in the domestic market decline from ad to bc.

Answer (C) is incorrect because domestic production increases from Oa to Ob.

Answer (D) is correct. Without the tariff, domestic production is determined by the intersection of the P w line with thedomestic supply curve at the quantity Oa. Domestic production increases from Oa to Ob as a result of the introduction ofthe tariff. Supply intersects the P t line at a higher price and at a greater domestic quantity, Ob.

[973] Gleim #: 7.2.46 -- Source: CMA 1285 1-28

Answer (A) is incorrect because foreign investment may be welcomed under the appropriate conditions.

Answer (B) is incorrect because the EU provides incentives to trade with other members, not nonmembers.

Answer (C) is incorrect because currencies are not affected by bloc membership.

Answer (D) is correct. A trading bloc provides trading incentives to member nations and discriminates againstnonmember nations. For example, the European Union calls for abolition of internal tariffs and import quotas, freemovement of capital and labor within the market, and implementation of common policies for the member nations.However, the EU also imposed a common system of tariffs on goods of nonmember nations.

[974] Gleim #: 7.2.47 -- Source: Publisher

Answer (A) is incorrect because export jobs may decline due to retaliation by other countries.

Answer (B) is incorrect because consumers will have to pay higher prices for the protected domestic goods and may haveto pay higher prices for foreign goods as well.

Answer (C) is incorrect because consumers will have to pay higher taxes. Prices for government purchases of protectedgoods will be higher.

Answer (D) is correct. Protectionism in the form of import restrictions can lead to a variety of economic and social costs,including higher prices to consumers for both domestic and imported goods, higher taxes, and retaliation by othercountries.

Gleim's CIA Test Prep: Part III: Business Analysis and Information TechnologyAnswer Explanations

(1312 questions)

Copyright 2008 Gleim Publications, Inc. Page 300Printed for Mamdouh Farag

Page 301: P.3 Answer Explanation

[975] Gleim #: 7.2.48 -- Source: CIA 595 IV-64

Answer (A) is incorrect because a tariff is a tax levied by a foreign government against certain imported products. A firmexporting to that country must accept lower profits, absorbing the tariff, or increase selling prices in the foreign country tocompensate. The tariff reduces profitability or competitiveness in the foreign market but does not exclude the firm fromexporting to that country.

Answer (B) is incorrect because a quota is a limit on the amount of goods that an importing country will accept in certainproduct categories. The effect of a quota is to restrict the quantity the firm can export to that country but not to exclude thefirm from selling in that market. The effect on revenues and profitability depends on market conditions in that country.

Answer (C) is correct. An embargo is a total ban on some kinds of imports. It is an extreme form of the import quota.Embargoes have the effect of totally excluding the exporting firm from selling in that country and are the most restrictivetype of import/export law.

Answer (D) is incorrect because exchange controls limit the amount of foreign exchange that can be transacted or theexchange rate against other currencies. These controls limit the ability of a firm selling in the country to repatriate itsexport earnings but do not exclude the firm from selling in that market.

[976] Gleim #: 7.2.49 -- Source: Publisher

Answer (A) is incorrect because these are not economic rationales for government intervention in trade.

Answer (B) is correct. The infant-industry argument contends that protective tariffs are needed to allow new domesticindustries to become established. Once such industries reach a maturity stage in their life cycles, the tariffs cansupposedly be removed.

Answer (C) is incorrect because these are not economic rationales for government intervention in trade.

Answer (D) is incorrect because these are not economic rationales for government intervention in trade.

[977] Gleim #: 7.2.50 -- Source: Publisher

Answer (A) is incorrect because the currency must be obtained, not the right to make an import.

Answer (B) is incorrect because countertrade is not a nontariff barrier, although it can sometimes be used to counterexchange controls. Countertrade is simply a barter arrangement.

Answer (C) is incorrect because the requirement to obtain currency is not a voluntary restraint.

Answer (D) is correct. Exchange controls limit foreign currency transactions and set exchange rates. The purpose is tolimit the ability of a firm to pay domestic currency to foreigners.

[978] Gleim #: 7.2.51 -- Source: Publisher

Answer (A) is incorrect because nothing indicates that weapons production is an infant industry.

Answer (B) is correct. Providing subsidies to military weapons manufacturers is an example of the essential-industryargument. The government protects essential domestic industries during peacetime so that a country is not dependent onforeign sources of supply during war.

Answer (C) is incorrect because production of military weapons is an essential industry.

Answer (D) is incorrect because protecting weapons manufacturers has nothing to do with protecting the national identity.

Gleim's CIA Test Prep: Part III: Business Analysis and Information TechnologyAnswer Explanations

(1312 questions)

Copyright 2008 Gleim Publications, Inc. Page 301Printed for Mamdouh Farag

Page 302: P.3 Answer Explanation

[979] Gleim #: 7.2.52 -- Source: Publisher

Answer (A) is incorrect because OPEC is an international oil cartel that controls oil production and prices.

Answer (B) is incorrect because NAFTA is a free-trade agreement among the USA, Canada, and Mexico.

Answer (C) is correct. The World Bank was created to provide credit for development purposes to underdeveloped anddeveloping countries.

Answer (D) is incorrect because the IMF manages a pool of currency from which participating countries can draw duringshort-term balance-of-payments difficulties.

[980] Gleim #: 7.2.53 -- Source: Publisher

Answer (A) is incorrect because transfer prices charged to foreign subsidiaries are often quite different from those chargeddomestic subsidiaries.

Answer (B) is incorrect because the existence of tariffs in the foreign country would necessitate that a lower transfer pricebe charged to avoid a high tariff.

Answer (C) is correct. Transfer pricing is an important aspect of the tax calculation for multinational corporations thattransfer inventories between branches in different countries. Transfer prices charged to foreign subsidiaries may differsubstantially from those charged to domestic subsidiaries for a variety of reasons. Limitations on taking profits out of aforeign country can be avoided by charging the foreign subsidiary a higher transfer price so that little or no profit exists tobe repatriated.

Answer (D) is incorrect because currency restrictions may limit transfer of profits to foreign vendors.

[981] Gleim #: 7.2.54 -- Source: CMA 687 1-21

Answer (A) is incorrect because the WTO is a worldwide agreement concerning trade barriers, not exchange rates. Today,moreover, exchange rates are not pegged (fixed) but are allowed to float.

Answer (B) is incorrect because the IMF was founded in 1944 to stabilize exchange rates.

Answer (C) is correct. International trade agreements provide regulatory authority for businesses in international trade.The WTO, which was established on January 1, 1995, is the product of the Uruguay Round of international tradenegotiations. It is a permanent body with a secretariat based in Geneva, Switzerland. The WTO Agreement is a permanentset of commitments by more than 120 nations designed to prohibit trade discrimination among member nations andbetween imported and domestic products.

Answer (D) is incorrect because the WTO is a worldwide agreement concerning trade barriers, not exchange rates.

Gleim's CIA Test Prep: Part III: Business Analysis and Information TechnologyAnswer Explanations

(1312 questions)

Copyright 2008 Gleim Publications, Inc. Page 302Printed for Mamdouh Farag

Page 303: P.3 Answer Explanation

[982] Gleim #: 7.2.55 -- Source: CMA 680 1-17

Answer (A) is incorrect because the value of the domestic currency is not formally tied to gold. While there may be a long-term relationship between gold and the value of the domestic currency, there are often inverse (or random) short-termfluctuations.

Answer (B) is incorrect because an exchange rate set by the government is called a fixed exchange rate. The oldinternational monetary system which used fixed exchange rates collapsed because of its inefficient handling of currencyprices.

Answer (C) is incorrect because the International Monetary Fund has little effect on the valuation of currencies.

Answer (D) is correct. Exchange rates are determined by the forces of supply and demand on the exchange markets.Often other forces try to intervene in this process of exchange rate determination, but these reflect only short-run policies.An example of this type of policy would be government or central bank intervention in the international money markets.

[983] Gleim #: 7.2.56 -- Source: CMA 680 1-20

Answer (A) is incorrect because this would raise the real cost to the company.

Answer (B) is incorrect because this would not maximize the amount of domestic currency the company will have.

Answer (C) is incorrect because this would not maximize the amount of domestic currency the company will have.

Answer (D) is correct. The proper action would be to increase collections and decrease payments. Collections should bemade quickly and converted into domestic currency to sustain the increase in their value as the domestic currencyappreciates. Decreasing payments would be profitable because, as the company exchanges domestic currency for foreigncurrency at a later date, it will receive more of the foreign currency, thus lowering its real cost.

[984] Gleim #: 7.2.57 -- Source: CMA 1282 1-12

Answer (A) is incorrect because a clean float system does not exist owing to central bank intervention.

Answer (B) is correct. Today’s international monetary system usually permits exchange rates to float freely. However,central banks occasionally intervene to avoid large fluctuations. Accordingly, the system is called a managed or dirty floatsystem.

Answer (C) is incorrect because as a result of the floating exchange rate system, rates are not stable.

Answer (D) is incorrect because the gold-based system was abandoned in 1973.

[985] Gleim #: 7.2.58 -- Source: CMA 1282 1-13

Answer (A) is correct. If a currency is overvalued relative to a foreign currency, it will purchase more of that currency,thereby encouraging imports of foreign goods. Similarly, exports are discouraged because the foreign currency willpurchase less of the overvalued currency. Therefore, an overvalued currency operates as a tax on exports and a subsidy toimports.

Answer (B) is incorrect because the effect of an undervalued currency is a tax on exports and a subsidy to imports.

Answer (C) is incorrect because overvaluation does not operate as a tax on imports.

Answer (D) is incorrect because overvaluation does not operate as a subsidy to exports.

Gleim's CIA Test Prep: Part III: Business Analysis and Information TechnologyAnswer Explanations

(1312 questions)

Copyright 2008 Gleim Publications, Inc. Page 303Printed for Mamdouh Farag

Page 304: P.3 Answer Explanation

[986] Gleim #: 7.2.59 -- Source: CMA 1285 1-31

Answer (A) is incorrect because foreign investments in the domestic country is a factor in the balance of payments but nottrade.

Answer (B) is incorrect because domestic investments in foreign countries is a factor in the balance of payments but nottrade.

Answer (C) is incorrect because exports increase the balance of trade.

Answer (D) is correct. The balance of payments embraces all payments made by one nation to another, including capitalmovements. The balance of trade is the difference between imports and exports of goods and services over a given period.In T-account form, exports are credits and imports debits. Assuming that a credit balance reflects a positive balance oftrade, imports will decrease a positive balance while exports will increase it.

[987] Gleim #: 7.2.60 -- Source: CMA 1285 1-35

Answer (A) is incorrect because the World Trade Organization (WTO) is an attempt by the signatory nations to reducetariffs and import quotas.

Answer (B) is incorrect because the World Bank provides credits for development purposes.

Answer (C) is incorrect because the U.S. Food for Peace program permits other countries to purchase U.S. farm productswith other currencies.

Answer (D) is correct. The resources of the IMF consist of a pool of currency from which short-term loans can be made tomember nations who are experiencing temporary balance of payments problems. Special drawing rights (SDRs) have alsobeen created on the books of IMF to serve as an additional reserve for member nations to use in settling accounts.

[988] Gleim #: 7.2.61 -- Source: CMA 686 1-23

Answer (A) is incorrect because efficient trade is not meaningful in this context.

Answer (B) is incorrect because diminishing returns is not meaningful in this context.

Answer (C) is incorrect because relative competition is not meaningful in this context.

Answer (D) is correct. The doctrine of comparative advantage relates to comparative costs within one country. It holdsthat a country should produce those products in which it has a comparative advantage, not necessarily those products inwhich it has an absolute advantage. The doctrine suggests that a country should produce those products for which thegreatest efficiencies are attainable even if it could also produce other goods more efficiently than another nation. In thelong run, importing a product in which a country has an absolute advantage but not a comparative advantage will result inan overall increase in global production.

Gleim's CIA Test Prep: Part III: Business Analysis and Information TechnologyAnswer Explanations

(1312 questions)

Copyright 2008 Gleim Publications, Inc. Page 304Printed for Mamdouh Farag

Page 305: P.3 Answer Explanation

[989] Gleim #: 7.2.62 -- Source: CMA 1286 1-16

Answer (A) is incorrect because the balance in the current account contains financial balances, but the balance of trade isconcerned only with the balance of transactions in goods and services.

Answer (B) is incorrect because the capital account records net capital movements, not the balance of trade.

Answer (C) is incorrect because the balance of all international transactions includes items that are not goods or services,such as net capital movements, government transactions, and remittances.

Answer (D) is correct. The balance of payments embraces all payments made by one nation to another, including capitalmovements. The balance of trade is the difference between imports and exports of goods and services over a given period.In T-account form, exports are credits and imports debits. Assuming that a credit balance reflects a positive balance oftrade, imports will decrease a positive balance while exports will increase it.

[990] Gleim #: 7.2.63 -- Source: CMA 1286 1-21

Answer (A) is incorrect because Japan and Germany should specialize in producing the product which they have acomparative advantage.

Answer (B) is correct. Japan has a comparative advantage in radio production because its cost of producing radios is asmaller fraction of its cost of producing TV sets (2/4 = .5) than is true for Germany (its fraction is 3/5 = .6). Germany hasa comparative advantage with regard to TV set production because its costs of producing TV sets (5/3 = 1.67) is a smallerfraction of its cost of producing radios than is true for Japan (its fraction is 4/2 = 2). Under the theory of comparativeadvantage, Japan should manufacture radios, Germany should make TV sets, and both should trade for what they do notproduce.

Answer (C) is incorrect because Japan and Germany should specialize in producing the product which they have acomparative advantage.

Answer (D) is incorrect because Japan and Germany should specialize in producing the product which they have acomparative advantage.

[991] Gleim #: 7.2.64 -- Source: CMA 1293 1-27

Answer (A) is incorrect because the balance of trade is the difference between imports and exports of goods.

Answer (B) is incorrect because beyond some level of output, returns diminish as more and more units of an input areadded to the production process.

Answer (C) is incorrect because relative competition is not a term relevant to international economics.

Answer (D) is correct. The relevant concept is comparative advantage, which compares the costs of inputs within a singlecountry. In contrast, the concept of absolute advantage compares the costs of inputs between countries. It is possible that acountry might have an absolute advantage with respect to every product, but comparative advantage is different fromabsolute advantage. A particular nation can have a comparative advantage even though it does not have an absoluteadvantage. For example, assume that Country A can produce Item X for $100 and Item Y for $200 and that Country B canproduce Item X for $50 and Item Y for $150. B has an absolute advantage in the production of both products; however, Bhas a comparative advantage in producing Item X (50 ÷ 100, or 50% of the A cost, compared with 150 ÷ 200, or 75% ofthe A cost for Item Y). A has a comparative advantage in producing Item Y (200 ÷ 150, or 133% of the B cost, versus 100÷ 50, or 200% for Item X). A nation will benefit by exporting goods in which it has a comparative advantage andimporting goods in which it does not have a comparative advantage. Total output will be maximized when each nationspecializes in the products in which it has the greatest comparative advantage or the least comparative disadvantage.

Gleim's CIA Test Prep: Part III: Business Analysis and Information TechnologyAnswer Explanations

(1312 questions)

Copyright 2008 Gleim Publications, Inc. Page 305Printed for Mamdouh Farag

Page 306: P.3 Answer Explanation

[992] Gleim #: 7.2.65 -- Source: CIA 1193 IV-68

Answer (A) is incorrect because military self-sufficiency is an argument for increasing trade barriers.

Answer (B) is incorrect because diversification for stability is an argument for increasing trade barriers by promotingindustrial diversification and less dependence on other nations for certain products.

Answer (C) is incorrect because protection of infant industries is an argument for increasing trade barriers for the purposeof allowing new domestic industries to establish themselves.

Answer (D) is correct. The general effect of free trade would be to maximize world output because resources in eachcountry would be deployed most efficiently according to the principle of comparative advantage. Comparative advantagemeans that a country can produce a greater output of certain goods for a given level of input than other goods. Thus, thatcountry should specialize in and export the goods it can produce most efficiently. Total world output will increase in thesecircumstances.

[993] Gleim #: 7.2.66 -- Source: CIA 1190 IV-58

Answer (A) is correct. The returns on the stock are presumably paid in foreign currency. Hence, the change in the valueof the foreign currency relative to the domestic currency does not affect the foreign company’s return. However, theweakening of the foreign currency reduces the amount of domestic currency it will buy, and the domestic company’s returnin domestic currency is correspondingly reduced.

Answer (B) is incorrect because the return to the domestic company is adversely affected by the exchange rate movement.

Answer (C) is incorrect because the return to the domestic company is adversely affected by the exchange rate movement.

Answer (D) is incorrect because the return to the domestic company was directly and adversely affected by the exchangerate movement in the short-run, but the return to the foreign company was not.

[994] Gleim #: 7.2.67 -- Source: CMA 694 1-5

Answer (A) is incorrect because world output will be maximized if each country specializes in those products in whichthey have a comparative advantage.

Answer (B) is incorrect because comparative advantage, not absolute advantage, determines the products in which acountry should specialize.

Answer (C) is correct. The concept of comparative advantage compares costs within a single country. It is the ability ofone nation to produce a product at a relatively lower opportunity cost (benefits forgone) than another nation. Absoluteadvantage compares the costs of inputs between countries. One country might have an absolute advantage with respect toevery product. However, total output will be maximized when each nation specializes in the products in which it has thelowest opportunity costs, that is, a comparative advantage.

Answer (D) is incorrect because given a reciprocal absolute advantage, a corresponding comparative advantage will exist.

Gleim's CIA Test Prep: Part III: Business Analysis and Information TechnologyAnswer Explanations

(1312 questions)

Copyright 2008 Gleim Publications, Inc. Page 306Printed for Mamdouh Farag

Page 307: P.3 Answer Explanation

[995] Gleim #: 7.2.68 -- Source: CIA 595 IV 68

Answer (A) is incorrect because, if selling prices are identical, the tax-minimizing strategy involves maximizing sales incountry B. However, to report the highest possible profits in the lower tax country, input costs must be minimized. Thetransfer price paid to the production facility in country A must therefore be set as low as possible.

Answer (B) is correct. The tax-minimizing strategy is to minimize taxable income where tax rates are high and tomaximize taxable income where tax rates are low. Consequently, the company should sell more in country B but set a lowtransfer price. This dual strategy minimizes sales and profits in country A, minimizes cost of sales in country B, andmaximizes sales and profits in country B.

Answer (C) is incorrect because a strategy of minimizing sales in country B and maximizing reported cost of goods sold incountry B would result in the lowest reported profit in the lower tax country, thereby maximizing taxes paid.

Answer (D) is incorrect because the tax-minimizing strategy involves maximizing sales revenue in the lower tax country,not minimizing it.

[996] Gleim #: 7.2.69 -- Source: IIA, adapted

Answer (A) is incorrect because unemployment and productivity rates will decline.

Answer (B) is incorrect because jobs in the home country are saved, lowering unemployment.

Answer (C) is incorrect because productivity rates decline due to the protection of inefficient industry.

Answer (D) is correct. With trade quotas, home jobs will be saved; hence, unemployment will decline. Since jobs will besaved for inefficient industries (less efficient than foreign competitors), productivity rates will decline because they willnot be specializing in those goods with which they have a comparative advantage.

[997] Gleim #: 7.2.70 -- Source: IIA, adapted

Answer (A) is incorrect because export opportunities apply to domestic products.

Answer (B) is correct. Revenue tariffs are usually applied to products that are not produced domestically. Their purposeis to provide the government with tax revenue.

Answer (C) is incorrect because import quotas are designed to restrict the amount of a commodity that can be imported ina period of time.

Answer (D) is incorrect because voluntary export restrictions, which have the same effect as import quotas, encourageforeign firms to limit their exports to a particular country.

[998] Gleim #: 7.2.71 -- Source: IIA, adapted

Answer (A) is correct. A value-added tax is collected on the basis of the value created by the firm. This is measured asthe difference between the value of its outputs and its inputs.

Answer (B) is incorrect because this is a description of how to calculate capital gains tax.

Answer (C) is incorrect because this is a description of an internal transfer price.

Answer (D) is incorrect because this is a description of how to calculate income tax.

Gleim's CIA Test Prep: Part III: Business Analysis and Information TechnologyAnswer Explanations

(1312 questions)

Copyright 2008 Gleim Publications, Inc. Page 307Printed for Mamdouh Farag

Page 308: P.3 Answer Explanation

[999] Gleim #: 7.3.72 -- Source: CIA 596 IV-74

Answer (A) is incorrect because, under a progressive tax system, both the amount of tax and the percentage of incomepaid in tax (the average tax rate) rise as income increases. In the case described, the individual pays a higher amount oftax but a lower percentage of income in tax.

Answer (B) is correct. The average tax rate of the individual has decreased from 34.8% (8,000 ÷ 23,000) to 33.3%(10,000 ÷ 30,000). Under a regressive tax system, the average tax rate falls as income rises, although the amount of taxpaid may rise.

Answer (C) is incorrect because marginal is not a type of tax system but a type of tax rate. The marginal tax rate is the taxrate paid on incremental income.

Answer (D) is incorrect because, under a proportional income tax system, the average tax rate is the same for all levels ofincome. The average tax rate of this individual falls as income rises.

[1000] Gleim #: 7.3.73 -- Source: CIA 1195 IV-70

Answer (A) is incorrect because general sales taxes tend to be regressive. The lower the taxpayer’s income, the higher theproportion that is usually paid in sales taxes, which are collected at a flat rate per dollar. Low-income taxpayers areunable to save as high a portion of their income as high-income taxpayers. Thus, the latter are less exposed to generalsales taxes because they avoid the tax on the amount saved.

Answer (B) is correct. Property taxes tend to be regressive. Taxpayers with lower incomes must pay a higher portion oftheir incomes for necessities, such as housing.

Answer (C) is incorrect because personal income taxes tend to be progressive. Higher tax rates are charged on higherincomes.

Answer (D) is incorrect because personal income taxes tend to be progressive. Higher tax rates are charged on higherincomes.

[1001] Gleim #: 7.3.74 -- Source: CIA 595 IV-70

Answer (A) is correct. The marginal tax rate is the tax applicable to the last unit of income, whereas the average tax rateis the total tax paid divided by taxable income. In a progressive tax system, higher incomes attract higher tax rates, so themarginal tax rate paid on the last unit of income exceeds the average tax rate.

Answer (B) is incorrect because, in a progressive tax system, the tax rate varies with the level of income, so the marginaland average tax rates differ.

Answer (C) is incorrect because, in a regressive tax system, the tax rate varies with the level of income, so the marginaland average tax rates differ.

Answer (D) is incorrect because, in a regressive tax system, the marginal tax rate is less than, not greater than, theaverage tax rate.

Gleim's CIA Test Prep: Part III: Business Analysis and Information TechnologyAnswer Explanations

(1312 questions)

Copyright 2008 Gleim Publications, Inc. Page 308Printed for Mamdouh Farag

Page 309: P.3 Answer Explanation

[1002] Gleim #: 7.3.75 -- Source: CIA 596 IV-77

Answer (A) is correct. A value-added tax (VAT) is collected on the basis of the value created by the firm. This tax ismeasured by the difference between the value of the firm’s sales and the value of its purchases. A VAT is in effect a retailsales tax. Because a consumer can avoid the tax by not purchasing, a VAT encourages saving and discouragesconsumption.

Answer (B) is incorrect because the difference between the selling price of a real estate property and the amount the firmoriginally paid for the property is a capital gain.

Answer (C) is incorrect because the value of a firm’s sales to related companies is the internal transfer price.

Answer (D) is incorrect because the profit earned on a firm’s sales is subject to the income tax.

[1003] Gleim #: 7.3.76 -- Source: CIA 1195 IV-69

Answer (A) is incorrect because, if only deductions and exemptions are indexed, real taxes will still increase withinflation as taxpayers experience bracket creep.

Answer (B) is incorrect because, if only tax brackets are adjusted, deductions and exemptions will have less real value.Real taxes will rise.

Answer (C) is correct. If tax brackets are not adjusted for inflation, increases in nominal but not real income may pushtaxpayers into tax brackets with higher (progressive) rates. Additionally, deductions and exemptions must be indexed sothat the real incidence of taxation is not increased by inflation. Increases in nominal but not real income will result inhigher real taxable income if deductions and exemptions have not been adjusted for inflation.

Answer (D) is incorrect because indexing does not result in nominal tax receipts growing more slowly than the rate ofinflation. Rather, indexing maintains the same real rate of taxation by ensuring that nominal tax receipts keep pace withinflation.

[1004] Gleim #: 7.3.77 -- Source: CIA 1196 IV-76

Answer (A) is incorrect because 437,500 is 25% of the earnings before tax.

Answer (B) is incorrect because 462,500 deducts labor costs as well as purchased inputs.

Answer (C) is correct. The value added is the difference between the value of the output and the value of the purchasedinputs. Value-added tax payable equals the value-added tax rate times value added, or 587,500 [(2,500,000 – 150,000) ×.25].

Answer (D) is incorrect because 625,000 is the value-added tax rate multiplied by the output of the firm.

Gleim's CIA Test Prep: Part III: Business Analysis and Information TechnologyAnswer Explanations

(1312 questions)

Copyright 2008 Gleim Publications, Inc. Page 309Printed for Mamdouh Farag

Page 310: P.3 Answer Explanation

[1005] Gleim #: 7.3.78 -- Source: CIA 594 IV-70

Answer (A) is incorrect because progressive taxes, for which the average tax rate rises as income rises, take both a largerpercentage of income and a larger absolute amount of income as income rises.

Answer (B) is incorrect because proportional taxes, for which the average tax rate is constant for all income levels, alwaystake a larger absolute amount of income as income rises.

Answer (C) is correct. Regressive taxes are those for which the average tax rate falls as income rises. They take a smallerpercentage of income as income rises, so they will not necessarily take a larger absolute amount of income as incomerises.

Answer (D) is incorrect because regenerative is not a term used to designate types of taxes.

[1006] Gleim #: 7.3.79 -- Source: CIA 594 IV-71

Answer (A) is incorrect because the marginal tax rate is 30%. The marginal tax rate equals the highest rate paid.

Answer (B) is incorrect because the marginal tax rate is 30%. The marginal tax rate equals the highest rate paid.

Answer (C) is correct. The average tax rate is calculated using the weighted-average method. The weight assigned toeach rate is determined by the proportion of taxable income subject to it. The average tax rate is 21% [(30,000 ÷ 50,000)× .15 + (20,000 ÷ 50,000) × .30].

Answer (D) is incorrect because 22.5% is a simple numerical average.

[1007] Gleim #: 7.3.80 -- Source: Publisher

Answer (A) is incorrect because a progressive tax is a tax in which individuals with higher (lower) incomes pay a higher(lower) percentage of their income in tax. For example, income taxes are progressive.

Answer (B) is correct. With a regressive tax, the percentage paid in taxes decreases as income increases. For example,excise taxes and payroll taxes are both regressive taxes. An excise tax is regressive because its burden fallsdisproportionally on lower-income persons. As personal income increases, the percentage of income paid declines becausean excise tax is a flat amount per quantity of the good or service purchased.

Answer (C) is incorrect because a proportional tax is a tax in which the individual pays a constant percentage in taxes,regardless of income level. A sales tax is a proportional tax.

Answer (D) is incorrect because a progressive tax is a tax in which individuals with higher (lower) incomes pay a higher(lower) percentage of their income in tax. For example, income taxes are progressive.

[1008] Gleim #: 7.3.81 -- Source: CIA 597 IV-76

Answer (A) is correct. A sales tax is regressive with respect to income even though the rate is the same regardless of thebuyer’s income. The reason is that a greater percentage of a low-income individual’s income is exposed to the tax. Ahigher-income individual should be able to save more and therefore shield a greater percentage of his/her income from thetax.

Answer (B) is incorrect because a larger portion of a lower-income person’s income is subject to the tax.

Answer (C) is incorrect because the general sales tax rate is uniform for all taxpayers.

Answer (D) is incorrect because, if the tax claims an increasing amount of income as income rises, it is progressive, notregressive.

Gleim's CIA Test Prep: Part III: Business Analysis and Information TechnologyAnswer Explanations

(1312 questions)

Copyright 2008 Gleim Publications, Inc. Page 310Printed for Mamdouh Farag

Page 311: P.3 Answer Explanation

[1009] Gleim #: 7.3.82 -- Source: Publisher

Answer (A) is incorrect because businesses must pay a value-added tax regardless of whether they make a profit.

Answer (B) is correct. A value-added tax is based on consumption. The tax is levied on the value added to goods by eachbusiness unit in the production and distribution chain. It is the consumer who ultimately bears the incidence of the taxbecause businesses subject to the tax will raise prices to offset the operating cost of the tax.

Answer (C) is incorrect because a value-added tax encourages savings. This tax is paid only on consumption, not savings.

Answer (D) is incorrect because the amount of value added is the difference between a firm’s sales and its purchases.

[1010] Gleim #: 7.3.83 -- Source: CIA 1194 IV 71

Answer (A) is correct. Because the taxpayer’s income increase is equal to the general inflation rate, no real increase inincome has occurred. However, some or all of the increase is subject to a higher tax rate. Accordingly, the taxpayer willexperience an increase in both nominal taxes payable and in real taxes payable.

Answer (B) is incorrect because real taxes payable and nominal taxes payable will increase.

Answer (C) is incorrect because real taxes payable and nominal taxes payable will increase.

Answer (D) is incorrect because real taxes payable and nominal taxes payable will increase.

[1011] Gleim #: 7.3.84 -- Source: CIA 1194 IV 72

Answer (A) is correct. A value-added tax is similar to a retail sales tax on consumer goods. It is applied to the differencebetween the value of a firm’s sales and the value of its purchases from other firms, that is, to the value added to the input.In effect, a value-added tax is a national tax that penalizes consumption.

Answer (B) is incorrect because the difference between the selling price of a real estate property and the amount the firmoriginally paid for it is taxed as capital gain.

Answer (C) is incorrect because the value of a firm’s sales to related companies is the internal transfer price.

Answer (D) is incorrect because the profit earned on a firm’s sales is subject to income tax.

[1012] Gleim #: 7.3.85 -- Source: CIA 595 IV 66

Answer (A) is incorrect because at least 1.00 of operating income will be needed to pay 1.00 of either interest ordividends.

Answer (B) is incorrect because 1.67 of pretax income is required to pay 1.00 of dividends.

Answer (C) is correct. Interest is deductible from operating income to obtain taxable income but dividends are not. Thus,only 1.00 of pretax income is needed to pay 1.00 of interest, but 1.67 of pretax income [1.00 ÷ (1.0 – .4)] is needed to pay1.00 of dividends if the enterprise is in a 40% tax bracket.

Answer (D) is incorrect because 1.00 of operating income is needed to pay 1.00 of interest, and 1.67 of pretax income isneeded to pay 1.00 of dividends.

Gleim's CIA Test Prep: Part III: Business Analysis and Information TechnologyAnswer Explanations

(1312 questions)

Copyright 2008 Gleim Publications, Inc. Page 311Printed for Mamdouh Farag

Page 312: P.3 Answer Explanation

[1013] Gleim #: 7.3.86 -- Source: CIA 595 IV-69

Answer (A) is incorrect because value-added taxes are not levied on personal interest income.

Answer (B) is incorrect because value-added taxes are not levied on corporate dividend income.

Answer (C) is incorrect because value-added taxes are not levied on the value of a firm’s assets.

Answer (D) is correct. A value-added tax is levied on the value a firm adds to a good or service. This amount is measuredas the difference between the value of a firm’s sales and the value of its purchases from other firms. A value-added tax istherefore similar to a retail sales tax. It is equivalent to a national sales tax on consumer goods. By penalizingconsumption, it encourages saving and investment.

[1014] Gleim #: 7.3.87 -- Source: CIA 1195 IV 71

Answer (A) is incorrect because price elasticity of demand is inversely correlated with the revenue effects of an increasein excise taxes.

Answer (B) is correct. A higher excise tax increases the selling price of the product. This price increase will have a lessnegative effect on sales volume for products with less elastic demand. Examples of products with low elasticity of demandinclude gasoline, tobacco, and alcohol. The tax revenue generated by an increase in excise taxes is therefore higher if thetax is levied on products with less elastic demand.

Answer (C) is incorrect because price elasticity of demand is inversely correlated with the revenue effects of an increasein excise taxes.

Answer (D) is incorrect because price elasticity of demand is inversely correlated with the revenue effects of an increasein excise taxes.

[1015] Gleim #: 7.3.88 -- Source: IIA, adapted

Answer (A) is correct. An investment tax credit is an incentive because it directly reduces future tax payments. Thus,corporate investments will have higher net present values, all else equal, than without tax credits.

Answer (B) is incorrect because tax credits reduce the corporate tax bill. With more profits available for dividends,inflation will not be restrained.

Answer (C) is incorrect because, if other factors are held constant, future net cash inflows from an investment areincreased by tax credits. Hence, tax credits make investment more attractive.

Answer (D) is incorrect because investment tax credits are not taxes levied on projects.

Gleim's CIA Test Prep: Part III: Business Analysis and Information TechnologyAnswer Explanations

(1312 questions)

Copyright 2008 Gleim Publications, Inc. Page 312Printed for Mamdouh Farag

Page 313: P.3 Answer Explanation

[1016] Gleim #: 7.3.89 -- Source: IIA, adapted

Answer (A) is incorrect because progressive taxes, for which the average tax rate rises as income rises, take both a largerpercentage of income and a larger absolute amount of income as income rises.

Answer (B) is incorrect because proportional taxes, for which the average tax rate is constant for all income levels, alwaystake a larger absolute amount of income as income rises.

Answer (C) is correct. Regressive taxes are those for which the average tax rate falls as income rises. They take a smallerpercentage of income as income rises, so they will not necessarily take a larger absolute amount of income as incomerises.

Answer (D) is incorrect because a flat tax would have the same percentage tax rate regardless of income and wouldtherefore take a larger absolute amount of income as income rises.

[1017] Gleim #: 7.4.90 -- Source: CIA 597 IV-59

Answer (A) is incorrect because the potential labor force is not a factor in the calculation. Rather, real GDP is determinedby actual inputs and their productivity.

Answer (B) is incorrect because real GDP is adjusted for inflation.

Answer (C) is incorrect because national income accounting does not address the quality of output.

Answer (D) is correct. Real GDP increases when resource inputs and their productivity increase. Thus, to the extent thatreal GDP depends on labor inputs, real GDP equals total worker hours (labor input) times labor productivity (real outputper worker per hour).

[1018] Gleim #: 7.4.91 -- Source: CIA 596 IV-68

Answer (A) is incorrect because, if a government increases the demand for its own currency, the currency will appreciate.This action will alter the trade balance and the level of aggregate demand.

Answer (B) is correct. Aggregate demand is the amount of real domestic output that domestic consumers, foreign buyers,governments, and businesses will want to purchase at each price level. One factor that changes aggregate demand is netexport spending. Exchange rates are among the determinants of net export spending. When a government intervenes in theforeign currency market to purchase its own currency, it causes an appreciation of that currency. One result is that thetrade balance will be affected. Exports will fall as domestic goods become more costly from the perspective of foreignconsumers. Imports will rise as foreign goods become less costly for domestic consumers. Consequently, net exports willdecline and domestic aggregate demand will also decline.

Answer (C) is incorrect because a sale of the currency by the domestic government will cause the currency to depreciate.This action will alter the trade balance and the level of aggregate demand.

Answer (D) is incorrect because, if the government intervenes and causes a depreciation of the domestic currency, exportswill rise and imports will fall. Net exports will therefore rise if the currency appreciates, so aggregate demand willincrease.

Gleim's CIA Test Prep: Part III: Business Analysis and Information TechnologyAnswer Explanations

(1312 questions)

Copyright 2008 Gleim Publications, Inc. Page 313Printed for Mamdouh Farag

Page 314: P.3 Answer Explanation

[1019] Gleim #: 7.4.92 -- Source: CIA 595 IV-57

Answer (A) is incorrect because the sale of final goods is included in GDP, and the sale of intermediate goods is excluded.

Answer (B) is correct. The sale of final goods is included in, and the sale of intermediate goods is excluded from, GDP.The purpose of this treatment is to avoid double counting. The value of final goods already includes any intermediatetransactions involved in their production.

Answer (C) is incorrect because the sale of final goods is included in GDP, and the sale of intermediate goods is excluded.

Answer (D) is incorrect because the sale of final goods is included in GDP, and the sale of intermediate goods isexcluded.

[1020] Gleim #: 7.4.93 -- Source: CIA 596 IV-60

Answer (A) is correct. GDP is the total value of goods and services produced within the boundaries of a country. It maybe measured using an expenditures approach or an income approach. Under the income approach, GDP equals all incomederived from the production of the year’s output, with an adjustment for net income earned abroad (a positive or negativeamount in theory). Two types of nonincome charges or allocations must be added to incomes (wages, rents, interest, andprofits). Depreciation reflects the consumption of fixed capital during the period. It is the part of the year’s receipts thatmust be allocated to replace the machinery, plant, etc., used up in the production of GDP. Indirect business taxes, such assales, excise, and property taxes, are treated by businesses as a cost of production and form part of the total price of goodsand services. Thus, they are not paid as wages, rents, interest, and profits. Accordingly, GDP may be measured as the sumof consumption of fixed capital, indirect business taxes, wages, rents, interest, and profits (proprietors’ income, corporatetaxes, dividends, and undistributed corporate profits), with an adjustment for net income earned abroad.

Answer (B) is incorrect because nonincome charges and the adjustment for net income earned abroad must also beincluded in the calculation.

Answer (C) is incorrect because interest income is added, not subtracted. Also, the adjustment for net income earnedabroad must be considered.

Answer (D) is incorrect because profits are added, not subtracted. Also, nonincome charges must be included in thecalculation.

[1021] Gleim #: 7.4.94 -- Source: CIA 1196 IV-59

Answer (A) is incorrect because net domestic product is calculated net of the capital consumption allowance.

Answer (B) is incorrect because net domestic product includes only final goods. The inclusion of intermediate goodswould involve double counting. Also, net domestic product is calculated net of the capital consumption allowance.

Answer (C) is correct. Net domestic product is the market value of all final goods and services produced within theboundaries of a country within 1 year minus the capital consumption allowance.

Answer (D) is incorrect because net domestic product does not include intermediate goods.

Gleim's CIA Test Prep: Part III: Business Analysis and Information TechnologyAnswer Explanations

(1312 questions)

Copyright 2008 Gleim Publications, Inc. Page 314Printed for Mamdouh Farag

Page 315: P.3 Answer Explanation

[1022] Gleim #: 7.4.95 -- Source: CMA 1293 1-25

Answer (A) is incorrect because the capital account includes capital movements only; the direction of capital movementsis influenced by the prevailing interest rates in each nation.

Answer (B) is incorrect because official reserves are assets held by central banks and are not necessarily related to currenttransactions.

Answer (C) is incorrect because this is a nonsense answer.

Answer (D) is correct. The balance of payments represents all international payments made by one nation to another,including those for imports, exports, investments, unilateral transfers such as pensions and gifts, and capital movements.The principal accounts are the current account and the capital account. The current account includes a net trade balance ingoods, net investment receipts or payments, net receipts or payments for services, and the balance of unilateral transfers.

[1023] Gleim #: 7.4.96 -- Source: CMA 1288 1-19

Answer (A) is incorrect because the current account includes exports and imports of services as well as goods.

Answer (B) is incorrect because the trade balance concerns goods only.

Answer (C) is correct. The balance of payments accounts include all international payments made by one nation toanother, including capital movements, imports, exports, and unilateral transfers. The net of exports and imports is thecurrent account balance. The current account balance does not include capital transactions. The capital account reflectsmovements of financial capital (investments).

Answer (D) is incorrect because the various components are never in balance. In reality, the important consideration is thetotal, and that is more of a long-run than an annual problem.

[1024] Gleim #: 7.4.97 -- Source: CMA 691 1-18

Answer (A) is incorrect because imports are debited.

Answer (B) is incorrect because expenditures of tourists abroad are treated as imports and are therefore debited.

Answer (C) is incorrect because earnings of foreigners represent outflows of foreign exchange and are debited.

Answer (D) is correct. Basically, exports of goods and services are credited to the balance of payments accounts andimports are debited. Similarly, capital movements may be debited or credited. For example, transfers of capital fromforeigners, such as loans, are credits in the accounts. In effect, these loans constitute exports of debt instruments andincrease the supply of foreign currency available.

Gleim's CIA Test Prep: Part III: Business Analysis and Information TechnologyAnswer Explanations

(1312 questions)

Copyright 2008 Gleim Publications, Inc. Page 315Printed for Mamdouh Farag

Page 316: P.3 Answer Explanation

[1025] Gleim #: 7.4.98 -- Source: CMA 1288 1-21

Answer (A) is incorrect because the current account has a deficit, not a surplus, of Pta 7,000.

Answer (B) is incorrect because investment by foreigners should be subtracted from, not added to, domestic investment inforeign countries. Furthermore, interest payments on foreign loans affect the current, not the capital account, and thecapital account has a trade deficit, not a surplus.

Answer (C) is incorrect because investment by foreigners should be subtracted from, not added to, domestic investment inforeign countries.

Answer (D) is correct. The balance of trade is the balance of imports and exports of goods. The current account alsoconsiders trade in services (none for this economy), unilateral transfers (e.g., gifts), and investment receipts and payments.It does not include capital transactions. Thus, investments by foreigners in the domestic economy and investments made inforeign countries will not be included in the current account. These transactions will be reflected in the capital account.The capital account will therefore have a deficit of 4,900 (6,300 – 1,400) pesetas because investment in foreign countriesis greater than investment by foreigners. The current account will have a deficit of 7,000 pesetas (15,760 – 20,300 + 1,240– 3,700).

[1026] Gleim #: 7.4.99 -- Source: CMA 1285 1-30

Answer (A) is correct. Currency devaluations result in a change in the balance of payments. A devaluation means thatother currencies will buy more of the devaluing nation’s currency, and the prices of goods denominated in the devaluedcurrency are therefore cheaper. A devaluation usually results in an increase in exports, a decrease in imports (caused byhigher relative input prices), and an improved balance of trade.

Answer (B) is incorrect because a devaluation will encourage exports.

Answer (C) is incorrect because a devaluation will discourage imports and may encourage domestic inflation. Foreigngoods will be more expensive.

Answer (D) is incorrect because devaluation most likely occurs when a country is losing its reserves.

[1027] Gleim #: 7.4.100 -- Source: CIA 596 IV-69

Answer (A) is incorrect because the total balance of payments surplus or deficit, which is the sum of the current andcapital account balances, is what determines changes in official reserves and pressures for exchange rates to change.

Answer (B) is incorrect because the total balance of payments surplus or deficit, which is the sum of the current andcapital account balances, is what determines changes in official reserves and pressures for exchange rates to change.

Answer (C) is incorrect because, given an appreciation of the currency, the short-run effects are likely to be a balance ofpayments surplus and growing official reserves.

Answer (D) is correct. In a freely floating exchange-rate system, exchange rates automatically adjust so as to eliminatebalance of payments surpluses or deficits. For example, if Country Z demand for country X’s currency increases, the resultis a Country Z deficit at the existing exchange rate because demand now exceeds the supply of X’s currency at that rate.However, the system of floating exchange rates allows the change in the relative strength of the currencies to be reflectedin their exchange rate. The appreciation of X’s currency against Z’s currency, that is, the increase in the amount of Z’scurrency exchangeable for a unit of X’s currency, makes Z’s products cheaper to buyers in country X. Furthermore, X’sproducts are more expensive to Z’s buyers. Consequently, Z’s imports will fall, Z’s exports will rise, and the balance ofpayments deficit will decrease.

Gleim's CIA Test Prep: Part III: Business Analysis and Information TechnologyAnswer Explanations

(1312 questions)

Copyright 2008 Gleim Publications, Inc. Page 316Printed for Mamdouh Farag

Page 317: P.3 Answer Explanation

[1028] Gleim #: 7.4.101 -- Source: Publisher

Answer (A) is incorrect because money declines in value by 33.33%.

Answer (B) is correct. If the price level increases 50%, an item that used to cost $1.00 will increase to $1.50. A personwith $1.50 could have previously purchased 1.5 units of product. After the price level increases, that person can buy only1 unit, or a decline of .5 units. Thus, the value of money declines by 33.33% (.5 ÷ 1.5).

Answer (C) is incorrect because 50% is the increase in price level, which is calculated using a different base from the oneused to calculate the decline in monetary value.

Answer (D) is incorrect because money declines in value by 33.33%.

[1029] Gleim #: 7.4.102 -- Source: CMA 676 1-34

Answer (A) is incorrect because increasing productivity will lower the price of domestic exports which will increase theamount of exports demanded, and thus reduce the balance of payments deficit.

Answer (B) is incorrect because as the rate of inflation slows down, prices charged to foreign buyers are reduced, whichwill increase exports (and reduce the balance of payments deficit).

Answer (C) is incorrect because if more money were given to trading, more domestic exports would be bought, thuscorrecting a balance of payments deficit.

Answer (D) is correct. A balance of payments deficit exists when the fixed or managed exchange rate is too high. “Toohigh” is when the fixed price is higher than the equilibrium price would be if market forces were at work. To correct abalance of payments deficit, the price of dollars must decrease or other means must be undertaken to raise the real valueto the fixed level. If the value of domestic currency is increased, the deficit will grow.

[1030] Gleim #: 7.4.103 -- Source: CMA 1281 1-16

Answer (A) is incorrect because it excludes capital inflows and outflows, grants, and remittances.

Answer (B) is incorrect because it excludes capital inflows and outflows, grants, and remittances.

Answer (C) is correct. The balance of payments is defined as the excess of imports, private capital outflows, grants, andremittances over exports and private capital inflows. When there is a surplus in the balance of payments, more domesticgoods may have been sold abroad than were imported, and/or foreigners may have invested more capital in the domesticcountry than domestic citizens invested abroad. For this reason, a surplus is considered a favorable balance of payments.Just the opposite is true for a deficit in the balance of payments.

Answer (D) is incorrect because it excludes exports completely.

Gleim's CIA Test Prep: Part III: Business Analysis and Information TechnologyAnswer Explanations

(1312 questions)

Copyright 2008 Gleim Publications, Inc. Page 317Printed for Mamdouh Farag

Page 318: P.3 Answer Explanation

[1031] Gleim #: 7.4.104 -- Source: CMA 682 1-12

Answer (A) is incorrect because receipt of dividends by a domestic corporation from its foreign subsidiary increases thedemand for domestic currency.

Answer (B) is incorrect because buying of domestic shares by a foreign investor represents an increase in demand fordomestic assets by foreigners.

Answer (C) is incorrect because domestic export of military equipment to a foreign country reflects an increase indomestic goods and services demanded by foreigners.

Answer (D) is correct. Because debits represent unfavorable items, expenditures by domestic residents vacationingabroad require debits. Buying foreign goods and services means using domestic currency to acquire foreign currency, thusincreasing the demand for foreign currency.

[1032] Gleim #: 7.4.105 -- Source: CMA 682 1-13

Answer (A) is incorrect because a domestic bank’s payment of $5,000 in interest to foreigners reflects an increase indemand for foreign currency by domestic citizens.

Answer (B) is incorrect because a domestic subsidiary’s remittance of a dividend of $1 million to its parent company in aforeign country is a private capital outflow and would appear as a debit in the balance of payments account.

Answer (C) is incorrect because a domestic exporter’s purchase of marine insurance from a foreign insurance companyrepresents imports by the domestic country of insurance services.

Answer (D) is correct. Exports help a country’s balance of payments and are therefore considered a credit in the domesticbalance of payments account. Thus, the domestic farmer’s export of grain to a foreign country is recorded as a credit.

[1033] Gleim #: 7.4.106 -- Source: CIA 594 IV-57

Answer (A) is incorrect because GDP does measure national economic performance. GDP can be measured as the total ofall output in the economy or total income.

Answer (B) is incorrect because GDP does measure market-oriented activity. GDP can be measured as the total of alloutput in the economy or total income.

Answer (C) is correct. GDP is the total market value of all final goods and services produced within the boundaries of theU.S. in one year, without regard to ownership of the means of production. It is a monetary measure of quantitative data,not qualitative characteristics. Thus, improvements in product quality are not measured by GDP.

Answer (D) is incorrect because GDP does measure the size of national output. GDP can be measured as the total of alloutput in the economy or total income.

[1034] Gleim #: 7.4.107 -- Source: CIA 1193 IV-64

Answer (A) is incorrect because an increase in consumption expenditures increases GDP.

Answer (B) is correct. By definition, GDP determined using the expenditures approach equals the sum of consumption,investment, government purchases of goods and services, and net exports (exports – imports). Thus, a rise in imports willcause a fall in net exports and GDP.

Answer (C) is incorrect because an increase in exports increases GDP.

Answer (D) is incorrect because an increase in inflation increases GDP.

Gleim's CIA Test Prep: Part III: Business Analysis and Information TechnologyAnswer Explanations

(1312 questions)

Copyright 2008 Gleim Publications, Inc. Page 318Printed for Mamdouh Farag

Page 319: P.3 Answer Explanation

[1035] Gleim #: 7.4.108 -- Source: CIA 1194 IV-59

Answer (A) is incorrect because the income or allocations approach adds the money income from production of the year’soutput. This approach to GDP measurement adds nonincome charges or allocations, wages, rents, interest, and profits.From this amount, net American income earned abroad is subtracted.

Answer (B) is incorrect because the income or allocations approach adds the money income from production of the year’soutput. This approach to GDP measurement adds nonincome charges or allocations, wages, rents, interest, and profits.From this amount, net American income earned abroad is subtracted.

Answer (C) is incorrect because the output approach does not include interest, but includes expenditures by foreigners.

Answer (D) is correct. GDP is the total value of all goods and services produced in the U.S., regardless of the ownershipof the means of production. GDP can be measured using an income or output (expenditures) approach. The outputapproach measures the amount spent to purchase the year’s total output. Using this approach, GDP is calculated by addingconsumption expenditures by households, investment expenditures by businesses, government purchases of goods andservices, and expenditures by foreigners.

[1036] Gleim #: 7.4.109 -- Source: CIA 1195 IV-57

Answer (A) is correct. GDP can be calculated using an income approach or an expenditures approach because what isspent on a product or service is income to those who contributed resources to its production and marketing. Thus, theamount spent to purchase the GDP is identical with the income derived from its production and sale. Under the output orexpenditures approach, GDP equals the sum of consumption expenditures by households, investment by business,government purchases of goods and services, and expenditures by foreigners (net exports).

Answer (B) is incorrect because consumption, investment, government purchases, and expenditures by foreigners (netexports) are added to determine GDP.

Answer (C) is incorrect because consumption, investment, government purchases, and expenditures by foreigners (netexports) are added to determine GDP.

Answer (D) is incorrect because consumption, investment, government purchases, and expenditures by foreigners (netexports) are added to determine GDP.

[1037] Gleim #: 7.5.110 -- Source: CIA 592 II-13

Answer (A) is incorrect because questionnaires usually provide for yes/no responses and therefore provide less detailedevidence than some other procedures.

Answer (B) is incorrect because questionnaires tend to be lengthy, and their completion is time-consuming.

Answer (C) is incorrect because an auditor need not be present.

Answer (D) is correct. An internal control questionnaire consists of a series of questions about the controls designed toprevent or detect errors or irregularities. Answers to the questions help the internal auditor to identify specific internalcontrol policies and procedures relevant to specific assertions and to design tests of controls to evaluate the effectivenessof their design and operation. The questionnaire provides a framework to assure that specific concerns are not overlooked,but it is not a sufficient means of understanding the entire system. Thus, the evidence obtained is indirect and requirescorroboration by means of observation, interviews, flowcharting, examination of documents, etc.

Gleim's CIA Test Prep: Part III: Business Analysis and Information TechnologyAnswer Explanations

(1312 questions)

Copyright 2008 Gleim Publications, Inc. Page 319Printed for Mamdouh Farag

Page 320: P.3 Answer Explanation

[1038] Gleim #: 7.5.111 -- Source: CIA 592 II-25

Answer (A) is incorrect because detail testing provides direct evidence that the approvals were not received. Indirectevidence establishes immediately collateral facts from which the main fact may be inferred.

Answer (B) is incorrect because circumstantial evidence tends to prove a fact by proving other events or circumstancesthat afford a basis for a reasonable inference of the occurrence of the fact. Thus, it is also indirect evidence.

Answer (C) is correct. Corroborative evidence is evidence from a different source that supplements and confirms otherevidence. For example, oral testimony that a certain procedure was not performed may be corroborated by the absence ofdocumentation.

Answer (D) is incorrect because subjective evidence is opinion-oriented and is not dependable for reaching engagementconclusions. No subjective evidence is present in this situation.

[1039] Gleim #: 7.5.112 -- Source: CIA 1188 I-28

Answer (A) is incorrect because, if the original writing is available, oral testimony cannot contradict the content of thewriting.

Answer (B) is correct. The best (primary) evidence is the most persuasive evidence. Reliability and the best evidence ruleare closely related. The best evidence rule is ordinarily applied only to documentary evidence, especially to proof of thecontent of a writing. If the original writing is available, the best evidence rule prohibits a party from proving the content ofa writing through oral testimony. Therefore, the original writing is the most competent evidence.

Answer (C) is incorrect because the contract itself is the best evidence.

Answer (D) is incorrect because the contract itself is the best evidence.

[1040] Gleim #: 7.5.113 -- Source: CIA 1182 I-7

Answer (A) is correct. Secondary evidence according to the legal view is acceptable if primary evidence (the strongestevidence, e.g., original documents) has been destroyed or is not reasonably procurable. Secondary evidence must be aproper representation of primary evidence, e.g., copies of a contract.

Answer (B) is incorrect because circumstantial evidence inferentially establishes one fact by proving another collateralfact.

Answer (C) is incorrect because hearsay is an out-of-court statement offered in evidence to prove the truth of the matterasserted.

Answer (D) is incorrect because, except for testimony by experts, witnesses may normally testify as to facts only.

[1041] Gleim #: 7.6.114 -- Source: Publisher

Answer (A) is incorrect because legality is a required element of a contract.

Answer (B) is incorrect because consideration is a required element of a contract.

Answer (C) is incorrect because legal capacity is a required element of a contract.

Answer (D) is correct. The four essential elements of a contract are an agreement (offer and acceptance), consideration,legal capacity of the parties to contract, and a legal objective or purpose. A writing is not required to enter into a contract.However, some contracts are not enforceable unless a writing evidences the contract.

Gleim's CIA Test Prep: Part III: Business Analysis and Information TechnologyAnswer Explanations

(1312 questions)

Copyright 2008 Gleim Publications, Inc. Page 320Printed for Mamdouh Farag

Page 321: P.3 Answer Explanation

[1042] Gleim #: 7.6.115 -- Source: Publisher

Answer (A) is incorrect because an oral contract is usually enforceable. Consideration must be legally sufficient and mustbe bargained for but need not have equal market value.

Answer (B) is incorrect because most contracts are informal (simple), and if a term is missing, it can be implied by thecourt, with the exception of a quantity term.

Answer (C) is correct. Contracts require each of the following:Offer and acceptance1.Mutual assent (meeting of the minds)2.Consideration (bargained-for exchange)3.Legality (legal purpose)4.Capacity of parties (legal ability)5.

Answer (D) is incorrect because promises can be unilateral or divisible.

[1043] Gleim #: 7.6.116 -- Source: Publisher

Answer (A) is incorrect because a promise satisfies the element of legal sufficiency. Monetary value is relativelyunimportant.

Answer (B) is incorrect because consideration is an exchange of legal benefit that may not have actual benefit.

Answer (C) is incorrect because contracts can be unilateral, which involves only one promise.

Answer (D) is correct. Consideration must be legally sufficient and intended as a bargained-for exchange. A promisee hasprovided legally sufficient consideration if (s)he incurs a legal detriment or if the promisor receives a legal benefit. Anessential aspect of consideration is that it be bargained for, and given in exchange for, the consideration provided by theother party. That is, consideration is mutual.

[1044] Gleim #: 7.6.117 -- Source: Publisher

Answer (A) is incorrect because past consideration does not satisfy the consideration requirement for the formation of acontract.

Answer (B) is correct. Lamar has a preexisting legal duty to find Aunt. Consideration does not exist if an existing dutywas imposed by law or a person is already under a contractual agreement to render a specified performance. Lamar willsuffer no new legal detriment by finding Aunt; thus, no contractual obligation exists.

Answer (C) is incorrect because Lamar has a preexisting legal duty to find Aunt.

Answer (D) is incorrect because Lamar has not made a valid contract with Uncle.

[1045] Gleim #: 7.6.118 -- Source: E. Rahm

Answer (A) is incorrect because the sheriff had a preexisting legal duty to capture the criminal.

Answer (B) is incorrect because a promise to make a gift is not enforceable. It lacks the bargain element.

Answer (C) is correct. A valid contract exists because Anxious Father offered to pay Albert $4,000 for taking hisdaughter to the dance. This is a unilateral contract supported by the consideration of $4,000. Albert will be entitled to the$4,000 after he takes the daughter to the dance.

Answer (D) is incorrect because past consideration is not consideration.

Gleim's CIA Test Prep: Part III: Business Analysis and Information TechnologyAnswer Explanations

(1312 questions)

Copyright 2008 Gleim Publications, Inc. Page 321Printed for Mamdouh Farag

Page 322: P.3 Answer Explanation

[1046] Gleim #: 8.1.1 -- Source: CIA 1189 I-24

Answer (A) is incorrect because the “paper trail” is less extensive in an information system. Combining processing andcontrols within the system reduces documentary evidence.

Answer (B) is incorrect because information assets are more likely to be under the control of the information systemfunction.

Answer (C) is correct. Using a computer does not change the basic concepts and objectives of control. However, the useof computers may modify the control techniques used. The processing of transactions may be combined with controlactivities previously performed separately, or control function may be combined within the information system activity.

Answer (D) is incorrect because documentation is more important in an information system. Information is more likely tobe stored in machine-readable form than in hard copy.

[1047] Gleim #: 8.1.2 -- Source: Publisher

Answer (A) is incorrect because control over report distribution (output), correction of input errors, and authorization ofinput are application controls.

Answer (B) is correct. General controls are policies and procedures that relate to many information systems applicationand support the effective functioning of application controls by helping to ensure the continued proper operation ofinformation systems. General controls include controls over (1) data center and network operations, (2) systems softwareacquisition and maintenance, (3) access security, and (4) application systems acquisition, development, and maintenance.Control over report distribution (output), correction of input errors, and authorization of input are application controls.

Answer (C) is incorrect because control over report distribution (output), correction of input errors, and authorization ofinput are application controls.

Answer (D) is incorrect because control over report distribution (output), correction of input errors, and authorization ofinput are application controls.

[1048] Gleim #: 8.1.3 -- Source: CIA 590 I-20

Answer (A) is correct. This separation is an organizational control. Organizational controls concern the propersegregation of duties and responsibilities within the information systems department. Although proper segregation isdesirable, functions that would be considered incompatible if performed by a single individual in a manual activity areoften performed through the use of an information systems program or series of programs. Thus, compensating controlsmay be necessary, such as library controls, effective supervisions, and rotation of personnel. Segregating test programsmakes concealment of unauthorized changes in production programs more difficult.

Answer (B) is incorrect because physical security (e.g., climate control and restrictions on physical access) is anotheraspect of organizational control.

Answer (C) is incorrect because input controls validate the completeness, accuracy, and appropriateness of input.

Answer (D) is incorrect because concurrency controls manage situations in which two or more programs attempt to use afile or database at the same time.

Gleim's CIA Test Prep: Part III: Business Analysis and Information TechnologyAnswer Explanations

(1312 questions)

Copyright 2008 Gleim Publications, Inc. Page 322Printed for Mamdouh Farag

Page 323: P.3 Answer Explanation

[1049] Gleim #: 8.1.4 -- Source: CIA 596 III-75

Answer (A) is correct. In distributed or cooperative systems, the responsibility for ensuring that adequate backups aretaken is the responsibility of user management. The systems are under the control of users, not a central informationprocessing department.

Answer (B) is incorrect because distributed environments have no systems programmers comparable to those at centralsites for traditional systems.

Answer (C) is incorrect because distributed environments may not have data entry clerks. Users typically perform theirown data entry.

Answer (D) is incorrect because, in distributed environments, there are no tape librarians.

[1050] Gleim #: 8.1.5 -- Source: CIA 1196 III-55

Answer (A) is incorrect because applications development is responsible for developing systems, not providing help toend-users.

Answer (B) is incorrect because the responsibility of systems programming is to implement and maintain system-levelsoftware such as operating systems, access control software, and database systems software.

Answer (C) is correct. Help desks are usually a responsibility of computer operations because of the operational nature oftheir functions. A help desk logs reported problems, resolves minor problems, and forwards more difficult problems to theappropriate information systems resources, such as a technical support unit or vendor assistance.

Answer (D) is incorrect because user departments usually lack the expertise to solve computer problems.

[1051] Gleim #: 8.1.6 -- Source: CIA1195 I-32

Answer (A) is incorrect because effective control requires that programmers not be able to make undetected, unrecordedchanges in data or programs. Thus, programmers should not have access to the production library.

Answer (B) is incorrect because programmers should be responsible for making program changes, and users should beresponsible for testing the changes. Hence, users should not have access to the test library. Accountability for changeswould be diminished. Moreover, users may lack the competence to make appropriate changes.

Answer (C) is correct. The program librarian is accountable for, and has custody of, the programs in the productionlibrary.

Answer (D) is incorrect because, if the operator has access to both program libraries, (s)he may be able to makeunauthorized and undetected changes to the computer programs.

[1052] Gleim #: 8.1.7 -- Source: CIA 597 III-46

Answer (A) is incorrect because external risks should be evaluated to determine the center’s location.

Answer (B) is incorrect because biometric access systems control physical access to the data center. These devices identifysuch unique physical qualities as fingerprints, voice patterns, and retinal patterns.

Answer (C) is correct. Authorization tables for operating system access address logical controls, not physical controls.

Answer (D) is incorrect because power supply systems and surge protection are included in data center design. Thus, twoseparate power lines, line conditioning equipment, and backup battery power or a generator are typical elements in thedesign.

Gleim's CIA Test Prep: Part III: Business Analysis and Information TechnologyAnswer Explanations

(1312 questions)

Copyright 2008 Gleim Publications, Inc. Page 323Printed for Mamdouh Farag

Page 324: P.3 Answer Explanation

[1053] Gleim #: 8.1.8 -- Source: CIA 1193 III-20

Answer (A) is correct. The advent of less expensive and smaller computers has permitted the development of a differentalternative to centralization or decentralization. In a distributed data processing system, the organization’s processingneeds are examined in their totality. The decision is not whether an application should be done centrally or locally but,rather, which parts of the application are better performed by smaller local computers and which parts are betterperformed at some other, possibly centralized, site. In essence, the best distribution of processing tasks within applicationareas is sought. The key distinction between decentralized and distributed systems is the interconnection among the nodes(sites) in the network.

Answer (B) is incorrect because time-sharing systems are terminal-oriented systems that are connected to a centralprocessing site.

Answer (C) is incorrect because an online processing system operates under direct control of the CPU.

Answer (D) is incorrect because a personal computing system is a microcomputer resource dedicated to a single user,usually in a stand-alone configuration.

[1054] Gleim #: 8.1.9 -- Source: CIA 1196 III-51

Answer (A) is incorrect because client-server technology used in downsizing is less reliable.

Answer (B) is incorrect because security is usually better on a mainframe.

Answer (C) is incorrect because downsizing applications often increases their complexity. The data files becomefragmented across multiple systems.

Answer (D) is correct. The purpose of downsizing is to reduce costs of applications by abandoning larger, more expensivesystems in favor of smaller, less expensive systems that are more versatile. However, downsized applications are lessreliable than their mainframe predecessors because they are new and have not been used extensively. Typically,downsized client-server implementations lack the monitoring and control features that permit recovery from minorprocessing interruptions.

[1055] Gleim #: 8.1.10 -- Source: CIA 1193 I-24

Answer (A) is incorrect because a computer system decreases processing errors.

Answer (B) is incorrect because the conversion to a new system does not reduce the number of risk exposures.

Answer (C) is incorrect because processing time is decreased.

Answer (D) is correct. In a manual system with appropriate internal control, separate individuals are responsible forauthorizing transactions, recording transactions, and custody of assets. These checks and balances prevent fraud and detectinaccurate or incomplete transactions. In a computer environment, however, this segregation of duties is not alwaysfeasible. For example, a computer may print checks, record disbursements, and generate information for reconciling theaccount balance.

Gleim's CIA Test Prep: Part III: Business Analysis and Information TechnologyAnswer Explanations

(1312 questions)

Copyright 2008 Gleim Publications, Inc. Page 324Printed for Mamdouh Farag

Page 325: P.3 Answer Explanation

[1056] Gleim #: 8.1.11 -- Source: CPA 586 A-58

Answer (A) is incorrect because the choice of language would have little effect on internal control.

Answer (B) is correct. Segregation of duties is a basic category of control activities (AU 319). Functions are incompatibleif a person is in a position both to perpetrate and conceal fraud or errors. Hence, the duties of authorizing transactions,recording transactions, and custody of assets should be assigned to different people. Those employees that operate theserver may be able to override the controls to change records to conceal a theft of cash.

Answer (C) is incorrect because the limitation on access would be considered a strength.

Answer (D) is incorrect because restrictions on the amount of data that can be stored and on the length of time that datacan be stored do not constitute a control weakness.

[1057] Gleim #: 8.1.12 -- Source: CPA 1195 A-14

Answer (A) is incorrect because, although periodic management review is appropriate, reports and systems documentationwill not prevent or detect unauthorized modifications.

Answer (B) is correct. Programmers and analysts can modify programs, data files, and controls. Thus, they should haveno access to programs used to process transactions. Segregation of programming and operations is a control necessary toprevent unauthorized modifications of programs.

Answer (C) is incorrect because user participation relates to new systems, not modification of existing systems.

Answer (D) is incorrect because programmers may have access through data communications. Thus, physical security isnot sufficient to prevent unauthorized modifications.

[1058] Gleim #: 8.1.13 -- Source: CPA 586 A-1

Answer (A) is incorrect because use of generalized audit software is only one of many ways of auditing through acomputer.

Answer (B) is correct. In a manual payroll system, a paper trail of documents is created to provide audit evidence thatcontrols over each step in processing are in place and functioning. One element of a computer system that differentiates itfrom a manual system is that a transaction trail useful for auditing purposes might exist only for a brief time or only incomputer-readable form.

Answer (C) is incorrect because conversion to a computer system may actually increase the chance of fraud by eliminatingsegregation of incompatible functions and other controls.

Answer (D) is incorrect because automatic updating indicates that processing is not in batch mode.

Gleim's CIA Test Prep: Part III: Business Analysis and Information TechnologyAnswer Explanations

(1312 questions)

Copyright 2008 Gleim Publications, Inc. Page 325Printed for Mamdouh Farag

Page 326: P.3 Answer Explanation

[1059] Gleim #: 8.1.14 -- Source: Publisher

Answer (A) is incorrect because halt and error condition controls include explicit instructions that should appear in runmanuals for handling halts and error messages for programs.

Answer (B) is correct. A batch total is the sum of information that results from adding a field from each record in a batchor group. It provides for control over the movement or processing of the batch and is considered an application control.

Answer (C) is incorrect because library security controls include the organization and operation of a library to precludemisplacement or theft of tapes, programs, and other file media. In addition, internal and external labels should be used sothat the proper files are processed.

Answer (D) is incorrect because operating controls also include provisions for backup and reconstruction of files if dataare lost as a result of processing errors or catastrophes.

[1060] Gleim #: 8.1.15 -- Source: CIA 1189 II-25

Answer (A) is incorrect because systems development controls concern systems analysis, design, and implementation.

Answer (B) is incorrect because hardware controls are incorporated into the equipment.

Answer (C) is incorrect because applications controls pertain to specific programs. They include input, processing, andoutput controls.

Answer (D) is correct. Organizational control concerns the proper segregation of duties and responsibilities within theinformation systems department. For example, programmers should not have access to the equipment, and operatorsshould not have programming ability. Although proper segregation is desirable, functions that would be consideredincompatible if performed by a single individual in a manual activity are often performed through the use of aninformation systems program or series of programs. Therefore, compensating controls may be necessary, such as librarycontrols, effective supervision, and rotation of personnel.

[1061] Gleim #: 8.1.16 -- Source: CPA 585 A-26

Answer (A) is incorrect because data conversion may be assigned to computer operations.

Answer (B) is incorrect because surveillance of screen display messages may be assigned to computer operations.

Answer (C) is correct. Systems analysts are specifically qualified to analyze and design computer information systems.They survey the existing system, analyze the organization’s information requirements, and design new systems to meetthose needs. These design specifications will guide the preparation of specific programs by computer programmers. Theconsole operator should not be assigned programming duties, much less responsibility for systems design, and thus nothave the opportunity to make changes in programs and systems as (s)he operates the equipment.

Answer (D) is incorrect because minor maintenance according to a schedule may be assigned to computer operations.

Gleim's CIA Test Prep: Part III: Business Analysis and Information TechnologyAnswer Explanations

(1312 questions)

Copyright 2008 Gleim Publications, Inc. Page 326Printed for Mamdouh Farag

Page 327: P.3 Answer Explanation

[1062] Gleim #: 8.1.17 -- Source: CMA 695 4-22

Answer (A) is incorrect because librarians maintain control over documentation, programs, and data files; they shouldhave no access to equipment, but they can assist in data processing operations.

Answer (B) is correct. Separation of duties is a general control that is vital in a computerized environment. Someseparation of duties common in noncomputerized environments may not be feasible in a computer environment. However,certain tasks should not be combined. Systems analysts and programmers should be separate from computer operators.Both programmers and analysts may be able to modify programs, files, and controls, and should therefore have no accessto those programs nor to computer equipment. Operators should not be assigned programming duties or responsibility forsystems design, and should have no opportunity to make changes in programs and systems.

Answer (C) is incorrect because a separate information officer outside of the accounting function would not be as critical aseparation of duties as that between programmers and processors.

Answer (D) is incorrect because programmers usually handle all types of programs.

[1063] Gleim #: 8.1.18 -- Source: CMA 1290 4-24

Answer (A) is incorrect because online processing is a method of processing data that permits both immediate posting(updating) and inquiry of master files as transactions occur.

Answer (B) is incorrect because interactive processing is a method of processing data immediately upon input.

Answer (C) is incorrect because time sharing is the processing of a program by the CPU until an input or output operationis required. In time sharing, the CPU spends a fixed amount of time on each program.

Answer (D) is correct. Distributed processing is characterized by a merger of computer and telecommunicationstechnology. Distributed systems permit not only remote access to a computer but also the performance of local processingat local sites. The result is greater flexibility in systems design and the possibility of an optimal distribution of processingtasks.

[1064] Gleim #: 8.1.19 -- Source: CMA 691 4-26

Answer (A) is incorrect because timeliness is not necessarily an element of this type of system.

Answer (B) is incorrect because timeliness is not necessarily an element of this type of system.

Answer (C) is incorrect because timeliness is not necessarily an element of this type of system.

Answer (D) is correct. An online processing system is in direct communication with the computer, giving it the capabilityto handle transactions as they are entered. An online system permits both immediate posting (updating) and inquiry ofmaster files as transactions occur. In an online system, data are immediately available to users upon entry.

[1065] Gleim #: 8.1.20 -- Source: CMA 692 4-1

Answer (A) is incorrect because batch processing provides as much of an audit trail as any computerized operation.

Answer (B) is incorrect because batch processing refers to the input of data, not inquiry.

Answer (C) is correct. Batch processing is the accumulation and grouping of transactions for processing on a delayedbasis. The batch approach is suitable for applications that can be processed against the master file at intervals and involvelarge volumes of similar items, such as payroll, sales, inventory, and billing.

Answer (D) is incorrect because batch processing can also be performed on a decentralized basis.

Gleim's CIA Test Prep: Part III: Business Analysis and Information TechnologyAnswer Explanations

(1312 questions)

Copyright 2008 Gleim Publications, Inc. Page 327Printed for Mamdouh Farag

Page 328: P.3 Answer Explanation

[1066] Gleim #: 8.1.21 -- Source: CPA 1185 A-52

Answer (A) is incorrect because, although personal computer systems may be part of a local area network (LAN), theyneed not be.

Answer (B) is incorrect because, although midrange computer systems may be part of a local area network (LAN), theyneed not be.

Answer (C) is incorrect because batching of transactions requires assembly of data at one place and a delay in updating.

Answer (D) is correct. Real-time processing involves processing an input record and receiving the output soon enough toaffect a current decision-making process. In a real-time system, the user interacts with the system to control an ongoingactivity. Online indicates that the decision maker is in direct communication with the computer. Online, real-time systemsusually permit access to the server from multiple remote terminals.

[1067] Gleim #: 8.1.22 -- Source: CIA 589 III-24

Answer (A) is incorrect because a dumb workstation usually lacks data processing capacity.

Answer (B) is correct. The workstation is usually said to be intelligent if it has data processing capacity, and storedprograms can be modified. Such a workstation usually includes a personal computer, an output device, disk storage, andcommunications links. Intelligent workstations are often connected in a local area network and can therefore communicatewith each other. Thus, they are not mere stand-alone personal computers.

Answer (C) is incorrect because a noninteractive workstation does not permit the user to direct system activities.

Answer (D) is incorrect because desktop publishing is the capability to produce high-quality printing using specializedsoftware.

[1068] Gleim #: 8.1.23 -- Source: CISA

Answer (A) is incorrect because an objective of application controls is that authorized transactions are completelyprocessed once and only once.

Answer (B) is incorrect because an objective of application controls is that transaction data is complete and accurate.

Answer (C) is correct. Application controls provide reasonable assurance that the recording, processing, and reporting ofdata are properly performed. Review and approval procedures for new systems are among the general controls known assystem software acquisition and maintenance controls.

Answer (D) is incorrect because an objective of application controls is that processing results are received by the intendeduser.

Gleim's CIA Test Prep: Part III: Business Analysis and Information TechnologyAnswer Explanations

(1312 questions)

Copyright 2008 Gleim Publications, Inc. Page 328Printed for Mamdouh Farag

Page 329: P.3 Answer Explanation

[1069] Gleim #: 8.1.24 -- Source: CMA 693 4-6

Answer (A) is incorrect because transaction controls and general controls do not ensure the completeness and authenticityof the output.

Answer (B) is incorrect because activity listings and prenumbered forms do not ensure control of the output in a system,and output controls are needed to ensure the accuracy of the output.

Answer (C) is incorrect because input controls are needed to ensure the completeness and authenticity of the output.

Answer (D) is correct. Input controls provide reasonable assurance that data received for processing have been properlyauthorized, converted into machine-sensible form, and identified, and that data have not been lost, suppressed, added,duplicated, or otherwise improperly changed. Output controls provide assurance that the processing result (such asreports, tape and disk, magnetic files, invoices, or printed output) is accurate and that only authorized personnel receivethe output.

[1070] Gleim #: 8.1.25 -- Source: CIA 579 II-5

Answer (A) is incorrect because the most economical point is the earliest time after data has been entered into the system.

Answer (B) is correct. The most economical point at which to correct input errors is at the earliest time after the data hasbeen entered into the system. The earliest point is the entry of data into each field of a record. Errors can easily berectified because the proper information is already available.

Answer (C) is incorrect because the most economical point is the earliest time after data has been entered into the system.

Answer (D) is incorrect because the most economical point is the earliest time after data has been entered into the system.

[1071] Gleim #: 8.1.26 -- Source: CIA, adapted

Answer (A) is incorrect because application audits should be about the same difficulty with or without an adequatelystaffed help desk.

Answer (B) is incorrect because preparation of documentation is a development function, not a help desk function.

Answer (C) is incorrect because the likelihood of use of unauthorized program code is a function of change control, not ofa help desk.

Answer (D) is correct. The biggest risk in not having an adequately staffed help desk is that users will unknowinglypersist in making errors in their interaction with the information systems.

[1072] Gleim #: 8.1.27 -- Source: CIA, adapted

Answer (A) is incorrect because the reviews of jobs processed will disclose access, but will not prevent it.

Answer (B) is incorrect because comparison of production programs and controlled copies will disclose changes, but willnot prevent them.

Answer (C) is incorrect because periodic running of test data will detect changes, but will not prevent them.

Answer (D) is correct. When duties are separated, users cannot obtain a detailed knowledge of programs and computeroperators cannot gain unsupervised access to production programs.

Gleim's CIA Test Prep: Part III: Business Analysis and Information TechnologyAnswer Explanations

(1312 questions)

Copyright 2008 Gleim Publications, Inc. Page 329Printed for Mamdouh Farag

Page 330: P.3 Answer Explanation

[1073] Gleim #: 8.2.28 -- Source: D. Payne

Answer (A) is incorrect because utility programs are application programs that are usually attached to larger programs.They perform various activities, such as sorting data, merging files, converting data from one medium to another, andprinting.

Answer (B) is correct. An operating system (e.g., UNIX or Windows) is required in all computerized systems to overseethe elements of the CPU and the interaction of the hardware components.

Answer (C) is incorrect because a compiler converts (compiles) a program written in a source language, such asFORTRAN, into machine language.

Answer (D) is incorrect because an assembler translates an assembly language program into machine language. Assemblylanguage uses mnemonic codes for each machine language instruction.

[1074] Gleim #: 8.2.29 -- Source: CPA 1185 A-52

Answer (A) is incorrect because access from multiple locations is more typical of larger computer systems than of personalcomputer systems.

Answer (B) is incorrect because data compression systems encode data to take up less storage space.

Answer (C) is incorrect because batching of transactions requires assembly of data at one place and a delay in updating.

Answer (D) is correct. Real-time processing involves processing an input record and receiving the output soon enough toaffect a current decision-making process. In a real-time system, the user interacts with the system to control an ongoingactivity. Online indicates that the decision maker is in direct communication with the computer. Online, real-time systemsusually permit access to the main computer from multiple remote terminals.

[1075] Gleim #: 8.2.30 -- Source: CPA 1194 A-37

Answer (A) is incorrect because the transactions within the batch are typically not contingent upon one another.

Answer (B) is incorrect because edit checks can be incorporated into batch processing environments. However, the editchecks are used to test the transactions in batches.

Answer (C) is correct. Transactions in a batch computer system are grouped together, or batched, prior to processing.Batches may be processed either daily, weekly, or even monthly. Thus, considerable time may elapse between theinitiation of the transaction and the discovery of an error.

Answer (D) is incorrect because a batch of transactions is typically processed uniformly.

[1076] Gleim #: 8.2.31 -- Source: CPA 594 A-16

Answer (A) is incorrect because the focus on programming is an advantage of using a server. A software program allowstransactions to be processed uniformly.

Answer (B) is correct. In a manual system, one individual is usually assigned responsibility for maintaining andsafeguarding the records. However, in a server environment, the data files may be subject to change by others withoutdocumentation or an indication of who made the changes.

Answer (C) is incorrect because it describes a disadvantage of a manual system.

Answer (D) is incorrect because the method of maintaining the files is independent of the ability to compare thisinformation in the file with the physical count of assets.

Gleim's CIA Test Prep: Part III: Business Analysis and Information TechnologyAnswer Explanations

(1312 questions)

Copyright 2008 Gleim Publications, Inc. Page 330Printed for Mamdouh Farag

Page 331: P.3 Answer Explanation

[1077] Gleim #: 8.2.32 -- Source: Publisher

Answer (A) is incorrect because in time sharing, the CPU spends a fixed amount of time on each program.

Answer (B) is incorrect because multitasking is multiprogramming on a single-user operating system. It is the process ofhaving multiple programs active at a given time, although the CPU is executing instructions from only one program at atime.

Answer (C) is correct. Multiprocessing greatly increases system efficiency by executing multiple programs on the samecomputer at the same time. In systems with only one CPU, although multiple programs may be active simultaneously,program instructions can only be executed for one of these programs at a time.

Answer (D) is incorrect because batch processing entails execution of a list of instructions from beginning to end withoutinterruption.

[1078] Gleim #: 8.2.33 -- Source: Publisher

Answer (A) is incorrect because personal computers may cost less than $1,000. Relative to personal computers, midrangecomputers are more costly, more powerful, have more memory, and are able to interface with more peripheral equipment.Mainframes are large computers with many peripheral devices and large memories. There is virtually no upper limit onthe cost of a mainframe.

Answer (B) is incorrect because, although personal computers have become extremely popular, e.g., for word processing,databases, other business-related activities, and Internet use, large mainframes are still necessary for simulations andprocessing not possible on other smaller computers. Midrange computers fill the gap between personal and mainframecomputers, and a relatively strong demand also exists for these types of processors, particularly for servers used innetworks.

Answer (C) is incorrect because all three computers ordinarily may be programmed in higher-level languages.

Answer (D) is correct. Advances in technology have resulted in less expensive computers and increased computingpower. The cost to process transactions on all kinds of computers has therefore decreased.

[1079] Gleim #: 8.2.34 -- Source: CIA 1196 III-76

Answer (A) is incorrect because creating permanent audit trails of EDI transaction sequences is likely to be accomplishedwith write once, read many times (WORM) devices.

Answer (B) is incorrect because maintaining images of documents with graphical components is likely to be done withredundant arrays of inexpensive disks (RAID). This technology is a magnetic medium that provides a primary storagemethod for imaging systems.

Answer (C) is incorrect because recording the front and back of checks in banking applications is likely to be done with amicroform such as microfilm.

Answer (D) is correct. CD-ROM (compact disk, read-only memory) is a fixed optical medium appropriate for storage ofvery large quantities of unchanging information. Researching standards is the best use of CD-ROM technology for anaccounting department because the data are static enough for periodic updates to remain sufficiently current. CD-ROMscommonly use indexing and searching facilities that make reference works usable. However, the use of CD-ROMs willdecline as the prices of erasable optical disks become more attractive.

Gleim's CIA Test Prep: Part III: Business Analysis and Information TechnologyAnswer Explanations

(1312 questions)

Copyright 2008 Gleim Publications, Inc. Page 331Printed for Mamdouh Farag

Page 332: P.3 Answer Explanation

[1080] Gleim #: 8.2.35 -- Source: CIA 597 III-67

Answer (A) is incorrect because parallel testing is an approach to implementing a new system.

Answer (B) is incorrect because an ITF is an audit tool that uses a fictitious entity against which data transactions areprocessed.

Answer (C) is correct. Performance monitoring is the systematic measurement and evaluation of operating results such astransaction rates, response times, and incidence of error conditions. Performance monitoring will reveal trends in capacityusage so that capacity can be upgraded before response deteriorates to the point that users behave in unintended orundesirable ways.

Answer (D) is incorrect because program code comparison software is used to detect unauthorized changes in programs.

[1081] Gleim #: 8.2.36 -- Source: CIA 594 III-21

Answer (A) is correct. An emulator is a hardware device that permits one system to imitate another, that is, to use thesame data and programs and obtain the same results as the other system. A translator is a program that translates from oneprogramming language into another.

Answer (B) is incorrect because shareware does not transfer data. Shareware is a program that can be freely copied andtested before purchase. If the party obtaining the shareware continues to use it, there is an obligation to send payment tothe author. Shareware typically is found on bulletin boards and online information systems.

Answer (C) is incorrect because there are facilities to transfer data and programs between disparate operating systems.

Answer (D) is incorrect because there are facilities to transfer data and programs between some environments.

[1082] Gleim #: 8.2.37 -- Source: CIA 597 III-75

Answer (A) is incorrect because any identification method may fail to record the movement of some parts.

Answer (B) is correct. Bar-code scanning is a form of optical character recognition. Bar codes are a series of bars ofdifferent widths that represent critical information about the item. They can be read and the information can be instantlyrecorded using a scanner. Thus, bar coding records the movement of parts with minimal labor costs.

Answer (C) is incorrect because each vendor has its own part-numbering scheme.

Answer (D) is incorrect because each vendor has its own identification method, although vendors in the same industryoften cooperate to minimize the number of bar-code systems they use.

[1083] Gleim #: 8.2.38 -- Source: CIA 597 III-79

Answer (A) is incorrect because WORM is an optical storage technique often used as an archival medium.

Answer (B) is incorrect because DAT is primarily used as a backup medium in imaging systems and as a master for CD-ROM.

Answer (C) is correct. CD-ROM is cheaper to produce and ship than the existing paper, yet it permits large volumes oftext and images to be reproduced. Users of the electronic equipment are likely to have access to CD-ROM readers on PCsfor using such documentation.

Answer (D) is incorrect because COM is used for frequent access to archived documents, such as canceled checks inbanking applications.

Gleim's CIA Test Prep: Part III: Business Analysis and Information TechnologyAnswer Explanations

(1312 questions)

Copyright 2008 Gleim Publications, Inc. Page 332Printed for Mamdouh Farag

Page 333: P.3 Answer Explanation

[1084] Gleim #: 8.2.39 -- Source: CIA, adapted

Answer (A) is correct. Optical character recognition (OCR) software converts images of paper documents, as read by ascanning device, into text document computer files.

Answer (B) is incorrect because EDI is the electronic communication between computers of two entities.

Answer (C) is incorrect because bar code scanning uses technology to read bar codes.

Answer (D) is incorrect because shareware is software made available for a fee.

[1085] Gleim #: 8.2.40 -- Source: CIA, adapted

Answer (A) is incorrect because external risks should be evaluated to determine the center’s location.

Answer (B) is incorrect because biometric access systems control physical access to the data center.

Answer (C) is correct. Authorization tables for operating system access address logical controls, not physical controls.

Answer (D) is incorrect because power supply systems and surge protection are included in data center design.

[1086] Gleim #: 8.2.41 -- Source: CIA 590 III-28

Answer (A) is incorrect because it is not meaningful in this context.

Answer (B) is incorrect because a monitor is a video display device that is the most common device used for human-computer interaction.

Answer (C) is incorrect because a video display terminal is a video display device that is the most common device usedfor human-computer interaction.

Answer (D) is correct. Point-of-sale terminals capture data by optical scanning or by keying. The data are thentransmitted to a CPU. The system permits collection of sales data, updating and ordering of inventory, pricing at the pointof sale, and checking of customer credit cards.

[1087] Gleim #: 8.2.42 -- Source: CMA 1294 4-11

Answer (A) is correct. A graphical user interface is part of an operating system with which users may interact. It usesgraphic icons to represent activities, programs, and files. The computer mouse is used to make selections. Windows is agraphical user interface shell initially developed by Microsoft to run in conjunction with DOS. Newer operating systemsalso have this feature. Thus, windowing is the characteristic that allows a computer to display more than one program onthe screen at the same time. Each program has its own section of the screen, but only one program is active.

Answer (B) is incorrect because distributed processing is a means of assigning computer processing to various segments ofa business, with some aspects centralized and some decentralized.

Answer (C) is incorrect because context switching does not relate to the various segments on a computer screen.

Answer (D) is incorrect because a file extension is a means of extending a logical collection of records.

Gleim's CIA Test Prep: Part III: Business Analysis and Information TechnologyAnswer Explanations

(1312 questions)

Copyright 2008 Gleim Publications, Inc. Page 333Printed for Mamdouh Farag

Page 334: P.3 Answer Explanation

[1088] Gleim #: 8.2.43 -- Source: CIA 1193 II-23

Answer (A) is correct. The only difference between the computerized and hard copy form is how the working papers arestored. Electronic working papers are saved on either disks or hard drive, whereas hard copies are stored in a file cabinet.Unlike computerized working papers, hard copies are not subject to computer controls and security procedures.

Answer (B) is incorrect because evidential support is retained and provided on the basis of the nature of the finding andnot the media used for storing working papers.

Answer (C) is incorrect because this capability is not an exclusive function of computerized working papers.

Answer (D) is incorrect because, though the nature of the preliminary survey may change in some cases, the requirementfor this phase of the audit is not eliminated by computerized working papers.

[1089] Gleim #: 8.3.44 -- Source: CMA 1287 5-4

Answer (A) is incorrect because systems analysis is the process of determining user problems and needs, surveying theorganization’s present system, and analyzing the facts.

Answer (B) is incorrect because a feasibility study determines whether a proposed system is technically, operationally,and economically feasible.

Answer (C) is correct. Systems maintenance must be undertaken by systems analysts and applications programmerscontinually throughout the life of a system. Maintenance is the redesign of the system and programs to meet new needs orto correct design flaws. These changes should be part of a regular program of preventive maintenance.

Answer (D) is incorrect because implementation involves training and educating users, testing, conversion, and follow-up.

[1090] Gleim #: 8.3.45 -- Source: CISA

Answer (A) is incorrect because standards must be established.

Answer (B) is incorrect because a technical systems programmer has a role in the development and modification of theoperating system but not necessarily in applications development. The technical support in this area would be provided bysystems analysts rather than programmers.

Answer (C) is correct. Effective systems development requires participation by top management. This can be achievedthrough a steering committee composed of higher-level representatives of system users. The committee approves orrecommends projects and reviews their progress. Studies of the economic, operational, and technical feasibility of newapplications necessarily entail evaluations of existing systems. Another necessary control is the establishment of standardsfor system design and programming. Standards represent user and system requirements determined during systemsanalysis.

Answer (D) is incorrect because a technical systems programmer has a role in the development and modification of theoperating system but not necessarily in applications development.

Gleim's CIA Test Prep: Part III: Business Analysis and Information TechnologyAnswer Explanations

(1312 questions)

Copyright 2008 Gleim Publications, Inc. Page 334Printed for Mamdouh Farag

Page 335: P.3 Answer Explanation

[1091] Gleim #: 8.3.46 -- Source: CIA 592 I-29

Answer (A) is incorrect because the needs of the organization should be the overriding factor in systems development.

Answer (B) is incorrect because the equipment selection should be a function of the processing needs, not vice versa.

Answer (C) is correct. The top-down method begins with analysis of broad organizational goals, objectives, and policiesas a basis for the design process. This step requires an understanding of the entity’s environment and significant activities.The next step is to determine the decisions made by managers and the information required to make them. The necessaryreports, databases, inputs, processing methods, and equipment specifications can then be defined. The weakness of thetop-down approach is that it tends to concentrate on managers’ information needs at the expense of the design of efficienttransaction processing at the operational level.

Answer (D) is incorrect because functional controls should be designed for the new system.

[1092] Gleim #: 8.3.47 -- Source: CIA 1195 III-65

Answer (A) is incorrect because obsolete data fields must be recognized by developers or users. Once recognized, obsoletedata fields can be treated consistently in CASE procedures.

Answer (B) is incorrect because using CASE will not ensure user commitment to new systems if they are poorly designedor otherwise do not meet users’ needs.

Answer (C) is incorrect because, although it has the potential to accelerate system development, CASE cannot ensure thatall programs are optimized for efficiency. In fact, some CASE-developed modules may need to be optimized by hand toachieve acceptable performance.

Answer (D) is correct. CASE is an automated technology (at least in part) for developing and maintaining software andmanaging projects. A benefit of using CASE technology is that it can ensure that data integrity rules, including those forvalidation and access, are applied consistently across all files.

[1093] Gleim #: 8.3.48 -- Source: CIA 592 III-38

Answer (A) is correct. CASE applies the computer to software design and development. It maintains on the computer alibrary of standard program modules and all of the system documentation, e.g., data flow diagrams, data dictionaries, andpseudocode (structured English); permits development of executable input and output screens; and generates programcode in at least skeletal form. Thus, CASE facilitates the creation, organization, and maintenance of documentation andpermits some automation of the coding process. However, information requirements must be determined prior to usingCASE.

Answer (B) is incorrect because CASE may generate program logic design.

Answer (C) is incorrect because CASE may generate computer program code.

Answer (D) is incorrect because CASE may generate program documentation.

Gleim's CIA Test Prep: Part III: Business Analysis and Information TechnologyAnswer Explanations

(1312 questions)

Copyright 2008 Gleim Publications, Inc. Page 335Printed for Mamdouh Farag

Page 336: P.3 Answer Explanation

[1094] Gleim #: 8.3.49 -- Source: CIA 597 III-71

Answer (A) is correct. An object-oriented approach is intended to produce reusable code. Because code segments can bereused in other programs, the time and cost of writing software should be reduced.

Answer (B) is incorrect because object technology has the potential to support faster maintenance of programs written inobject-oriented, but not procedural, languages.

Answer (C) is incorrect because object technology is being applied to relational, but not hierarchical, databases.

Answer (D) is incorrect because object technology is typically implemented in a prototyping environment.

[1095] Gleim #: 8.3.50 -- Source: CIA 596 III-70

Answer (A) is incorrect because neural networking involves hardware or software that imitates the processing activities ofthe human brain.

Answer (B) is correct. Prototyping produces the first model(s) of a new system. This technique usually employs asoftware tool for quick development of a model of the user interface (such as by report or screen), interaction of users withthe system (for example, a menu-screen approach or data entry), and processing logic (the executable module).Prototyping stimulates user participation because the model allows quick exploration of concepts and development ofsolutions with quick results.

Answer (C) is incorrect because reengineering salvages reusable components of existing systems and restructures them todevelop new systems or to improve the old systems.

Answer (D) is incorrect because an application generator is software that can be used to develop an application simply bydescribing its requirements to the computer rather than by writing a procedural program.

[1096] Gleim #: 8.3.51 -- Source: CIA 597 III-72

Answer (A) is incorrect because, instead of traditional design documents, items such as the business model, narratives ofprocess functions, iterative development screens, computer processes and reports, and product descriptions guides areproduced in object-oriented development.

Answer (B) is incorrect because, in general, object-oriented development systems include tracking systems for changesmade in objects and hierarchies.

Answer (C) is incorrect because object-oriented systems are usually developed in client-server environments, so thepotential exists for continuous monitoring of system use. However, continuous monitoring typically occurs during systemoperation, not during development.

Answer (D) is correct. In object-oriented development, all objects in a class inherit the properties of higher classes in thehierarchy. Thus, changes in one object may affect many other objects, and the extent and effects of errors significantlyincrease. Testing one object provides no assurance that the objects are properly coordinated. Accordingly, user acceptancetesting to verify correct functioning of the whole system becomes more important.

Gleim's CIA Test Prep: Part III: Business Analysis and Information TechnologyAnswer Explanations

(1312 questions)

Copyright 2008 Gleim Publications, Inc. Page 336Printed for Mamdouh Farag

Page 337: P.3 Answer Explanation

[1097] Gleim #: 8.3.52 -- Source: CISA

Answer (A) is incorrect because not enough information is given to conclude that priority should be given to the securitysystem.

Answer (B) is incorrect because the MIS manager should not be the only decision maker.

Answer (C) is incorrect because the question indicates that development of both systems is not possible.

Answer (D) is correct. The needs assessment and cost-benefit analysis should be conducted by those responsible formaking the decision. In this case, the information systems steering committee is the appropriate decision maker.

[1098] Gleim #: 8.3.53 -- Source: CIA 1193 I-26

Answer (A) is incorrect because service on a management decision-making committee is an operating responsibility andwould impair audit objectivity.

Answer (B) is correct. Auditor objectivity is not impaired when (s)he recommends standards of control for systems orreviews procedures before implementation. However, drafting procedures for systems and designing, installing, andoperating systems are not audit functions. Thus, reviewing the methodology used by an organization is an appropriateactivity that enables the internal auditor to determine whether (s)he can rely on the systems development activity to designand implement appropriate automated controls within applications.

Answer (C) is incorrect because making recommendations for automated procedures is an operating responsibility.

Answer (D) is incorrect because recommending operational procedures is an operating responsibility.

[1099] Gleim #: 8.3.54 -- Source: CIA 1189 II-29

Answer (A) is incorrect because access controls authorize access to data files.

Answer (B) is incorrect because access controls authorize access to program files.

Answer (C) is incorrect because processing controls ensure the completeness, accuracy, and validity of updating.

Answer (D) is correct. Input controls provide reasonable assurance that data received for computer processing have beenproperly authorized and are in a form suitable for processing, i.e., complete, accurate, and valid. Input controls also relateto rejection, correction, and resubmission of data that were initially incorrect.

[1100] Gleim #: 8.3.55 -- Source: CIA 1193 I-29

Answer (A) is incorrect because having customers specify the name for each item they order would let the companycorrect erroneous order codes once they had been detected, but would not, in general, detect erroneous codes.

Answer (B) is correct. Self-checking digits may be used to detect incorrect codes. The digit is generated by applying analgorithm to the code. During the input process, the check digit is recomputed by applying the same algorithm to the codeactually entered.

Answer (C) is incorrect because separating the parts of the order code with hyphens would make the characters easier toread, but would not cure the problem of transposed characters.

Answer (D) is incorrect because using a master file reference for all order codes would verify the existence of items, butwould not detect erroneous order codes in which transposed characters in an order code match other items.

Gleim's CIA Test Prep: Part III: Business Analysis and Information TechnologyAnswer Explanations

(1312 questions)

Copyright 2008 Gleim Publications, Inc. Page 337Printed for Mamdouh Farag

Page 338: P.3 Answer Explanation

[1101] Gleim #: 8.3.56 -- Source: CIA 597 I-19

Answer (A) is incorrect because self-checking digits detect inaccurate identification numbers. They are an effectivecontrol to ensure that the appropriate part has been identified. However, the control objective is to ensure that datatransfer is complete.

Answer (B) is correct. Batch control totals for the data transferred can be reconciled with the batch control totals in theexisting file. This comparison provides information on the completion of the data transfer. Batch totals may include recordcounts, totals of certain critical amounts, or hash totals. A hash total is a control total without a defined meaning, such asthe total of employee numbers or invoice numbers, that is used to verify the completeness of data. Thus, the hash total forthe employee listing by the personnel department could be compared with the total generated during the payroll run.

Answer (C) is incorrect because passwords help ensure that only authorized personnel make the transfer, not that datatransfer is complete.

Answer (D) is incorrect because field checks are effective input controls, but they do not ensure completeness of datatransfer.

[1102] Gleim #: 8.3.57 -- Source: CIA 596 I-58

Answer (A) is incorrect because reasonableness, limit, and range checks are based upon known limits for giveninformation. For example, the hours worked per week is not likely to be greater than 45.

Answer (B) is correct. Validity checks are tests of identification numbers or transaction codes for validity by comparisonwith items already known to be correct or authorized. For example, Social Security numbers on payroll input records canbe compared with Social Security numbers authorized by the personnel department.

Answer (C) is incorrect because a record count is a control total of the number of records processed during the operationof a program. Financial totals summarize dollar amounts in an information field in a group of records.

Answer (D) is incorrect because a hash total is the number obtained from totaling the same field value for each transactionin a batch. The total has no meaning or value other than as a comparison with another hash total.

[1103] Gleim #: 8.3.58 -- Source: CIA 1192 II-31

Answer (A) is correct. An echo check is a hardware control that provides for a peripheral device to return (echo) a signalsent by the CPU. For example, the CPU sends a signal to the printer, and the printer, just prior to printing, sends a signalback to the CPU verifying that the proper print position has been activated.

Answer (B) is incorrect because a protection ring prevents accidental writing on a tape file. A real-time system would notuse tape files.

Answer (C) is incorrect because hash totals are used to control data sent to a batch system, not a real-time system.

Answer (D) is incorrect because integrated test facilities are useful in testing real-time systems but cannot be used toensure completeness of data transmissions.

Gleim's CIA Test Prep: Part III: Business Analysis and Information TechnologyAnswer Explanations

(1312 questions)

Copyright 2008 Gleim Publications, Inc. Page 338Printed for Mamdouh Farag

Page 339: P.3 Answer Explanation

[1104] Gleim #: 8.3.59 -- Source: CIA 590 II-20

Answer (A) is incorrect because memory protection prohibits programs from accessing memory outside their designatedranges.

Answer (B) is correct. A parity check adds the bits in a character or message and checks the sum to determine if it is oddor even, depending on whether the computer has odd or even parity. This check verifies that all data have been transferredwithout loss. For example, if the computer has even parity, a bit will be added to a binary coded character or message thatcontains an odd number of bits. No bit is added if a character or message in binary form has an even number of bits.

Answer (C) is incorrect because for hardware, validity checking verifies that a machine-level instruction is a validinstruction; for applications, validity checking verifies that transaction data are complete, authorized, and reasonable.

Answer (D) is incorrect because range checking verifies that input data values are within pre-determined ranges.

[1105] Gleim #: 8.3.60 -- Source: CMA 680 5-15

Answer (A) is incorrect because R is not a hexadecimal character. Hexadecimal characters are 0-9 and A-F representing 0to 15 in decimal.

Answer (B) is incorrect because the probability of a computer malfunction’s resulting in the printing of an R is slight.

Answer (C) is incorrect because 2.5 would not be appropriate for a quantity of shirts sold.

Answer (D) is correct. The probable explanation for reporting a quantity using a character other than a digit is that thedata were incorrectly encoded and the computer program did not perform a field check, which would have detected theerror. A field check tests whether a field consists of the proper characters, whether alphabetic, numeric, special, orcombinations thereof.

[1106] Gleim #: 8.3.61 -- Source: CMA 687 5-5

Answer (A) is incorrect because an edit routine is a program initiated prior to regular input to discover errors in databefore entry so that the errors can be corrected.

Answer (B) is incorrect because a completeness check tests whether all data items for a transaction have been entered bythe terminal operator.

Answer (C) is incorrect because the dialogue approach is another screen prompting method for data entry. It is mostappropriate when information is received orally, e.g., by phone.

Answer (D) is correct. To avoid data entry errors in online systems, a preformatted screen approach may be used. It is ascreen prompting approach that involves the display on a monitor of a set of boxes for entry of specified data items. Theformat may even be in the form of a copy of a transaction document. This technique is best suited to conversion of datafrom a source document.

Gleim's CIA Test Prep: Part III: Business Analysis and Information TechnologyAnswer Explanations

(1312 questions)

Copyright 2008 Gleim Publications, Inc. Page 339Printed for Mamdouh Farag

Page 340: P.3 Answer Explanation

[1107] Gleim #: 8.3.62 -- Source: CIA 595 I-27

Answer (A) is incorrect because oral verification also would address the problem.

Answer (B) is incorrect because assigning a sequential number to the customer’s order helps build an audit trail but doesnot address the product identification issue.

Answer (C) is correct. A self-checking digit detects incorrect codes. The digit is generated by applying an algorithm tothe code. During input, the digit is recomputed by applying the algorithm to the code actually entered. Oral verificationalso addresses the problem of incorrectly identifying the product number. Assigning a sequential number to the customer’sorder helps build an audit trail but does not address the product identification issue.

Answer (D) is incorrect because assigning a sequential number to the customer’s order helps build an audit trail but doesnot address the product identification issue.

[1108] Gleim #: 8.3.63 -- Source: CIA 596 I-57

Answer (A) is incorrect because a record count determines the number of documents entered into a process.

Answer (B) is incorrect because an echo check tests the reliability of computer hardware. For example, the CPU sends asignal to a printer that is echoed just prior to printing. The signal verifies that the proper print position has been activated.

Answer (C) is incorrect because a self-checking number is generated by applying an algorithm to an identification number.

Answer (D) is correct. A limit, reasonableness, or range test determines whether an amount is within a predeterminedlimit for given information. It can only detect certain errors (i.e., those that exceed the acceptable limit).

[1109] Gleim #: 8.3.64 -- Source: CIA 582 IV-12

Answer (A) is incorrect because key verification does not detect errors in the source documents.

Answer (B) is incorrect because, although widely used, key verification effectively doubles the work and is expensive.

Answer (C) is correct. Key verification is a procedure to determine if the keying process was performed properly.Information from source documents is rekeyed on a special keyboard by another operator and compared with thatpreviously recorded.

Answer (D) is incorrect because key verification is a manual process.

[1110] Gleim #: 8.3.65 -- Source: CIA 597 III-37

Answer (A) is incorrect because reviewing the computer processing logs is an output control.

Answer (B) is correct. Output controls often include comparing output totals with input and processing totals; reviewingcomputer logs; auditing output reports to ensure that totals, formats, and details are accurate and reconcilable; andspecifying authorized recipients by formal means. The data control group also performs important output control functions.Matching the input data with information held on master or suspense files is a processing control, not an output control. Itensures that data are complete and accurate during updating.

Answer (C) is incorrect because periodically reconciling output reports is an output control.

Answer (D) is incorrect because maintaining formal procedures and documentation specifying authorized recipients is anoutput control.

Gleim's CIA Test Prep: Part III: Business Analysis and Information TechnologyAnswer Explanations

(1312 questions)

Copyright 2008 Gleim Publications, Inc. Page 340Printed for Mamdouh Farag

Page 341: P.3 Answer Explanation

[1111] Gleim #: 8.3.66 -- Source: CIA 1195 III-36

Answer (A) is correct. One risk of end-user computing is that documentation may be poor and that important knowledgemay be limited to one person. The command sequences should have been documented so that other analysts could use andmodify them readily.

Answer (B) is incorrect because the inability of other analysts to understand the command sequences is not a function ofinadequate data backup procedures.

Answer (C) is incorrect because the inability of other analysts to understand the command sequences is not a function ofinadequate testing.

Answer (D) is incorrect because the inability of other analysts to understand the command sequences is not a function ofinadequate use of anti-virus software.

[1112] Gleim #: 8.3.67 -- Source: CIA 1195 III-34

Answer (A) is incorrect because inadequate backups are not the cause of reuse of erroneous code.

Answer (B) is correct. Change control manages changes in information system resources and procedures. It includes aformal change request procedure; assessments of change requests on technical and business grounds; scheduling changes;testing, installing, and monitoring changes; and reporting the status of recorded changes. The analysts were reusingerroneous code that should have been but was not corrected.

Answer (C) is incorrect because inadequate access control is not the cause of reuse of erroneous code.

Answer (D) is incorrect because inadequate testing is not the cause of reuse of erroneous code.

[1113] Gleim #: 8.3.68 -- Source: CIA 593 III-46

Answer (A) is incorrect because verifying that the program move request is authorized ensures that a change wasauthorized, not that the source and executable code match.

Answer (B) is incorrect because requiring program, system, and parallel testing of the code ensures that the code meetstest specifications, not that the source and executable code match.

Answer (C) is incorrect because authorizing programmer access to test libraries only ensures that programmers do nothave access to production libraries, not that the source and executable code match.

Answer (D) is correct. Recompiling source code into the production load library ensures that the source and executablecodes match because the executable code is created from the source code.

[1114] Gleim #: 8.3.69 -- Source: CIA 1196 III-36

Answer (A) is incorrect because EUC systems typically increase flexibility and responsiveness to management’sinformation requests. Such systems are more easily modified.

Answer (B) is incorrect because EUC systems typically reduce application development cycle time.

Answer (C) is correct. End-user developed applications may not be subject to an independent outside review by systemsanalysts and are not created in the context of a formal development methodology. These applications may lack appropriatestandards, controls, quality assurance procedures, and documentation. A risk of end-user applications is that managementmay rely on them as much as traditional applications.

Answer (D) is incorrect because EUC systems typically result in reduced application development and maintenance costs.

Gleim's CIA Test Prep: Part III: Business Analysis and Information TechnologyAnswer Explanations

(1312 questions)

Copyright 2008 Gleim Publications, Inc. Page 341Printed for Mamdouh Farag

Page 342: P.3 Answer Explanation

[1115] Gleim #: 8.3.70 -- Source: CIA 1196 III-37

Answer (A) is correct. Systems analysis is one step that is not absolutely required in the development of a system. Thedesire to produce a system quickly may result in this step being eliminated or poorly implemented. A system is oftenproduced and then analyzed to see if it will satisfy the needs of the organization. In an EUC application, the systemsanalysis is often incomplete or omitted.

Answer (B) is incorrect because, without programming, there would be no system.

Answer (C) is incorrect because, without computer operations, the system would not be able to do anything.

Answer (D) is incorrect because, without users, there would be no need for the system.

[1116] Gleim #: 8.3.71 -- Source: CIA 586 II-39

Answer (A) is incorrect because the use of GAS is normally more efficient. Less time is required to write instructions toaccomplish a function than to manually select and examine items.

Answer (B) is incorrect because the program is generalized, i.e., designed to be used on a variety of systems withoutsignificant modifications.

Answer (C) is incorrect because an advantage is that GAS requires minimal knowledge of computer technology.

Answer (D) is correct. Diversity of programming languages, computers, systems designs, and differing data structuresmakes generalized audit software impossible to apply in certain situations.

[1117] Gleim #: 8.3.72 -- Source: CIA 595 III-74

Answer (A) is incorrect because the end user never acts as a liaison between the IS department and the rest of theorganization.

Answer (B) is incorrect because programmers design, write, test, and document the specific programs developed bysystems analysts. It is not their responsibility to act as a liaison between the IS department and the rest of the organization.

Answer (C) is incorrect because programmers design, write, test, and document the specific programs developed bysystems analysts. It is not their responsibility to act as a liaison between the IS department and the rest of the organization.

Answer (D) is correct. Systems analysts are specifically qualified to analyze and design computer information systems.They survey the existing system, analyze the organization’s information requirements, and design new systems to meetthose needs. Systems analysts communicate with the entire organization and act as a liaison between the organization andthe IS department.

[1118] Gleim #: 8.3.73 -- Source: CIA 588 II-31

Answer (A) is incorrect because writing and revising computer programs are appropriate functions for programmers.

Answer (B) is incorrect because distributing computer reports is an appropriate function of the control group.

Answer (C) is incorrect because maintaining custody and related record keeping for computer programs is appropriate fora computer librarian.

Answer (D) is correct. Computer operators need access to operator instructions. Otherwise, they could not perform theirduties. Operators, however, should not have the authority to change computer programs.

Gleim's CIA Test Prep: Part III: Business Analysis and Information TechnologyAnswer Explanations

(1312 questions)

Copyright 2008 Gleim Publications, Inc. Page 342Printed for Mamdouh Farag

Page 343: P.3 Answer Explanation

[1119] Gleim #: 8.3.74 -- Source: CIA 1191 III-31

Answer (A) is incorrect because the life cycle approach is better suited for structured projects.

Answer (B) is incorrect because the life cycle approach is better suited for large projects.

Answer (C) is incorrect because the life cycle approach is better suited for complex projects.

Answer (D) is correct. The life cycle approach is best employed when systems are large and highly structured, usersunderstand the tasks to be performed by the information system, and the developers have directly applicable experience indesigning similar systems. In the life cycle process, each stage of development is highly structured, and requirements areclearly defined. However, when the task is unstructured, prototyping may be the better approach. Prototyping (anexperimental assurance process) is costly and time-consuming and thus is not currently the most common approach. Itentails developing and putting into operation successively more refined versions of the system until sufficient informationis obtained to produce satisfactory design.

[1120] Gleim #: 8.3.75 -- Source: CIA 1192 III-39

Answer (A) is incorrect because possible vendors for the system and their reputation for quality would be determined afterthe feasibility study.

Answer (B) is incorrect because exposure to computer viruses and other intrusions is part of the information requirementsphase.

Answer (C) is incorrect because methods of implementation such as parallel or cut-over would be determined during theimplementation and operations stage.

Answer (D) is correct. The feasibility study should consider the activity to be automated, the needs of the user, the type ofequipment required, the cost, and the potential benefit to the specific area and the company in general. Thus, technicalfeasibility and cost are determined during this stage.

[1121] Gleim #: 8.3.76 -- Source: CIA 1192 III-87

Answer (A) is correct. The life cycle approach is best employed when systems are large and highly structured, usersunderstand the tasks to be performed by the information system, and the developers have directly applicable experience indesigning similar systems. In the life cycle process, each stage of development is highly structured, and requirements areclearly defined.

Answer (B) is incorrect because the lower the user understanding of tasks and the smaller the project size, the lessappropriate life cycle methodologies are for the problem.

Answer (C) is incorrect because the lower the user understanding of tasks and the more uncertain the requirements, theless appropriate life cycle methodologies are for the problem.

Answer (D) is incorrect because the more uncertain the requirements, the less appropriate life cycle methodologies are forthe problem.

Gleim's CIA Test Prep: Part III: Business Analysis and Information TechnologyAnswer Explanations

(1312 questions)

Copyright 2008 Gleim Publications, Inc. Page 343Printed for Mamdouh Farag

Page 344: P.3 Answer Explanation

[1122] Gleim #: 8.3.77 -- Source: CIA 1191 III-29

Answer (A) is correct. Prototyping (an experimental assurance process) is costly and time-consuming and thus is notcurrently the most common approach. It entails developing and putting into operation successively more refined versionsof the system until sufficient information is obtained to produce satisfactory design. Prototyping is the best approach inthese circumstances because the requirements are difficult to specify in advance and are likely to change significantlyduring development.

Answer (B) is incorrect because the system development life cycle model is appropriate for highly structured operationalapplications whose requirements can be defined in advance. Thus, it is not suitable for the bank’s application.

Answer (C) is incorrect because structured analysis and design technique is a specific application of the systemdevelopment life cycle model.

Answer (D) is incorrect because hierarchy-input-process-output is a specific application of the system development lifecycle model.

[1123] Gleim #: 8.3.78 -- Source: CIA 593 III-47

Answer (A) is incorrect because the first time CASE is used, the bank will have to acquire all the CASE developmentsoftware.

Answer (B) is incorrect because the first time CASE is used, the bank will have to train programmers in its uses.

Answer (C) is correct. CASE applies computers to software design and development. It permits creation and maintenanceof systems documentation on the computer and the automation of a part of the programming effort. Using CASE wouldimprove management of the development process because the CASE software maintains the links between the differentcomponents, provides built-in project management tools, and supplies automated testing aids.

Answer (D) is incorrect because using CASE does not reduce the need for testing to verify that program code matchesspecifications. However, the testing itself is more manageable because of automated testing aids provided by CASE.

[1124] Gleim #: 8.3.79 -- Source: CIA 1192 III-90

Answer (A) is incorrect because overall development costs are higher when requirements change frequently in a life cyclemethodology.

Answer (B) is incorrect because life cycle methodologies are unable to give users a functioning system quickly.

Answer (C) is incorrect because life cycle methodologies require lengthy application development time to achieve afunctioning system.

Answer (D) is correct. The systems development life cycle approach is the most common methodology applied to thedevelopment of large, highly structured application systems. The life cycle approach is based on the idea that aninformation system has a finite life span that is limited by the changing needs of the organization. This cycle isanalytically divisible into stages. A new system life cycle begins when the inadequacy of the current system leads to adecision to develop a new or improved system. This method is a structured process for controlling the creative activityrequired to devise, develop, and implement an information system. The process is described in varying terms by differentwriters, but the nature and sequence of the steps are essentially the same. Life cycle methodologies provide enhancedmanagement and control of the development process because they provide structure for a creative process by dividing itinto manageable steps and specifying what must be produced in each phase.

Gleim's CIA Test Prep: Part III: Business Analysis and Information TechnologyAnswer Explanations

(1312 questions)

Copyright 2008 Gleim Publications, Inc. Page 344Printed for Mamdouh Farag

Page 345: P.3 Answer Explanation

[1125] Gleim #: 8.3.80 -- Source: CPA 1192 A-20

Answer (A) is incorrect because hours worked is an example of a financial (amount) total.

Answer (B) is incorrect because total debits and total credits is a financial total.

Answer (C) is incorrect because net pay is a financial total.

Answer (D) is correct. The three types of control totals are record counts, financial (amount) totals, and hash totals.Record counts establish the number of source documents and reconcile it to the number of output records. Financial(amount) totals compute dollar or amount totals from source documents (e.g., the total dollar amount of invoicesprocessed) and reconcile them with the output records. Hash totals add numbers on input documents that are not normallyadded (e.g., department numbers for payroll processing) and reconcile them with output records.

[1126] Gleim #: 8.3.81 -- Source: CPA AUD R98-4

Answer (A) is incorrect because a hash total is ordinarily the sum of a numeric field.

Answer (B) is incorrect because 4 is a record count.

Answer (C) is incorrect because 204 is an invoice number.

Answer (D) is correct. Input controls in batch computer systems are used to determine that no data are lost or added to thebatch. Depending on the sophistication of a particular system, control may be accomplished by using record counts, batchtotals, or hash totals. The hash total is a control total without a defined meaning, such as the total of employee numbers orinvoice numbers, that is used to verify the completeness of data. The hash total of the invoice numbers is 810.

[1127] Gleim #: 8.3.82 -- Source: CIA 1190 III-23

Answer (A) is incorrect because password authorization is a general control over access to the system or parts of it.

Answer (B) is correct. Self-checking digits may be used to detect incorrect identification numbers. The digit is generatedby applying an algorithm to the ID number. During the input process, the check digit is recomputed by applying the samealgorithm to the code actually entered.

Answer (C) is incorrect because hash totals (otherwise meaningless control totals, e.g., of identification numbers) areappropriate for batch processing.

Answer (D) is incorrect because backup and recovery procedures are general controls.

[1128] Gleim #: 8.3.83 -- Source: CPA AUD R98-5

Answer (A) is correct. Check digit verification is used to identify incorrect identification numbers. The digit is generatedby applying an algorithm to the ID number. During input, the check digit is recomputed by applying the same algorithm tothe entered ID number.

Answer (B) is incorrect because a record count is a control total of the number of transactions in a batch.

Answer (C) is incorrect because a hash total is a control total that is the sum of a field without a defined meaning.

Answer (D) is incorrect because a redundant data check searches for duplicate information in a database.

Gleim's CIA Test Prep: Part III: Business Analysis and Information TechnologyAnswer Explanations

(1312 questions)

Copyright 2008 Gleim Publications, Inc. Page 345Printed for Mamdouh Farag

Page 346: P.3 Answer Explanation

[1129] Gleim #: 8.3.84 -- Source: CIA 1195 III-63

Answer (A) is correct. A validation check at data entry verifying that a quantity field contains only numbers is anexample of a programmatic means of ensuring the accuracy of an input value. Thus, it is an input control.

Answer (B) is incorrect because the purpose of an audit trail control is to ensure that a chronological record of all relevantevents in a system has been recorded.

Answer (C) is incorrect because a processing control ensures that data are complete and accurate during updating.

Answer (D) is incorrect because a data security control ensure that only authorized individuals are permitted to access anduse a system.

[1130] Gleim #: 8.3.85 -- Source: CIA 1187 I-30

Answer (A) is correct. Validity checks are tests of identification numbers or transaction codes for validity by comparisonwith items already known to be correct or authorized. For example, Social Security numbers on payroll input records canbe compared with Social Security numbers authorized by the personnel department.

Answer (B) is incorrect because a range check is based on known limits for given information.

Answer (C) is incorrect because a reasonableness test is based on known limits for given information.

Answer (D) is incorrect because a parity check adds the bits in a character or message and checks the sum to determine ifit is odd or even, depending on whether the computer has odd or even parity.

[1131] Gleim #: 8.3.86 -- Source: CMA 687 5-4

Answer (A) is incorrect because sign checks test data for the appropriate arithmetic sign. For instance, hours worked in apayroll should always be a positive number.

Answer (B) is incorrect because reasonableness tests verify that amounts fall within predetermined limits.

Answer (C) is correct. The program controls listed prescreen or edit data prior to processing, but the sequence check ismost likely to be used only in batch processing. A sequence check tests to determine that records are in proper order. Forexample, a payroll input file can be sorted into Social Security number order. A sequence check can then be performed toverify record order. This control would not apply in a real-time operation because records are not processed sequentially.

Answer (D) is incorrect because a redundancy check requires sending additional data items to serve as a check on theother transmitted data; for example, part of a customer name can be matched against the name associated with thetransmitted customer number.

[1132] Gleim #: 8.3.87 -- Source: CMA 1287 5-16

Answer (A) is correct. A completeness test checks that all data elements are entered before processing. An interactivesystem can be programmed to notify the user to enter the number before accepting the receiving report.

Answer (B) is incorrect because a sequence check tests for the ordering, not omission, of records.

Answer (C) is incorrect because a limit or reasonableness test checks the values of data items against established limits.

Answer (D) is incorrect because a compatibility test (field check) determines whether characters are appropriate to a field.

Gleim's CIA Test Prep: Part III: Business Analysis and Information TechnologyAnswer Explanations

(1312 questions)

Copyright 2008 Gleim Publications, Inc. Page 346Printed for Mamdouh Farag

Page 347: P.3 Answer Explanation

[1133] Gleim #: 8.3.88 -- Source: CIA 1193 II-25

Answer (A) is correct. During each program run in a series, the computer accumulates the totals of transactions that havebeen processed. The run-to-run check reconciles them with the totals forwarded from the previous program run. Run-to-run totals thus ensure completeness of update.

Answer (B) is incorrect because computer matching compares transaction data with referenced fields or records.

Answer (C) is incorrect because computer sequence checks identify changes or breaks in a numerical sequence.

Answer (D) is incorrect because one-for-one checking usually requires manual comparisons of input data elements withprocessing results.

[1134] Gleim #: 8.3.89 -- Source: CIA 1189 II-31

Answer (A) is incorrect because input controls are designed to provide reasonable assurance that data received forprocessing have been properly authorized and are in a suitable form. Input controls also include those that relate torejection, correction, and resubmission of data that were initially incorrect.

Answer (B) is incorrect because processing controls provide reasonable assurance that all transactions are processed asauthorized, no authorized transactions are omitted, and no unauthorized transactions are added.

Answer (C) is correct. Application controls include input, processing, and output controls. Output controls are designedto assure the accuracy of processing results and that only authorized personnel receive the output. For example, comparingthe control total of input documents with the total documents processed is an output control.

Answer (D) is incorrect because access controls prevent unauthorized and improper use of data and programs.

[1135] Gleim #: 8.3.90 -- Source: CMA 685 5-14

Answer (A) is incorrect because programmers should not have access to operational materials.

Answer (B) is incorrect because editing routines check for arithmetic errors prior to processing, and debugging shoulduncover errors in programs.

Answer (C) is correct. Complete, up-to-date documentation of all programs and associated operating procedures isnecessary for efficient operation of a computer installation. Maintenance of programs is important to provide for continuityand consistency of data processing services to users. Program documentation (the program run manual) consists ofproblem statements, systems flowcharts, operating instructions, record layouts, program flowcharts, program listings, testdata, and approval and change sheets.

Answer (D) is incorrect because documentation cannot ensure computer security.

[1136] Gleim #: 8.3.91 -- Source: CIA 593 II-27

Answer (A) is incorrect because monitoring concerns controlling the use of resources.

Answer (B) is incorrect because consistency with strategic plans concern evaluation of the system.

Answer (C) is correct. The accountant’s successor could not use the forecasting model because of inadequatedocumentation. By requiring that documentation standards exist and are followed, the company will enable newemployees to understand internally developed programs when the developer leaves the organization.

Answer (D) is incorrect because maintaining adequate backups for spreadsheet models is necessary, but lack of adequatebackup is not the reason the accountant’s successor could not use the forecasting model.

Gleim's CIA Test Prep: Part III: Business Analysis and Information TechnologyAnswer Explanations

(1312 questions)

Copyright 2008 Gleim Publications, Inc. Page 347Printed for Mamdouh Farag

Page 348: P.3 Answer Explanation

[1137] Gleim #: 8.3.92 -- Source: CIA 596 III-36

Answer (A) is incorrect because production and test programs can be separated only if a specific procedure exists forplacing programs in production libraries. Thus, maintaining the separation requires its own procedure, which maydecrease development efficiency.

Answer (B) is correct. Separating production and test versions of programs facilitates restricting access to productionprograms to the individuals, such as computer operators, who need access. The effect is to separate the incompatiblefunctions of operators and programmers.

Answer (C) is incorrect because separating production and test versions of programs is independent of facilitating userinput on proposed changes.

Answer (D) is incorrect because separating production and test versions of programs restricts access to programs.

[1138] Gleim #: 8.3.93 -- Source: CIA 1196 III-49

Answer (A) is incorrect because security administration is concerned with access to data.

Answer (B) is correct. Change control is the process of authorizing, developing, testing, and installing coded changes soas to minimize the impact on processing and the risk to the system.

Answer (C) is incorrect because problem tracking is concerned with collecting data to be analyzed for corrective action.

Answer (D) is incorrect because problem escalation-procedures are a means of categorizing problems so that the leastskilled person can address them.

[1139] Gleim #: 8.3.94 -- Source: CIA 597 III-49

Answer (A) is incorrect because database reviews determine whether users have gained access to database areas for whichthey have no authorization or whether they are employing programs that provide them with unauthorized privileges toview or change information.

Answer (B) is incorrect because compliance reviews determine whether an organization has complied with applicableinternal and external procedures and regulations.

Answer (C) is correct. Program change control includes (1) maintaining records of change authorizations, code changes,and test results; (2) adhering to a systems development methodology (including documentation); (3) authorizingchangeovers of subsidiary and headquarters’ interfaces; and (4) restricting access to authorized source and executablecodes.

Answer (D) is incorrect because network security audits review the physical and logical controls over the network.

Gleim's CIA Test Prep: Part III: Business Analysis and Information TechnologyAnswer Explanations

(1312 questions)

Copyright 2008 Gleim Publications, Inc. Page 348Printed for Mamdouh Farag

Page 349: P.3 Answer Explanation

[1140] Gleim #: 8.3.95 -- Source: CIA 591 I-36

Answer (A) is incorrect because programmed checks determine the potential accuracy of input data (e.g., a range check).

Answer (B) is incorrect because batch control is used to ensure the completeness and accuracy of input and updating.

Answer (C) is correct. General controls include organizational controls, such as a policy (an implementation control) thatrequires new programs and changes in programs (after adequate testing) to be formally approved before being put intooperation (implemented). This policy is reflected in the maintenance of approval and change sheets with appropriateauthorizations.

Answer (D) is incorrect because one-for-one checking is a technique used to check individual documents for accuracy andcompleteness of data input or update.

[1141] Gleim #: 8.3.96 -- Source: CIA 596 III-61

Answer (A) is incorrect because inability to afford adequate uninterruptible power supply systems is a risk in allcomputing environments.

Answer (B) is incorrect because lack of a GUI is a risk in all computing environments.

Answer (C) is correct. The risk of allowing end users to develop their own applications is decentralization of control.End-user developed applications may not be subject to an independent outside review by systems analysts and are notcreated in the context of a formal development methodology. These applications may lack appropriate standards, controls,and quality assurance procedures. Moreover, when end users create their own applications and files, private informationsystems may proliferate in which data are largely uncontrolled. These systems may contain the same information, but end-user applications may update and define the data in different ways. Thus, determining the location of data and ensuringdata consistency become more difficult because the applications are difficult to integrate.

Answer (D) is incorrect because lack of adequate utility programs is a risk in all computing environments.

[1142] Gleim #: 8.3.97 -- Source: CIA 595 III-40

Answer (A) is incorrect because a screen saver is software used to prevent the burning of an image onto the monitor’sscreen.

Answer (B) is incorrect because the unattended workstation has likely already had the required password provided to gainaccess.

Answer (C) is incorrect because data is only encrypted when stored in a file. While the data or file is being modified by auser, the data is not encrypted. Thus, an unattended workstation would leave data susceptible to unauthorized access.

Answer (D) is correct. Automatic log-off of inactive users is a utility that disconnects a workstation from the mainframeor server after a certain amount of time. Once the workstation has been disconnected, the user must log back into thesystem.

Gleim's CIA Test Prep: Part III: Business Analysis and Information TechnologyAnswer Explanations

(1312 questions)

Copyright 2008 Gleim Publications, Inc. Page 349Printed for Mamdouh Farag

Page 350: P.3 Answer Explanation

[1143] Gleim #: 8.3.98 -- Source: CIA, adapted

Answer (A) is incorrect because review of the computer processing logs is an output control to ensure that data areaccurate and complete.

Answer (B) is correct. Matching the input data with information held on master or suspense files is a processing control,not an output control, to ensure that data are complete and accurate during updating.

Answer (C) is incorrect because periodic reconciliation of output reports is an output control to ensure that data areaccurate and complete.

Answer (D) is incorrect because maintaining formal procedures and documentation specifying authorized recipients is anoutput control to ensure proper distribution.

[1144] Gleim #: 8.3.99 -- Source: CIA, adapted

Answer (A) is incorrect because programmed cutoff controls mitigate the risk of recording transactions in the wrongperiod.

Answer (B) is incorrect because redundant hardware is a control over hardware malfunction.

Answer (C) is correct. Activity logging provides an audit trail of user activity.

Answer (D) is incorrect because transaction error logging controls transactions rather than user terminal activity.

[1145] Gleim #: 8.3.100 -- Source: CIA, adapted

Answer (A) is correct. Implementation controls occur in the systems development process at various points to ensure thatimplementation is properly controlled and managed.

Answer (B) is incorrect because hardware controls ensure that computer hardware is physically secure and check forequipment malfunction.

Answer (C) is incorrect because computer operations controls apply to the work of the computer department and helpensure that programmed procedures are consistently and correctly applied to the storage and processing of data.

Answer (D) is incorrect because data security controls ensure that data files on either disk or tape are not subject tounauthorized access, change, or destruction.

[1146] Gleim #: 8.3.101 -- Source: CIA, adapted

Answer (A) is incorrect because sequence checks determine that records are in the proper order.

Answer (B) is correct. Self-checking digits may be used to detect incorrect identification numbers. A check digit is anextra reference number that follows an identification code and bears a mathematical relationship to the other digits. Theextra digit is input with the data. The identification code can be subjected to an algorithm and compared to the checkdigit.

Answer (C) is incorrect because cross-footing compares an amount to the sum of its components.

Answer (D) is incorrect because field checks are tests of the characters in a field to verify that they are of an appropriatetype for that field (i.e., alphabetic versus numeric).

Gleim's CIA Test Prep: Part III: Business Analysis and Information TechnologyAnswer Explanations

(1312 questions)

Copyright 2008 Gleim Publications, Inc. Page 350Printed for Mamdouh Farag

Page 351: P.3 Answer Explanation

[1147] Gleim #: 8.3.102 -- Source: CIA, adapted

Answer (A) is incorrect because a sequence check determines that records are in proper order.

Answer (B) is incorrect because a check digit is used to detect an incorrect ID number.

Answer (C) is incorrect because a record count is a control total of the number of records processed during the operationof a program.

Answer (D) is correct. With a format check, the computer checks the characteristics of the character content, length, orsign of the individual data fields. Field checks, sign checks and range checks are examples of format checks.

[1148] Gleim #: 8.3.103 -- Source: CIA, adapted

Answer (A) is incorrect because system documentation is not eliminated or deferred by using rapid applicationdevelopment.

Answer (B) is incorrect because project management involves development teams.

Answer (C) is correct. The new system would be developed module by module until completed.

Answer (D) is incorrect because object development might not be of use; if it were, it would increase usage of previouscode.

[1149] Gleim #: 8.3.104 -- Source: CIA, adapted

Answer (A) is correct. An expert system is a knowledge-intensive computer program that captures the expertise of ahuman in limited domains of knowledge.

Answer (B) is incorrect because a neural network is software that attempts to emulate the processing patterns of thebiological brain.

Answer (C) is incorrect because intelligent agents are software programs that use a built-in or learned knowledge base tocarry out specific, repetitive, and predictable tasks for an individual user, business process, or software application. Onthe Internet, an intelligent agent is generally a program that gathers information or performs some other service withoutthe user’s immediate presence and on some regular schedule.

Answer (D) is incorrect because fuzzy logic is rule-based artificial intelligence that tolerates imprecision by using non-specific terms called membership functions to solve problems.

Gleim's CIA Test Prep: Part III: Business Analysis and Information TechnologyAnswer Explanations

(1312 questions)

Copyright 2008 Gleim Publications, Inc. Page 351Printed for Mamdouh Farag

Page 352: P.3 Answer Explanation

[1150] Gleim #: 8.3.105 -- Source: CIA, adapted

Answer (A) is incorrect because, instead of traditional design documents, items such as the business model, narratives ofprocess functions, iterative development screens, computer processes and reports, and product descriptions guides areproduced in object-oriented development, but the existence of specific documents does not affect the importance of useracceptance testing.

Answer (B) is incorrect because, in general, object-oriented development systems do include tracking systems for changesmade to objects and hierarchies.

Answer (C) is incorrect because object-oriented systems are usually developed in client/server environments, there is thepotential for continuous monitoring of system use, but continuous monitoring typically occurs during system operation, notduring development.

Answer (D) is correct. User acceptance testing is more important in object-oriented development because of the fact thatall objects in a class inherit the properties of the hierarchy, which means that changes to one object may affect otherobjects, which increases the importance of user acceptance testing to verify correct functioning of the whole system.

[1151] Gleim #: 8.3.106 -- Source: CIA, adapted

Answer (A) is incorrect because this practice is a wise control, but it does not address the issue of the integrity ofuploaded data. Backups cannot prevent or detect data-upload problems, but can only help correct data errors that a poorupload caused.

Answer (B) is incorrect because this control may be somewhat helpful in preventing fraud in data uploads, but it is oflittle use in preventing errors.

Answer (C) is correct. The same edits and validation routines used for online data entry should be applied to datauploaded from a desktop computer. This could help prevent data errors.

Answer (D) is incorrect because this control is detective in nature, but the error could have already caused erroneousreports and management decisions. Having users try to find errors in uploaded data would be costly.

[1152] Gleim #: 8.3.107 -- Source: CIA, adapted

Answer (A) is correct. The appropriate setup for access in the computer environment is for users to have update accessfor production data. Users need to update data through applications programs, but have have no need to change productionprograms.

Answer (B) is incorrect because application programmers should not be able to change production programs. They shouldsubmit changes to the change control unit.

Answer (C) is incorrect because application programmers should never have update access to production data. Users haveno need to change production programs.

Answer (D) is incorrect because application programmers should not be able to change production programs. They shouldsubmit changes to the change control unit. Application programmers should never have update access to production data.

Gleim's CIA Test Prep: Part III: Business Analysis and Information TechnologyAnswer Explanations

(1312 questions)

Copyright 2008 Gleim Publications, Inc. Page 352Printed for Mamdouh Farag

Page 353: P.3 Answer Explanation

[1153] Gleim #: 8.3.108 -- Source: CIA, adapted

Answer (A) is incorrect because key verification ensures the accuracy of selected fields by requiring a different individualto re-key them.

Answer (B) is incorrect because sequence checks are used to ensure the completeness of input or update data by checkingthe use of pre-assigned document serial numbers.

Answer (C) is incorrect because computer matching entails checking selected fields of input data with information held ina suspense or master file.

Answer (D) is correct. Users can gain access to databases from terminals only through established recognition andauthorization procedures; thus, unauthorized access is prevented.

[1154] Gleim #: 8.3.109 -- Source: CIA 1192 I-33

Answer (A) is incorrect because installing a logging system for program access would permit detection of unauthorizedaccess but not prevent it.

Answer (B) is incorrect because monitoring physical access to program library media would control only unauthorizedphysical access.

Answer (C) is correct. An important operating control is to establish a library to preclude misplacement, misuse, or theftof data files, programs and documentation. A librarian should perform this custodianship function and be appropriatelyaccountable. Restricting physical and logical access secures programs from unauthorized use, whether in person orremotely via terminals.

Answer (D) is incorrect because denying all remote access via terminals would likely be inefficient and would not secureprogram libraries against physical access.

[1155] Gleim #: 8.4.110 -- Source: Publisher

Answer (A) is incorrect because generalized audit software is written to interface with many different client systems.

Answer (B) is incorrect because the purposes and users of this software must be defined before it is written.

Answer (C) is incorrect because generalized audit software purchased “off the shelf” requires less computer expertise thanspecialized software created by the auditor.

Answer (D) is correct. Specialized audit software is written in a procedure- or problem-oriented language to fulfill aspecific set of audit tasks. The purposes and users of the software are well defined before the software is written. Auditorsdevelop specialized audit software for the following reasons:

Unavailability of alternative software1.Functional limitations of alternative software2.Efficiency considerations3.Increased understanding of systems4.Opportunity for easy implementation5.Increased auditor independence and prestige6.

Gleim's CIA Test Prep: Part III: Business Analysis and Information TechnologyAnswer Explanations

(1312 questions)

Copyright 2008 Gleim Publications, Inc. Page 353Printed for Mamdouh Farag

Page 354: P.3 Answer Explanation

[1156] Gleim #: 8.4.111 -- Source: CPA 1188 A-57

Answer (A) is correct. The question relates to using the computer as an audit tool. To use a personal computer for thispurpose effectively and efficiently, the auditor must have the appropriate hardware and software.

Answer (B) is incorrect because access to data does not relate directly to the efficient and effective use of a personalcomputer.

Answer (C) is incorrect because access to data does not relate directly to the efficient and effective use of a personalcomputer.

Answer (D) is incorrect because the selection of the appropriate sample size does not apply when using softwareapplications because the entire population can be tested.

[1157] Gleim #: 8.4.112 -- Source: CPA 593 A-40

Answer (A) is incorrect because parallel simulation involves using an auditor’s program to reproduce the logic ofmanagement’s program.

Answer (B) is incorrect because computer software makes accessing company files much faster and easier.

Answer (C) is incorrect because many audit spreadsheet programs are available.

Answer (D) is correct. The auditor is required to evaluate the adequacy and effectiveness of the system of internal controland to assess risk to plan the audit. This assessment is a matter of professional judgment that cannot be accomplished witha computer.

[1158] Gleim #: 8.4.113 -- Source: CIA 586 I-34

Answer (A) is incorrect because independence is jeopardized when an operator is involved in the process.

Answer (B) is correct. Independence can be preserved when the auditor acquires general audit software (GAS) from anexternal source rather than relying on auditee-developed audit software. Also, efficiency is enhanced to the extent GAScan be used (as compared to manual auditing or writing special audit programs).

Answer (C) is incorrect because printing out the entire file is both unnecessary and inefficient.

Answer (D) is incorrect because overreliance on an auditee’s programmer impairs independence.

[1159] Gleim #: 8.4.114 -- Source: Publisher

Answer (A) is incorrect because identifying gaps is a function of major CAATs software packages.

Answer (B) is correct. CAATs software has the ability to extract data files containing specified criteria, such as the sizeof a data field, but it does not enable the auditor to change the format of this information within management’s computersystem. However, the auditor can specify the format of data fields in CAATs software-produced reports.

Answer (C) is incorrect because merging files is a function of major CAATs software packages.

Answer (D) is incorrect because aging is a function of major CAATs software packages.

Gleim's CIA Test Prep: Part III: Business Analysis and Information TechnologyAnswer Explanations

(1312 questions)

Copyright 2008 Gleim Publications, Inc. Page 354Printed for Mamdouh Farag

Page 355: P.3 Answer Explanation

[1160] Gleim #: 8.4.115 -- Source: CIA 1196 III-61

Answer (A) is incorrect because utility programs are provided by manufacturers of equipment to perform routineprocessing tasks.

Answer (B) is incorrect because an electronic bulletin board system is a centralized information source and messageswitching system and therefore does not provide a repository to store messages for each auditor.

Answer (C) is correct. Electronic mail permits transfer, receipt, and storage of messages within or between computersystems. The “mail” consists of electronically transmitted messages. A user’s “mailbox” is the storage area allocated formessages. The advantages of electronic mail are high-speed transmission, reduction of message preparation costs, and thepossibility of sending or reading messages at a convenient time.

Answer (D) is incorrect because a PBX is a telecommunications system that routes calls to particular extensions within anorganization.

[1161] Gleim #: 8.4.116 -- Source: Publisher

Answer (A) is incorrect because the Internet is a useful audit tool for gathering and disseminating audit-relatedinformation.

Answer (B) is correct. Users transmitting sensitive information across the Internet must understand the threats that arisethat could compromise the confidentiality of the data. Security measures, such as encryption technology, need to be takento ensure that the information is viewed only by those authorized to view it.

Answer (C) is incorrect because the major use of the Internet by internal auditors is electronic communication.

Answer (D) is incorrect because web browsing leaves an electronic record of the user’s search path.

[1162] Gleim #: 8.4.117 -- Source: CIA 595 III-69

Answer (A) is incorrect because privileged software may be needed on the mainframe to modify programs and data.

Answer (B) is incorrect because privileged access may be necessary to modify the final versions of applications.

Answer (C) is correct. Since certain utility software may have privileged access to software and data stored on themainframe, management must control the use of this utility software. Management should limit the use of this software toonly those individuals with appropriate authority.

Answer (D) is incorrect because authorized users must access sensitive programs and data through their workstations thatare connected to the mainframe.

Gleim's CIA Test Prep: Part III: Business Analysis and Information TechnologyAnswer Explanations

(1312 questions)

Copyright 2008 Gleim Publications, Inc. Page 355Printed for Mamdouh Farag

Page 356: P.3 Answer Explanation

[1163] Gleim #: 8.5.118 -- Source: CIA 591 II-33

Answer (A) is incorrect because changes in organizational reporting requirements are not new issues related to advancingcomputer technology.

Answer (B) is incorrect because controls over library tape procedures have not been materially changed by advancingcomputer technology.

Answer (C) is correct. Advancing computer technology presents more complex audit environments. With the advent ofsystems that permit remote access, the risk that unauthorized parties may obtain or tamper with important information isincreased.

Answer (D) is incorrect because changes in organizational behavior are not directly associated with auditorresponsibilities in advancing technology.

[1164] Gleim #: 8.5.119 -- Source: CIA 1196 III-67

Answer (A) is correct. Senior management is responsible for risk assessment, including identification of risks andconsideration of their significance, the likelihood of their occurrence, and how they should be managed. Seniormanagement is also responsible for establishing organizational policies regarding computer security and implementing acompliance structure. Thus, senior management should assess the risks to the integrity, confidentiality, and availability ofinformation systems data and resources.

Answer (B) is incorrect because assignment of access privileges is the responsibility of security management.

Answer (C) is incorrect because determining ownership of data is the responsibility of security management. In thiscontext, ownership of data or resources mean responsibility for ultimate use or disposition.

Answer (D) is incorrect because training employees in security matters is the responsibility of security management.

[1165] Gleim #: 8.5.120 -- Source: Publisher

Answer (A) is incorrect because risk assessment is a proper response to technology challenges.

Answer (B) is incorrect because internal control is a proper response to technology challenges.

Answer (C) is correct. A series of responses to technology challenges is outlined by eSAC. These responses include riskassessment of e-commerce, internal control objectives, and e-assurance services. While businesses must respond totechnology challenges, the response does not necessarily require the minimization of changes.

Answer (D) is incorrect because e-assurance services is a proper response to technology challenges.

[1166] Gleim #: 8.5.121 -- Source: Publisher

Answer (A) is correct. According to eSAC, one of the key technology challenges is privacy. Privacy is recognizedinternationally as a fundamental human right. The globalization of business through the Internet has resulted in legislationto protect personal information, but the requirements of different nations vary.

Answer (B) is incorrect because the use and development of software has outstripped the gains in computer processingand speed.

Answer (C) is incorrect because open systems are subject to more risk than closed, private systems.

Answer (D) is incorrect because, with the advancement of technology, system boundaries are becoming indistinct asinteractions with suppliers, customers, partners, and associates increases.

Gleim's CIA Test Prep: Part III: Business Analysis and Information TechnologyAnswer Explanations

(1312 questions)

Copyright 2008 Gleim Publications, Inc. Page 356Printed for Mamdouh Farag

Page 357: P.3 Answer Explanation

[1167] Gleim #: 8.5.122 -- Source: Publisher

Answer (A) is incorrect because confidentiality is usually an issue with regard to trade secrets and other intellectualproperty.

Answer (B) is correct. Accountability is the control attribute that identifies the source of a transaction. It specifiesemployees’ roles, actions, and responsibilities. Thus, the person who caused a transaction is identifiable. Fundamentalconcepts of accountability are data ownership, identification, and authentication.

Answer (C) is incorrect because physical security is the restriction of access to processing and storage devices.

Answer (D) is incorrect because privacy is the attribute most often applicable to personal information about employeesand customers.

[1168] Gleim #: 8.6.123 -- Source: Publisher

Answer (A) is correct. COBIT (Control Objectives for Information and related Technology) is an IT control frameworkcopyrighted by the Information Systems Audit and Control Foundation (ISACF). COBIT is a set of guidelines to assistmanagement and business process owners in implementing adequate controls over IT processes and resources. It isdesigned to be an IT governance tool that facilitates understanding and managing the risks and benefits associated withinformation and related IT.

Answer (B) is incorrect because it refers to the eSAC model.

Answer (C) is incorrect because Electronic Systems Assurance and Control (eSAC) is the update of the previous SystemsAuditability and Control.

Answer (D) is incorrect because COBIT is copyrighted by ISACF. The Committee of Sponsoring Organizations publishedCOSO.

[1169] Gleim #: 8.6.124 -- Source: Publisher

Answer (A) is incorrect because management is a distinct audience of COBIT.

Answer (B) is incorrect because users are a distinct audience of COBIT.

Answer (C) is correct. COBIT is targeted at three distinct audiences: management, users, and auditors. Managementmust balance risks and control costs in the volatile IT environment. Users need assurance about the security of, andcontrols over, internal or third-party IT services. Auditors must be able to support their opinions conveyed to managementand others about the state of internal control. Because shareholders are not directly involved with IT services used in theday-to-day management of an organization, they are not a targeted audience of COBIT.

Answer (D) is incorrect because auditors are a distinct audience of COBIT.

Gleim's CIA Test Prep: Part III: Business Analysis and Information TechnologyAnswer Explanations

(1312 questions)

Copyright 2008 Gleim Publications, Inc. Page 357Printed for Mamdouh Farag

Page 358: P.3 Answer Explanation

[1170] Gleim #: 8.6.125 -- Source: CIA 1191 III-32

Answer (A) is incorrect because capacity of the hard disk is a measure of memory.

Answer (B) is incorrect because main memory storage capacity is a measure of memory.

Answer (C) is correct. Processing speed is commonly calculated in terms of arithmetic-logic operations performed persecond. This method is a function of the main processing chip’s cycle speed stated in gigahertz.

Answer (D) is incorrect because read only memory is main memory that ordinarily cannot be modified by the user. It is nota performance measure.

[1171] Gleim #: 8.6.126 -- Source: CIA, adapted

Answer (A) is incorrect because legibility of image data is important to its use, but is independent of using the wrongimage.

Answer (B) is incorrect because accuracy of image data is important to its use, but is independent of using the wrongimage.

Answer (C) is correct. Data integrity is a protectibility objective. If index data for image processing systems arecorrupted, users will likely be relying on the wrong images.

Answer (D) is incorrect because maintaining the initial sequence of index data may not be possible as the image data ismodified and images are added/dropped.

[1172] Gleim #: 8.6.127 -- Source: CIA, adapted

Answer (A) is incorrect because write-once-read-many (WORM) is an optical storage technique often used as an archivalmedium.

Answer (B) is incorrect because digital audiotape (DAT) is primarily used as a backup medium in imaging systems and asa master for CD-ROM.

Answer (C) is correct. Compact-disc/read-only memory (CD-ROM) would be cheaper to produce and ship than theexisting paper yet permit large volumes of text and images to be reproduced. Users of the electronic equipment are likelyto have access to CD-ROM readers on personal computers so that they could use the documentation on CD-ROM.

Answer (D) is incorrect because computer-output-to-microform (COM) is used for frequent access to archived documentssuch as canceled checks in banking applications.

[1173] Gleim #: 9.1.1 -- Source: Publisher

Answer (A) is incorrect because auditor documentation is not as crucial as data integrity.

Answer (B) is incorrect because efficiency does not affect the basis for critical auditor decisions using informationprovided by the system.

Answer (C) is correct. Controls are intended to ensure the integrity, confidentiality, and availability of information. Anauditor relies on the integrity of the system’s data and programs in making critical decisions throughout the audit process.

Answer (D) is incorrect because rejected and suspense item controls represent a portion of the techniques used to ensuredata integrity.

Gleim's CIA Test Prep: Part III: Business Analysis and Information TechnologyAnswer Explanations

(1312 questions)

Copyright 2008 Gleim Publications, Inc. Page 358Printed for Mamdouh Farag

Page 359: P.3 Answer Explanation

[1174] Gleim #: 9.1.2 -- Source: CIA 1195 I-31

Answer (A) is incorrect because potential loss, the probability thereof, and the cost and effectiveness of security measuresare important elements of the analysis.

Answer (B) is incorrect because potential loss, the probability thereof, and the cost and effectiveness of security measuresare important elements of the analysis.

Answer (C) is incorrect because potential loss, the probability thereof, and the cost and effectiveness of security measuresare important elements of the analysis.

Answer (D) is correct. Potential loss is the amount of dollar damages associated with a security problem or loss of assets.Potential loss times the probability of occurrence is an estimate (expected value) of the exposure associated with lack ofsecurity. It represents a potential benefit associated with the implementation of security measures. To perform a cost-benefit analysis, the costs should be considered. Thus, all three items need to be addressed.

[1175] Gleim #: 9.1.3 -- Source: CIA 1195 I-28

Answer (A) is correct. Access should be limited to those whose activities necessitate access to the computer system.Moreover, the degree of access allowed should be consistent with an individual’s responsibilities. Restricting access toparticular individuals rather than groups or departments clearly establishes specific accountability. Not everyone in agroup will need access or the same degree of access. Thus, passwords assigned to individuals should be required foridentification of users by the system. Furthermore, data should be restricted at the field level, not the workstation level. Itmay be possible to limit access to a workstation, but most workstations are connected to larger mainframe or networkdatabases. Thus, the security at the workstation level only would be insufficient.

Answer (B) is incorrect because access should be restricted to particular individuals on a need-to-know basis, data shouldbe restricted at the field level, and use should be limited to necessary functions performed by the accountable individual.

Answer (C) is incorrect because access should be restricted to particular individuals on a need-to-know basis, data shouldbe restricted at the field level, and use should be limited to necessary functions performed by the accountable individual.

Answer (D) is incorrect because access should be restricted to particular individuals on a need-to-know basis, data shouldbe restricted at the field level, and use should be limited to necessary functions performed by the accountable individual.

[1176] Gleim #: 9.1.4 -- Source: CIA 592 III-27

Answer (A) is incorrect because use of an integrated test facility (ITF) is a technique by which an auditor selectstransactions and processing functions and applies the transactions to a fictitious entity during a normal processing cyclealong with regular transactions. This technique cannot determine whether the data themselves are legitimate.

Answer (B) is incorrect because tracing follows the path of a transaction during processing but is inadequate to determinewhether a transaction is legitimate.

Answer (C) is incorrect because transaction selection uses an independent computer program to monitor and selecttransactions for internal audit review. Like tracing, it fails to determine whether a transaction is legitimate. It would be anappropriate technique to apply to transactions suspected to be illegitimate.

Answer (D) is correct. An access log should be used to record all attempts to use the system. The date and time, codesused, mode of access, and data involved are recorded. The system should monitor unsuccessful attempts because repeatedattempts could suggest that someone is trying random or patterned character sequences in order to identify a password.

Gleim's CIA Test Prep: Part III: Business Analysis and Information TechnologyAnswer Explanations

(1312 questions)

Copyright 2008 Gleim Publications, Inc. Page 359Printed for Mamdouh Farag

Page 360: P.3 Answer Explanation

[1177] Gleim #: 9.1.5 -- Source: CIA 593 II-23

Answer (A) is incorrect because, although there is a migration of control of this type away from applications to othersoftware, the large bulk of these controls still reside in application software.

Answer (B) is correct. Utility programs perform routine functions (e.g., sorting and copying), are available to all users,and are promptly available for many different applications. Utility programs are one of the more serious weaknesses indata access security because some can bypass normal access controls.

Answer (C) is incorrect because access control software has as one of its primary objectives improving data access securityfor all data on the system.

Answer (D) is incorrect because most database management systems provide for improved data access security while theyare running.

[1178] Gleim #: 9.1.6 -- Source: CIA 1195 I-36

Answer (A) is incorrect because users often choose passwords that are easily guessed.

Answer (B) is incorrect because a program to test passwords is useful but less effective than see-through authentication.

Answer (C) is correct. See-through authentication techniques, such as the one described, require the user to have two ofthe three important elements to authenticate oneself to the system, i.e., a possession (the card used to generate thepassword), knowledge (the new password), or a personal characteristic (e.g., fingerprints).

Answer (D) is incorrect because limiting access to times and locations is helpful in certain environments but not when thesystem allows dial-up access.

[1179] Gleim #: 9.1.7 -- Source: J. Brooks

Answer (A) is incorrect because an echo check relates to the accuracy of signals sent from or to a terminal.

Answer (B) is correct. A device authorization table restricts file access to those physical devices that logically needaccess. Because it is illogical for anyone to access the accounts receivable file from a manufacturing terminal, the deviceauthorization table will deny access even when a valid password is used.

Answer (C) is incorrect because dial-up terminals provide less security than hardwired terminals. Any terminal may dialinto the communications port using public telephones.

Answer (D) is incorrect because, although data encryption (transmitting data in code form) might make the data unusable,it would not prevent access.

[1180] Gleim #: 9.1.8 -- Source: CIA 1195 III-32

Answer (A) is correct. The objective of security software is to control access to information system resources, such asprogram libraries, data files, and proprietary software. Security software identifies and authenticates users, controls accessto information, and records and investigates security related events and data.

Answer (B) is incorrect because security software will control the use of utilities, not their installation.

Answer (C) is incorrect because antivirus software detects the presence of viruses.

Answer (D) is incorrect because security software may be a tool to establish, but does not monitor, separation of duties.

Gleim's CIA Test Prep: Part III: Business Analysis and Information TechnologyAnswer Explanations

(1312 questions)

Copyright 2008 Gleim Publications, Inc. Page 360Printed for Mamdouh Farag

Page 361: P.3 Answer Explanation

[1181] Gleim #: 9.1.9 -- Source: CIA 597 I-5

Answer (A) is incorrect because asynchronous transmission is a method of data transmission, not a means of safeguardingdata. It is used for slow, irregular transmissions, such as from a keyboard terminal. Each character is marked by a startand stop code.

Answer (B) is correct. Encryption software uses a fixed algorithm to manipulate plaintext and an encryption key (a set ofrandom data bits used as a starting point for application of the algorithm) to introduce variation. Although data may beaccessed by tapping into the transmission line, the encryption key is necessary to understand the data being sent.

Answer (C) is incorrect because, although fiber-optic transmission lines are difficult to tap, their use will not prevent theftof unencrypted data by someone who has access to them.

Answer (D) is incorrect because use of passwords will control access at the sending location and the head-office computer.However, passwords will not prevent someone from tapping the transmission line.

[1182] Gleim #: 9.1.10 -- Source: CIA 1196 III-78

Answer (A) is incorrect because no encryption approach absolutely guarantees the secrecy of data.

Answer (B) is incorrect because keys may also be distributed electronically via secure key transporters.

Answer (C) is correct. Encryption software uses a fixed algorithm to manipulate plaintext and an encryption key (a set ofrandom data bits used as a starting point for application of the algorithm) to introduce variation. The machine instructionsnecessary to encrypt and decrypt data constitute system overhead. As a result, processing speed may be slowed.

Answer (D) is incorrect because periodic password changes are needed. Passwords are the typical means of validatingusers’ access to unencrypted data.

[1183] Gleim #: 9.1.11 -- Source: CIA 1190 I-34

Answer (A) is incorrect because password security for access to the system permits all departmental employees access toall documents in the system.

Answer (B) is incorrect because this system uses no floppy disks.

Answer (C) is incorrect because periodic server backup and storage in a secure area is a good security/backup procedure,but it would not prevent access to sensitive documents online.

Answer (D) is correct. Different passwords may be required to access the system, to read certain files, and to performcertain other functions. Required entry of passwords for access to individual documents is the best single control overunauthorized access to sensitive documents.

[1184] Gleim #: 9.1.12 -- Source: CIA 1196 III-62

Answer (A) is incorrect because authentication using fingerprints is a biometric measure.

Answer (B) is incorrect because authentication using retina patterns is a biometric measure.

Answer (C) is incorrect because authentication using speech patterns is a biometric measure.

Answer (D) is correct. Biometric technologies are automated methods of establishing an individual’s identity usingphysiological or behavioral traits. These characteristics include fingerprints, retina patterns, hand geometry, signaturedynamics, speech, and keystroke dynamics.

Gleim's CIA Test Prep: Part III: Business Analysis and Information TechnologyAnswer Explanations

(1312 questions)

Copyright 2008 Gleim Publications, Inc. Page 361Printed for Mamdouh Farag

Page 362: P.3 Answer Explanation

[1185] Gleim #: 9.1.13 -- Source: Publisher

Answer (A) is incorrect because a firewall cannot adequately protect a system against computer viruses.

Answer (B) is correct. A firewall is a device that separates two networks and prevents passage of specific types ofnetwork traffic while maintaining a connection between the networks. Generally, an Internet firewall is designed toprotect a system from unauthenticated logins from outside users, although it may provide several other features as well.

Answer (C) is incorrect because industrial spies need not leak information through the firewall. A telephone or floppy diskare much more common means of sharing confidential information.

Answer (D) is incorrect because a firewall cannot adequately protect against a Trojan horse (a program, such as a game,that appears friendly but that actually contains applications destructive to the computer system) or any other program thatcan be executed in the system by an internal user.

[1186] Gleim #: 9.1.14 -- Source: CIA 1195 III-37

Answer (A) is correct. Antivirus software designed to identify and remove known viruses is sometimes known as avaccine. A vaccine works only for known viruses and may not be completely effective for variants of those viruses.

Answer (B) is incorrect because having antivirus software is unlikely to make software installation overly complex.

Answer (C) is incorrect because antivirus software need not interfere with system operations. Its execution can bescheduled in advance so as not to interfere with running programs.

Answer (D) is incorrect because antivirus software can be set to execute at times when it would not consume too manysystem resources, e.g., at startup.

[1187] Gleim #: 9.1.15 -- Source: CIA 594 I-32

Answer (A) is incorrect because passwords concern authorization, not accuracy of data.

Answer (B) is incorrect because passwords do not prevent physical access to the computer.

Answer (C) is incorrect because passwords concern authorization, not completeness of data.

Answer (D) is correct. The use of passwords is an effective control in an online system to prevent unauthorized access tocomputer files. Lists of authorized users are maintained in the computer. The entry of passwords or ID numbers; aprearranged act of personal questions; and use of badges, magnetic cards, or optically scanned cards may be combined toavoid unauthorized access.

[1188] Gleim #: 9.1.16 -- Source: CIA 595 III-37

Answer (A) is incorrect because it is not an objective of logical security control.

Answer (B) is incorrect because it is not an objective of logical security control.

Answer (C) is correct. The primary objective of security controls for information systems is to restrict access to data andresources (both hardware and software) to only authorized individuals.

Answer (D) is incorrect because it is not an objective of logical security control.

Gleim's CIA Test Prep: Part III: Business Analysis and Information TechnologyAnswer Explanations

(1312 questions)

Copyright 2008 Gleim Publications, Inc. Page 362Printed for Mamdouh Farag

Page 363: P.3 Answer Explanation

[1189] Gleim #: 9.1.17 -- Source: Publisher

Answer (A) is incorrect because, although passwords should be changed periodically, changing a long, non-reusablepassword weekly encourages employees to write each new password down in order to remember it, a practice notconsidered conducive to the control structure.

Answer (B) is incorrect because the duties of authorization and control over assets should be separated.

Answer (C) is incorrect because, although failed log-in attempts should be logged, it is considered bad practice to recordthe password of failed log-in attempts. Employees often mistype their passwords, and therefore access to the log by aninfiltrator could facilitate breaking into a user’s account.

Answer (D) is correct. Management’s network security policy should include measures to ensure that old and unusedaccounts are removed promptly. If employees’ accounts expire semiannually, reasonable assurance is provided thataccounts in use by unauthorized employees do not exist.

[1190] Gleim #: 9.1.18 -- Source: Publisher

Answer (A) is incorrect because a password should not be displayed.

Answer (B) is incorrect because a minimum length requirement is more likely.

Answer (C) is correct. Security measures include changing passwords frequently, that is, establishing a relatively shortmaximum retention period; not displaying or printing passwords; setting minimum lengths; prohibiting the use of certainwords, character strings, or names; mandating a minimum retention period, so users cannot promptly change passwordsback to their old and convenient values; and retaining old passwords to prevent their use.

Answer (D) is incorrect because retention of old passwords prevents their reuse.

[1191] Gleim #: 9.1.19 -- Source: CIA 595 III-45

Answer (A) is correct. Utility programs can be used to gain access to almost any file. However, gaining access to a file ofpasswords would be useless if those passwords were encrypted prior to placing them in the file.

Answer (B) is incorrect because the utility program could gain access to all passwords in the file, regardless of thehierarchy.

Answer (C) is incorrect because if the user has access to utility programs, (s)he already has access (is logged on) to thesystem.

Answer (D) is incorrect because a peer-to-peer network relates to connectivity of similar networks.

Gleim's CIA Test Prep: Part III: Business Analysis and Information TechnologyAnswer Explanations

(1312 questions)

Copyright 2008 Gleim Publications, Inc. Page 363Printed for Mamdouh Farag

Page 364: P.3 Answer Explanation

[1192] Gleim #: 9.1.20 -- Source: CIA 1194 I-27

Answer (A) is incorrect because the built-in access controls should be retained until replaced with a more comprehensiveand cost-effective system.

Answer (B) is correct. Access control software provides comprehensive and coordinated security. It permits authorizedusers to gain access only for purposes of performing their assigned duties and restricts employees from performingincompatible functions. A comprehensive system is more cost-effective than programming access controls into eachapplication.

Answer (C) is incorrect because utility software does not usually perform security functions.

Answer (D) is incorrect because a comprehensive system is more cost-effective than programming access controls intoeach application.

[1193] Gleim #: 9.1.21 -- Source: CIA 1194 I-28

Answer (A) is incorrect because discussing the password removal process does not determine whether ex-employees arestill using or are able to use their passwords to access the databases.

Answer (B) is incorrect because the computer logs should be compared with current payroll lists.

Answer (C) is correct. To determine if ex-employees are accessing the company’s automated database, the auditor shouldobtain the log showing database accesses. This log should be compared with current payroll lists to see if anyone not onthe payroll is still accessing or is able to access the databases.

Answer (D) is incorrect because reviewing the access control software does not indicate whether ex-employees can accessor are accessing the databases.

[1194] Gleim #: 9.1.22 -- Source: Publisher

Answer (A) is incorrect because biometric technologies do not eliminate the need for specialists who evaluate and monitorsecurity needs.

Answer (B) is incorrect because checking to see if information is reasonable is related to input controls, not accesscontrols.

Answer (C) is correct. The purpose of passwords is to prevent access by unauthorized users just as the more sophisticatedcontrol of employee biometric attributes. The use of passwords is an effective control in an online system to preventunauthorized access to computer systems. However, biometric technologies are more sophisticated and difficult tocompromise.

Answer (D) is incorrect because virus protection software prevents damage to data in a system, not access to a system.

[1195] Gleim #: 9.1.23 -- Source: CPA AUD R02-20

Answer (A) is correct. To be effective, passwords should consist of random letters, symbols, and numbers and should notcontain words or phrases. Accordingly, computer system users should avoid employing words for or in their passwords.

Answer (B) is incorrect because words should not be used for or in passwords.

Answer (C) is incorrect because words should not be used for or in passwords.

Answer (D) is incorrect because words should not be used for or in passwords.

Gleim's CIA Test Prep: Part III: Business Analysis and Information TechnologyAnswer Explanations

(1312 questions)

Copyright 2008 Gleim Publications, Inc. Page 364Printed for Mamdouh Farag

Page 365: P.3 Answer Explanation

[1196] Gleim #: 9.1.24 -- Source: CPA AUD R02-9

Answer (A) is incorrect because the term "Heuristic terminal" is not meaningful in this context.

Answer (B) is incorrect because the term "Perimeter switch" is not meaningful in this context.

Answer (C) is incorrect because, in a computer program, a default setting is a value that a parameter will automaticallyassume unless specifically overridden.

Answer (D) is correct. Businesses and others require that documents sent over the Internet be authentic. To authenticate adocument, a company or other user may transmit a complete plaintext document along with an encrypted portion of thesame document or another standard text that serves as a digital signature. If the plaintext document is tampered with, thetwo will not match.

[1197] Gleim #: 9.1.25 -- Source: CIA 1196 III-77

Answer (A) is correct. RSA is an encryption standard licensed to hardware and software vendors. Public-key encryptionrequires management of fewer keys for a given client-server environment than does private-key encryption. However,compared with DES, RSA entails more complex computations and therefore has a higher processing overhead. RSArequires two keys: The public key for encrypting messages is widely known, but the private key for decrypting messages iskept secret by the recipient.

Answer (B) is incorrect because DES is a shared private-key method developed by the U.S. government. It encrypts datainto 64-bit blocks using a 56-bit key. DES requires only a single key for each pair of parties that want to send each otherencrypted messages. DES is being replaced by AES, Advanced Encryption Standard, as the method of choice by the U.S.government.

Answer (C) is incorrect because a modem is used for telecommunications.

Answer (D) is incorrect because a cypher lock is a physical device.

[1198] Gleim #: 9.1.26 -- Source: Publisher

Answer (A) is incorrect because a firewall tries to prevent access from specific types of traffic to an internal network.After someone has obtained information from the site, a firewall cannot prevent its use.

Answer (B) is incorrect because an access log only records attempted usage of a system.

Answer (C) is incorrect because passwords prevent unauthorized users from accessing the system. If information hasalready been obtained, a password cannot prevent its use.

Answer (D) is correct. Encryption technology converts data into a code. Encoding data before transmission overcommunications lines makes it more difficult for someone with access to the transmission to understand or modify itscontents.

[1199] Gleim #: 9.1.27 -- Source: CISA

Answer (A) is correct. In a public key system, the public key is used to encrypt the message prior to transmission,whereas the private key is needed to decrypt (decode) the message.

Answer (B) is incorrect because the private key, not the public key, is needed to decrypt (decode) the message.

Answer (C) is incorrect because the public key is not needed.

Answer (D) is incorrect because the private key is needed to decrypt (decode) the message.

Gleim's CIA Test Prep: Part III: Business Analysis and Information TechnologyAnswer Explanations

(1312 questions)

Copyright 2008 Gleim Publications, Inc. Page 365Printed for Mamdouh Farag

Page 366: P.3 Answer Explanation

[1200] Gleim #: 9.1.28 -- Source: CIA 596 III-44

Answer (A) is incorrect because developing an information security policy for the organization is an appropriate duty ofthe information security officer.

Answer (B) is correct. The information security officer should not know user passwords. They are normally stored on acomputer in encrypted format, and users change them directly.

Answer (C) is incorrect because commenting on security controls in new applications is an appropriate duty of theinformation security officer.

Answer (D) is incorrect because monitoring and investigating unsuccessful access attempts is an appropriate duty of theinformation security officer.

[1201] Gleim #: 9.1.29 -- Source: CMA 693 4-10

Answer (A) is correct. The list or tables of authorized users or devices are sometimes called access control matrices;however, they are more commonly referred to as device authorization tables. A device authorization table restricts fileaccess to those physical devices that logically need access. A device authorization table can contain such items asauthorized user code numbers, passwords, lists of all files and programs, and a record of the type of access each user isentitled to have to each file and program.

Answer (B) is incorrect because it is not a complete list of the contents of an access control matrix.

Answer (C) is incorrect because it is not information stored in an access control matrix.

Answer (D) is incorrect because it is not a list of items stored in an access control matrix.

[1202] Gleim #: 9.1.30 -- Source: CIA, adapted

Answer (A) is incorrect because physical controls limit access to an area and do not include passwords.

Answer (B) is incorrect because edit controls test the validity of data.

Answer (C) is incorrect because digital controls are examples of physical controls.

Answer (D) is correct. Passwords are a form of access controls since they limit access to computer systems and theinformation stored in them.

[1203] Gleim #: 9.1.31 -- Source: CISA

Answer (A) is incorrect because authentication involves verifying the identity of the user. This process does notnecessarily confirm the functions the user is authorized to perform.

Answer (B) is correct. Identification is the process of uniquely distinguishing one user from all others. Authentication isthe process of determining that individuals are who they say they are. For example, a password may identify but notauthenticate its user if it is known by more than one individual.

Answer (C) is incorrect because user identification to the system does not imply that the system has verified the identity ofthe user.

Answer (D) is incorrect because this procedure is an application control for accuracy of the transaction.

Gleim's CIA Test Prep: Part III: Business Analysis and Information TechnologyAnswer Explanations

(1312 questions)

Copyright 2008 Gleim Publications, Inc. Page 366Printed for Mamdouh Farag

Page 367: P.3 Answer Explanation

[1204] Gleim #: 9.1.32 -- Source: CIA, adapted

Answer (A) is incorrect because data encryption provides adequate security for notebook computers.

Answer (B) is correct. Password protection for a screen-saver program can be easily bypassed.

Answer (C) is incorrect because removable hard drives would provide adequate security.

Answer (D) is incorrect because security is promoted by physically locking the notebook computer in a case.

[1205] Gleim #: 9.1.33 -- Source: CIA, adapted

Answer (A) is correct. While most delete programs erase file pointers, they do not remove the underlying data. Thecompany must use special utilities that fully erase the data. This is important because of the potential for confidential dataon the microcomputers.

Answer (B) is incorrect because this could create a liability for the company if a virus destroyed the purchasing party’sdata or programs. However, the purchasing party should use anti-virus software to detect and eliminate any viruses. Thisconcern, while important, is not as serious as the one in the answer choice which states: “Whether deleted files on thehard disk drive have been completely erased.”

Answer (C) is incorrect because the purchasing party has a responsibility to insure that all their software is properlylicensed. If the company represented that all the software was properly licensed, this could create a liability. However, thisliability is not as serious as the implication from the answer choice which states: “Whether deleted files on the hard diskdrive have been completely erased.”

Answer (D) is incorrect because terminal emulation software is widely available.

[1206] Gleim #: 9.2.34 -- Source: CIA 1193 I-32

Answer (A) is incorrect because the practice of not retaining daily transaction data is unsound in that the bank loses aday’s transactions for each backup that is unreadable.

Answer (B) is correct. At appropriate intervals, the disk files should be copied on magnetic tape so that restart procedurescan begin at those points if data are lost or destroyed. However, not retaining each day’s transaction files is risky becauseinformation processed since the last backup file was created will be lost.

Answer (C) is incorrect because the practice of not retaining daily transaction data certainly minimizes complexity but atthe expense of losing transaction data if the online file must be restored from the backup.

Answer (D) is incorrect because checkpoint/restart information is not needed. The backups are created after all processingis finished for the day.

Gleim's CIA Test Prep: Part III: Business Analysis and Information TechnologyAnswer Explanations

(1312 questions)

Copyright 2008 Gleim Publications, Inc. Page 367Printed for Mamdouh Farag

Page 368: P.3 Answer Explanation

[1207] Gleim #: 9.2.35 -- Source: CIA 589 II-25

Answer (A) is incorrect because comparison of batch totals is a control over the completeness of processing, not a recoveryprocedure.

Answer (B) is incorrect because internal labels may avoid destruction of data but do not aid in recovery.

Answer (C) is correct. The grandfather-father-son approach normally employs magnetic tapes to furnish backup in a batchprocessing system. The procedure involves creation and retention of three generations of master files so that lost ordestroyed data may be regenerated from the remaining master files and transaction data. In this case, a master file (thegrandfather) and the first week’s transactions are used to generate a second master file (the father). This file and thesecond week’s transactions are the basis for the current master file (the son). Online systems employ rollback andrecovery procedures; i.e., the master file is periodically dumped onto a storage medium. Reconstruction is then possibleusing the backup copy and the transactions log.

Answer (D) is incorrect because validation may avoid destruction of data but does not aid in recovery.

[1208] Gleim #: 9.2.36 -- Source: CIA 596 III-39

Answer (A) is correct. The disaster plan should embrace data center recovery, critical application recovery, and networkrecovery. It should be updated and current with regard to recent test results and new applications, equipment, and networkconfigurations. The plan should also ensure that backup facilities are still able to process critical applications and thatend-user responsibility is established. Another essential component of a disaster recovery plan is that backup/restartprocedures have been anticipated and provided for in the application systems.

Answer (B) is incorrect because whether change control procedures can be bypassed is not usually a consideration indisaster recovery planning.

Answer (C) is incorrect because planned rather than actual changes in equipment capacities are not relevant in disasterrecovery planning.

Answer (D) is incorrect because ensuring that service level agreements with owners of critical applications are adequate isnot a function of disaster recovery planning.

[1209] Gleim #: 9.2.37 -- Source: CIA 593 I-42

Answer (A) is incorrect because a “hot site” has all needed assets in place and is not vendor-dependent.

Answer (B) is correct. Organizations should maintain contingency plans for operations in the case of a disaster. Theseplans usually include off-site storage of important backup data and an arrangement for the continuation of operations atanother location. A “cold site” has all needed assets in place except the needed computer equipment and is vendor-dependent for timely delivery of equipment.

Answer (C) is incorrect because a “cold and hot site” combination allows the “hot site” to be used until the “cold site” isprepared and is thus not too vendor-dependent.

Answer (D) is incorrect because excess capacity would ensure that needed assets are available and would not be vendor-dependent.

Gleim's CIA Test Prep: Part III: Business Analysis and Information TechnologyAnswer Explanations

(1312 questions)

Copyright 2008 Gleim Publications, Inc. Page 368Printed for Mamdouh Farag

Page 369: P.3 Answer Explanation

[1210] Gleim #: 9.2.38 -- Source: CMA 687 5-6

Answer (A) is incorrect because it is not an error that a checkpoint/restart procedure is primarily designed to recover from.

Answer (B) is incorrect because it is not an error that a checkpoint/restart procedure is primarily designed to recover from.

Answer (C) is incorrect because it is not an error that a checkpoint/restart procedure is primarily designed to recover from.

Answer (D) is correct. Checkpoint/restart procedures involve capturing all the values of data and program indicators atspecified points and storing these values in another file. If processing is interrupted, it can be resumed at the lastcheckpoint rather than at the beginning of the run.

[1211] Gleim #: 9.2.39 -- Source: CPA AUD R02-6

Answer (A) is incorrect because converting all data from EDI format to an internal company format would reduce theability to process transactions with outside companies.

Answer (B) is incorrect because a Trojan horse program is a computer program that appears to be legitimate but performssome illicit activity when it is run.

Answer (C) is incorrect because the use of an uninterruptible power supply assures continued processing rather thanrecovery from a disaster.

Answer (D) is correct. Off-site storage of duplicate copies of critical files protects them from a fire or other disaster at thecomputing facility. The procedure is part of an overall disaster recovery plan.

[1212] Gleim #: 9.2.40 -- Source: CIA 595 III-71

Answer (A) is incorrect because personnel employed at the site are not related to the company, and would therefore not befamiliar with company operations.

Answer (B) is incorrect because depending on the location of the cold site, traveling expenses could increase dramatically.

Answer (C) is correct. A cold site backup facility is a shell facility where the user can quickly install computer equipmentand resume operations in the event of a disaster. The facility has all of the needed assets in place except the neededcomputer equipment and is vendor-dependent for timely delivery of equipment. Accordingly, an advantage of using athird-party cold site is the elimination of the need for additional equipment.

Answer (D) is incorrect because it is not an advantage of using a third-party cold site; a few hours of downtime could bevery costly to the organization.

[1213] Gleim #: 9.2.41 -- Source: CIA 595 III-72

Answer (A) is correct. When a company decentralizes its information processing, many changes in equipment, data, andsoftware occur as a result of the different processing methods. Since the company has not revised its contingency plansince the time processing was centralized, the plan is most likely out of date as a result of the changes in equipment, data,and software.

Answer (B) is incorrect because it is not a reason that would cause a contingency plan to be out of date.

Answer (C) is incorrect because it is not a reason that would cause a contingency plan to be out of date.

Answer (D) is incorrect because it is not a reason that would cause a contingency plan to be out of date.

Gleim's CIA Test Prep: Part III: Business Analysis and Information TechnologyAnswer Explanations

(1312 questions)

Copyright 2008 Gleim Publications, Inc. Page 369Printed for Mamdouh Farag

Page 370: P.3 Answer Explanation

[1214] Gleim #: 9.2.42 -- Source: CIA 595 III-73

Answer (A) is incorrect because headquarters could monitor the network on a real-time basis and have completeawareness of the state of processing.

Answer (B) is correct. If data stored at one regional center is to mirror the data stored at another center, the most efficientmethod to ensure each center has the most current data is to transfer data across a network. Consequently, the cost ofnetwork traffic would increase dramatically. The complexity of the network would also increase as the network wouldneed to provide a great deal of security when transferring data.

Answer (C) is incorrect because adequate controls will ensure that the mirrored data will not affect the source data.

Answer (D) is incorrect because the location of the data is not relevant to the insurance agents. The agents will retrievecustomer data through the network.

[1215] Gleim #: 9.2.43 -- Source: CPA AUD R98-6

Answer (A) is incorrect because a cold site is a shell facility suitable for quick installation of computer equipment.Installing computer equipment would take more time in a cold site than in a hot site.

Answer (B) is incorrect because it is a fabricated term that does not describe actual facilities.

Answer (C) is incorrect because it is a fabricated term that does not describe actual facilities.

Answer (D) is correct. A hot site is a service bureau that is a fully operational processing facility and is promptlyavailable in the case of a power outage or disaster.

[1216] Gleim #: 9.2.44 -- Source: Publisher

Answer (A) is correct. A company uses a hot site backup when fast recovery is critical. The hot site includes all software,hardware, and other equipment necessary for a company to carry out operations. Hot sites are expensive to maintain andmay be shared with other organizations with similar needs.

Answer (B) is incorrect because a warm site provides an intermediate level of backup and causes more downtime than ahot site.

Answer (C) is incorrect because a cold site is a shell facility suitable for quick installation of computer equipment.Disaster recovery would take more time in a cold site than a hot site.

Answer (D) is incorrect because there is no backup site called a quick site.

[1217] Gleim #: 9.2.45 -- Source: CIA 1190 III-17

Answer (A) is incorrect because it does not require extensive backup and recovery procedures.

Answer (B) is correct. Database systems require a more elaborate backup procedure than other systems. A databasesystem for online entry would require almost continuous backup if data loss is to be minimized as transactions areprocessed on a continuous basis, and without tangible source documentation. Backup procedures that could be employedfor online order entry include dual logging and rollback and recovery.

Answer (C) is incorrect because it does not require extensive backup and recovery procedures.

Answer (D) is incorrect because it does not require extensive backup and recovery procedures.

Gleim's CIA Test Prep: Part III: Business Analysis and Information TechnologyAnswer Explanations

(1312 questions)

Copyright 2008 Gleim Publications, Inc. Page 370Printed for Mamdouh Farag

Page 371: P.3 Answer Explanation

[1218] Gleim #: 9.2.46 -- Source: CIA, adapted

Answer (A) is incorrect because the absence of processing interruptions indicates nothing about the interruptions thatmight occur in the future, especially those that are not under the organization’s control.

Answer (B) is incorrect because a contingency plan may have comprehensive documentation, but until the plan is tested,an organization has no indication of its effectiveness.

Answer (C) is incorrect because audit signoff is one indicator of plan quality, but until the plan is tested, an organizationhas no indication of its effectiveness.

Answer (D) is correct. The only way to know whether contingency planning has been effective is to test the plan, bysimulating an interruption or by conducting a paper test with a walk-through of recovery procedures.

[1219] Gleim #: 9.2.47 -- Source: CIA, adapted

Answer (A) is incorrect because exception reporting can be used to control correctness and timeliness of updates butcannot minimize the impact of an interruption.

Answer (B) is correct. An advantage of distributed processing is fail-soft protection, that is, the ability to continueprocessing at all sites except a nonfunctioning one.

Answer (C) is incorrect because backup procedures are intended to prevent the recovery process from introducing anyerroneous changes into the system after computer failure.

Answer (D) is incorrect because data file security is intended to prevent unauthorized changes to data files.

[1220] Gleim #: 9.3.48 -- Source: CISA, adapted

Answer (A) is incorrect because attenuation of the signal is not an advantage of using fiber optics.

Answer (B) is incorrect because attenuation of the signal is not an advantage of using fiber optics.

Answer (C) is incorrect because attenuation of the signal is not an advantage of using fiber optics.

Answer (D) is correct. A fiber-optic cable uses light impulses that travel through clear, flexible tubing half the size of ahuman hair. Fiber-optic cables are not subject to electrical interference and are highly reliable. They provide for extremelyflexible and fast data transmission. The signal remains strong across long distances; i.e., it does not tend to weaken(attenuate).

[1221] Gleim #: 9.3.49 -- Source: CIA 592 III-37

Answer (A) is incorrect because asynchronous communication is a mode of transmission. Communication is in disjointedsegments, typically character by character, preceded by a start code and ended by a stop code.

Answer (B) is incorrect because synchronous communication is a mode of transmission in which a continuous stream ofblocks of characters result in faster communications.

Answer (C) is incorrect because a communication channel is a transmission link between devices in a network. The termis also used for a small processor that controls input-output devices.

Answer (D) is correct. A protocol is a set of formal rules or conventions governing communication between a sendingdevice and a receiving device. It prescribes the manner by which data are transmitted between these communicationsdevices. In essence, a protocol is the envelope within which each message is transmitted throughout a datacommunications network.

Gleim's CIA Test Prep: Part III: Business Analysis and Information TechnologyAnswer Explanations

(1312 questions)

Copyright 2008 Gleim Publications, Inc. Page 371Printed for Mamdouh Farag

Page 372: P.3 Answer Explanation

[1222] Gleim #: 9.3.50 -- Source: CIA 1195 III-58

Answer (A) is incorrect because the company would be responsible for all maintenance of the equipment, although itcould contract for service.

Answer (B) is correct. A PBX has the advantage of using existing telephone lines and therefore not needing specialwiring. Moreover, equipment can be moved without necessitating rewiring. However, because PBX-based systems usetelephone wiring (most often copper wire), they cannot easily handle large volumes of data.

Answer (C) is incorrect because PBXs use telephone wiring. LANs typically require their own coaxial cabling.

Answer (D) is incorrect because PBX-based systems do not require rewiring when devices are moved.

[1223] Gleim #: 9.3.51 -- Source: CIA 1195 III-40

Answer (A) is correct. Companies can use public switched lines (phone lines) on a per-transmission basis. This option isthe most cost-effective way for low-volume users to conduct telecommunications.

Answer (B) is incorrect because fast-packet switches receive transmissions from various devices, break the data intopackets, and route them over a network to their destination. They are typically installed by telecommunication utilitycompanies and other large companies that have their own networks.

Answer (C) is incorrect because electronic mail systems do not allow for voice transmissions.

Answer (D) is incorrect because large organizations would use a wide area network.

[1224] Gleim #: 9.3.52 -- Source: CIA 1196 III-72

Answer (A) is correct. To prepare the company for changes resulting from the enhanced external network services,management should take appropriate action. A number of bottlenecks may limit the benefits that can be derived from theexternal network. For example, conversion from analog to digital technology is necessary to achieve rapid improvementsin bandwidth and speed and to improve access to telecommunications services. Furthermore, applications, systemssoftware, and communications protocols must be able to process information in a format and in a manner acceptable toend users. Communications security also has heightened importance as greater amounts of data are transmitted fromremote sites.

Answer (B) is incorrect because resistance to change, inflexible organizational structures, and skepticism about thetechnology should be expected and must be successfully managed if the company is to reap the benefits.

Answer (C) is incorrect because a company’s disaster recovery plan should be enhanced to ensure the reliability of thenetwork.

Answer (D) is incorrect because network management may now be primarily a function, yet it will become more of apartnership arrangement with the communications carrier.

Gleim's CIA Test Prep: Part III: Business Analysis and Information TechnologyAnswer Explanations

(1312 questions)

Copyright 2008 Gleim Publications, Inc. Page 372Printed for Mamdouh Farag

Page 373: P.3 Answer Explanation

[1225] Gleim #: 9.3.53 -- Source: CMA 695 4-19

Answer (A) is correct. A LAN is a local distributed computer system, often housed within a single building. Computers,communication devices, and other equipment are linked by cable. Special software facilitates efficient datacommunication among the hardware devices.

Answer (B) is incorrect because a LAN is more than a system to allow computer users to share information; it is aninterconnection of a computer system.

Answer (C) is incorrect because a LAN is not a library.

Answer (D) is incorrect because a LAN does not require specialized hardware.

[1226] Gleim #: 9.3.54 -- Source: Publisher

Answer (A) is correct. Voice recognition input devices provide an alternative to keyboard input. These systems comparethe speaker’s voice patterns with prerecorded patterns. Advanced systems now have large vocabularies and shortertraining periods. They allow for dictation and are not limited to simple commands.

Answer (B) is incorrect because a voice output device converts digital data into speech using prerecorded sounds.

Answer (C) is incorrect because PCS services use lower-power, higher-frequency radio waves than cell phones.

Answer (D) is incorrect because newer pager systems permit transmission of text messages.

[1227] Gleim #: 9.3.55 -- Source: CIA 1196 III-56

Answer (A) is correct. The Internet facilitates information transfer between computers. Gateways are hardware orsoftware products that allow translation between two different protocol families. For example, a gateway can be used toexchange messages between different email systems.

Answer (B) is incorrect because routers are used to determine the best path for data.

Answer (C) is incorrect because bridges connect LANs.

Answer (D) is incorrect because repeaters strengthen signals.

[1228] Gleim #: 9.3.56 -- Source: Publisher

Answer (A) is incorrect because HTML is the acronym for HyperText Markup Language.

Answer (B) is incorrect because the language is relatively easy to learn. Almost anyone can learn and use HTML, not justcomputer programmers.

Answer (C) is correct. HTML is the most popular language for authoring Web pages. It is hardware and softwareindependent, which means that it can be read by several different applications and on many different kinds of computeroperating systems. HTML uses tags to mark information for proper display on Web pages.

Answer (D) is incorrect because a number of other languages can be used for Internet transmissions, including JAVA andXML.

Gleim's CIA Test Prep: Part III: Business Analysis and Information TechnologyAnswer Explanations

(1312 questions)

Copyright 2008 Gleim Publications, Inc. Page 373Printed for Mamdouh Farag

Page 374: P.3 Answer Explanation

[1229] Gleim #: 9.3.57 -- Source: CIA, adapted

Answer (A) is incorrect because asynchronous modems handle data streams from peripheral devices to a central processor.

Answer (B) is incorrect because authentication techniques confirm that valid users have access to the system.

Answer (C) is incorrect because call-back procedures are used to ensure incoming calls are from authorized locations.

Answer (D) is correct. Encryption involves using a fixed algorithm to manipulate plaintext. The information is sent in itsmanipulated form and the receiver translates the information back into plain text. Cryptographic devices protect data intransmission over communication lines.

[1230] Gleim #: 9.3.58 -- Source: CIA, adapted

Answer (A) is correct. A gateway, often implemented via software, translates between two or more different protocolfamilies and makes connections between dissimilar networks possible.

Answer (B) is incorrect because a bridge joins network segments so that they appear to be one physical segment.

Answer (C) is incorrect because a router connects two or more network segments, such that the segments maintain theirseparate logical identities.

Answer (D) is incorrect because a wiring concentrator accepts twisted-pair cabling from each of several personalcomputers in the same local area network (LAN).

[1231] Gleim #: 9.3.59 -- Source: CMA 695 4-13

Answer (A) is incorrect because a star network routes all data through the host computer.

Answer (B) is correct. In a distributed system, an organization’s processing needs are examined in their totality. Thedecision is not whether an application should be done centrally or locally, but rather which parts are better performed bysmall local computers as intelligent terminals, and which parts are better performed at some other, possibly centralized,site. The key distinction between decentralized and distributed systems is the interconnection among the nodes in thenetwork. A ring network links all communication channels to form a loop and each link passes communications throughits neighbor to the appropriate location.

Answer (C) is incorrect because a bus network attaches all channel messages along one common line with communicationto the appropriate location via direct access.

Answer (D) is incorrect because a tree configuration is organized along hierarchical lines to a host computer.

Gleim's CIA Test Prep: Part III: Business Analysis and Information TechnologyAnswer Explanations

(1312 questions)

Copyright 2008 Gleim Publications, Inc. Page 374Printed for Mamdouh Farag

Page 375: P.3 Answer Explanation

[1232] Gleim #: 9.3.60 -- Source: CIA 1194 III-32

Answer (A) is correct. Controllers are hardware units designed to operate (control) specific input or output units, e.g.,terminals and printers. These devices eliminate the need for the CPU to operate such devices. The term “buffers andchannels” is also sometimes used to describe devices that link the CPU with input/output devices and storage units.Controllers may hold large blocks of logical records to compensate for the differences in the rates of flow of data betweeninput/output devices and the central processing unit.

Answer (B) is incorrect because compensating for the differences in the lengths of records in a file is achieved by settingaside space in the file control block for the longest record that will be encountered.

Answer (C) is incorrect because there is no compensating for the differences in the execution time in various programinstructions.

Answer (D) is incorrect because multiplexors and concentrators compensate for the differences in the volume of data thatcan be transmitted between communication devices.

[1233] Gleim #: 9.4.61 -- Source: CIA 1186 III-33

Answer (A) is correct. A record is a collection of related data items (fields). A field (data item) is a group of charactersrepresenting one unit of information.

Answer (B) is incorrect because a file is a group or set of related records ordered to facilitate processing.

Answer (C) is incorrect because a byte is a group of bits (binary digits). It represents one character.

Answer (D) is incorrect because occurrences is not a meaningful term in this context.

[1234] Gleim #: 9.4.62 -- Source: CIA 592 III-28

Answer (A) is correct. Data organized in files and used by the organization’s various applications programs arecollectively known as a database. In a database system, storage structures are created that render the applicationsprograms independent of the physical or logical arrangement of the data. Each data item has a standard definition, name,and format, and related items are linked by a system of pointers. The programs therefore need only to specify data itemsby name, not by location. A database management system handles retrieval and storage. Because separate files fordifferent applications programs are unnecessary, data redundancy can be substantially reduced.

Answer (B) is incorrect because conversion to a database is often costly and time consuming.

Answer (C) is incorrect because a traditional flat-file system, not a database, has multiple occurrences of data items.

Answer (D) is incorrect because, given the absence of data redundancy and the quick propagation of data errorsthroughout applications, backup and recovery procedures are just as critical in a database as in a flat-file system.

Gleim's CIA Test Prep: Part III: Business Analysis and Information TechnologyAnswer Explanations

(1312 questions)

Copyright 2008 Gleim Publications, Inc. Page 375Printed for Mamdouh Farag

Page 376: P.3 Answer Explanation

[1235] Gleim #: 9.4.63 -- Source: CIA 1188 III-36

Answer (A) is correct. In a single flat file structure, all attributes and field lengths in a record are identical to those in theother records. The structure is typically a table or spreadsheet with records for rows and attributes for columns.

Answer (B) is incorrect because a hierarchical or tree structure is used to express relationships in which one attribute oritem is related to many others in layers of subordinate records.

Answer (C) is incorrect because a network structure expresses complex relationships in which many attributes are relatedto many others.

Answer (D) is incorrect because a relational structure is not unlike the flat structure but is far more sophisticated. It givesthe system the ability to handle many data relationships that were not anticipated by the designers. It uses a series oftables in which each table defines a relationship.

[1236] Gleim #: 9.4.64 -- Source: CIA 592 III-32

Answer (A) is incorrect because field refers to a single data item.

Answer (B) is incorrect because file refers to multiple records.

Answer (C) is incorrect because database refers to multiple files.

Answer (D) is correct. A record is a collection of related data items (fields). A field (data item) is a group of charactersrepresenting one unit of information. The part number, part description, etc., are represented by fields.

[1237] Gleim #: 9.4.65 -- Source: CIA 593 III-57

Answer (A) is incorrect because a database is an organized collection of files.

Answer (B) is correct. A data item (or field) is a group of characters. It is used to represent individual attributes of anentity, such as an employee’s address. A field is an item in a record.

Answer (C) is incorrect because a file is a collection of records.

Answer (D) is incorrect because a record is a collection of data items.

[1238] Gleim #: 9.4.66 -- Source: CPA AUD R98-3

Answer (A) is incorrect because 1 is the number of batches.

Answer (B) is correct. Input controls in batch computer systems are used to determine that no data are lost or added to thebatch. Depending on the sophistication of a particular system, control may be accomplished by using record counts, batchtotals, or hash totals. A record count establishes the number of source documents and reconciles it to the number of outputrecords. The total number of invoices processed is an example of a record count. In this case, the record count is 4.

Answer (C) is incorrect because 810 is a hash total of the invoice numbers.

Answer (D) is incorrect because 900 is the total quantity of items.

Gleim's CIA Test Prep: Part III: Business Analysis and Information TechnologyAnswer Explanations

(1312 questions)

Copyright 2008 Gleim Publications, Inc. Page 376Printed for Mamdouh Farag

Page 377: P.3 Answer Explanation

[1239] Gleim #: 9.4.67 -- Source: CIA 594 III-17

Answer (A) is correct. The primary record key uniquely identifies each record in a file. Because there is only one recordfor each vendor in an accounts payable master file, the vendor number would be the appropriate key.

Answer (B) is incorrect because purchase order files can have multiple purchase orders made out to the same vendor. Theprimary key in purchase order files would be the purchase order number because it is the only unique identifier for therecord.

Answer (C) is incorrect because purchase order files can have multiple purchase orders made out to the same vendor. Theprimary key in purchase order files would be the purchase order number because it is the only unique identifier for therecord.

Answer (D) is incorrect because not all of the answer choices are correct.

[1240] Gleim #: 9.4.68 -- Source: CIA 1194 III-29

Answer (A) is incorrect because a direct-access file has no index file.

Answer (B) is correct. If records are stored sequentially on a direct-access storage device, the records can be accesseddirectly using the indexed-sequential-access method. An index of key fields is maintained that lists the physical locationof the record corresponding to each key field.

Answer (C) is incorrect because a sequential file has no index file.

Answer (D) is incorrect because a text file would probably not be used in this situation.

[1241] Gleim #: 9.4.69 -- Source: CIA 595 III-75

Answer (A) is incorrect because extraction selects data containing specified criteria from a data file; it does not combinetables.

Answer (B) is correct. In CAAT software packages, joining is the combining of data files based on a common dataelement. For example, if rows in a table containing information about specified parts have been selected, the result can bejoined with a table that contains information about suppliers. The join operation may combine the two tables using thesupplier number (assuming both tables contained this element) to provide information about the suppliers of particularparts.

Answer (C) is incorrect because sorting allows the auditor to organize data by any data field, not combine tables.

Answer (D) is incorrect because summarization reports on the information contained in several tables but does notcombine the tables.

Gleim's CIA Test Prep: Part III: Business Analysis and Information TechnologyAnswer Explanations

(1312 questions)

Copyright 2008 Gleim Publications, Inc. Page 377Printed for Mamdouh Farag

Page 378: P.3 Answer Explanation

[1242] Gleim #: 9.4.70 -- Source: CIA 1188 III-31

Answer (A) is incorrect because the ability to respond immediately to customers requires direct access.

Answer (B) is incorrect because the ability to respond immediately to customers requires direct access.

Answer (C) is correct. A record key is an attribute that uniquely identifies or distinguishes each record from the others.An index is a table listing storage locations for attributes, often including those other than the unique record key attribute.A pointer is a data item that indicates the physical address of the next logically related record.

Answer (D) is incorrect because internal labels are used to indicate various things to the computer, such as the contents ofvarious types of data storage media, the beginning of each file (with identification information), and the end of each file.However, they do not provide information for locating specific records in a file. An inverted file structure (inverted list) isan index based on a secondary key, for example, years of experience rather than an employee number (the primary key).

[1243] Gleim #: 9.4.71 -- Source: CIA 597 III-50

Answer (A) is correct. Joining is the combining of two or more relational tables based on a common data element. Forexample, if a supplier table contains information about suppliers and a parts table contains information about parts, thetwo tables can be joined using the supplier number (assuming both tables contain this attribute) to give information aboutthe supplier of particular parts.

Answer (B) is incorrect because the three basic operations in a relational database are selecting, joining, and projecting.

Answer (C) is incorrect because projecting is the basic operation in a relational database that results in a subset consistingof columns (fields) in a table. This operation creates a new table containing only the required information.

Answer (D) is incorrect because a pointer is a data element attached to a record that gives the address of another record.

[1244] Gleim #: 9.4.72 -- Source: CIA 596 III-65

Answer (A) is incorrect because the snapshot technique makes duplicates to be stored at multiple locations.

Answer (B) is incorrect because the replication technique makes duplicates to be stored at multiple locations. Changes areperiodically copied and sent to each location. If a database is small, storing multiple copies may be cheaper than retrievingrecords from a central site.

Answer (C) is correct. A distributed database is stored in two or more physical sites. The two basic methods ofdistributing a database are partitioning and replication. However, normalization is a process of database design, notdistribution. Normalization is the term for determining how groups of data items in a relational structure are arranged inrecords in a database. This process relies on “normal forms,” that is, conceptual definitions of data records and specifieddesign rules. Normalization is intended to prevent inconsistent updating of data items. It is a process of breaking down acomplex data structure by creating smaller, more efficient relations, thereby minimizing or eliminating the repeatinggroups in each relation.

Answer (D) is incorrect because fragmentation or partitioning stores specific records where they are most needed. Forexample, a financial institution may store a particular customer’s data at the branch where (s)he usually transacts his orher business. If the customer executes a transaction at another branch, the pertinent data is retrieved via communicationslines.

Gleim's CIA Test Prep: Part III: Business Analysis and Information TechnologyAnswer Explanations

(1312 questions)

Copyright 2008 Gleim Publications, Inc. Page 378Printed for Mamdouh Farag

Page 379: P.3 Answer Explanation

[1245] Gleim #: 9.4.73 -- Source: CIA 1196 III-75

Answer (A) is incorrect because inconsistent processing occurs when a transaction has different effects depending onwhen it is processed. Data locking ensures consistent processing.

Answer (B) is incorrect because rollback failure is the inability of the software to undo the effects of a transaction thatcould not be run to completion. A rollback failure is not caused by data locking. However, data locking may lead tosituations in which rollback is required.

Answer (C) is incorrect because unrecoverable transactions are not a typical symptom of locking procedures.

Answer (D) is correct. In a distributed processing system, the data and resources a transaction may update or use shouldbe held in their current status until the transaction is complete. A deadly embrace occurs when two transactions need thesame resource at the same time. If the system does not have a method to cope with the problem efficiently, response timeworsens or the system eventually fails. The system should have an algorithm for undoing the effects of one transaction andreleasing the resources it controls so that the other transaction can run to completion.

[1246] Gleim #: 9.4.74 -- Source: CMA 695 4-20

Answer (A) is incorrect because each organizational unit develops programs to make use of elements of a broad database.

Answer (B) is incorrect because data handling techniques are still the responsibility of the data processing department; itis the use of the data that is departmentalized.

Answer (C) is incorrect because the DBMS is no safer than any other database system.

Answer (D) is correct. A fundamental characteristic of databases is that applications are independent of the databasestructure; when writing programs or designing applications to use the database, only the name of the desired item isnecessary. Programs can be developed for the user’s specific needs without concern for data capture problems. Referencecan be made to the items using the data manipulation language, after which the DBMS takes care of locating andretrieving the desired items. The physical or logical structure of the database can be completely altered without having tochange any of the programs using the data items; only the schema requires alteration.

[1247] Gleim #: 9.4.75 -- Source: CISA, adapted

Answer (A) is incorrect because the advantage of a DBMS is that data can be used concurrently by multiple users.

Answer (B) is correct. In this kind of system, applications use the same database. There is no need to pass files betweenapplications.

Answer (C) is incorrect because, when a DBMS is used, the physical structure of the data is independent of user needs.

Answer (D) is incorrect because, when a DBMS is used, the data are defined independently of the needs of any oneprogram.

Gleim's CIA Test Prep: Part III: Business Analysis and Information TechnologyAnswer Explanations

(1312 questions)

Copyright 2008 Gleim Publications, Inc. Page 379Printed for Mamdouh Farag

Page 380: P.3 Answer Explanation

[1248] Gleim #: 9.4.76 -- Source: CIA 588 II-29

Answer (A) is incorrect because designing the content and organization of the database is a responsibility of the databaseadministrator.

Answer (B) is correct. The database administrator (DBA) is the person who has overall responsibility for developing andmaintaining the database. One primary responsibility is for designing the content of the database. Another responsibilityof the DBA is to protect and control the database. A third responsibility is to monitor and improve the efficiency of thedatabase. The responsibility of developing applications to access the database belongs to systems analysts andprogrammers.

Answer (C) is incorrect because protecting the database and its software is a responsibility of the database administrator.

Answer (D) is incorrect because monitoring and improving the efficiency of the database is a responsibility of the databaseadministrator.

[1249] Gleim #: 9.4.77 -- Source: Publisher

Answer (A) is correct. The DA handles administrative issues that arise regarding the database. The DA acts as anadvocate by suggesting new applications and standards. One of the DA’s responsibilities is to monitor the databaseindustry for new developments. In contrast, the database administrator (DBA) deals with the technical aspects of thedatabase.

Answer (B) is incorrect because it is an example of the database administrator’s responsibilities.

Answer (C) is incorrect because it is an example of the database administrator’s responsibilities.

Answer (D) is incorrect because it is an example of the database administrator’s responsibilities.

[1250] Gleim #: 9.4.78 -- Source: CIA 589 III-39

Answer (A) is correct. The data dictionary is a file (possibly manual but usually computerized) in which the records relateto specified data items. It contains definitions of data items, the list of programs used to process them, and the reports inwhich data are found. Only certain persons or entities are permitted to retrieve data or to modify data items. Accordingly,these access limitations are also found in the data dictionary.

Answer (B) is incorrect because the schema describes the structure of the database.

Answer (C) is incorrect because an encryptor encodes data.

Answer (D) is incorrect because a decision table is a type of logic diagram that presents in matrix form the decision pointsand related actions reflected in a computer program.

[1251] Gleim #: 9.4.79 -- Source: CIA, adapted

Answer (A) is correct. The schema is a description of the overall logical structure of the database using data-definitionlanguage (DDL), which is the connection between the logical and physical structure of the database. DDL is used todefine, or determine, the database.

Answer (B) is incorrect because data control language (DCL) is used to specify privileges and security rules.

Answer (C) is incorrect because data manipulation language (DML) provides programmers with a facility to update thedatabase.

Answer (D) is incorrect because data query language (DQL) is used for ad hoc queries.

Gleim's CIA Test Prep: Part III: Business Analysis and Information TechnologyAnswer Explanations

(1312 questions)

Copyright 2008 Gleim Publications, Inc. Page 380Printed for Mamdouh Farag

Page 381: P.3 Answer Explanation

[1252] Gleim #: 9.4.80 -- Source: CIA, adapted

Answer (A) is incorrect because most query tools include the capability of presenting the results of queries graphically.

Answer (B) is incorrect because query tools include data dictionary access because that is how they know what tableattributes to present to users.

Answer (C) is correct. The least likely feature of a query tool would be a data validity checker because the databasesystem has already enforced any validity constraints at the time the data were inserted in the database. Any further datavalidity checking would be a function of a user application program rather than a query.

Answer (D) is incorrect because query tools typically have query-by-example interface.

[1253] Gleim #: 10.1.1 -- Source: Publisher

Answer (A) is incorrect because authentication is a security issue related to e-commerce.

Answer (B) is correct. E-commerce is the purchase and sale of goods and services by electronic means. E-commerce mayoccur via online transactions on public networks, electronic data interchange (EDI), and email. Security for e-commerceissues include the correct identification of transacting parties (authentication), determining who may rightfully makedecisions (authorization), and verification of payment data. While proliferation of computer viruses is a general securityissue with regard to information systems, it is not a specific risk associated with e-commerce.

Answer (C) is incorrect because authorization is a security issue related to e-commerce.

Answer (D) is incorrect because verification of payment data is a security issue related to e-commerce.

[1254] Gleim #: 10.1.2 -- Source: CIA 594 III-26

Answer (A) is incorrect because email can send text or document files, but the term encompasses a wide range oftransfers. EDI specifically applies to the system described in the question.

Answer (B) is incorrect because electronic funds transfer (EFT) refers to the transfer of money.

Answer (C) is correct. Electronic data interchange is the electronic transfer of documents between businesses. EDI wasdeveloped to enhance just-in-time (JIT) inventory management. Advantages include speed, reduction of clerical errors,and elimination of repetitive clerical tasks and their costs.

Answer (D) is incorrect because electronic data processing (EDP) is a generic term for computerized processing oftransaction data within organizations.

Gleim's CIA Test Prep: Part III: Business Analysis and Information TechnologyAnswer Explanations

(1312 questions)

Copyright 2008 Gleim Publications, Inc. Page 381Printed for Mamdouh Farag

Page 382: P.3 Answer Explanation

[1255] Gleim #: 10.1.3 -- Source: CPA AUD R99-9

Answer (A) is correct. EDI transactions are typically transmitted and processed in real time. Thus, EDI compresses thebusiness cycle by eliminating delays. The time required to receive and process an order, ship goods, and receive paymentis greatly reduced compared with that of a typical manual system. Accordingly, more rapid receipt of payment minimizesreceivables and improves cash flow.

Answer (B) is incorrect because use of a sophisticated processing system would increase the need to test computercontrols.

Answer (C) is incorrect because computer technology allows all transactions to be tested rather than just a sample.

Answer (D) is incorrect because EDI often uses a VAN (value-added network) as a third-party service provider, andreliance on controls provided by the VAN may be critical.

[1256] Gleim #: 10.1.4 -- Source: CIA 593 III-38

Answer (A) is correct. EDI is the communication of electronic documents directly from a computer in one entity to acomputer in another entity. EDI for business documents between unrelated parties has the potential to increase the risk ofunauthorized third-party access to systems because more outsiders will have access to internal systems.

Answer (B) is incorrect because systematic programming errors are the result of misspecification of requirements or lackof correspondence between specifications and programs.

Answer (C) is incorrect because inadequate knowledge bases are a function of lack of care in building them.

Answer (D) is incorrect because a benefit of EDI is to improve the efficiency and effectiveness of system use.

[1257] Gleim #: 10.1.5 -- Source: CIA 596 I-53

Answer (A) is correct. The float period is the time lag between transmittal of a regular check (a negotiable instrument)and its clearance through regular banking channels. Float is eliminated by EFT.

Answer (B) is incorrect because payment schedules may be based on the time required to process invoices, preparechecks, and transmit checks. Using EFT, payment is instantaneous, and payment schedules can be based on other criteria,e.g., discounts for prompt payment.

Answer (C) is incorrect because EFT allows for more effective control of payments and transfers among accounts.

Answer (D) is incorrect because integration of EDI and EFT eliminates manual input of transaction data, a process thatintroduces errors into the accounting system.

[1258] Gleim #: 10.1.6 -- Source: CIA 596 I-54

Answer (A) is incorrect because approval information is needed to provide an audit trail.

Answer (B) is incorrect because disaster recovery plans are needed to ensure that the company can continue to function ifthe system crashes.

Answer (C) is incorrect because individuals should not be able to update data without proper identification andauthentication.

Answer (D) is correct. One of the benefits of an EDI-EFT system is that it can provide remote access at any time from anyplace if telecommunications links are available. However, appropriate controls should prevent unauthorized access.

Gleim's CIA Test Prep: Part III: Business Analysis and Information TechnologyAnswer Explanations

(1312 questions)

Copyright 2008 Gleim Publications, Inc. Page 382Printed for Mamdouh Farag

Page 383: P.3 Answer Explanation

[1259] Gleim #: 10.1.7 -- Source: CIA 596 III-64

Answer (A) is incorrect because unauthorized access and activity is a risk specific to EFT.

Answer (B) is incorrect because inaccurate transaction processing (including duplication) is a risk specific to EFT.

Answer (C) is correct. EFT is a service provided by financial institutions worldwide that is based on EDI technology.EFT transaction costs are lower than for manual systems because documents and human intervention are eliminated fromthe transactions process.

Answer (D) is incorrect because inadequate backup and recovery capabilities is a risk specific to EFT.

[1260] Gleim #: 10.1.8 -- Source: CPA AUD R99-14

Answer (A) is correct. An audit trail allows for the tracing of a transaction from initiation to conclusion. Network andsender/recipient acknowledgments relate to the transaction flow and provide for the tracking of transactions.

Answer (B) is incorrect because message directories and header segments provide information controlling the message,such as originating and destination stations, message type and priority level, which are part of the message and not theaudit trail.

Answer (C) is incorrect because, although contingency and disaster recovery plans are important controls, they do notrelate to the audit trail.

Answer (D) is incorrect because, although maintaining control over security and mailbox codes is an important control, itdoes not relate to the audit trail.

[1261] Gleim #: 10.1.9 -- Source: Publisher

Answer (A) is incorrect because the AICPA-led consortium that developed XBRL has promoted the application as a freelylicensed product.

Answer (B) is incorrect because XBRL will facilitate the exchange of information, for example, for reporting to the SEC.

Answer (C) is correct. XBRL stands for eXtensible Business Reporting Language. It is being developed for business andaccounting applications. It is an XML-based application used to create, exchange, and analyze financial reportinginformation and is being developed for worldwide use.

Answer (D) is incorrect because XBRL will allow exchange of data across many platforms and will soon be integratedinto accounting software applications and products.

Gleim's CIA Test Prep: Part III: Business Analysis and Information TechnologyAnswer Explanations

(1312 questions)

Copyright 2008 Gleim Publications, Inc. Page 383Printed for Mamdouh Farag

Page 384: P.3 Answer Explanation

[1262] Gleim #: 10.1.10 -- Source: CPA AUD R99-11

Answer (A) is correct. In general, preventive controls are more important than detective controls because the benefitstypically outweigh the costs. In electronic processing, once a transaction is accepted, there is often little opportunity toapply detective controls. Thus, it is important to prevent errors or frauds before they happen.

Answer (B) is incorrect because the basic control objectives are the same regardless of the nature of the processing: toensure the integrity of the information and to safeguard the assets.

Answer (C) is incorrect because, to gather sufficient evidence in a sophisticated computer system, it is often necessary torely on the controls. Control risk may be assessed at below the maximum if relevant controls are identified and tested andif the resulting evidential matter provides the degree of assurance necessary to support the assessed level of control risk.

Answer (D) is incorrect because the level of segregation of duties achieved in a manual system is usually not feasible in acomputer system.

[1263] Gleim #: 10.1.11 -- Source: CIA 1193 III-43

Answer (A) is correct. If the company acknowledges messages initiated externally, the alleged sender will have theopportunity to recognize that it had not sent the message and will then be able to notify the company of the potentialforgery. Then corrective action can be taken by the company.

Answer (B) is incorrect because permitting only authorized employees to have access to transmission facilities controls forunauthorized access to the facilities but would not detect forged EDI messages.

Answer (C) is incorrect because delaying action on orders until a second order is received for the same goods defeats thepurpose of using EDI, namely, rapid communication followed by rapid response.

Answer (D) is incorrect because writing all incoming messages to a write-once/read-many device is a good practice, but itwill not detect forgeries.

[1264] Gleim #: 10.1.12 -- Source: Publisher

Answer (A) is correct. SET (Secured Electronic Transaction) is a trademarked protocol that provides a common securitystandards, especially with regard to Internet card purchases. It is supported by Visa and MasterCard through anorganization called SETCo. SET encrypts the details of payment transactions at all times so as to ensure privacy and dataintegrity. It also ensures that the identities of buyers and sellers are authenticated using digital signatures and digitalcertificates. SET is based on a hierarchy of certificate authorities that parallels financial relationships outside cyberspace.

Answer (B) is incorrect because SET is based in a hierarchy of certificate authorities that parallels financial relationshipsoutside cyberspace.

Answer (C) is incorrect because SET authenticates sellers and buyers using digital signatures and certificates.

Answer (D) is incorrect because SET is supported by Visa and MasterCard through an organization called SETCo.

Gleim's CIA Test Prep: Part III: Business Analysis and Information TechnologyAnswer Explanations

(1312 questions)

Copyright 2008 Gleim Publications, Inc. Page 384Printed for Mamdouh Farag

Page 385: P.3 Answer Explanation

[1265] Gleim #: 10.1.13 -- Source: CPA AUD R99-8

Answer (A) is incorrect because the audit trail is typically less apparent in an electronic environment than in a manualenvironment.

Answer (B) is incorrect because a key control is management’s establishment and monitoring of access controls.

Answer (C) is correct. The processing and transmission of electronic transactions, such as EFTs, virtually eliminateshuman interaction. This process not only helps eliminate errors but also allows for the rapid detection and recovery fromerrors when they do occur.

Answer (D) is incorrect because source documents are often eliminated in EFT transactions.

[1266] Gleim #: 10.1.14 -- Source: CIA 596 III-57

Answer (A) is incorrect because EDI transmits document data, not the actual document.

Answer (B) is correct. Electronic data interchange is the electronic transfer of documents between businesses. EDI wasdeveloped to enhance just-in-time (JIT) inventory management. Advantages include speed, reduction of clerical errors,and elimination of repetitive clerical tasks and their costs. Improved business relationships result because of the mutualbenefits conferred by EDI. Accordingly, some organizations require EDI.

Answer (C) is incorrect because liability for protection of a trading partner’s proprietary business data is a major risk thatmust be addressed by the control structure.

Answer (D) is incorrect because backup and contingency planning requirements are not diminished by use of EDItechnology. Materials purchased from non-EDI vendors are delivered to the receiving dock and recorded manually onreceiving reports. Copies of these reports are given to the purchasing and accounts payable departments. The internalaudit department is scheduled to complete a full audit of the purchasing and accounts payable cycle before the end of theyear. However, there are severe time pressures because other matters delayed the start of the audit.

[1267] Gleim #: 10.1.15 -- Source: CIA 1192 III-30

Answer (A) is incorrect because a request for an airline reservation requires an online, real-time reservations system.

Answer (B) is incorrect because withdrawal of cash from an automated teller is accomplished via online transactions tocopies of master files.

Answer (C) is incorrect because the transfer of summary data to headquarters may be accomplished with point-to-pointcommunications, known as distributed computing.

Answer (D) is correct. EDI is the communication of electronic documents directly from a computer in one entity to acomputer in another entity. Placement of order entry transactions from a customer to its supplier is an accepted use of EDIbetween trading partners.

Gleim's CIA Test Prep: Part III: Business Analysis and Information TechnologyAnswer Explanations

(1312 questions)

Copyright 2008 Gleim Publications, Inc. Page 385Printed for Mamdouh Farag

Page 386: P.3 Answer Explanation

[1268] Gleim #: 10.1.16 -- Source: CIA 596 I-55

Answer (A) is incorrect because a large volume of repetitive purchases from the same vendors suggests that EDI should beimplemented.

Answer (B) is correct. EDI is advantageous in a JIT environment because it provides the capacity for instantaneousordering. Moreover, a JIT environment already is characterized by the close vendor-purchaser cooperation required by anEDI system. Another reason for implementing an EDI system is that the purchaser has a large volume of transactions withthe same vendor(s). Otherwise, EDI may not be cost efficient.

Answer (C) is incorrect because a large volume of repetitive purchases from the same vendors suggests that EDI should beimplemented.

Answer (D) is incorrect because a large volume of custom purchases is inappropriate for EDI. The absence ofstandardization imposes unacceptable costs.

[1269] Gleim #: 10.1.17 -- Source: CIA 596 I-47

Answer (A) is incorrect because the number of vendors does not indicate the size of the purchases.

Answer (B) is incorrect because the amount of purchases is equally divided between the EDI and non-EDI systems anddoes not provide a basis for prioritizing risks.

Answer (C) is correct. Sound controls mitigate the risks associated with EDI. The question states that the internalauditing department’s prior involvement consisted of assessing and testing the EDI system. This review found nosignificant problems. Accordingly, the risk of the EDI system is decreased.

Answer (D) is incorrect because failure to examine EDI purchase controls increases risk.

[1270] Gleim #: 10.1.18 -- Source: CIA 596 III-59

Answer (A) is incorrect because using a third-party service provider does not require encryption.

Answer (B) is incorrect because using a third-party service provider does not require encryption.

Answer (C) is incorrect because use of public-switched data networks is not a requirement of EDI.

Answer (D) is correct. An auditor should review trading partner agreements and contracts with third-party serviceproviders. These documents should contain necessary clauses and appropriately limit liabilities. Moreover, legal counselshould have reviewed the agreements or contracts. An auditor should also determine whether the third-party serviceprovider’s operations and controls have been independently reviewed (e.g., by public accountants).

[1271] Gleim #: 10.1.19 -- Source: CIA 593 III-31

Answer (A) is incorrect because poor physical access controls represent a secondary exposure for compromise of remotedata communications lines.

Answer (B) is incorrect because network viruses represent a secondary exposure for compromise of remote datacommunications lines.

Answer (C) is incorrect because poor system documentation represent a secondary exposure for compromise of remotedata communications lines.

Answer (D) is correct. Leased telephone circuits represent a direct exposure to the risk of breached data integrity. Theyuse public lines that can be easily identified and tapped.

Gleim's CIA Test Prep: Part III: Business Analysis and Information TechnologyAnswer Explanations

(1312 questions)

Copyright 2008 Gleim Publications, Inc. Page 386Printed for Mamdouh Farag

Page 387: P.3 Answer Explanation

[1272] Gleim #: 10.1.20 -- Source: CIA 1196 III-63

Answer (A) is incorrect because improper change control procedures is a risk common to all information technologyenvironments. This risk is not higher than for other systems.

Answer (B) is correct. Unauthorized access to money transfer activities or data is an inherent and unique risk of EFTsystems. An unauthorized person may attempt to read, alter, or delete information in data files or to enter authorized fundtransfers. Hence, in the financial services industry, protection of confidential customer transactions is especiallyimportant. Moreover, unauthorized transfers subject a financial institution to a direct risk of serious loss.

Answer (C) is incorrect because insufficient online edit checks is a risk common to all information technologyenvironments. This risk is not higher than for other systems.

Answer (D) is incorrect because inadequate backups and disaster recovery procedures is a risk common to all informationtechnology environments. This risk is not higher than for other systems.

[1273] Gleim #: 10.1.21 -- Source: CIA 1193 III-47

Answer (A) is incorrect because determining whether the software was developed in a controlled environment is feasibleonly if the company develops it.

Answer (B) is incorrect because, if the company developed and maintained its own software, internal audit would beresponsible for evaluating that the software is backed up adequately to permit recovery in the event of a system failure.

Answer (C) is incorrect because, if the company purchased, leased, or paid for the use of the software, internal auditwould be responsible for evaluating that the software was acquired with legal counsel review of contract terms.

Answer (D) is correct. An EDI application should meet business objectives and satisfy user and control requirements. Theinternal auditors should consider the organization’s important EDI applications because they represent significant riskexposures and control problems. This role is within the scope of work of the internal auditors, who are charged withexamining and evaluating internal control and the quality of performance in carrying out assigned responsibilities.

[1274] Gleim #: 10.1.22 -- Source: CIA 597 III-52

Answer (A) is incorrect because unauthorized access to the EDI system should be prevented by procedures that ensure theeffective use of passwords, and data integrity and privacy should be maintained through the use of encryption andauthentication measures.

Answer (B) is correct. Tracking of customers’ functional acknowledgments, when required, will help to ensure successfultransmission of EDI transactions. Some possible controls include the provision of end-to-end acknowledgments,particularly when multiple, interconnected networks are involved, and maintenance of a tickler file of outstandingfunctional acknowledgments, with issuance of warnings for those that are overdue.

Answer (C) is incorrect because contractual issues should be resolved by the company and its trading partners before EDIis implemented.

Answer (D) is incorrect because the risk that EDI data may not be completely and accurately processed is minimized bysystem-based controls, not by acknowledgments from trading partners.

Gleim's CIA Test Prep: Part III: Business Analysis and Information TechnologyAnswer Explanations

(1312 questions)

Copyright 2008 Gleim Publications, Inc. Page 387Printed for Mamdouh Farag

Page 388: P.3 Answer Explanation

[1275] Gleim #: 10.1.23 -- Source: CIA 591 III-50

Answer (A) is incorrect because employee access to the computer system via an ID and an encrypted password isconsidered acceptable. Encrypted passwords further decrease the likelihood of unauthorized access.

Answer (B) is incorrect because message sequencing detects unauthorized access by numbering each message andincrementing each message by one more than the last one sent. This procedure will detect a gap or duplicate.

Answer (C) is incorrect because allowing certain types of transactions (such as payroll transactions) to be made only atspecific terminals minimizes the likelihood of unauthorized access.

Answer (D) is correct. The system should employ automatic dial-back to prevent intrusion by unauthorized parties. Thisprocedure accepts an incoming modem call, disconnects, and automatically dials back a prearranged number to establish apermanent connection for data transfer or inquiry.

[1276] Gleim #: 10.1.24 -- Source: CIA 596 I-46

Answer (A) is correct. An exception report (error listing) should be issued so that company personnel can investigate thediscrepancy, determine its cause, and take appropriate corrective action.

Answer (B) is incorrect because the company should not pay for goods not received.

Answer (C) is incorrect because the company should first determine the cause of the discrepancy.

Answer (D) is incorrect because the company should not pay for goods not received.

[1277] Gleim #: 10.1.25 -- Source: CIA 593 III-59

Answer (A) is incorrect because catalog phone orders do not involve EDI. They are not computer to computer.

Answer (B) is incorrect because the ordering of parts by computer is not OLRT. Processing does not occur.

Answer (C) is incorrect because the catalog phone orders are OLRT, and the ordering of parts by computer is EDI.

Answer (D) is correct. The catalog sales firm is using an online processing system known as an online, real-time system(OLRT). OLRT systems handle transactions as they are entered, processing the input and providing output soon enough toaffect a current decision making process, e.g., inventory and credit status of phone call orders. The sending of upcomingproduction schedules and parts orders by and manufacturer’s computer to a supplier’s computer is an example ofelectronic data interchange (EDI). EDI is the communication of electronic documents directly from one entity’s computerto another’s computer.

[1278] Gleim #: 10.1.26 -- Source: CIA 1193 III-51

Answer (A) is correct. A well-managed company seeks to minimize the sum of the costs of holding and orderinginventory. An EDI system reduces the costs of ordering, thereby permitting the company to order more frequently. Morefrequent ordering reduces inventory and the associated holding costs.

Answer (B) is incorrect because the company would not need to ensure that it maintained a 25-day buffer stock if it canorder more frequently.

Answer (C) is incorrect because tracking materials through production is not an EDI, which is the intercompany exchangeof business information.

Answer (D) is incorrect because scheduling production is not an application of EDI.

Gleim's CIA Test Prep: Part III: Business Analysis and Information TechnologyAnswer Explanations

(1312 questions)

Copyright 2008 Gleim Publications, Inc. Page 388Printed for Mamdouh Farag

Page 389: P.3 Answer Explanation

[1279] Gleim #: 10.1.27 -- Source: CIA 1196 I-11

Answer (A) is correct. The number of systems personnel employed may reflect differences in operating philosophy(outsourcing vs. in-house development of applications). However, the number of personnel in each company is a lessserious concern than the compatibility of hardware and software.

Answer (B) is incorrect because Company A has little EDI experience. Hence, the greater the number of vendors that mustbe connected with Company A, the greater the risk exposure.

Answer (C) is incorrect because the difficulty and expense of conversion will be increased if the computer systems havesignificant compatibility problems.

Answer (D) is incorrect because the greater the complexity of the systems to be integrated, the greater the risk exposure.

[1280] Gleim #: 10.1.28 -- Source: CIA 1193 III-49

Answer (A) is correct. If the company gives the supplier more information about use of the materials, the supplier may beable to plan its production more effectively. It could then reduce its inventory of the materials and its inventory costs, thuspermitting it to charge a lower price.

Answer (B) is incorrect because the company could demand that the supplier reduce the price of the materials, but thesupplier could then decline to supply them.

Answer (C) is incorrect because other suppliers may also charge a high price.

Answer (D) is incorrect because if the special materials are needed in the primary product line, it is unlikely that thecompany would discontinue it before investigating alternatives, e.g., working with the supplier to help the suppliermanage its inventory.

[1281] Gleim #: 10.1.29 -- Source: CIA 597 III-51

Answer (A) is correct. Marked benefits arise when EDI is tied to strategic efforts that alter, not mirror, previouspractices. Applying EDI to an inefficient process results in continuing to do things the wrong way, only faster. Hence, theinitial phase of EDI implementation includes understanding the organization’s mission and an analysis of its activities aspart of an integrated solution to the organization’s needs.

Answer (B) is incorrect because the prerequisite for EDI success is an understanding of the mission of the business andthe processes and flows that support its goals, followed by cooperation with external partners. Purchasing new hardware isa subsequent step.

Answer (C) is incorrect because, before applying EDI technology to the business, EDI must be viewed as part of an overallintegrated solution to organizational requirements.

Answer (D) is incorrect because EDI is not a solution by itself. Instead of considering how to transmit and receivetransactions, a company must first analyze the entire process.

Gleim's CIA Test Prep: Part III: Business Analysis and Information TechnologyAnswer Explanations

(1312 questions)

Copyright 2008 Gleim Publications, Inc. Page 389Printed for Mamdouh Farag

Page 390: P.3 Answer Explanation

[1282] Gleim #: 10.1.30 -- Source: CIA 1193 III-46

Answer (A) is incorrect because the company and its customers may obtain their EDI-related software from the samevendor but still have software incompatibility problems if they do not synchronize their installation of updated versions.

Answer (B) is correct. EDI entails the exchange of common business data converted into standard message formats. Thus,two crucial requirements are that the participants agree on transaction formats and that translation software be developedto convert messages into a form understandable by other companies. Thus, if one company changes its software, its tradingpartners must also do so.

Answer (C) is incorrect because, as business requirements change, it may not be possible to use the same software in thesame ways indefinitely.

Answer (D) is incorrect because, even if the company and its customers each write their own versions, synchronizationproblems will arise from updates.

[1283] Gleim #: 10.1.31 -- Source: CIA 1193 III-45

Answer (A) is correct. Before sending or receiving EDI messages, a company should execute a trading partner agreementwith its customers and suppliers. All parties should understand their responsibilities, the messages each will initiate, howthey will interpret messages, the means of authenticating and verifying the completeness and accuracy of messages, themoment when the contract between the parties is effective, the required level of security, etc.

Answer (B) is incorrect because the company may intend to reduce inventory levels, but that intention is unrelated to thetiming of its first EDI messages.

Answer (C) is incorrect because the company may want to demand or encourage all its customers and suppliers toimplement EDI capabilities, but that request is independent of sending and receiving messages.

Answer (D) is incorrect because it is not possible to evaluate the effectiveness of EDI transmissions until after they occur.

[1284] Gleim #: 10.2.32 -- Source: CIA 591 I-40

Answer (A) is incorrect because contingency planning concerns the arrangements for alternative processing facilities inthe event of equipment failure.

Answer (B) is incorrect because a feasibility study is one of the phases in the systems development life cycle.

Answer (C) is correct. Planning is as important for the information systems function as for any other part of theorganization. The master plan for this function should be consistent with the strategic plan for the organization andinclude goals and objectives, an inventory of current capacity, and a forecast of future needs. The plan is the basis fordetermining hardware needs.

Answer (D) is incorrect because exception reports are meant to list errata and bring them to the attention of management.

Gleim's CIA Test Prep: Part III: Business Analysis and Information TechnologyAnswer Explanations

(1312 questions)

Copyright 2008 Gleim Publications, Inc. Page 390Printed for Mamdouh Farag

Page 391: P.3 Answer Explanation

[1285] Gleim #: 10.2.33 -- Source: CIA 593 III-42

Answer (A) is incorrect because greater online access may or may not be helpful, depending on the user organization’sneeds.

Answer (B) is incorrect because marketplace creates competitive pressures for enhanced functions in systems.

Answer (C) is correct. An important management challenge is to integrate the planning, design, and implementation ofcomplex application systems with the strategy of the organization, which will permit the best possible response to quicklychanging information requirements.

Answer (D) is incorrect because more pervasive use of automated controls may be independent of responding quickly tochanging information requirements.

[1286] Gleim #: 10.2.34 -- Source: CIA, adapted

Answer (A) is incorrect because contingency planning refers to the arrangements for alternative processing facilities in theevent of equipment failure.

Answer (B) is incorrect because the feasibility study is one of the phases in the systems development life cycle.

Answer (C) is correct. The plan should include goals and objectives, an inventory of current capacity, and a forecast offuture needs.

Answer (D) is incorrect because exception reports are meant to highlight problems and bring them to the attention ofmanagement.

[1287] Gleim #: 10.3.35 -- Source: Publisher

Answer (A) is correct. The benefits of ERP may significantly derive from the business process reengineering that isneeded for its implementation. Using ERP software that reflects the best practices forces the linked subunits in theorganization not only to redesign and improve their processes but also to conform to one standard.

Answer (B) is incorrect because the disadvantages of ERP are its extent and complexity, which make customization of thesoftware difficult and costly.

Answer (C) is incorrect because ERP software is costly and complex. It is usually installed only by the largest enterprises.

Answer (D) is incorrect because implementing an ERP system is likely to encounter significant resistance because of itscomprehensiveness.

Gleim's CIA Test Prep: Part III: Business Analysis and Information TechnologyAnswer Explanations

(1312 questions)

Copyright 2008 Gleim Publications, Inc. Page 391Printed for Mamdouh Farag

Page 392: P.3 Answer Explanation

[1288] Gleim #: 10.3.36 -- Source: Publisher

Answer (A) is incorrect because an MRP II system does not integrate all the subsystems internal to the organization (back-office functions), such as human resources and customer service.

Answer (B) is correct. Manufacturing resource planning (MRP II) continued the evolution begun with MRP. It is aclosed-loop manufacturing system that integrates all facets of manufacturing, including production, sales, inventories,schedules, and cash flows. The same system is used for accounting and finance functions, which use the same transactionsand numbers. MRP II uses an MPS (master production schedule), a statement of the anticipated manufacturing schedulefor selected items for selected periods. MRP also uses the MPS. Thus, MRP is a component of an MRP II system.

Answer (C) is incorrect because MRP II includes forecasting and planning capacities for generating cash and otherbudgets.

Answer (D) is incorrect because MRP, MRP II, and traditional ERP do not provide for front-office functions, that is,connections with customers, suppliers, owners, creditors, and strategic allies.

[1289] Gleim #: 10.3.37 -- Source: Publisher

Answer (A) is incorrect because ERP II but not traditional ERP is capable of customer tracking of the order’s progress andautomatic replenishment of inventory by a supplier.

Answer (B) is incorrect because ERP II but not traditional ERP is capable of customer tracking of the order’s progress andautomatic replenishment of inventory by a supplier.

Answer (C) is correct. The traditional ERP system is one in which subsystems share data and coordinate their activities.Thus, if marketing receives an order, it can quickly verify that inventory is sufficient to notify shipping to process theorder. Otherwise, production is notified to manufacture more of the product, with a consequent automatic adjustment ofoutput schedules. If materials are inadequate for this purpose, the system will issue a purchase order. If more labor isneeded, human resources will be instructed to reassign or hire employees. However, the subsystems in a traditional ERPsystem are internal to the organization. Hence, they are often called back-office functions. The information produced isprincipally (but not exclusively) intended for internal use by the organization’s managers.

The current generation of ERP software (ERP II) has added front-office functions. Consequently, ERP II, but nottraditional ERP, is capable of customer tracking of the order’s progress and automatic replenishment of inventory by asupplier.

Answer (D) is incorrect because ERP II but not traditional ERP is capable of customer tracking of the order’s progress andautomatic replenishment of inventory by a supplier.

[1290] Gleim #: 10.3.38 -- Source: Publisher

Answer (A) is correct. Current ERP systems have a client-server configuration with possibly scores or hundreds of client(user) computers. Clients may be thin or fat. So-called thin clients have little processing ability, but fat clients may havesubstantial processing power. The system may have multiple servers to run applications and contain databases. Thenetwork architecture may be in the form of a local area network (LAN) or wide area network (WAN), or users mayconnect with the server(s) via the Internet. An ERP system may use almost any of the available operating systems anddatabase management systems.

Answer (B) is incorrect because an ERP system also may have thin clients connected via a LAN.

Answer (C) is incorrect because an ERP system also may have thin clients connected via a LAN or the Internet with oneserver.

Answer (D) is incorrect because an ERP system also may have fat clients connected via a WAN or the Internet to multipleservers.

Gleim's CIA Test Prep: Part III: Business Analysis and Information TechnologyAnswer Explanations

(1312 questions)

Copyright 2008 Gleim Publications, Inc. Page 392Printed for Mamdouh Farag

Page 393: P.3 Answer Explanation

[1291] Gleim #: 10.3.39 -- Source: Publisher

Answer (A) is incorrect because an ERP system uses a central database and a database management system. Afundamental characteristic of a database is that applications are independent of the physical structure of the database.Writing programs or designing applications to use the database requires only the names of desired data items, not theirlocations.

Answer (B) is incorrect because an ERP system eliminates data redundancy.

Answer (C) is incorrect because an ERP system is characterized by one-time data updating for all organizational functions.

Answer (D) is correct. An advantage of an ERP system is the elimination of data redundancy through the use of a centraldatabase. In principle, information about an item of data is stored once, and all functions have access to it. Thus, when theitem (such as a price) is updated, the change is effectively made for all functions. The result is reliability (data integrity).

[1292] Gleim #: 10.3.40 -- Source: Publisher

Answer (A) is incorrect because customer relationship management applications in ERP II extend to customer service,finance-related matters, sales, and database creation and maintenance. Integrated data are helpful in better understandingcustomer needs, such as product preference or location of retail outlets.

Answer (B) is incorrect because partner relationship management applications connect the organization not only with suchpartners as customers and suppliers but also with owners, creditors, and strategic allies (for example, other members of ajoint venture).

Answer (C) is correct. An advanced planning and scheduling system may be an element of a supply chain managementapplication for a manufacturer. It controls the flow of materials and components within the chain. Schedules are createdgiven projected costs, lead times, and inventories.

Answer (D) is incorrect because an advanced planning scheduling system is used by a manufacturer to control flowsthrough the supply chain. Other software permits customers to obtain information about order availability.

[1293] Gleim #: 10.3.41 -- Source: Publisher

Answer (A) is incorrect because inventory control is a back-office function.

Answer (B) is incorrect because human resources is a back-office function.

Answer (C) is incorrect because purchasing is a back-office function.

Answer (D) is correct. The current generation of ERP software (ERP II) has added front-office functions. Customerrelationship management applications in ERP II extend to customer service, finance-related matters, sales, and databasecreation and maintenance. Integrated data are helpful in better understanding customer needs, such as product preferenceor location of retail outlets. Thus, the organization may be able to optimize its sales forecasts, product line, and inventorylevels.

Gleim's CIA Test Prep: Part III: Business Analysis and Information TechnologyAnswer Explanations

(1312 questions)

Copyright 2008 Gleim Publications, Inc. Page 393Printed for Mamdouh Farag

Page 394: P.3 Answer Explanation

[1294] Gleim #: 10.3.42 -- Source: CIA 1193 IV-25

Answer (A) is incorrect because the EOQ model focuses on the trade-off between carrying and ordering costs.

Answer (B) is correct. Materials requirements planning (MRP) is a system that translates a production schedule intorequirements for each component needed to meet the schedule. It is usually implemented in the form of a computer-basedinformation system designed to plan and control raw materials used in production. It assumes that forecasted demand isreasonably accurate and that suppliers can deliver based upon this accurate schedule. MRP is a centralized push-throughsystem; output based on forecasted demand is pushed through to the next department or to inventory.

Answer (C) is incorrect because linear programming is a decision model concerned with allocating scarce resources tomaximize profit or minimize costs.

Answer (D) is incorrect because JIT is a decentralized demand-pull system. It is driven by actual demand.

[1295] Gleim #: 10.3.43 -- Source: Publisher

Answer (A) is incorrect because the difficulty of installing ERP systems is a disadvantage to the auditor.

Answer (B) is incorrect because ERP systems often require the client to depart from the traditional functional division ofduties such as accounting, finance, and marketing. The result is increased audit risk.

Answer (C) is correct. ERP systems have a variety of controls and report generation functions that allow the auditor toabstract and monitor data collected and processed. Some ERP systems have built-in audit functions.

Answer (D) is incorrect because ERP systems are very costly and therefore have been implemented only by largeorganizations. However, the trend is for more and more organizations to install these systems.

[1296] Gleim #: 10.4.44 -- Source: CPA AUD R02-5

Answer (A) is incorrect because a hoax virus is a false notice about the existence of a computer virus. It is usuallydisseminated through use of distribution lists and is sent by email or via an internal network.

Answer (B) is incorrect because a web crawler (a spider or bot) is a computer program created to access and readinformation on websites. The results are included as entries in the index of a search engine.

Answer (C) is correct. A Trojan horse is a computer program that appears friendly, for example, a game, but that actuallycontains an application destructive to the computer system.

Answer (D) is incorrect because a killer application is one that is so useful that it may justify widespread adoption of anew technology.

Gleim's CIA Test Prep: Part III: Business Analysis and Information TechnologyAnswer Explanations

(1312 questions)

Copyright 2008 Gleim Publications, Inc. Page 394Printed for Mamdouh Farag

Page 395: P.3 Answer Explanation

[1297] Gleim #: 10.4.45 -- Source: CIA 591 III-24

Answer (A) is incorrect because comparing software with authorized versions is a detective control used to determinewhether only authorized versions of the software are being used on the system.

Answer (B) is incorrect because executing virus exterminator programs is a corrective control against a computer virus.

Answer (C) is correct. Preventive controls are designed to prevent errors before they occur. Detective and correctivecontrols attempt to identify and correct errors. Preventive controls are usually more cost beneficial than detective orcorrective controls. Allowing only authorized software from known sources to be used on the system is a preventivemeasure. The authorized software from known sources is expected to be free of viruses.

Answer (D) is incorrect because preparing and testing a plan for virus recovery is a corrective control against a computervirus.

[1298] Gleim #: 10.4.46 -- Source: CIA 1196 III-40

Answer (A) is incorrect because power surges are caused by hardware or power supply problems.

Answer (B) is correct. The effects of computer viruses range from harmless messages to complete destruction of all datawithin the system. A symptom of a virus would be the unexplained loss of or change to data.

Answer (C) is incorrect because inadequate back-up, recovery, and contingency plans are operating policy weaknesses.

Answer (D) is incorrect because copyright violations represent policy or compliance problems.

[1299] Gleim #: 10.4.47 -- Source: CIA 595 III-67

Answer (A) is incorrect because, if viruses infect a system, the company should restore the system with authorizedsoftware, but this procedure does not minimize the likelihood of initial infection.

Answer (B) is incorrect because, if viruses infect programs that the company created, it should recompile the programsfrom source code backups, but this procedure does not minimize the likelihood of initial infection.

Answer (C) is incorrect because instituting program change control procedures is good practice but does not minimize thelikelihood of the system being infected initially.

Answer (D) is correct. Software from recognized sources should be tested in quarantine (for example, in atest/development machine or a stand-alone personal computer) because even vendor-supplied software may be infectedwith viruses. The software should be run with a vaccine program and tested for the existence of logic bombs, etc.

[1300] Gleim #: 10.4.48 -- Source: CIA 1196 III-41

Answer (A) is incorrect because viruses are spread through the distribution of contaminated programs.

Answer (B) is incorrect because backing up files does not increase the chances of a virus entering the computer system.

Answer (C) is correct. Viruses are spread through shared data. Downloading public-domain software carries a risk thatcontaminated data may enter the computer.

Answer (D) is incorrect because original copies of purchased software on hard disk drives should be free of viruses.

Gleim's CIA Test Prep: Part III: Business Analysis and Information TechnologyAnswer Explanations

(1312 questions)

Copyright 2008 Gleim Publications, Inc. Page 395Printed for Mamdouh Farag

Page 396: P.3 Answer Explanation

[1301] Gleim #: 10.4.49 -- Source: CIA 1195 III-37

Answer (A) is correct. Antivirus software designed to identify and remove known viruses is sometimes known as avaccine. A vaccine works only for known viruses and may not be effective for variants of those viruses or new viruses.

Answer (B) is incorrect because having antivirus software is unlikely to make software installation overly complex.

Answer (C) is incorrect because antivirus software need not interfere with system operations. Its execution can bescheduled in advance so as not to interfere with running programs.

Answer (D) is incorrect because antivirus software can be set to execute at times when it would not consume too manysystem resources, e.g., at startup.

[1302] Gleim #: 10.4.50 -- Source: CIA 594 III-29

Answer (A) is incorrect because running a different program as a test may cause the virus to spread and do additionaldamage.

Answer (B) is incorrect because rebooting the system may cause the virus to spread and do additional damage.

Answer (C) is correct. The described condition is a symptom of a virus. Many viruses will spread and cause additionaldamage. Use of an appropriate antivirus program may identify and even eliminate a viral infection. Ways to minimizecomputer virus risk in a networked system include restricted access, regularly updated passwords, periodic testing ofsystems with virus detection software, and the use of antivirus software on all shareware prior to introducing it into thenetwork.

Answer (D) is incorrect because backing up hard disk files may cause the virus to spread and do additional damage.

[1303] Gleim #: 10.4.51 -- Source: CIA 1190 III-19

Answer (A) is incorrect because the programmer would most likely be denied access to the center.

Answer (B) is incorrect because the programmer would not know the necessary passwords.

Answer (C) is incorrect because collusion is less likely than individual wrongdoing.

Answer (D) is correct. Viruses are a form of computer sabotage. They are programs hidden within other programs thathave the capacity to duplicate themselves and infect other systems. Sharing of storage media or participation in computernetworks creates exposure to viruses. Viruses may result in actions ranging from harmless pranks to erasure of files andprograms. A back door is a shortcut created in an operating system that permits a programmer simple access to the system.

Gleim's CIA Test Prep: Part III: Business Analysis and Information TechnologyAnswer Explanations

(1312 questions)

Copyright 2008 Gleim Publications, Inc. Page 396Printed for Mamdouh Farag

Page 397: P.3 Answer Explanation

[1304] Gleim #: 10.4.52 -- Source: CIA 1195 III-61

Answer (A) is correct. Viruses are harmful programs that disrupt memory and processing functions and may destroy data.They spread from network to network, from infected diskettes, or from infected machines. Hence, connecting allnetworked personal computers through a host computer to outside networks increases the exposure of all of a company’scomputers to viruses.

Answer (B) is incorrect because whether uploaded files are properly edited and validated is independent of whetherexternal links to other networks exist.

Answer (C) is incorrect because whether data downloaded to the personal computers is sufficiently timely is independentof whether external links to other networks exist.

Answer (D) is incorrect because whether software maintenance on the personal computers becomes more costly isindependent of whether external links to other networks exist.

[1305] Gleim #: 10.4.53 -- Source: Publisher

Answer (A) is correct. A denial-of-service (DOS) attack is an attempt to overload a system (e.g., a network or Webserver) with false messages so that it cannot function (a system crash). A distributed DOS attack comes from multiplesources, for example, the machines of innocent parties infected by Trojan horses. When activated, these programs sendmessages to the target and leave the connection open. A DOS may establish as many network connections as possible toexclude other users, overload primary memory, or corrupt file systems.

Answer (B) is incorrect because a man-in-the-middle attack takes advantage of network packet sniffing and routing andtransport protocols to access packets flowing through a network.

Answer (C) is incorrect because a brute-force attack uses password cracking software to try large numbers of letter andnumber combinations to access a network.

Answer (D) is incorrect because password-cracking software is used to access a network by using a large number of letterand number combinations.

[1306] Gleim #: 10.4.54 -- Source: Publisher

Answer (A) is incorrect because a brute-force attack uses password cracking software to try large numbers of letter andnumber combinations to access a network.

Answer (B) is incorrect because sniffing is use of software to eavesdrop on information sent by a user to the host computerof a website.

Answer (C) is incorrect because a man-in-the-middle attack takes advantage of network packet sniffing and routing andtransport protocols to access packets flowing through a network.

Answer (D) is correct. Passwords, user account numbers, and other information may be stolen using techniques such asTrojan horses, IP spoofing, and packet sniffers. Spoofing is identity misrepresentation in cyberspace, for example, byusing a false website to obtain information about visitors.

Gleim's CIA Test Prep: Part III: Business Analysis and Information TechnologyAnswer Explanations

(1312 questions)

Copyright 2008 Gleim Publications, Inc. Page 397Printed for Mamdouh Farag

Page 398: P.3 Answer Explanation

[1307] Gleim #: 10.4.55 -- Source: Publisher

Answer (A) is incorrect because a host IDS provides maximum protection only when the software is installed on eachcomputer. It may operate in the following ways: The aggressive response is to monitor every call on the operating systemand application as it occurs. A less effective method of preventing attacks is analysis of access log files. A host IDS mayalso identify questionable processes and verify the security of system files.

Answer (B) is correct. A network IDS works by using sensors to examine packets traveling on the network. Each sensormonitors only the segment of the network to which it is attached. A packet is examined if it matches a signature. Stringsignatures (certain strings of text) are potential signs of attack. Port signatures alert the IDS that a point subject tofrequent intrusion attempts may be under attack. A header signature is a suspicious combination in a packet header.

Answer (C) is incorrect because an IDS is not limited to knowledge-based detection. Knowledge-based detection is basedon information about the system’s weaknesses and searches for intrusions that take advantage of them.

Answer (D) is incorrect because an IDS is not limited to behavior-based detection. Behavior-based detection presumesthat an attack will cause an observable anomaly. Actual and normal system behavior (a model of expected operations) arecompared. A discrepancy results in an alert.

[1308] Gleim #: 10.4.56 -- Source: CIA 593 II-20

Answer (A) is incorrect because hiring policies can provide assurance of qualified personnel for operation of the system,but they cannot prevent introduction of viruses from bulletin boards or from outside sources.

Answer (B) is incorrect because software programs can identify and neutralize known viruses but may not recognize andproperly neutralize new strains of a computer virus.

Answer (C) is correct. Acceptably safe computing can be achieved by carefully crafted policies and procedures used inconjunction with antivirus and access control software.

Answer (D) is incorrect because physical protection devices can reduce access but cannot prevent introduction of virusesby errant employees or from outside sources.

[1309] Gleim #: 10.4.57 -- Source: CIA, adapted

Answer (A) is incorrect because a logic bomb is a mechanism for releasing a system attack of some kind, which istriggered when a particular condition (for example, a certain date or system operation) occurs.

Answer (B) is incorrect because a virus is a code fragment (not an independent program) that reproduces by attaching toanother program.

Answer (C) is correct. A worm is an independent program that reproduces by copying itself from one system to anotherover a network and consumes computer and network resources.

Answer (D) is incorrect because a Trojan horse is an independent program that appears to perform a useful function, buthides another unauthorized program inside it.

Gleim's CIA Test Prep: Part III: Business Analysis and Information TechnologyAnswer Explanations

(1312 questions)

Copyright 2008 Gleim Publications, Inc. Page 398Printed for Mamdouh Farag

Page 399: P.3 Answer Explanation

[1310] Gleim #: 10.4.58 -- Source: CIA, adapted

Answer (A) is incorrect because statement III is also correct.

Answer (B) is incorrect because statement II is not correct, but statement III is correct.

Answer (C) is correct. A confidential mail message should not be retained on the server once the user has downloaded itto a personal computer. The security of electronic mail is limited to the capability of the computer system on which itoperates. Several electronic mail administrators and locations with varying levels of security exists in larger organizations.

Answer (D) is incorrect because statement II is not correct, but statement I is correct.

[1311] Gleim #: 10.4.59 -- Source: CIA, adapted

Answer (A) is incorrect because there is no limitation on the number access ports.

Answer (B) is correct. The most difficult aspect of using Internet resources is locating the best information given thelarge number of information sources.

Answer (C) is incorrect because the only equipment required for accessing Internet resources is a computer, a modem, atelephone or other access line, and basic communication software.

Answer (D) is incorrect because organizations routinely provide Internet access to their employees, and individuals canobtain access through individual subscription to commercial service providers.

[1312] Gleim #: 10.4.60 -- Source: CIA 596 III-73

Answer (A) is incorrect because use of unlicensed software increases the risk of viral infection, and its use can be detectedby software checking routines. Moreover, the cost of the software is not relevant. Any software may contain a virus.

Answer (B) is incorrect because use of unlicensed software increases the risk of viral infection, and its use can be detectedby software checking routines. Moreover, the cost of the software is not relevant. Any software may contain a virus.

Answer (C) is incorrect because use of unlicensed software increases the risk of viral infection, and its use can be detectedby software checking routines. Moreover, the cost of the software is not relevant. Any software may contain a virus.

Answer (D) is correct. Antivirus measures should include strict adherence to software acquisition policies. Unlicensedsoftware is less likely to have come from reputable vendors and to have been carefully tested. Special software is availableto test software in use to determine whether it has authorized.

Gleim's CIA Test Prep: Part III: Business Analysis and Information TechnologyAnswer Explanations

(1312 questions)

Copyright 2008 Gleim Publications, Inc. Page 399Printed for Mamdouh Farag